ΠœΠΎΠΌΠ΅Π½Ρ‚ ΠΈΠ½Π΅Ρ€Ρ†ΠΈΠΈ ΠΊΠ²Π°Π΄Ρ€Π°Ρ‚Π°: Как ΠΏΠΎΡΡ‡ΠΈΡ‚Π°Ρ‚ΡŒ ΠΌΠΎΠΌΠ΅Π½Ρ‚ ΠΈΠ½Π΅Ρ€Ρ†ΠΈΠΈ ΠΊΠ²Π°Π΄Ρ€Π°Ρ‚Π°? : ΠœΠ΅Ρ…Π°Π½ΠΈΠΊΠ° ΠΈ Π’Π΅Ρ…Π½ΠΈΠΊΠ°

Π‘ΠΎΠ΄Π΅Ρ€ΠΆΠ°Π½ΠΈΠ΅

ΠœΠΎΠΌΠ΅Π½Ρ‚ ΠΈΠ½Π΅Ρ€Ρ†ΠΈΠΈ ΠΈ ΠΌΠΎΠΌΠ΅Π½Ρ‚ сопротивлСния

05-12-2012: ΠΠ΄ΠΎΠ»ΡŒΡ„ Π‘Ρ‚Π°Π»ΠΈΠ½

Π‘Ρ‹Π»ΠΎ Π±Ρ‹ Π½Π΅ΠΏΠ»ΠΎΡ…ΠΎ ΠΎΠ±ΡŠΡΡΠ½ΠΈΡ‚ΡŒ Π½Π° наглядном ΠΏΡ€ΠΈΠΌΠ΅Ρ€Π΅ для особо ΠΎΠ΄Π°Ρ€Π΅Π½Π½Ρ‹Ρ…, Ρ‚ΠΈΠΏΠ° мСня, Ρ‡Ρ‚ΠΎ Ρ‚Π°ΠΊΠΎΠ΅ ΠΌΠΎΠΌΠ΅Π½Ρ‚ ΠΈΠ½Π΅Ρ€Ρ†ΠΈΠΈ ΠΈ с Ρ‡Π΅ΠΌ Π΅Π³ΠΎ Сдят. На спСциализированных сайтах ΠΊΠ°ΠΊ-Ρ‚ΠΎ всё ΠΎΡ‡Π΅Π½ΡŒ Π·Π°ΠΏΡƒΡ‚Π°Π½Π½ΠΎ, Π° Ρƒ Π”ΠΎΠΊΠ° Π΅ΡΡ‚ΡŒ явный Ρ‚Π°Π»Π°Π½Ρ‚ довСсти ΠΈΠ½Ρ„ΠΎΡ€ΠΌΠ°Ρ†ΠΈΡŽ, Π±Ρ‹Ρ‚ΡŒ ΠΌΠΎΠΆΠ΅Ρ‚ Π½Π΅ ΡΠ°ΠΌΡƒΡŽ ΡΠ»ΠΎΠΆΠ½ΡƒΡŽ, Π½ΠΎ ΠΎΡ‡Π΅Π½ΡŒ Π³Ρ€Π°ΠΌΠΎΡ‚Π½ΠΎ ΠΈ понятно


05-12-2012: Π”ΠΎΠΊΡ‚ΠΎΡ€ Π›ΠΎΠΌ

Π’ ΠΏΡ€ΠΈΠ½Ρ†ΠΈΠΏΠ΅, Ρ‡Ρ‚ΠΎ Ρ‚Π°ΠΊΠΎΠ΅ ΠΌΠΎΠΌΠ΅Π½Ρ‚ ΠΈΠ½Π΅Ρ€Ρ†ΠΈΠΈ ΠΈ ΠΎΡ‚ΠΊΡƒΠ΄Π° ΠΎΠ½ взялся, достаточно ΠΏΠΎΠ΄Ρ€ΠΎΠ±Π½ΠΎ объяснСно Π² ΡΡ‚Π°Ρ‚ΡŒΠ΅ “ΠžΡΠ½ΠΎΠ²Ρ‹ сопромата, расчСтныС Ρ„ΠΎΡ€ΠΌΡƒΠ»Ρ‹”, здСсь лишь ΠΏΠΎΠ²Ρ‚ΠΎΡ€ΡŽΡΡŒ: “W – это ΠΌΠΎΠΌΠ΅Π½Ρ‚ сопротивлСния ΠΏΠΎΠΏΠ΅Ρ€Π΅Ρ‡Π½ΠΎΠ³ΠΎ сСчСния Π±Π°Π»ΠΊΠΈ, Π΄Ρ€ΡƒΠ³ΠΈΠΌΠΈ словами, ΠΏΠ»ΠΎΡ‰Π°Π΄ΡŒ сТимаСмой ΠΈΠ»ΠΈ растягиваСмой части сСчСния Π±Π°Π»ΠΊΠΈ, умноТСнная Π½Π° ΠΏΠ»Π΅Ρ‡ΠΎ дСйствия Ρ€Π°Π²Π½ΠΎΠ΄Π΅ΠΉΡΡ‚Π²ΡƒΡŽΡ‰Π΅ΠΉ силы”. ΠœΠΎΠΌΠ΅Π½Ρ‚ сопротивлСния Π½Π΅ΠΎΠ±Ρ…ΠΎΠ΄ΠΈΠΌΠΎ Π·Π½Π°Ρ‚ΡŒ для расчСтов конструкции Π½Π° ΠΏΡ€ΠΎΡ‡Π½ΠΎΡΡ‚ΡŒ, Ρ‚.Π΅. ΠΏΠΎ ΠΏΡ€Π΅Π΄Π΅Π»ΡŒΠ½Ρ‹ΠΌ напряТСниям.

ΠœΠΎΠΌΠ΅Π½Ρ‚ ΠΈΠ½Π΅Ρ€Ρ†ΠΈΠΈ Π½Π΅ΠΎΠ±Ρ…ΠΎΠ΄ΠΈΠΌΠΎ Π·Π½Π°Ρ‚ΡŒ для опрСдСлСния ΡƒΠ³Π»ΠΎΠ² ΠΏΠΎΠ²ΠΎΡ€ΠΎΡ‚Π° ΠΏΠΎΠΏΠ΅Ρ€Π΅Ρ‡Π½ΠΎΠ³ΠΎ сСчСния ΠΈ ΠΏΡ€ΠΎΠ³ΠΈΠ±Π° (смСщСния) Ρ†Π΅Π½Ρ‚Ρ€Π° тяТСсти ΠΏΠΎΠΏΠ΅Ρ€Π΅Ρ‡Π½ΠΎΠ³ΠΎ сСчСния, Ρ‚Π°ΠΊ ΠΊΠ°ΠΊ ΠΌΠ°ΠΊΡΠΈΠΌΠ°Π»ΡŒΠ½Ρ‹Π΅ Π΄Π΅Ρ„ΠΎΡ€ΠΌΠ°Ρ†ΠΈΠΈ Π²ΠΎΠ·Π½ΠΈΠΊΠ°ΡŽΡ‚ Π² самом Π²Π΅Ρ€Ρ…Π½Π΅ΠΌ ΠΈ Π² самом Π½ΠΈΠΆΠ½Π΅ΠΌ слоС ΠΈΠ·Π³ΠΈΠ±Π°Π΅ΠΌΠΎΠΉ конструкции, Ρ‚ΠΎ ΠΎΠΏΡ€Π΅Π΄Π΅Π»ΠΈΡ‚ΡŒ ΠΌΠΎΠΌΠ΅Π½Ρ‚ ΠΈΠ½Π΅Ρ€Ρ†ΠΈΠΈ ΠΌΠΎΠΆΠ½ΠΎ, ΡƒΠΌΠ½ΠΎΠΆΠΈΠ² ΠΌΠΎΠΌΠ΅Π½Ρ‚ сопротивлСния Π½Π° расстояниС ΠΎΡ‚ Ρ†Π΅Π½Ρ‚Ρ€Π° тяТСсти сСчСния Π΄ΠΎ Π²Π΅Ρ€Ρ…Π½Π΅Π³ΠΎ ΠΈΠ»ΠΈ Π½ΠΈΠΆΠ½Π΅Π³ΠΎ слоя, поэтому для ΠΏΡ€ΡΠΌΠΎΡƒΠ³ΠΎΠ»ΡŒΠ½Ρ‹Ρ… сСчСний I=Wh/2. ΠŸΡ€ΠΈ ΠΎΠΏΡ€Π΅Π΄Π΅Π»Π΅Π½ΠΈΠΈ ΠΌΠΎΠΌΠ΅Π½Ρ‚Π° ΠΈΠ½Π΅Ρ€Ρ†ΠΈΠΈ сСчСний слоТных гСомСтричСских Ρ„ΠΎΡ€ΠΌ сначала слоТная Ρ„ΠΈΠ³ΡƒΡ€Π° разбиваСтся Π½Π° ΠΏΡ€ΠΎΡΡ‚Π΅ΠΉΡˆΠΈΠ΅, Π·Π°Ρ‚Π΅ΠΌ ΠΎΠΏΡ€Π΅Π΄Π΅Π»ΡΡŽΡ‚ΡΡ ΠΏΠ»ΠΎΡ‰Π°Π΄ΠΈ сСчСния этих Ρ„ΠΈΠ³ΡƒΡ€ ΠΈ ΠΌΠΎΠΌΠ΅Π½Ρ‚Ρ‹ ΠΈΠ½Π΅Ρ€Ρ†ΠΈΠΈ ΠΏΡ€ΠΎΡΡ‚Π΅ΠΉΡˆΠΈΡ… Ρ„ΠΈΠ³ΡƒΡ€, Π·Π°Ρ‚Π΅ΠΌ ΠΏΠ»ΠΎΡ‰Π°Π΄ΠΈ ΠΏΡ€ΠΎΡΡ‚Π΅ΠΉΡˆΠΈΡ… Ρ„ΠΈΠ³ΡƒΡ€ ΡƒΠΌΠ½ΠΎΠΆΠ°ΡŽΡ‚ΡΡ Π½Π° ΠΊΠ²Π°Π΄Ρ€Π°Ρ‚ расстояния ΠΎΡ‚ ΠΎΠ±Ρ‰Π΅Π³ΠΎ Ρ†Π΅Π½Ρ‚Ρ€Π° тяТСсти сСчСния Π΄ΠΎ Ρ†Π΅Π½Ρ‚Ρ€Π° тяТСсти ΠΏΡ€ΠΎΡΡ‚Π΅ΠΉΡˆΠ΅ΠΉ Ρ„ΠΈΠ³ΡƒΡ€Ρ‹. ΠœΠΎΠΌΠ΅Π½Ρ‚ ΠΈΠ½Π΅Ρ€Ρ†ΠΈΠΈ ΠΏΡ€ΠΎΡΡ‚Π΅ΠΉΡˆΠ΅ΠΉ Ρ„ΠΈΠ³ΡƒΡ€Ρ‹ Π² составС слоТного сСчСния Ρ€Π°Π²Π΅Π½ ΠΌΠΎΠΌΠ΅Π½Ρ‚Ρƒ ΠΈΠ½Π΅Ρ€Ρ†ΠΈΠΈ Ρ„ΠΈΠ³ΡƒΡ€Ρ‹ + ΠΊΠ²Π°Π΄Ρ€Π°Ρ‚ расстояния ΡƒΠΌΠ½ΠΎΠΆΠ΅Π½Π½Ρ‹ΠΉ Π½Π° ΠΏΠ»ΠΎΡ‰Π°Π΄ΡŒ. Π—Π°Ρ‚Π΅ΠΌ ΠΏΠΎΠ»ΡƒΡ‡Π΅Π½Π½Ρ‹Π΅ ΠΌΠΎΠΌΠ΅Π½Ρ‚Ρ‹ ΠΈΠ½Π΅Ρ€Ρ†ΠΈΠΈ ΡΡƒΠΌΠΌΠΈΡ€ΡƒΡŽΡ‚ΡΡ ΠΈ получаСтся ΠΌΠΎΠΌΠ΅Π½Ρ‚ ΠΈΠ½Π΅Ρ€Ρ†ΠΈΠΈ слоТного сСчСния. Но это максимально ΡƒΠΏΡ€ΠΎΡ‰Π΅Π½Π½Ρ‹Π΅ Ρ„ΠΎΡ€ΠΌΡƒΠ»ΠΈΡ€ΠΎΠ²ΠΊΠΈ (хотя, ΡΠΎΠ³Π»Π°ΡˆΡƒΡΡŒ, всС Ρ€Π°Π²Π½ΠΎ выглядит достаточно ΠΌΡƒΠ΄Ρ€Π΅Π½ΠΎ).
Π‘ΠΎ Π²Ρ€Π΅ΠΌΠ΅Π½Π΅ΠΌ Π½Π°ΠΏΠΈΡˆΡƒ ΠΎΡ‚Π΄Π΅Π»ΡŒΠ½ΡƒΡŽ ΡΡ‚Π°Ρ‚ΡŒΡŽ.


05-12-2012: Π“ΠΈΠ²ΠΈ

Π’ ΠΏΡ€ΠΈΠ½Ρ†ΠΈΠΏΠ΅ всС ΠΏΡ€Π΅Π΄Π΅Π»ΡŒΠ½ΠΎ ясно, Π½ΠΎ здСсь ΠΏΡ€ΠΎΡ‰Π΅ www.kataltim.ru


20-04-2013: Petr

НС Π½ΡƒΠΆΠ½ΠΎ ΠΏΠΎΠ»Π½ΠΎΡΡ‚ΡŒΡŽ Π΄ΠΎΠ²Π΅Ρ€ΡΡ‚ΡŒ ΠΏΠΎΠ΄Π°Π½Π½ΠΎΠΉ Π² сайтах ΠΈΠ½Ρ„ΠΎΡ€ΠΌΠ°Ρ†ΠΈΠΈ. Π•Ρ‘ Π½ΠΈΠΊΡ‚ΠΎ ΠΏΠΎ-Ρ…ΠΎΡ€ΠΎΡˆΠ΅ΠΌΡƒ Π½Π΅ провСряСт. И ссылки Π½Π° Π½Π΅Ρ‘ Π½Π΅ Π΄Π°ΡŽΡ‚ΡΡ. Π’Π°ΠΊ Π² Π’Π°Π±Π»ΠΈΡ†Π΅ 1. “Π€ΠΎΡ€ΠΌΡ‹ сСчСния, ΠΏΠ»ΠΎΡ‰Π°Π΄ΠΈ сСчСний, ΠΌΠΎΠΌΠ΅Π½Ρ‚Ρ‹ ΠΈΠ½Π΅Ρ€Ρ†ΠΈΠΈ ΠΈ ΠΌΠΎΠΌΠ΅Π½Ρ‚Ρ‹ сопротивлСния для конструкций достаточно простых гСомСтричСских Ρ„ΠΎΡ€ΠΌ” для тонкостСнной Ρ‚Ρ€ΡƒΠ±Ρ‹ даСтся ΠΎΠΏΡ€Π΅Π΄Π΅Π»Π΅Π½ΠΈΠ΅, Ρ‡Ρ‚ΠΎ ΠΎΡ‚Π½ΠΎΡˆΠ΅Π½ΠΈΠ΅ Π΄ΠΈΠ°ΠΌΠ΅Ρ‚Ρ€Π° ΠΊ Ρ‚ΠΎΠ»Ρ‰ΠΈΠ½Π΅ ΠΎΠ±ΠΎΠ»ΠΎΡ‡ΠΊΠΈ Π΄ΠΎΠ»ΠΆΠ½ΠΎ Π±Ρ‹Ρ‚ΡŒ большС 10. По Π΄Ρ€ΡƒΠ³ΠΈΠΌ источникам – Π΄ΠΎΠ»ΠΆΠ½ΠΎ Π±Ρ‹Ρ‚ΡŒ большС 20!!! (Н.М. БСляСв. Π‘ΠΎΠΏΡ€ΠΎΡ‚ΠΈΠ²Π»Π΅Π½ΠΈΠ΅ ΠΌΠ°Ρ‚Π΅Ρ€ΠΈΠ°Π»ΠΎΠ². М.1996. стр.160. ΠΈΠ»ΠΈ Н.И.Π‘Π΅Π·ΡƒΡ…ΠΎΠ². ΠžΡΠ½ΠΎΠ²Ρ‹ Ρ‚Π΅ΠΎΡ€ΠΈΠΈ упругости, пластичности ΠΈ ползучСсти.М.1961.стр.390)


21-04-2013: Π”ΠΎΠΊΡ‚ΠΎΡ€ Π›ΠΎΠΌ

Π’Π΅Ρ€Π½ΠΎ. Π”ΠΎΠ²Π΅Ρ€ΡΡ‚ΡŒ нСльзя. Но логичСскоС ΠΌΡ‹ΡˆΠ»Π΅Π½ΠΈΠ΅ ΠΏΠΎΠΊΠ° Π½ΠΈΠΊΡ‚ΠΎ Π½Π΅ отмСнял. Π‘Π°ΠΌΡ‹ΠΉ ΠΏΡ€Π°Π²ΠΈΠ»ΡŒΠ½Ρ‹ΠΉ Π²Π°Ρ€ΠΈΠ°Π½Ρ‚ – Ρ€Π°ΡΡΡ‡ΠΈΡ‚Ρ‹Π²Π°Ρ‚ΡŒ ΠΌΠΎΠΌΠ΅Π½Ρ‚ ΠΈΠ½Π΅Ρ€Ρ†ΠΈΠΈ ΠΈΠ»ΠΈ ΠΌΠΎΠΌΠ΅Π½Ρ‚ сопротивлСния для любой Ρ‚Ρ€ΡƒΠ±Ρ‹ ΠΏΠΎ Ρ„ΠΎΡ€ΠΌΡƒΠ»Π°ΠΌ, ΠΏΡ€ΠΈΠ²Π΅Π΄Π΅Π½Π½Ρ‹ΠΌ для ΠΎΠ±Ρ‹Ρ‡Π½ΠΎΠΉ Ρ‚Ρ€ΡƒΠ±Ρ‹ (Π½Π° 1 ΠΏΡƒΠ½ΠΊΡ‚ Π²Ρ‹ΡˆΠ΅). Π€ΠΎΡ€ΠΌΡƒΠ»Ρ‹, ΠΏΡ€ΠΈΠ²ΠΎΠ΄ΠΈΠΌΡ‹Π΅ для тонкостСнной Ρ‚Ρ€ΡƒΠ±Ρ‹, Π² любом случаС Π±ΡƒΠ΄ΡƒΡ‚ ΠΏΡ€ΠΈΠ±Π»ΠΈΠΆΠ΅Π½Π½Ρ‹ΠΌΠΈ ΠΈ годятся Ρ‚ΠΎΠ»ΡŒΠΊΠΎ для ΠΏΠ΅Ρ€Π²ΠΈΡ‡Π½ΠΎΠ³ΠΎ расчСта ΠΈ ΠΎΠ± этом Π·Π°Π±Ρ‹Π²Π°Ρ‚ΡŒ нСльзя.
Π’ΠΏΡ€ΠΎΡ‡Π΅ΠΌ ΠΏΠ°Ρ€Π°ΠΌΠ΅Ρ‚Ρ€Ρ‹ максимально допустимой Ρ‚ΠΎΠ»Ρ‰ΠΈΠ½Ρ‹ стСнки исправил.


25-06-2013: Баня

трСбуСтся ΠΎΠΏΡ€Π΅Π΄Π΅Π»ΠΈΡ‚ΡŒ ΠΌΠΎΠΌΠ΅Π½Ρ‚ ΠΈΠ½Π΅Ρ€Ρ†ΠΈΠΈ для слоТного нСстандартного сСчСния. сСчСниС: ΠΏΡ€ΡΠΌΠΎΡƒΠ³ΠΎΠ»ΡŒΠ½ΠΈΠΊ с двумя ΠΏΠ°Π·Π°ΠΌΠΈ. внСшнС ΠΏΠΎΡ…ΠΎΠΆΠ΅ Π½Π° Π±ΡƒΠΊΠ²Ρƒ “Π¨”. Π½Π΅ получаСтся Π½Π°ΠΉΡ‚ΠΈ ΠΊΠ°ΠΊΡƒΡŽ Π»ΠΈΠ±ΠΎ ΠΈΠ½Ρ„ΠΎΡ€ΠΌΠ°Ρ†ΠΈΡŽ. Π±ΡƒΠ΄Ρƒ ΠΏΡ€ΠΈΠ·Π½Π°Ρ‚Π΅Π»Π΅Π½ Π·Π° ΠΊΠ°ΠΊΡƒΡŽ Π½ΠΈΠ±ΡƒΠ΄ΡŒ ΠΈΠ½Ρ„ΠΎΡ€ΠΌΠ°Ρ†ΠΈΡŽ


25-06-2013: Π”ΠΎΠΊΡ‚ΠΎΡ€ Π›ΠΎΠΌ

ΠŸΠΎΡΠΌΠΎΡ‚Ρ€ΠΈΡ‚Π΅ ΡΡ‚Π°Ρ‚ΡŒΡŽ “РасчСт прочности ΠΏΠΎΡ‚ΠΎΠ»ΠΎΡ‡Π½ΠΎΠ³ΠΎ профиля для гипсокартона” (http://doctorlom. 3)*3,14/32.
ΠžΠ±ΡŠΡΡΠ½ΠΈΡ‚Π΅, поТалуйста, ΠΏΡ€Π°Π²ΠΈΠ»ΡŒΠ½ΠΎΡΡ‚ΡŒ этой Ρ„ΠΎΡ€ΠΌΡƒΠ»Ρ‹ (ΠΈΠ»ΠΈ Π½Π΅ΠΏΡ€Π°Π²ΠΈΠ»ΡŒΠ½ΠΎΡΡ‚ΡŒ).


04-11-2014: Π”ΠΎΠΊΡ‚ΠΎΡ€ Π›ΠΎΠΌ

Π€ΠΎΡ€ΠΌΡƒΠ»Π° ΠΈΠ· ΠΏΡ€ΠΈΠ²Π΅Π΄Π΅Π½Π½ΠΎΠ³ΠΎ Π²Π°ΠΌΠΈ источника Π½Π΅ΠΏΡ€Π°Π²ΠΈΠ»ΡŒΠ½Π°Ρ (Сю ΠΌΠΎΠΆΠ½ΠΎ ΠΏΠΎΠ»ΡŒΠ·ΠΎΠ²Π°Ρ‚ΡŒΡΡ Ρ‚ΠΎΠ»ΡŒΠΊΠΎ для ΠΏΡ€ΠΈΠ±Π»ΠΈΠ·ΠΈΡ‚Π΅Π»ΡŒΠ½Ρ‹Ρ… вычислСний) ΠΈ ΠΏΡ€ΠΎΠ²Π΅Ρ€ΠΈΡ‚ΡŒ это Π»Π΅Π³ΠΊΠΎ.
Π§Ρ‚ΠΎΠ±Ρ‹ ΠΎΠΏΡ€Π΅Π΄Π΅Π»ΠΈΡ‚ΡŒ ΠΌΠΎΠΌΠ΅Π½Ρ‚ ΠΈΠ½Π΅Ρ€Ρ†ΠΈΠΈ сСчСния Ρ‚Ρ€ΡƒΠ±Ρ‹, достаточно Π²Ρ‹Ρ‡Π΅ΡΡ‚ΡŒ ΠΈΠ· ΠΌΠΎΠΌΠ΅Π½Ρ‚Π° ΠΈΠ½Π΅Ρ€Ρ†ΠΈΠΈ стСрТня ΠΊΡ€ΡƒΠ³Π»ΠΎΠ³ΠΎ сСчСния (Ρ‚ΡƒΡ‚ ΠΏΡ€ΠΈ вычислСниях ΠΈΡΠΏΠΎΠ»ΡŒΠ·ΡƒΠ΅Ρ‚ΡΡ Π½Π°Ρ€ΡƒΠΆΠ½Ρ‹ΠΉ Π΄ΠΈΠ°ΠΌΠ΅Ρ‚Ρ€ Ρ‚Ρ€ΡƒΠ±Ρ‹) ΠΌΠΎΠΌΠ΅Π½Ρ‚ ΠΈΠ½Π΅Ρ€Ρ†ΠΈΠΈ отвСрстия (Π²Π½ΡƒΡ‚Ρ€Π΅Π½Π½ΠΈΠΉ Π΄ΠΈΠ°ΠΌΠ΅Ρ‚Ρ€, вСдь Π²Π½ΡƒΡ‚Ρ€ΠΈ Ρ‚Ρ€ΡƒΠ±Ρ‹ Π½ΠΈΠΊΠ°ΠΊΠΎΠ³ΠΎ ΠΌΠ°Ρ‚Π΅Ρ€ΠΈΠ°Π»Π° Π½Π΅Ρ‚, Π½Π° Ρ‚ΠΎ ΠΎΠ½Π° ΠΈ Ρ‚Ρ€ΡƒΠ±Π°). ПослС ΠΏΡ€ΠΎΡΡ‚Π΅ΠΉΡˆΠΈΡ… матСматичСских ΠΏΡ€Π΅ΠΎΠ±Ρ€Π°Π·ΠΎΠ²Π°Π½ΠΈΠΉ ΠΌΡ‹ ΠΏΠΎΠ»ΡƒΡ‡ΠΈΠΌ Ρ„ΠΎΡ€ΠΌΡƒΠ»Ρƒ ΠΌΠΎΠΌΠ΅Π½Ρ‚Π° ΠΈΠ½Π΅Ρ€Ρ†ΠΈΠΈ Ρ‚Ρ€ΡƒΠ±Ρ‹, ΠΏΡ€ΠΈΠ²Π΅Π΄Π΅Π½Π½ΡƒΡŽ Π² Ρ‚Π°Π±Π»ΠΈΡ†Π΅.
А для Ρ‚ΠΎΠ³ΠΎ, Ρ‡Ρ‚ΠΎΠ±Ρ‹ ΠΎΠΏΡ€Π΅Π΄Π΅Π»ΠΈΡ‚ΡŒ ΠΌΠΎΠΌΠ΅Π½Ρ‚ сопротивлСния, Π½ΡƒΠΆΠ½ΠΎ ΠΌΠΎΠΌΠ΅Π½Ρ‚ ΠΈΠ½Π΅Ρ€Ρ†ΠΈΠΈ Ρ€Π°Π·Π΄Π΅Π»ΠΈΡ‚ΡŒ Π½Π° максимальноС расстояниС ΠΎΡ‚ Ρ†Π΅Π½Ρ‚Ρ€Π° тяТСсти Π΄ΠΎ самой дальнСй Ρ‚ΠΎΡ‡ΠΊΠΈ сСчСния, соотвСтствСнно Π½Π° D/2, ΠΈΠ»ΠΈ ΡƒΠΌΠ½ΠΎΠΆΠΈΡ‚ΡŒ Π½Π° 2/D.

Π’ ΠΈΡ‚ΠΎΠ³Π΅ ΠΏΠΎΠ»ΡƒΡ‡ΠΈΡ‚ΡŒ ΡƒΠΊΠ°Π·Π°Π½Π½ΡƒΡŽ Π²Π°ΠΌΠΈ Ρ„ΠΎΡ€ΠΌΡƒΠ»Ρƒ Π½Π΅Π²ΠΎΠ·ΠΌΠΎΠΆΠ½ΠΎ ΠΈ Ρ‡Π΅ΠΌ Ρ‚ΠΎΠ»Ρ‰Π΅ Π±ΡƒΠ΄Π΅Ρ‚ стСнка Ρ‚Ρ€ΡƒΠ±Ρ‹, Ρ‚Π΅ΠΌ большС Π±ΡƒΠ΄Π΅Ρ‚ ΠΏΠΎΠ³Ρ€Π΅ΡˆΠ½ΠΎΡΡ‚ΡŒ ΠΏΡ€ΠΈ использовании этой Ρ„ΠΎΡ€ΠΌΡƒΠ»Ρ‹.


04-11-2014: Π Π°Π΄ΠΈΠΊ

Бпасибо, док!


11-11-2014: Ильгам

НС смог Π½Π°ΠΉΡ‚ΠΈ ΠΈΠ½Ρ„ΠΎ ΠΎ Ρ‚ΠΎΠΌ Π² ΠΊΠ°ΠΊΠΈΡ… Π΅Π΄ΠΈΠ½ΠΈΡ†Π°Ρ… (ΠΌΠΌ, см, ΠΌ) всС значСния Π² Ρ„ΠΎΡ€ΠΌΡƒΠ»Π°Ρ….

ΠŸΠΎΠΏΡ€ΠΎΠ±ΠΎΠ²Π°Π» ΠΏΠΎΡΡ‡ΠΈΡ‚Π°Ρ‚ΡŒ Wz для ΡƒΠ³ΠΎΠ»ΠΊΠ° 210Ρ…90ΠΌΠΌ (Ссли Ρƒ швСл.24П ΡΡ€Π΅Π·Π°Ρ‚ΡŒ Π²Π΅Ρ€Ρ…Π½ΡŽΡŽ ΠΏΠΎΠ»ΠΊΡƒ), ΠΏΠΎΠ»ΡƒΡ‡ΠΈΠ»ΠΎΡΡŒ 667,5 см3, ΠΏΡ€ΠΈ условии Ρ‡Ρ‚ΠΎ всС значСния Π² см.
Для ΠΏΡ€ΠΈΠΌΠ΅Ρ€Π°, Ρƒ швСл.24П (Π΄ΠΎ срСзания ΠΏΠΎΠ»ΠΊΠΈ) Wx(Wz)=243 см3.


11-11-2014: Π”ΠΎΠΊΡ‚ΠΎΡ€ Π›ΠΎΠΌ

Π­Ρ‚ΠΎ ΠΎΠ±Ρ‰ΠΈΠ΅ Ρ„ΠΎΡ€ΠΌΡƒΠ»Ρ‹. Π’ ΠΊΠ°ΠΊΠΈΡ… Π΅Π΄ΠΈΠ½ΠΈΡ†Π°Ρ… подставитС значСния, Π² Ρ‚Π°ΠΊΠΈΡ… ΠΈ ΠΏΠΎΠ»ΡƒΡ‡ΠΈΡ‚Π΅ Ρ€Π΅Π·ΡƒΠ»ΡŒΡ‚Π°Ρ‚, Ρ‚ΠΎΠ»ΡŒΠΊΠΎ само собой ΡƒΠΆΠ΅ Π² кубичСских. Но Ссли Π½Π°Ρ‡Π°Π»ΠΈ ΠΏΠΎΠ΄ΡΡ‚Π°Π²Π»ΡΡ‚ΡŒ, Π½Π°ΠΏΡ€ΠΈΠΌΠ΅Ρ€, Π² сантимСтрах, Ρ‚ΠΎ Ρ‚Π°ΠΊ ΠΈ Π½ΡƒΠΆΠ½ΠΎ ΠΏΡ€ΠΎΠ΄ΠΎΠ»ΠΆΠ°Ρ‚ΡŒ.
Π£ ΡˆΠ²Π΅Π»Π»Π΅Ρ€Π° Π±Π΅Π· ΠΏΠΎΠ»ΠΊΠΈ ΠΌΠΎΠΌΠ΅Π½Ρ‚ сопротивлСния ΠΏΠΎ ΡƒΠΌΠΎΠ»Ρ‡Π°Π½ΠΈΡŽ Π½Π΅ ΠΌΠΎΠΆΠ΅Ρ‚ Π±Ρ‹Ρ‚ΡŒ большС Ρ‡Π΅ΠΌ Ρƒ Ρ†Π΅Π»ΠΎΠ³ΠΎ ΡˆΠ²Π΅Π»Π»Π΅Ρ€Π°.

Для ΠΏΡ€ΠΈΠ±Π»ΠΈΠ·ΠΈΡ‚Π΅Π»ΡŒΠ½ΠΎΠ³ΠΎ опрСдСлСния ΠΌΠΎΠΌΠ΅Π½Ρ‚Π° сопротивлСния ΡˆΠ²Π΅Π»Π»Π΅Ρ€Π° Π±Π΅Π· ΠΏΠΎΠ»ΠΊΠΈ Π²Ρ‹ ΠΌΠΎΠΆΠ΅Ρ‚Π΅ Π²ΠΎΡΠΏΠΎΠ»ΡŒΠ·ΠΎΠ²Π°Ρ‚ΡŒΡΡ Ρ„ΠΎΡ€ΠΌΡƒΠ»Π°ΠΌΠΈ для Π½Π΅Ρ€Π°Π²Π½ΠΎΠΏΠΎΠ»ΠΎΡ‡Π½ΠΎΠ³ΠΎ ΡƒΠ³ΠΎΠ»ΠΊΠ° (Ρ‚ΠΎΠ»ΡŒΠΊΠΎ для опрСдСлСния Wz, для Wy эти Ρ„ΠΎΡ€ΠΌΡƒΠ»Ρ‹ Π½Π΅ ΠΏΠΎΠ΄ΠΎΠΉΠ΄ΡƒΡ‚).


04-01-2015: Valerij

Если сСчСниС Ρ‚Ρ€ΡƒΠ±Ρ‹ ослаблСно нСсколькими Π·Π½Π°Ρ‡ΠΈΡ‚Π΅Π»ΡŒΠ½Ρ‹ΠΌΠΈ отвСрстиями, ΠΊΠ°ΠΊ ΡƒΡ‡Π΅ΡΡ‚ΡŒ это ΠΏΡ€ΠΈ расчётС ΠΌΠΎΠΌΠ΅Π½Ρ‚Π° ΠΈΠ½Π΅Ρ€Ρ†ΠΈΠΈ ΠΈ ΠΌΠΎΠΌΠ΅Π½Ρ‚Π° сопротивлСния? Π’Ρ€ΡƒΠ±Π° 32.39см ΠΈ Π² Π½Π΅ΠΉ 9 ΠΎΡ‚Π². Π΄ΠΈΠ°ΠΌ.2.8см Π² сСчСнии(шаг ΠΎΡ‚Π²Π΅Ρ€ΠΌΡ‚ΠΈΠΉ 10см. ΠΏΠΎ Π΄Π»ΠΈΠ½Π΅ Ρ‚Ρ€ΡƒΠ±Ρ‹).


05-01-2015: Π”ΠΎΠΊΡ‚ΠΎΡ€ Π›ΠΎΠΌ

Для опрСдСлСния ΠΌΠΎΠΌΠ΅Π½Ρ‚Π° ΠΈΠ½Π΅Ρ€Ρ†ΠΈΠΈ Π²Π°ΠΌ Π½ΡƒΠΆΠ½ΠΎ Π²Ρ‹Ρ‡Π΅ΡΡ‚ΡŒ ΠΈΠ· ΠΌΠΎΠΌΠ΅Π½Ρ‚Π° ΠΈΠ½Π΅Ρ€Ρ†ΠΈΠΈ Ρ‚Ρ€ΡƒΠ±Ρ‹ ΠΌΠΎΠΌΠ΅Π½Ρ‚ ΠΈΠ½Π΅Ρ€Ρ†ΠΈΠΈ вашСго отвСрстия. Для этого Π½ΡƒΠΆΠ½ΠΎ ΠΎΠΏΡ€Π΅Π΄Π΅Π»ΠΈΡ‚ΡŒ ΠΏΠ»ΠΎΡ‰Π°Π΄ΡŒ сСчСния отвСрстия ΠΈ Π·Π°Ρ‚Π΅ΠΌ ΡƒΠΌΠ½ΠΎΠΆΠΈΡ‚ΡŒ Π΅Π΅ Π½Π° ΠΊΠ²Π°Π΄Ρ€Π°Ρ‚ расстояния Π΄ΠΎ Ρ†Π΅Π½Ρ‚Ρ€Π° Ρ‚Ρ€ΡƒΠ±Ρ‹ плюс собствСнный ΠΌΠΎΠΌΠ΅Π½Ρ‚ ΠΈΠ½Π΅Ρ€Ρ†ΠΈΠΈ отвСрстия. Π‘ΠΎΠ»ΡŒΡˆΠ΅ подробностСй Π² ΡΡ‚Π°Ρ‚ΡŒΠ΅ “ΠœΠΎΠΌΠ΅Π½Ρ‚Ρ‹ ΠΈΠ½Π΅Ρ€Ρ†ΠΈΠΈ ΠΏΠΎΠΏΠ΅Ρ€Π΅Ρ‡Π½Ρ‹Ρ… сСчСний”.
Если расчСт Π½Π΅ Ρ‚Ρ€Π΅Π±ΡƒΠ΅Ρ‚ особой точности ΠΈ Π΄ΠΈΠ°ΠΌΠ΅Ρ‚Ρ€ отвСрстия Π² 5 ΠΈ Π±ΠΎΠ»Π΅Π΅ Ρ€Π°Π· мСньшС Π΄ΠΈΠ°ΠΌΠ΅Ρ‚Ρ€Π° Ρ‚Ρ€ΡƒΠ±Ρ‹ (Π²Ρ€ΠΎΠ΄Π΅ ваш случай, Ссли 32.39 – это Π½Π°Ρ€ΡƒΠΆΠ½Ρ‹ΠΉ Π΄ΠΈΠ°ΠΌΠ΅Ρ‚Ρ€), Ρ‚ΠΎ сСгмСнт отвСрстия ΠΌΠΎΠΆΠ½ΠΎ привСсти ΠΊ ΠΏΡ€ΡΠΌΠΎΡƒΠ³ΠΎΠ»ΡŒΠ½ΠΈΠΊΡƒ. Если отвСрстиС Π½Π΅ сквозноС, Ρ‚ΠΎ слСдуСт Π΄ΠΎΠΏΠΎΠ»Π½ΠΈΡ‚Π΅Π»ΡŒΠ½ΠΎ ΠΎΠΏΡ€Π΅Π΄Π΅Π»ΠΈΡ‚ΡŒ ΠΏΠΎΠ»ΠΎΠΆΠ΅Π½ΠΈΠ΅ Ρ†Π΅Π½Ρ‚Ρ€Π° тяТСсти Ρ‚Ρ€ΡƒΠ±Ρ‹ с отвСрстиСм для Ρ‚ΠΎΠ³ΠΎ, Ρ‡Ρ‚ΠΎΠ±Ρ‹ ΠΏΠΎΡ‚ΠΎΠΌ Π²Ρ‹Ρ‡ΠΈΡΠ»ΠΈΡ‚ΡŒ Π½ΠΎΠ²ΠΎΠ΅ Π·Π½Π°Ρ‡Π΅Π½ΠΈΠ΅ ΠΌΠΎΠΌΠ΅Π½Ρ‚Π° сопротивлСния.

Но ΠΈ это Π΅Ρ‰Π΅ Π½Π΅ всС. Π’Π°ΠΌ слСдуСт ΡƒΡ‡Π΅ΡΡ‚ΡŒ, Ρ‡Ρ‚ΠΎ Π²ΠΎΠ·Π»Π΅ отвСрстий Π²ΠΎΠ·Π½ΠΈΠΊΠ°ΡŽΡ‚ Π·Π½Π°Ρ‡ΠΈΡ‚Π΅Π»ΡŒΠ½Ρ‹Π΅ Π»ΠΎΠΊΠ°Π»ΡŒΠ½Ρ‹Π΅ напряТСния.


09-10-2015: Борис

НСравноплСчий ΡƒΠ³ΠΎΠ»ΠΎΠΊ.ΠŸΡ€ΠΈ вычислСнии Wy Π½Π΅ y,Π° H-y


09-10-2015: Π”ΠΎΠΊΡ‚ΠΎΡ€ Π›ΠΎΠΌ

НС ΠΏΠΎΠΉΠΌΡƒ, ΠΎ Ρ‡Π΅ΠΌ Π²Ρ‹. ΠžΠΏΡ€Π΅Π΄Π΅Π»Π΅Π½ΠΈΠ΅ ΠΌΠΎΠΌΠ΅Π½Ρ‚Π° сопротивлСния ΠΎΡ‚Π½ΠΎΡΠΈΡ‚Π΅Π»ΡŒΠ½ΠΎ оси Ρƒ Π² Ρ‚Π°Π±Π»ΠΈΡ†Π°Ρ… Π²ΠΎΠΎΠ±Ρ‰Π΅ Π½Π΅ приводится.


09-10-2015: Борс

Для Ρ‚Ρ€Π΅ΡƒΠ³ΠΎΠ»ΡŒΠ½ΠΈΠΊΠΎΠ² ΠΏΡ€ΠΈ вычислСнии WzΠΏ h Π² ΠΊΠ²Π°Π΄Ρ€Π°Ρ‚Π΅.


09-10-2015: Борис

ΠŸΠ°Ρ€Π΄ΠΎΠ½,Wz


09-10-2015: Π”ΠΎΠΊΡ‚ΠΎΡ€ Π›ΠΎΠΌ

ВсС Π²Π΅Ρ€Π½ΠΎ. Π’Π΅ΠΏΠ΅Ρ€ΡŒ понял, ΠΎ Ρ‡Π΅ΠΌ Π²Ρ‹. Π‘ΠΎΠ»Π΅Π΅ ΠΊΠΎΡ€Ρ€Π΅ΠΊΡ‚Π½ΠΎ Π±Ρ‹Π»ΠΎ Π±Ρ‹ ΡƒΠΊΠ°Π·Π°Ρ‚ΡŒ ΠΌΠΎΠΌΠ΅Π½Ρ‚ сопротивлСния для Π²Π΅Ρ€Ρ…Π½Π΅ΠΉ ΠΈ для Π½ΠΈΠΆΠ½Π΅ΠΉ части сСчСния, Π° я ΡƒΠΊΠ°Π·Π°Π» Ρ‚ΠΎΠ»ΡŒΠΊΠΎ для Π½ΠΈΠΆΠ½Π΅ΠΉ. Ну Π° ΠΏΡ€ΠΈ ΠΎΠΏΡ€Π΅Π΄Π΅Π»Π΅Π½ΠΈΠΈ ΠΌΠΎΠΌΠ΅Π½Ρ‚Π° сопротивлСния Ρ‚Ρ€Π΅ΡƒΠ³ΠΎΠ»ΡŒΠ½ΠΈΠΊΠΎΠ² банально ΠΏΡ€ΠΎΠΏΡƒΡ‰Π΅Π½ ΠΊΠ²Π°Π΄Ρ€Π°Ρ‚.

Π˜ΡΠΏΡ€Π°Π²ΠΈΠ». Бпасибо Π·Π° Π²Π½ΠΈΠΌΠ°Ρ‚Π΅Π»ΡŒΠ½ΠΎΡΡ‚ΡŒ.


28-04-2016: Jama

ЗдравствуСтС! ΠšΡ‚ΠΎ ΠΌΠΎΠΆΠ΅Ρ‚ ΠΏΠΎΠΌΠΎΡ‡ΡŒ ΠΎ ΠΏΡ€Π°Π²ΠΈΠ»ΡŒΠ½ΠΎΡΡ‚ΠΈ расчСта http://ej.kubagro.ru/2011/02/pdf/19.pdf
я Π½Π΅ ΠΌΠΎΠ³Ρƒ понят ΠΎΡ‚ΠΊΡƒΠ΄Π° Π·Π½Π°Ρ‡Π΅Π½ΠΈΠ΅ бСрСтся ΠΌΠΎΠΌΠ΅Π½Ρ‚ сопротивлСния. ΠŸΠΎΠΌΠΎΠ³ΠΈΡ‚Π΅ поТалуйста!


28-04-2016: Π”ΠΎΠΊΡ‚ΠΎΡ€ Π›ΠΎΠΌ

Π§Ρ‚ΠΎ ΠΈΠΌΠ΅Π½Π½ΠΎ Π²Π°ΠΌ Π½Π΅ понятно (Π²Ρ‹Ρ‡ΠΈΡ‚Ρ‹Π²Π°Ρ‚ΡŒ вСсь Π΄ΠΎΠΊΡƒΠΌΠ΅Π½Ρ‚ Ρƒ мСня Π½Π΅Ρ‚ Π²Ρ€Π΅ΠΌΠ΅Π½ΠΈ). Если Ρ€Π΅Ρ‡ΡŒ ΠΎ Π±Π°Π»ΠΊΠ΅, Π»Π΅ΠΆΠ°Ρ‰Π΅ΠΉ Π½Π° ΡƒΠΏΡ€ΡƒΠ³ΠΎΠΌ основании, Ρ‚ΠΎ скорСС всСго Π±Π°Π»ΠΊΠ° эта ΠΈΠΌΠ΅Π΅Ρ‚ ΠΏΡ€ΡΠΌΠΎΡƒΠ³ΠΎΠ»ΡŒΠ½ΠΎΠ΅ сСчСниС (см. Ρ‚Π°Π±Π»ΠΈΡ†Ρƒ 1).


29-08-2016: Максим

ЗдравствуйтС ! Π˜ΠΌΠ΅Π΅Ρ‚ΡΡ ΡˆΠ²Π΅Π»Π»Π΅Ρ€ β„– 12. Π’ Π²Π΅Ρ€Ρ…Π½ΠΈΠΉ пояс Π±ΡƒΠ΄ΡƒΡ‚ Π²ΠΊΡ€ΡƒΡ‡ΠΈΠ²Π°Ρ‚ΡŒΡΡ саморСзы ΠΈ Π²ΠΈΠ½Ρ‚Ρ‹ для крСплСния ΠΊΡ€ΠΎΠ²Π»ΠΈ. Как ΡƒΡ‡Π΅ΡΡ‚ΡŒ ослаблСниС ΡˆΠ²Π΅Π»Π»Π΅Ρ€Π°, Ρ‚.Π΅ ΠΊΠ°ΠΊ ΠΎΠΏΡ€Π΅Π΄Π΅Π»ΠΈΡ‚ΡŒ W ослаблСнного сСчСния.


29-08-2016: Π”ΠΎΠΊΡ‚ΠΎΡ€ Π›ΠΎΠΌ

Если максимально ΡƒΠΏΡ€ΠΎΡΡ‚ΠΈΡ‚ΡŒ, Ρ‚ΠΎ:
Π‘Π½Π°Ρ‡Π°Π»Π° опрСдСляСтС ΠΌΠΎΠΌΠ΅Π½Ρ‚ ΠΈΠ½Π΅Ρ€Ρ†ΠΈΠΈ отвСрстия (для упрощСния расчСтов Π΅Π³ΠΎ ΠΌΠΎΠΆΠ½ΠΎ ΠΏΡ€ΠΈΠ½ΠΈΠΌΠ°Ρ‚ΡŒ ΠΏΡ€ΡΠΌΠΎΡƒΠ³ΠΎΠ»ΡŒΠ½Ρ‹ΠΌ). Π—Π°Ρ‚Π΅ΠΌ ΠΈΠ· ΠΌΠΎΠΌΠ΅Π½Ρ‚Π° ΠΈΠ½Π΅Ρ€Ρ†ΠΈΠΈ ΡˆΠ²Π΅Π»Π»Π΅Ρ€Π° Π²Ρ‹Ρ‡ΠΈΡ‚Π°Π΅Ρ‚Π΅ ΠΌΠΎΠΌΠ΅Π½Ρ‚ ΠΈΠ½Π΅Ρ€Ρ†ΠΈΠΈ отвСрстия, Π·Π°Ρ‚Π΅ΠΌ Π΄Π΅Π»ΠΈΡ‚Π΅ ΠΏΠΎΠ»ΡƒΡ‡Π΅Π½Π½Ρ‹ΠΉ ΠΌΠΎΠΌΠ΅Π½Ρ‚ ΠΈΠ½Π΅Ρ€Ρ†ΠΈΠΈ Π½Π° ΠΏΠΎΠ»ΠΎΠ²ΠΈΠ½Ρƒ высоты ΡˆΠ²Π΅Π»Π»Π΅Ρ€Π° ΠΈ ΠΏΠΎΠ»ΡƒΡ‡Π°Π΅Ρ‚Π΅ ΠΌΠΎΠΌΠ΅Π½Ρ‚ сопротивлСния.


21-03-2017: ΠΈΠ³ΠΎΡ€ΡŒ

здравствуйтС,Π‘Π΅Ρ€Π³Π΅ΠΉ. я ΠΏΡ€ΠΎΡ‡ΠΈΡ‚Π°Π» Π½Π΅ΠΊΠΎΡ‚ΠΎΡ€Ρ‹Π΅ ваши ΡΡ‚Π°Ρ‚ΡŒΠΈ,ΠΎΡ‡Π΅Π½ΡŒ интСрСсно ΠΈ понятно(Π² основном).я Ρ…ΠΎΡ‚Π΅Π» Π±Ρ‹ Ρ€Π°ΡΡΡ‡ΠΈΡ‚Π°Ρ‚ΡŒ Π±Π°Π»ΠΊΡƒ Π΄Π²ΡƒΡ‚Π°Π²Ρ€ΠΎΠ²ΠΎΠ³ΠΎ сСчСния,Π½ΠΎ Π½Π΅ ΠΌΠΎΠ³Ρƒ Π½Π°ΠΉΡ‚ΠΈ Ix ΠΈ Wx. Π΄Π΅Π»ΠΎ Π² Ρ‚ΠΎΠΌ Ρ‡Ρ‚ΠΎ ΠΎΠ½Π° Π½Π΅ стандартная,я Π΅Ρ‘ Π±ΡƒΠ΄Ρƒ Π΄Π΅Π»Π°Ρ‚ΡŒ сам,ΠΈΠ· Π΄Π΅Ρ€Π΅Π²Π°.ΠΌΠΎΠΆΠ΅Ρ‚Π΅ Π»ΠΈ Π²Ρ‹ ΠΌΠ½Π΅ ΠΏΠΎΠΌΠΎΡ‡ΡŒ? я ΠΎΠΏΠ»Π°Ρ‡Ρƒ.Ρ‚ΠΎΠ»ΡŒΠΊΠΎ я Π½Π΅ смогу ΠΎΠΏΠ»Π°Ρ‚ΠΈΡ‚ΡŒ элСктронными срСдствами Ρ‚.ΠΊ. Π½Π΅ знаю ΠΊΠ°ΠΊ этим ΠΏΠΎΠ»ΡŒΠ·ΠΎΠ²Π°Ρ‚ΡŒΡΡ.


21-03-2017: Π”ΠΎΠΊΡ‚ΠΎΡ€ Π›ΠΎΠΌ

Π˜Π³ΠΎΡ€ΡŒ, я ΠΎΡ‚ΠΏΡ€Π°Π²ΠΈΠ» Π²Π°ΠΌ письмо.


30-08-2017: Али

Π£Π²Π°ΠΆΠ°Π΅ΠΌΡ‹ΠΉ Π΄ΠΎΠΊΡ‚ΠΎΡ€, ТСлаю Π²Π°ΠΌ всСго Π½Π°ΠΉΠ»ΡƒΡ‡ΡˆΠ΅Π³ΠΎ. ΠŸΠΎΠΌΠΎΠ³ΠΈΡ‚Π΅ поТалуйста, ΠΊΠ°ΠΊΠΈΠΌΠΈ Ρ„ΠΎΡ€ΠΌΡƒΠ»Π°ΠΌΠΈ Π½ΡƒΠΆΠ½Ρ‹ для ΠΏΠΎΠ΄Π±ΠΎΡ€Π° ΠΈ ΠΏΡ€ΠΎΠ²Π΅Ρ€ΠΊΠΈ Π½Π° ΠΏΡ€ΠΎΡ‡Π½ΠΎΡΡ‚ΡŒ Π±Π°Π»ΠΊΡƒ ΡΠ»Π΅Π΄ΡƒΡŽΡ‰ΠΈΡ… сСчСний,:Π¨Π²Π΅Π»Π»Π΅Ρ€,ΡƒΠ³ΠΎΠ»ΠΎΠΊ ΠΈ Π±ΡƒΠ»ΡŒΠ±ΠΎΠ²Ρ‹ΠΉ ΠΏΡ€ΠΎΡ„ΠΈΠ»ΡŒ, имСя допускаСмый ΠΌΠΎΠΌΠ΅Π½Ρ‚ сопротивлСния W=58,58cm3. спасибо большоС ΠΈ ΠΆΠ΄Ρƒ Π²Π°ΡˆΡƒ ΠΏΠΎΠΌΠΎΡ‰ΡŒ.


31-08-2017: Π”ΠΎΠΊΡ‚ΠΎΡ€ Π›ΠΎΠΌ

ΠŸΠΎΡΠΌΠΎΡ‚Ρ€ΠΈΡ‚Π΅ ΡΡ‚Π°Ρ‚ΡŒΡŽ “РасчСт ΡΡ‚Π°Π»ΡŒΠ½Ρ‹Ρ… ΠΎΠ΄Π½ΠΎΠΏΡ€ΠΎΠ»Π΅Ρ‚Π½Ρ‹Ρ… Π±Π°Π»ΠΎΠΊ с ΡˆΠ°Ρ€Π½ΠΈΡ€Π½Ρ‹ΠΌΠΈ ΠΎΠΏΠΎΡ€Π°ΠΌΠΈ ΠΏΡ€ΠΈ ΠΈΠ·Π³ΠΈΠ±Π΅ согласно БП 16. 2/8 ΠΏΠΎΡ‡Π΅ΠΌΡƒ дСлСнная Π½Π° 8 ΠΈ ΠΏΠΎΡ‡Π΅ΠΌΡƒ ΠΈΠ½ΠΎΠ³Π΄Π° Π΄Π΅Π»ΠΈΠΌ Π½Π° 6 ΠΈ 24 ΠΈΡ‚Π΄ подскаТитС поТалуйста Ρ‚ΠΎΠ»ΡŒΠΊΠΎ это Π½Π΅ понял


Π’Π°Π±Π»ΠΈΡ†Π°. Изгиб. ΠžΡΠ΅Π²Ρ‹Π΅ ΠΌΠΎΠΌΠ΅Π½Ρ‚Ρ‹ ΠΈΠ½Π΅Ρ€Ρ†ΠΈΠΈ сСчСний (статичСскиС ΠΌΠΎΠΌΠ΅Π½Ρ‚Ρ‹ сСчСний), осСвыС ΠΌΠΎΠΌΠ΅Π½Ρ‚Ρ‹ сопротивлСния ΠΈ радиусы ΠΈΠ½Π΅Ρ€Ρ†ΠΈΠΈ плоских Ρ„ΠΈΠ³ΡƒΡ€.

Π›Π΅Π³Π΅Π½Π΄Π°:
  • Ο€ – матСматичСская константа (3,14)
  • d, D – Π΄ΠΈΠ°ΠΌΠ΅Ρ‚Ρ€
  • r – радиус
  • с – ΠΎΡ‚Π½ΠΎΡˆΠ΅Π½ΠΈΠ΅ 2Ρ… Π΄ΠΈΠ°ΠΌΠ΅Ρ‚Ρ€ΠΎΠ² Π΄Ρ€ΡƒΠ³ ΠΊ Π΄Ρ€ΡƒΠ³Ρƒ
  • s – Ρ‚ΠΎΠ»Ρ‰ΠΈΠ½Π°
Π›Π΅Π³Π΅Π½Π΄Π°:
  • h – высота
  • Ξ± – Π΄ΠΈΠ°ΠΌΠ΅Ρ‚Ρ€
  • b – ΡˆΠΈΡ€ΠΈΠ½Π°, Π΄Π»ΠΈΠ½Π°
  • О – Ρ†Π΅Π½Ρ‚Ρ€

Π€ΠΎΡ€ΠΌΠ° ΠΏΠΎΠΏΠ΅Ρ€Π΅Ρ‡Π½ΠΎΠ³ΠΎ сСчСния

ОсСвой ΠΌΠΎΠΌΠ΅Π½Ρ‚ ΠΈΠ½Π΅Ρ€Ρ†ΠΈΠΈ, J, см4

ΠœΠΎΠΌΠ΅Π½Ρ‚ сопротивлСния W, см3

Радиус ΠΈΠ½Π΅Ρ€Ρ†ΠΈΠΈ i, см

ΠšΡ€ΡƒΠ³
ΠšΠΎΠ»ΡŒΡ†ΠΎ

c=d1/d
ВонкостСнноС ΠΊΠΎΠ»ΡŒΡ†ΠΎ

s≀(D/10)
ΠŸΠΎΠ»ΡƒΠΊΡ€ΡƒΠ³

Vo=2d/3Ο€=0,2122d=0,4244r
ΠšΡ€ΡƒΠ³ΠΎΠ²ΠΎΠΉ сСгмСнт

ΠšΡ€ΡƒΠ³ΠΎΠ²ΠΎΠΉ сСктор

ΠšΡ€ΡƒΠ³ΠΎΠ²ΠΎΠ΅ ΠΏΠΎΠ»ΡƒΠΊΠΎΠ»ΡŒΡ†ΠΎ

Π‘Π΅ΠΊΡ‚ΠΎΡ€ ΠΊΡ€ΡƒΠ³ΠΎΠ²ΠΎΠ³ΠΎ ΠΊΠΎΠ»ΡŒΡ†Π°

ΠŸΡ€ΠΎΡ„ΠΈΠ»ΡŒ с симмСтричными закруглСниями

Эллипс

ΠšΠ²Π°Π΄Ρ€Π°Ρ‚

ΠŸΠΎΠ»Ρ‹ΠΉ ΠΊΠ²Π°Π΄Ρ€Π°Ρ‚

Β 

ΠŸΠΎΠ»Ρ‹ΠΉ тонкостСнный ΠΊΠ²Π°Π΄Ρ€Π°Ρ‚

s<(B/15)
ΠšΠ²Π°Π΄Ρ€Π°Ρ‚, поставлСнный Π½Π° Ρ€Π΅Π±Ρ€ΠΎ

Π‘Ρ€Π΅Π· Π²Π΅Ρ€Ρ…Π½Π΅Π³ΠΎ ΠΈ Π½ΠΈΠΆΠ½Π΅Π³ΠΎ ΡƒΠ³Π»ΠΎΠ² ΡƒΠ²Π΅Π»ΠΈΡ‡ΠΈΠ²Π°Π΅Ρ‚ Wx;

ΠΏΡ€ΠΈ срСзС ΡƒΠ³Π»ΠΎΠ² Π½Π° Π‘=1/18 Π΄ΠΈΠ°Π³ΠΎΠ½Π°Π»ΠΈ с ΠΊΠ°ΠΆΠ΄ΠΎΠΉ стороны

ΠΌΠΎΠΌΠ΅Π½Ρ‚ сопротивлСния увСличиваСтся Π΄ΠΎ Wx=0,124b3

ΠŸΠΎΠ»Ρ‹ΠΉ ΠΊΠ²Π°Π΄Ρ€Π°Ρ‚, поставлСнный Π½Π° Ρ€Π΅Π±Ρ€ΠΎ

ΠŸΡ€ΡΠΌΠΎΡƒΠ³ΠΎΠ»ΡŒΠ½ΠΈΠΊ

Β 

ΠŸΡ€ΡΠΌΠΎΡƒΠ³ΠΎΠ»ΡŒΠ½ΠΈΠΊ ΠΏΠΎΠ²Π΅Ρ€Π½ΡƒΡ‚Ρ‹ΠΉ

ΠŸΠΎΠ»Ρ‹ΠΉ ΠΏΡ€ΡΠΌΠΎΡƒΠ³ΠΎΠ»ΡŒΠ½ΠΈΠΊ

ΠŸΠΎΠ»Ρ‹ΠΉ тонкостСнный ΠΏΡ€ΡΠΌΠΎΡƒΠ³ΠΎΠ»ΡŒΠ½ΠΈΠΊ

Π‘Π΅Ρ‡Π΅Π½ΠΈΠ΅ ΠΈΠ· Π΄Π²ΡƒΡ… Ρ€Π°Π²Π½Ρ‹Ρ… ΠΏΡ€ΡΠΌΠΎΡƒΠ³ΠΎΠ»ΡŒΠ½ΠΈΠΊΠΎΠ²

Π’Ρ€Π΅ΡƒΠ³ΠΎΠ»ΡŒΠ½ΠΈΠΊΒ 

ΠŸΡ€ΠΈ вычислСнии напряТСния Π² Π²Π΅Ρ€ΡˆΠΈΠ½Π΅ Ρ‚Ρ€Π΅ΡƒΠ³ΠΎΠ»ΡŒΠ½ΠΈΠΊΠ°

ΠΏΡ€ΠΈ вычислСнии напряТСния Π² Ρ‚ΠΎΡ‡ΠΊΠ΅ основания

ΠŸΠΎΡΡ‚Π°Π²Π»Π΅Π½Π½Ρ‹ΠΉ Π½Π° Ρ€Π΅Π±Ρ€ΠΎ Ρ‚Ρ€Π΅ΡƒΠ³ΠΎΠ»ΡŒΠ½ΠΈΠΊ

ВрапСция

ΠŸΡ€ΠΈ вычислСнии напряТСний Π² Ρ‚ΠΎΡ‡ΠΊΠ°Ρ…

Π²Π΅Ρ€Ρ…Π½Π΅Π³ΠΎ основания

Π² Ρ‚ΠΎΡ‡ΠΊΠ°Ρ… Π½ΠΈΠΆΠ½Π΅Π³ΠΎ основания

ВрапСция

Π’Π°Π²Ρ€

Для Π½ΠΈΠΆΠ½ΠΈΡ… Π²ΠΎΠ»ΠΎΠΊΠΎΠ½

Для Π²Π΅Ρ€Ρ…Π½ΠΈΡ… Π²ΠΎΠ»ΠΎΠΊΠΎΠ½

ΠšΠΎΡ€Ρ‹Ρ‚Π½ΠΎΠ΅ сСчСниС 

ΠšΡ€Π΅ΡΡ‚ΠΎΠΎΠ±Ρ€Π°Π·Π½ΠΎΠ΅ сСчСниС

ΠŸΡ€Π°Π²ΠΈΠ»ΡŒΠ½Ρ‹ΠΉ ΡˆΠ΅ΡΡ‚ΠΈΡƒΠ³ΠΎΠ»ΡŒΠ½ΠΈΠΊ

ΠŸΡ€Π°Π²ΠΈΠ»ΡŒΠ½Ρ‹ΠΉ Π²ΠΎΡΡŒΠΌΠΈΡƒΠ³ΠΎΠ»ΡŒΠ½ΠΈΠΊ

4.

3. ΠœΠΎΠΌΠ΅Π½Ρ‚Ρ‹ ΠΈΠ½Π΅Ρ€Ρ†ΠΈΠΈ простых Ρ„ΠΈΠ³ΡƒΡ€

Как ΡƒΠΆΠ΅ ΠΎΡ‚ΠΌΠ΅Ρ‡Π°Π»ΠΎΡΡŒ Π²Ρ‹ΡˆΠ΅, ΠΊ числу простых плоских Ρ„ΠΈΠ³ΡƒΡ€ относятся Ρ‚Ρ€ΠΈ Ρ„ΠΈΠ³ΡƒΡ€Ρ‹: ΠΏΡ€ΡΠΌΠΎΡƒΠ³ΠΎΠ»ΡŒΠ½ΠΈΠΊ, Ρ‚Ρ€Π΅ΡƒΠ³ΠΎΠ»ΡŒΠ½ΠΈΠΊ ΠΈ ΠΊΡ€ΡƒΠ³. ΠŸΡ€ΠΎΡΡ‚Ρ‹ΠΌΠΈ эти Ρ„ΠΈΠ³ΡƒΡ€Ρ‹ ΡΡ‡ΠΈΡ‚Π°ΡŽΡ‚ΡΡ ΠΏΠΎΡ‚ΠΎΠΌΡƒ, Ρ‡Ρ‚ΠΎ ΠΏΠΎΠ»ΠΎΠΆΠ΅Π½ΠΈΠ΅ Ρ†Π΅Π½Ρ‚Ρ€Π° тяТСсти этих Ρ„ΠΈΠ³ΡƒΡ€ Π·Π°Ρ€Π°Π½Π΅Π΅ извСстно. ВсС ΠΎΡΡ‚Π°Π»ΡŒΠ½Ρ‹Π΅ Ρ„ΠΈΠ³ΡƒΡ€Ρ‹ ΠΌΠΎΠ³ΡƒΡ‚ Π±Ρ‹Ρ‚ΡŒ составлСны ΠΈΠ· этих простых Ρ„ΠΈΠ³ΡƒΡ€ ΠΈ ΡΡ‡ΠΈΡ‚Π°ΡŽΡ‚ΡΡ слоТными. Вычислим осСвыС ΠΌΠΎΠΌΠ΅Π½Ρ‚Ρ‹ ΠΈΠ½Π΅Ρ€Ρ†ΠΈΠΈ простых Ρ„ΠΈΠ³ΡƒΡ€ ΠΎΡ‚Π½ΠΎΡΠΈΡ‚Π΅Π»ΡŒΠ½ΠΎ ΠΈΡ… Ρ†Π΅Π½Ρ‚Ρ€Π°Π»ΡŒΠ½Ρ‹Ρ… осСй.

1. ΠŸΡ€ΡΠΌΠΎΡƒΠ³ΠΎΠ»ΡŒΠ½ΠΈΠΊ.Рассмотрим сСчСниС ΠΏΡ€ΡΠΌΠΎΡƒΠ³ΠΎΠ»ΡŒΠ½ΠΎΠ³ΠΎ профиля Ρ€Π°Π·ΠΌΠ΅Ρ€Π°ΠΌΠΈ(Рис.4.6). Π’Ρ‹Π΄Π΅Π»ΠΈΠΌ элСмСнт сСчСния двумя бСсконСчно Π±Π»ΠΈΠ·ΠΊΠΎ располоТСнными сСчСниями Π½Π° расстоянииот Ρ†Π΅Π½Ρ‚Ρ€Π°Π»ΡŒΠ½ΠΎΠΉ оси.

Рис.4.6

Вычислим ΠΌΠΎΠΌΠ΅Π½Ρ‚ ΠΈΠ½Π΅Ρ€Ρ†ΠΈΠΈ ΠΏΡ€ΡΠΌΠΎΡƒΠ³ΠΎΠ»ΡŒΠ½ΠΎΠ³ΠΎ сСчСния ΠΎΡ‚Π½ΠΎΡΠΈΡ‚Π΅Π»ΡŒΠ½ΠΎ оси :

. (4. 10)

ΠœΠΎΠΌΠ΅Π½Ρ‚ ΠΈΠ½Π΅Ρ€Ρ†ΠΈΠΈ ΠΏΡ€ΡΠΌΠΎΡƒΠ³ΠΎΠ»ΡŒΠ½ΠΎΠ³ΠΎ сСчСния ΠΎΡ‚Π½ΠΎΡΠΈΡ‚Π΅Π»ΡŒΠ½ΠΎ оси Π½Π°ΠΉΠ΄Π΅ΠΌ Π°Π½Π°Π»ΠΎΠ³ΠΈΡ‡Π½ΠΎ. Π—Π΄Π΅ΡΡŒ Π²Ρ‹Π²ΠΎΠ΄ Π½Π΅ приводится.

. (4.11)

Π¦Π΅Π½Ρ‚Ρ€ΠΎΠ±Π΅ΠΆΠ½Ρ‹ΠΉ ΠΌΠΎΠΌΠ΅Π½Ρ‚ ΠΈΠ½Π΅Ρ€Ρ†ΠΈΠΈ ΠΎΡ‚Π½ΠΎΡΠΈΡ‚Π΅Π»ΡŒΠ½ΠΎ осСй ΠΈΡ€Π°Π²Π΅Π½ Π½ΡƒΠ»ΡŽ, Ρ‚Π°ΠΊ ΠΊΠ°ΠΊ ΠΎΡΠΈΠΈΡΠ²Π»ΡΡŽΡ‚ΡΡ осями симмСтрии, Π°, ΡΠ»Π΅Π΄ΠΎΠ²Π°Ρ‚Π΅Π»ΡŒΠ½ΠΎ, Π³Π»Π°Π²Π½Ρ‹ΠΌΠΈ осями.

2. Π Π°Π²Π½ΠΎΠ±Π΅Π΄Ρ€Π΅Π½Π½Ρ‹ΠΉ Ρ‚Ρ€Π΅ΡƒΠ³ΠΎΠ»ΡŒΠ½ΠΈΠΊ.Рассмотрим сСчСниС Ρ‚Ρ€Π΅ΡƒΠ³ΠΎΠ»ΡŒΠ½ΠΎΠ³ΠΎ профиля Ρ€Π°Π·ΠΌΠ΅Ρ€Π°ΠΌΠΈ(Рис.4.7). Π’Ρ‹Π΄Π΅Π»ΠΈΠΌ элСмСнт сСчСния двумя бСсконСчно Π±Π»ΠΈΠ·ΠΊΠΎ располоТСнными сСчСниями Π½Π° расстоянииот Ρ†Π΅Π½Ρ‚Ρ€Π°Π»ΡŒΠ½ΠΎΠΉ оси. Π¦Π΅Π½Ρ‚Ρ€ тяТСсти Ρ‚Ρ€Π΅ΡƒΠ³ΠΎΠ»ΡŒΠ½ΠΈΠΊΠ° находится Π½Π° расстояниот основания. Π’Ρ€Π΅ΡƒΠ³ΠΎΠ»ΡŒΠ½ΠΈΠΊ принимаСтся Ρ€Π°Π²Π½ΠΎΠ±Π΅Π΄Ρ€Π΅Π½Π½Ρ‹ΠΌ, Ρ‚Π°ΠΊ Ρ‡Ρ‚ΠΎ ΠΎΡΡŒΡΠ΅Ρ‡Π΅Π½ΠΈΡ являСтся осью симмСтрии.

Рис.4.7

Вычислим ΠΌΠΎΠΌΠ΅Π½Ρ‚ ΠΈΠ½Π΅Ρ€Ρ†ΠΈΠΈ сСчСния ΠΎΡ‚Π½ΠΎΡΠΈΡ‚Π΅Π»ΡŒΠ½ΠΎ оси :

. (4. 12)

Π’Π΅Π»ΠΈΡ‡ΠΈΠ½Ρƒ ΠΎΠΏΡ€Π΅Π΄Π΅Π»ΠΈΠΌ ΠΈΠ· подобия Ρ‚Ρ€Π΅ΡƒΠ³ΠΎΠ»ΡŒΠ½ΠΈΠΊΠΎΠ²:

; ΠΎΡ‚ΠΊΡƒΠ΄Π° .

ΠŸΠΎΠ΄ΡΡ‚Π°Π²Π»ΡΡ выраТСния для Π² (4.12) ΠΈ интСгрируя, ΠΏΠΎΠ»ΡƒΡ‡ΠΈΠΌ:

. (4.13)

ΠœΠΎΠΌΠ΅Π½Ρ‚ ΠΈΠ½Π΅Ρ€Ρ†ΠΈΠΈ для Ρ€Π°Π²Π½ΠΎΠ±Π΅Π΄Ρ€Π΅Π½Π½ΠΎΠ³ΠΎ Ρ‚Ρ€Π΅ΡƒΠ³ΠΎΠ»ΡŒΠ½ΠΈΠΊΠ° ΠΎΡ‚Π½ΠΎΡΠΈΡ‚Π΅Π»ΡŒΠ½ΠΎ оси находится Π°Π½Π°Π»ΠΎΠ³ΠΈΡ‡Π½Ρ‹ΠΌ ΠΎΠ±Ρ€Π°Π·ΠΎΠΌ ΠΈ Ρ€Π°Π²Π΅Π½:

(4.14)

Π¦Π΅Π½Ρ‚Ρ€ΠΎΠ±Π΅ΠΆΠ½Ρ‹ΠΉ ΠΌΠΎΠΌΠ΅Π½Ρ‚ ΠΈΠ½Π΅Ρ€Ρ†ΠΈΠΈ ΠΎΡ‚Π½ΠΎΡΠΈΡ‚Π΅Π»ΡŒΠ½ΠΎ осСй ΠΈΡ€Π°Π²Π΅Π½ Π½ΡƒΠ»ΡŽ, Ρ‚Π°ΠΊ ΠΊΠ°ΠΊ ΠΎΡΡŒΡΠ²Π»ΡΠ΅Ρ‚ΡΡ осью симмСтрии сСчСния.

3. ΠšΡ€ΡƒΠ³. Рассмотрим сСчСниС ΠΊΡ€ΡƒΠ³Π»ΠΎΠ³ΠΎ профиля Π΄ΠΈΠ°ΠΌΠ΅Ρ‚Ρ€ΠΎΠΌ(Рис.4.8). Π’Ρ‹Π΄Π΅Π»ΠΈΠΌ элСмСнт сСчСния двумя бСсконСчно Π±Π»ΠΈΠ·ΠΊΠΎ располоТСнными концСнтричСскими окруТностями, располоТСнными Π½Π° расстоянииот Ρ†Π΅Π½Ρ‚Ρ€Π° тяТСсти ΠΊΡ€ΡƒΠ³Π°.

Рис.4.8

Вычислим полярный ΠΌΠΎΠΌΠ΅Π½Ρ‚ ΠΈΠ½Π΅Ρ€Ρ†ΠΈΠΈ ΠΊΡ€ΡƒΠ³Π°, воспользовавшись Π²Ρ‹Ρ€Π°ΠΆΠ΅Π½ΠΈΠ΅ΠΌ (4. 5):

. (4.15)

Π˜ΡΠΏΠΎΠ»ΡŒΠ·ΡƒΡ условиС инвариантности для суммы осСвых ΠΌΠΎΠΌΠ΅Π½Ρ‚ΠΎΠ² ΠΈΠ½Π΅Ρ€Ρ†ΠΈΠΈ ΠΎΡ‚Π½ΠΎΡΠΈΡ‚Π΅Π»ΡŒΠ½ΠΎ Π΄Π²ΡƒΡ… Π²Π·Π°ΠΈΠΌΠ½ΠΎ пСрпСндикулярных осСй (4.6) ΠΈ учитывая, Ρ‡Ρ‚ΠΎ для ΠΊΡ€ΡƒΠ³Π° Π² силу симмСтрии , опрСдСляСм Π²Π΅Π»ΠΈΡ‡ΠΈΠ½Ρƒ осСвых ΠΌΠΎΠΌΠ΅Π½Ρ‚ΠΎΠ² ΠΈΠ½Π΅Ρ€Ρ†ΠΈΠΈ:

. (4.16)

ΠžΡ‚ΠΊΡƒΠ΄Π°:

. (4.17)

Π¦Π΅Π½Ρ‚Ρ€ΠΎΠ±Π΅ΠΆΠ½Ρ‹ΠΉ ΠΌΠΎΠΌΠ΅Π½Ρ‚ ΠΈΠ½Π΅Ρ€Ρ†ΠΈΠΈ ΠΎΡ‚Π½ΠΎΡΠΈΡ‚Π΅Π»ΡŒΠ½ΠΎ осСй ΠΈΡ€Π°Π²Π΅Π½ Π½ΡƒΠ»ΡŽ, Ρ‚Π°ΠΊ ΠΊΠ°ΠΊ ΠΎΡΠΈΠΈΡΠ²Π»ΡΡŽΡ‚ΡΡ осями симмСтрии сСчСния.

4.4. Зависимости ΠΌΠ΅ΠΆΠ΄Ρƒ ΠΌΠΎΠΌΠ΅Π½Ρ‚Π°ΠΌΠΈ ΠΈΠ½Π΅Ρ€Ρ†ΠΈΠΈ ΠΎΡ‚Π½ΠΎΡΠΈΡ‚Π΅Π»ΡŒΠ½ΠΎ ΠΏΠ°Ρ€Π°Π»Π»Π΅Π»ΡŒΠ½Ρ‹Ρ… осСй

ΠŸΡ€ΠΈ вычислСнии ΠΌΠΎΠΌΠ΅Π½Ρ‚ΠΎΠ² ΠΈΠ½Π΅Ρ€Ρ†ΠΈΠΈ для слоТных Ρ„ΠΈΠ³ΡƒΡ€ слСдуСт Π·Π°ΠΏΠΎΠΌΠ½ΠΈΡ‚ΡŒ ΠΎΠ΄Π½ΠΎ ΠΏΡ€Π°Π²ΠΈΠ»ΠΎ: значСния для ΠΌΠΎΠΌΠ΅Π½Ρ‚ΠΎΠ² ΠΈΠ½Π΅Ρ€Ρ†ΠΈΠΈ ΠΌΠΎΠΆΠ½ΠΎ ΡΠΊΠ»Π°Π΄Ρ‹Π²Π°Ρ‚ΡŒ, Ссли ΠΎΠ½ΠΈ вычислСны ΠΎΡ‚Π½ΠΎΡΠΈΡ‚Π΅Π»ΡŒΠ½ΠΎ ΠΎΠ΄Π½ΠΎΠΉ ΠΈ Ρ‚ΠΎΠΉ ΠΆΠ΅ оси. Для слоТных Ρ„ΠΈΠ³ΡƒΡ€ Ρ‡Π°Ρ‰Π΅ всСго Ρ†Π΅Π½Ρ‚Ρ€Ρ‹ тяТСсти ΠΎΡ‚Π΄Π΅Π»ΡŒΠ½Ρ‹Ρ… простых Ρ„ΠΈΠ³ΡƒΡ€ ΠΈ всСй Ρ„ΠΈΠ³ΡƒΡ€Ρ‹ Π½Π΅ ΡΠΎΠ²ΠΏΠ°Π΄Π°ΡŽΡ‚. НС ΡΠΎΠ²ΠΏΠ°Π΄Π°ΡŽΡ‚, соотвСтствСнно, ΠΈ Ρ†Π΅Π½Ρ‚Ρ€Π°Π»ΡŒΠ½Ρ‹Π΅ оси для ΠΎΡ‚Π΄Π΅Π»ΡŒΠ½Ρ‹Ρ… простых Ρ„ΠΈΠ³ΡƒΡ€ ΠΈ всСй Ρ„ΠΈΠ³ΡƒΡ€Ρ‹. Π’ связи с этим ΡΡƒΡ‰Π΅ΡΡ‚Π²ΡƒΡŽΡ‚ ΠΏΡ€ΠΈΠ΅ΠΌΡ‹ привСдСния ΠΌΠΎΠΌΠ΅Π½Ρ‚ΠΎΠ² ΠΈΠ½Π΅Ρ€Ρ†ΠΈΠΈ ΠΊ ΠΎΠ΄Π½ΠΎΠΉ оси, Π½Π°ΠΏΡ€ΠΈΠΌΠ΅Ρ€, Ρ†Π΅Π½Ρ‚Ρ€Π°Π»ΡŒΠ½ΠΎΠΉ оси всСй Ρ„ΠΈΠ³ΡƒΡ€Ρ‹. Π­Ρ‚ΠΎ ΠΌΠΎΠΆΠ΅Ρ‚ Π±Ρ‹Ρ‚ΡŒ связано с ΠΏΠ°Ρ€Π°Π»Π»Π΅Π»ΡŒΠ½Ρ‹ΠΌ пСрСносом осСй ΠΈΠ½Π΅Ρ€Ρ†ΠΈΠΈ ΠΈ Π΄ΠΎΠΏΠΎΠ»Π½ΠΈΡ‚Π΅Π»ΡŒΠ½Ρ‹ΠΌΠΈ вычислСниями.

Рассмотрим ΠΎΠΏΡ€Π΅Π΄Π΅Π»Π΅Π½ΠΈΠ΅ ΠΌΠΎΠΌΠ΅Π½Ρ‚ΠΎΠ² ΠΈΠ½Π΅Ρ€Ρ†ΠΈΠΈ ΠΎΡ‚Π½ΠΎΡΠΈΡ‚Π΅Π»ΡŒΠ½ΠΎ ΠΏΠ°Ρ€Π°Π»Π»Π΅Π»ΡŒΠ½Ρ‹Ρ… осСй ΠΈΠ½Π΅Ρ€Ρ†ΠΈΠΈ, ΠΈΠ·ΠΎΠ±Ρ€Π°ΠΆΠ΅Π½Π½Ρ‹Ρ… Π½Π° рис.4.9.

Рис.4.9

ΠŸΡƒΡΡ‚ΡŒ осСвыС ΠΈ Ρ†Π΅Π½Ρ‚Ρ€ΠΎΠ±Π΅ΠΆΠ½Ρ‹ΠΉ ΠΌΠΎΠΌΠ΅Π½Ρ‚Ρ‹ ΠΈΠ½Π΅Ρ€Ρ†ΠΈΠΈ ΠΈΠ·ΠΎΠ±Ρ€Π°ΠΆΠ΅Π½Π½ΠΎΠΉ Π½Π° рис.4.9. Ρ„ΠΈΠ³ΡƒΡ€Ρ‹ ΠΎΡ‚Π½ΠΎΡΠΈΡ‚Π΅Π»ΡŒΠ½ΠΎ ΠΏΡ€ΠΎΠΈΠ·Π²ΠΎΠ»ΡŒΠ½ΠΎ Π²Ρ‹Π±Ρ€Π°Π½Π½Ρ‹Ρ… осСй ис Π½Π°Ρ‡Π°Π»ΠΎΠΌ ΠΊΠΎΠΎΡ€Π΄ΠΈΠ½Π°Ρ‚ Π² точкСизвСстны. ВрСбуСтся Π²Ρ‹Ρ‡ΠΈΡΠ»ΠΈΡ‚ΡŒ осСвыС ΠΈ Ρ†Π΅Π½Ρ‚Ρ€ΠΎΠ±Π΅ΠΆΠ½Ρ‹ΠΉ ΠΌΠΎΠΌΠ΅Π½Ρ‚Ρ‹ ΠΈΠ½Π΅Ρ€Ρ†ΠΈΠΈ Ρ„ΠΈΠ³ΡƒΡ€Ρ‹ ΠΎΡ‚Π½ΠΎΡΠΈΡ‚Π΅Π»ΡŒΠ½ΠΎ ΠΏΡ€ΠΎΠΈΠ·Π²ΠΎΠ»ΡŒΠ½Ρ‹Ρ… ΠΏΠ°Ρ€Π°Π»Π»Π΅Π»ΡŒΠ½Ρ‹Ρ… осСйис Π½Π°Ρ‡Π°Π»ΠΎΠΌ ΠΊΠΎΠΎΡ€Π΄ΠΈΠ½Π°Ρ‚ Π² Ρ‚ΠΎΡ‡ΠΊΠ΅. ΠžΡΠΈΠΈΠΏΡ€ΠΎΠ²Π΅Π΄Π΅Π½Ρ‹ Π½Π° расстоянияхисоотвСтствСнно ΠΎΡ‚ осСйи.

Π’ΠΎΡΠΏΠΎΠ»ΡŒΠ·ΡƒΠ΅ΠΌΡΡ выраТСниями для осСвых ΠΌΠΎΠΌΠ΅Π½Ρ‚ΠΎΠ² ΠΈΠ½Π΅Ρ€Ρ†ΠΈΠΈ (4. 4) ΠΈ для Ρ†Π΅Π½Ρ‚Ρ€ΠΎΠ±Π΅ΠΆΠ½ΠΎΠ³ΠΎ ΠΌΠΎΠΌΠ΅Π½Ρ‚Π° ΠΈΠ½Π΅Ρ€Ρ†ΠΈΠΈ (4.7). ΠŸΠΎΠ΄ΡΡ‚Π°Π²ΠΈΠΌ Π² эти выраТСния вмСсто Ρ‚Π΅ΠΊΡƒΡ‰ΠΈΡ… ΠΊΠΎΠΎΡ€Π΄ΠΈΠ½Π°Ρ‚ иэлСмСнта с бСсконСчно ΠΌΠ°Π»ΠΎΠΉ ΠΏΠ»ΠΎΡ‰Π°Π΄ΡŒΡŽ ΠΊΠΎΠΎΡ€Π΄ΠΈΠ½Π°Ρ‚Ρ‹ΠΈΠ² Π½ΠΎΠ²ΠΎΠΉ систСмС ΠΊΠΎΠΎΡ€Π΄ΠΈΠ½Π°Ρ‚. ΠŸΠΎΠ»ΡƒΡ‡ΠΈΠΌ:

. (4.18)

. (4.19)

.

(4.20)

Анализируя ΠΏΠΎΠ»ΡƒΡ‡Π΅Π½Π½Ρ‹Π΅ выраТСния, ΠΏΡ€ΠΈΡ…ΠΎΠ΄ΠΈΠΌ ΠΊ Π²Ρ‹Π²ΠΎΠ΄Ρƒ, Ρ‡Ρ‚ΠΎ ΠΏΡ€ΠΈ вычислСнии ΠΌΠΎΠΌΠ΅Π½Ρ‚ΠΎΠ² ΠΈΠ½Π΅Ρ€Ρ†ΠΈΠΈ ΠΎΡ‚Π½ΠΎΡΠΈΡ‚Π΅Π»ΡŒΠ½ΠΎ ΠΏΠ°Ρ€Π°Π»Π»Π΅Π»ΡŒΠ½Ρ‹Ρ… осСй ΠΊ ΠΌΠΎΠΌΠ΅Π½Ρ‚Π°ΠΌ ΠΈΠ½Π΅Ρ€Ρ†ΠΈΠΈ, вычислСнных ΠΎΡ‚Π½ΠΎΡΠΈΡ‚Π΅Π»ΡŒΠ½ΠΎ исходных осСй ΠΈΠ½Π΅Ρ€Ρ†ΠΈΠΈ, слСдуСт ΠΏΡ€ΠΈΠ·Π²ΠΎΠ΄ΠΈΡ‚ΡŒ Π΄ΠΎΠ±Π°Π²ΠΊΠΈ Π² Π²ΠΈΠ΄Π΅ Π΄ΠΎΠΏΠΎΠ»Π½ΠΈΡ‚Π΅Π»ΡŒΠ½Ρ‹Ρ… Ρ‡Π»Π΅Π½ΠΎΠ², ΠΊΠΎΡ‚ΠΎΡ€Ρ‹Π΅ ΠΌΠΎΠ³ΡƒΡ‚ ΠΎΠΊΠ°Π·Π°Ρ‚ΡŒΡΡ Π½Π°ΠΌΠ½ΠΎΠ³ΠΎ большС Π·Π½Π°Ρ‡Π΅Π½ΠΈΠΉ для ΠΌΠΎΠΌΠ΅Π½Ρ‚ΠΎΠ² ΠΈΠ½Π΅Ρ€Ρ†ΠΈΠΈ ΠΎΡ‚Π½ΠΎΡΠΈΡ‚Π΅Π»ΡŒΠ½ΠΎ исходных осСй. ΠŸΠΎΡΡ‚ΠΎΠΌΡƒ ΠΏΡ€Π΅Π½Π΅Π±Ρ€Π΅Π³Π°Ρ‚ΡŒ этими Π΄ΠΎΠΏΠΎΠ»Π½ΠΈΡ‚Π΅Π»ΡŒΠ½Ρ‹ΠΌΠΈ Ρ‡Π»Π΅Π½Π°ΠΌΠΈ Π½ΠΈ Π² ΠΊΠΎΠ΅ΠΌ случаС нСльзя.

РассмотрСнный случай прСдставляСт собой самый ΠΎΠ±Ρ‰ΠΈΠΉ случай ΠΏΠ°Ρ€Π°Π»Π»Π΅Π»ΡŒΠ½ΠΎΠ³ΠΎ пСрСноса осСй, ΠΊΠΎΠ³Π΄Π° Π² качСствС исходных Π±Ρ‹Π»ΠΈ взяты ΠΏΡ€ΠΎΠΈΠ·Π²ΠΎΠ»ΡŒΠ½Ρ‹Π΅ оси ΠΈΠ½Π΅Ρ€Ρ†ΠΈΠΈ. Π’ Π±ΠΎΠ»ΡŒΡˆΠΈΠ½ΡΡ‚Π²Π΅ расчСтов Π²ΡΡ‚Ρ€Π΅Ρ‡Π°ΡŽΡ‚ΡΡ частныС случаи опрСдСлСния ΠΌΠΎΠΌΠ΅Π½Ρ‚ΠΎΠ² ΠΈΠ½Π΅Ρ€Ρ†ΠΈΠΈ.

ΠŸΠ΅Ρ€Π²Ρ‹ΠΉ частный случай. Π˜ΡΡ…ΠΎΠ΄Π½Ρ‹Π΅ оси ΡΠ²Π»ΡΡŽΡ‚ΡΡ Ρ†Π΅Π½Ρ‚Ρ€Π°Π»ΡŒΠ½Ρ‹ΠΌΠΈ осями ΠΈΠ½Π΅Ρ€Ρ†ΠΈΠΈ Ρ„ΠΈΠ³ΡƒΡ€Ρ‹. Π’ΠΎΠ³Π΄Π°, ΠΈΡΠΏΠΎΠ»ΡŒΠ·ΡƒΡ основноС свойство для статичСского ΠΌΠΎΠΌΠ΅Π½Ρ‚Π° ΠΏΠ»ΠΎΡ‰Π°Π΄ΠΈ, ΠΌΠΎΠΆΠ½ΠΎ ΠΈΡΠΊΠ»ΡŽΡ‡ΠΈΡ‚ΡŒ ΠΈΠ· ΡƒΡ€Π°Π²Π½Π΅Π½ΠΈΠΉ (4.18)ο€­(4.20) Ρ‡Π»Π΅Π½Ρ‹ ΡƒΡ€Π°Π²Π½Π΅Π½ΠΈΠΉ, Π² ΠΊΠΎΡ‚ΠΎΡ€Ρ‹Π΅ Π²Ρ…ΠΎΠ΄ΠΈΡ‚ статичСский ΠΌΠΎΠΌΠ΅Π½Ρ‚ ΠΏΠ»ΠΎΡ‰Π°Π΄ΠΈ Ρ„ΠΈΠ³ΡƒΡ€Ρ‹. Π’ Ρ€Π΅Π·ΡƒΠ»ΡŒΡ‚Π°Ρ‚Π΅ ΠΏΠΎΠ»ΡƒΡ‡ΠΈΠΌ:

. (4.21)

. (4.22)

. (4.23)

Π—Π΄Π΅ΡΡŒ оси ΠΈο€­Ρ†Π΅Π½Ρ‚Ρ€Π°Π»ΡŒΠ½Ρ‹Π΅ оси ΠΈΠ½Π΅Ρ€Ρ†ΠΈΠΈ.

Π’Ρ‚ΠΎΡ€ΠΎΠΉ частный случай. Π˜ΡΡ…ΠΎΠ΄Π½Ρ‹Π΅ оси ΡΠ²Π»ΡΡŽΡ‚ΡΡ Π³Π»Π°Π²Π½Ρ‹ΠΌΠΈ осями ΠΈΠ½Π΅Ρ€Ρ†ΠΈΠΈ. Π’ΠΎΠ³Π΄Π°, учитывая, Ρ‡Ρ‚ΠΎ ΠΎΡ‚Π½ΠΎΡΠΈΡ‚Π΅Π»ΡŒΠ½ΠΎ Π³Π»Π°Π²Π½Ρ‹Ρ… осСй ΠΈΠ½Π΅Ρ€Ρ†ΠΈΠΈ Ρ†Π΅Π½Ρ‚Ρ€ΠΎΠ±Π΅ΠΆΠ½Ρ‹ΠΉ ΠΌΠΎΠΌΠ΅Π½Ρ‚ ΠΈΠ½Π΅Ρ€Ρ†ΠΈΠΈ Ρ€Π°Π²Π΅Π½ Π½ΡƒΠ»ΡŽ, ΠΏΠΎΠ»ΡƒΡ‡ΠΈΠΌ:

. (4.24)

. (4.25)

. (4.26)

Π—Π΄Π΅ΡΡŒ оси ΠΈο€­Π³Π»Π°Π²Π½Ρ‹Π΅ оси ΠΈΠ½Π΅Ρ€Ρ†ΠΈΠΈ.

Π’ΠΎΡΠΏΠΎΠ»ΡŒΠ·ΡƒΠ΅ΠΌΡΡ ΠΏΠΎΠ»ΡƒΡ‡Π΅Π½Π½Ρ‹ΠΌΠΈ выраТСниями ΠΈ рассмотрим нСсколько ΠΏΡ€ΠΈΠΌΠ΅Ρ€ΠΎΠ² вычислСния ΠΌΠΎΠΌΠ΅Π½Ρ‚ΠΎΠ² ΠΈΠ½Π΅Ρ€Ρ†ΠΈΠΈ для плоских Ρ„ΠΈΠ³ΡƒΡ€.

ΠŸΡ€ΠΈΠΌΠ΅Ρ€ 4.2.ΠžΠΏΡ€Π΅Π΄Π΅Π»ΠΈΡ‚ΡŒ осСвыС ΠΌΠΎΠΌΠ΅Π½Ρ‚Ρ‹ ΠΈΠ½Π΅Ρ€Ρ†ΠΈΠΈ Ρ„ΠΈΠ³ΡƒΡ€Ρ‹, ΠΏΡ€ΠΈΠ²Π΅Π΄Π΅Π½Π½ΠΎΠΉ Π½Π° рис. 4.10, ΠΎΡ‚Π½ΠΎΡΠΈΡ‚Π΅Π»ΡŒΠ½ΠΎ Ρ†Π΅Π½Ρ‚Ρ€Π°Π»ΡŒΠ½Ρ‹Ρ… осСйи.

Рис.4.10

РСшСниС:

Π’ ΠΏΡ€Π΅Π΄Ρ‹Π΄ΡƒΡ‰Π΅ΠΌ ΠΏΡ€ΠΈΠΌΠ΅Ρ€Π΅ 4.1 для ΠΈΠ·ΠΎΠ±Ρ€Π°ΠΆΠ΅Π½Π½ΠΎΠΉ Π½Π° рис.4.10 Ρ„ΠΈΠ³ΡƒΡ€Ρ‹ Π±Ρ‹Π»ΠΎ ΠΎΠΏΡ€Π΅Π΄Π΅Π»Π΅Π½ΠΎ ΠΏΠΎΠ»ΠΎΠΆΠ΅Π½ΠΈΠ΅ Ρ†Π΅Π½Ρ‚Ρ€Π° тяТСсти Π‘. ΠšΠΎΠΎΡ€Π΄ΠΈΠ½Π°Ρ‚Π° Ρ†Π΅Π½Ρ‚Ρ€Π° тяТСсти ΠΎΡ‚ΠΊΠ»Π°Π΄Ρ‹Π²Π°Π»Π°ΡΡŒ ΠΎΡ‚ оси ΠΈ составила. Вычислим расстоянияимСТду осямиии осямии. Π­Ρ‚ΠΈ расстояния составили соотвСтствСннои. Π’Π°ΠΊ ΠΊΠ°ΠΊ исходныС ΠΎΡΠΈΠΈΡΠ²Π»ΡΡŽΡ‚ΡΡ Ρ†Π΅Π½Ρ‚Ρ€Π°Π»ΡŒΠ½Ρ‹ΠΌΠΈ осями для простых Ρ„ΠΈΠ³ΡƒΡ€ Π² Π²ΠΈΠ΄Π΅ ΠΏΡ€ΡΠΌΠΎΡƒΠ³ΠΎΠ»ΡŒΠ½ΠΈΠΊΠΎΠ², для опрСдСлСния ΠΌΠΎΠΌΠ΅Π½Ρ‚Π° ΠΈΠ½Π΅Ρ€Ρ†ΠΈΠΈ Ρ„ΠΈΠ³ΡƒΡ€Ρ‹ ΠΎΡ‚Π½ΠΎΡΠΈΡ‚Π΅Π»ΡŒΠ½ΠΎ ΠΎΡΠΈΠ²ΠΎΡΠΏΠΎΠ»ΡŒΠ·ΡƒΠ΅ΠΌΡΡ Π²Ρ‹Π²ΠΎΠ΄Π°ΠΌΠΈ для ΠΏΠ΅Ρ€Π²ΠΎΠ³ΠΎ частного случая, Π² частности, Ρ„ΠΎΡ€ΠΌΡƒΠ»ΠΎΠΉ (4. 21).

см4.

ΠœΠΎΠΌΠ΅Π½Ρ‚ ΠΈΠ½Π΅Ρ€Ρ†ΠΈΠΈ ΠΎΡ‚Π½ΠΎΡΠΈΡ‚Π΅Π»ΡŒΠ½ΠΎ оси ΠΏΠΎΠ»ΡƒΡ‡ΠΈΠΌ ΠΏΡƒΡ‚Π΅ΠΌ слоТСния ΠΌΠΎΠΌΠ΅Π½Ρ‚ΠΎΠ² ΠΈΠ½Π΅Ρ€Ρ†ΠΈΠΈ простых Ρ„ΠΈΠ³ΡƒΡ€ ΠΎΡ‚Π½ΠΎΡΠΈΡ‚Π΅Π»ΡŒΠ½ΠΎ этой ΠΆΠ΅ оси, Ρ‚Π°ΠΊ ΠΊΠ°ΠΊ ΠΎΡΡŒΡΠ²Π»ΡΠ΅Ρ‚ΡΡ ΠΎΠ±Ρ‰Π΅ΠΉ Ρ†Π΅Π½Ρ‚Ρ€Π°Π»ΡŒΠ½ΠΎΠΉ осью для простых Ρ„ΠΈΠ³ΡƒΡ€ ΠΈ для всСй Ρ„ΠΈΠ³ΡƒΡ€Ρ‹.

см4.

Π¦Π΅Π½Ρ‚Ρ€ΠΎΠ±Π΅ΠΆΠ½Ρ‹ΠΉ ΠΌΠΎΠΌΠ΅Π½Ρ‚ ΠΈΠ½Π΅Ρ€Ρ†ΠΈΠΈ ΠΎΡ‚Π½ΠΎΡΠΈΡ‚Π΅Π»ΡŒΠ½ΠΎ осСй ΠΈΡ€Π°Π²Π΅Π½ Π½ΡƒΠ»ΡŽ, Ρ‚Π°ΠΊ ΠΊΠ°ΠΊ ось инСрцииявляСтся Π³Π»Π°Π²Π½ΠΎΠΉ осью (осью симмСтрии Ρ„ΠΈΠ³ΡƒΡ€Ρ‹).

ΠŸΡ€ΠΈΠΌΠ΅Ρ€ 4.3. Π§Π΅ΠΌΡƒ Ρ€Π°Π²Π΅Π½ Ρ€Π°Π·ΠΌΠ΅Ρ€ b (Π² см) Ρ„ΠΈΠ³ΡƒΡ€Ρ‹, ΠΈΠ·ΠΎΠ±Ρ€Π°ΠΆΠ΅Π½Π½ΠΎΠΉ Π½Π° рис. 4.11, Ссли ΠΌΠΎΠΌΠ΅Π½Ρ‚ ΠΈΠ½Π΅Ρ€Ρ†ΠΈΠΈ Ρ„ΠΈΠ³ΡƒΡ€Ρ‹ ΠΎΡ‚Π½ΠΎΡΠΈΡ‚Π΅Π»ΡŒΠ½ΠΎ оси Ρ€Π°Π²Π΅Π½ 1000 см4?

Рис.4.11

РСшСниС:

Π’Ρ‹Ρ€Π°Π·ΠΈΠΌ ΠΌΠΎΠΌΠ΅Π½Ρ‚ ΠΈΠ½Π΅Ρ€Ρ†ΠΈΠΈ ΠΎΡ‚Π½ΠΎΡΠΈΡ‚Π΅Π»ΡŒΠ½ΠΎ оси Ρ‡Π΅Ρ€Π΅Π· нСизвСстный Ρ€Π°Π·ΠΌΠ΅Ρ€ сСчСния, воспользовавшись Ρ„ΠΎΡ€ΠΌΡƒΠ»ΠΎΠΉ (4.21), учитывая, Ρ‡Ρ‚ΠΎ расстояниС ΠΌΠ΅ΠΆΠ΄Ρƒ осямииравно 7см:

см4. (а)

РСшая Π²Ρ‹Ρ€Π°ΠΆΠ΅Π½ΠΈΠ΅ (Π°) ΠΎΡ‚Π½ΠΎΡΠΈΡ‚Π΅Π»ΡŒΠ½ΠΎ Ρ€Π°Π·ΠΌΠ΅Ρ€Π° сСчСния , ΠΏΠΎΠ»ΡƒΡ‡ΠΈΠΌ:

см.

ΠŸΡ€ΠΈΠΌΠ΅Ρ€.4.4. Какая ΠΈΠ· Ρ„ΠΈΠ³ΡƒΡ€, ΠΈΠ·ΠΎΠ±Ρ€Π°ΠΆΠ΅Π½Π½Ρ‹Ρ… Π½Π° рис.4.12 , ΠΈΠΌΠ΅Π΅Ρ‚ больший ΠΌΠΎΠΌΠ΅Π½Ρ‚ ΠΈΠ½Π΅Ρ€Ρ†ΠΈΠΈ ΠΎΡ‚Π½ΠΎΡΠΈΡ‚Π΅Π»ΡŒΠ½ΠΎ оси , Ссли ΠΎΠ±Π΅ Ρ„ΠΈΠ³ΡƒΡ€Ρ‹ ΠΈΠΌΠ΅ΡŽΡ‚ ΠΎΠ΄ΠΈΠ½Π°ΠΊΠΎΠ²ΡƒΡŽ ΠΏΠ»ΠΎΡ‰Π°Π΄ΡŒΡΠΌ2?

Рис.4.12

РСшСниС:

1. Π’Ρ‹Ρ€Π°Π·ΠΈΠΌ ΠΏΠ»ΠΎΡ‰Π°Π΄ΠΈ Ρ„ΠΈΠ³ΡƒΡ€ Ρ‡Π΅Ρ€Π΅Π· ΠΈΡ… Ρ€Π°Π·ΠΌΠ΅Ρ€Ρ‹ ΠΈ ΠΎΠΏΡ€Π΅Π΄Π΅Π»ΠΈΠΌ:

Π°) Π΄ΠΈΠ°ΠΌΠ΅Ρ‚Ρ€ сСчСния для ΠΊΡ€ΡƒΠ³Π»ΠΎΠ³ΠΎ сСчСния:

см2; ΠžΡ‚ΠΊΡƒΠ΄Π°ΡΠΌ.

Π±) Ρ€Π°Π·ΠΌΠ΅Ρ€ стороны ΠΊΠ²Π°Π΄Ρ€Π°Ρ‚Π°:

; ΠžΡ‚ΠΊΡƒΠ΄Π°ΡΠΌ.

2. ВычисляСм ΠΌΠΎΠΌΠ΅Π½Ρ‚ ΠΈΠ½Π΅Ρ€Ρ†ΠΈΠΈ для ΠΊΡ€ΡƒΠ³Π»ΠΎΠ³ΠΎ сСчСния:

см4.

3. ВычисляСм ΠΌΠΎΠΌΠ΅Π½Ρ‚ ΠΈΠ½Π΅Ρ€Ρ†ΠΈΠΈ для сСчСния ΠΊΠ²Π°Π΄Ρ€Π°Ρ‚Π½ΠΎΠΉ Ρ„ΠΎΡ€ΠΌΡ‹:

см4.

Бравнивая ΠΏΠΎΠ»ΡƒΡ‡Π΅Π½Π½Ρ‹Π΅ Ρ€Π΅Π·ΡƒΠ»ΡŒΡ‚Π°Ρ‚Ρ‹, ΠΏΡ€ΠΈΡ…ΠΎΠ΄ΠΈΠΌ ΠΊ Π²Ρ‹Π²ΠΎΠ΄Ρƒ, Ρ‡Ρ‚ΠΎ наибольшим ΠΌΠΎΠΌΠ΅Π½Ρ‚ΠΎΠΌ ΠΈΠ½Π΅Ρ€Ρ†ΠΈΠΈ Π±ΡƒΠ΄Π΅Ρ‚ ΠΎΠ±Π»Π°Π΄Π°Ρ‚ΡŒ сСчСниС ΠΊΠ²Π°Π΄Ρ€Π°Ρ‚Π½ΠΎΠΉ Ρ„ΠΎΡ€ΠΌΡ‹ ΠΏΠΎ ΡΡ€Π°Π²Π½Π΅Π½ΠΈΡŽ с сСчСниС ΠΊΡ€ΡƒΠ³Π»ΠΎΠΉ Ρ„ΠΎΡ€ΠΌΡ‹ ΠΏΡ€ΠΈ ΠΎΠ΄ΠΈΠ½Π°ΠΊΠΎΠ²ΠΎΠΉ Ρƒ Π½ΠΈΡ… ΠΏΠ»ΠΎΡ‰Π°Π΄ΠΈ.

ΠŸΡ€ΠΈΠΌΠ΅Ρ€ 4.5.ΠžΠΏΡ€Π΅Π΄Π΅Π»ΠΈΡ‚ΡŒ полярный ΠΌΠΎΠΌΠ΅Π½Ρ‚ ΠΈΠ½Π΅Ρ€Ρ†ΠΈΠΈ (Π² см4) сСчСния ΠΏΡ€ΡΠΌΠΎΡƒΠ³ΠΎΠ»ΡŒΠ½ΠΎΠΉ Ρ„ΠΎΡ€ΠΌΡ‹ ΠΎΡ‚Π½ΠΎΡΠΈΡ‚Π΅Π»ΡŒΠ½ΠΎ Π΅Π³ΠΎ Ρ†Π΅Π½Ρ‚Ρ€Π° тяТСсти, Ссли ΡˆΠΈΡ€ΠΈΠ½Π° сСчСния см, высота сСчСниясм.

РСшСниС:

1. НайдСм ΠΌΠΎΠΌΠ΅Π½Ρ‚Ρ‹ ΠΈΠ½Π΅Ρ€Ρ†ΠΈΠΈ сСчСния ΠΎΡ‚Π½ΠΎΡΠΈΡ‚Π΅Π»ΡŒΠ½ΠΎ Π³ΠΎΡ€ΠΈΠ·ΠΎΠ½Ρ‚Π°Π»ΡŒΠ½ΠΎΠΉ ΠΈ Π²Π΅Ρ€Ρ‚ΠΈΠΊΠ°Π»ΡŒΠ½ΠΎΠΉΡ†Π΅Π½Ρ‚Ρ€Π°Π»ΡŒΠ½Ρ‹Ρ… осСй ΠΈΠ½Π΅Ρ€Ρ†ΠΈΠΈ:

см4;см4.

2. ΠžΠΏΡ€Π΅Π΄Π΅Π»ΡΠ΅ΠΌ полярный ΠΌΠΎΠΌΠ΅Π½Ρ‚ ΠΈΠ½Π΅Ρ€Ρ†ΠΈΠΈ сСчСния ΠΊΠ°ΠΊ сумму осСвых ΠΌΠΎΠΌΠ΅Π½Ρ‚ΠΎΠ² ΠΈΠ½Π΅Ρ€Ρ†ΠΈΠΈ:

см4.

ΠŸΡ€ΠΈΠΌΠ΅Ρ€ 4.6. ΠžΠΏΡ€Π΅Π΄Π΅Π»ΠΈΡ‚ΡŒ ΠΌΠΎΠΌΠ΅Π½Ρ‚ ΠΈΠ½Π΅Ρ€Ρ†ΠΈΠΈ Ρ„ΠΈΠ³ΡƒΡ€Ρ‹ Ρ‚Ρ€Π΅ΡƒΠ³ΠΎΠ»ΡŒΠ½ΠΎΠΉ Ρ„ΠΎΡ€ΠΌΡ‹ ΠΈΠ·ΠΎΠ±Ρ€Π°ΠΆΠ΅Π½Π½ΠΎΠΉ Π½Π° рис.4.13, ΠΎΡ‚Π½ΠΎΡΠΈΡ‚Π΅Π»ΡŒΠ½ΠΎ Ρ†Π΅Π½Ρ‚Ρ€Π°Π»ΡŒΠ½ΠΎΠΉ оси , Ссли ΠΌΠΎΠΌΠ΅Π½Ρ‚ ΠΈΠ½Π΅Ρ€Ρ†ΠΈΠΈ Ρ„ΠΈΠ³ΡƒΡ€Ρ‹ ΠΎΡ‚Π½ΠΎΡΠΈΡ‚Π΅Π»ΡŒΠ½ΠΎ осиравСн 2400 см4.

Рис.4.13

РСшСниС:

ΠœΠΎΠΌΠ΅Π½Ρ‚ ΠΈΠ½Π΅Ρ€Ρ†ΠΈΠΈ сСчСния Ρ‚Ρ€Π΅ΡƒΠ³ΠΎΠ»ΡŒΠ½ΠΎΠΉ Ρ„ΠΎΡ€ΠΌΡ‹ ΠΎΡ‚Π½ΠΎΡΠΈΡ‚Π΅Π»ΡŒΠ½ΠΎ Π³Π»Π°Π²Π½ΠΎΠΉ оси ΠΈΠ½Π΅Ρ€Ρ†ΠΈΠΈ Π±ΡƒΠ΄Π΅Ρ‚ мСньшС ΠΏΠΎ ΡΡ€Π°Π²Π½Π΅Π½ΠΈΡŽ с ΠΌΠΎΠΌΠ΅Π½Ρ‚ΠΎΠΌ ΠΈΠ½Π΅Ρ€Ρ†ΠΈΠΈ ΠΎΡ‚Π½ΠΎΡΠΈΡ‚Π΅Π»ΡŒΠ½ΠΎ осина Π²Π΅Π»ΠΈΡ‡ΠΈΠ½Ρƒ. ΠŸΠΎΡΡ‚ΠΎΠΌΡƒ присм ΠΌΠΎΠΌΠ΅Π½Ρ‚ ΠΈΠ½Π΅Ρ€Ρ†ΠΈΠΈ сСчСния ΠΎΡ‚Π½ΠΎΡΠΈΡ‚Π΅Π»ΡŒΠ½ΠΎ осинайдСм ΡΠ»Π΅Π΄ΡƒΡŽΡ‰ΠΈΠΌ ΠΎΠ±Ρ€Π°Π·ΠΎΠΌ:

см4.

ГСомСтричСскиС характСристики ΡΠΏΠ»ΠΎΡˆΠ½Ρ‹Ρ… сСчСний 017

Π’Π½ΠΈΠΌΠ°Π½ΠΈΠ΅! Π Π°Π·ΠΌΠ΅Ρ€ “с” игнорируСтся!

Π’Π½ΠΈΠΌΠ°Π½ΠΈΠ΅! Π Π°Π·ΠΌΠ΅Ρ€ “a” игнорируСтся!

Ошибка! ΠŸΡ€ΠΎΠ²Π΅Ρ€ΡŒΡ‚Π΅ ΠΏΡ€Π°Π²ΠΈΠ»ΡŒΠ½ΠΎΡΡ‚ΡŒ построСния Ρ‚Ρ€Π΅ΡƒΠ³ΠΎΠ»ΡŒΠ½ΠΈΠΊΠ° ΠΈ Ρ„ΠΎΡ€ΠΌΠ°Ρ‚ Π²Π²ΠΎΠ΄Π° Π΄Π°Π½Π½Ρ‹Ρ….

ΠŸΠ»ΠΎΡ‰Π°Π΄ΡŒ Ρ‚Ρ€Π΅ΡƒΠ³ΠΎΠ»ΡŒΠ½ΠΈΠΊΠ°:

Π¦Π΅Π½Ρ‚Ρ€ тяТСсти Ρ‚Ρ€Π΅ΡƒΠ³ΠΎΠ»ΡŒΠ½ΠΈΠΊΠ°:

Π Π°Π·ΠΌΠ΅Ρ€Ρ‹ Ρ‚Ρ€Π΅ΡƒΠ³ΠΎΠ»ΡŒΠ½ΠΈΠΊΠ°:

ΠœΠΎΠΌΠ΅Π½Ρ‚Ρ‹ ΠΈΠ½Π΅Ρ€Ρ†ΠΈΠΈ Ρ‚Ρ€Π΅ΡƒΠ³ΠΎΠ»ΡŒΠ½ΠΈΠΊΠ°:

ΠŸΠΎΠ»ΡΡ€Π½Ρ‹Π΅ ΠΌΠΎΠΌΠ΅Π½Ρ‚Ρ‹ ΠΈΠ½Π΅Ρ€Ρ†ΠΈΠΈ Ρ‚Ρ€Π΅ΡƒΠ³ΠΎΠ»ΡŒΠ½ΠΈΠΊΠ°:

Радиусы ΠΈΠ½Π΅Ρ€Ρ†ΠΈΠΈ Ρ‚Ρ€Π΅ΡƒΠ³ΠΎΠ»ΡŒΠ½ΠΈΠΊΠ°:

ΠœΠΎΠΌΠ΅Π½Ρ‚Ρ‹ сопротивлСния Ρ‚Ρ€Π΅ΡƒΠ³ΠΎΠ»ΡŒΠ½ΠΈΠΊΠ°:

Π’Π΅Ρ€Ρ…Π½ΠΈΠ΅ Π²ΠΎΠ»ΠΎΠΊΠ½Π°:

НиТниС волокна:

Π›Π΅Π²Ρ‹Π΅ Π²ΠΎΠ»ΠΎΠΊΠ½Π°:

ΠŸΡ€Π°Π²Ρ‹Π΅ Π²ΠΎΠ»ΠΎΠΊΠ½Π°:

ΠΎΠΏΡ€Π΅Π΄Π΅Π»Π΅Π½ΠΈΠ΅, Ρ„ΠΎΡ€ΠΌΡƒΠ»Ρ‹, ΠΏΡ€ΠΈΠΌΠ΅Ρ€Ρ‹ Ρ€Π΅ΡˆΠ΅Π½ΠΈΡ Π·Π°Π΄Π°Ρ‡.

РасчСт Π²ΠΈΠ½Ρ‚ΠΎΠ²Ρ‹Ρ… цилиндричСских ΠΏΡ€ΡƒΠΆΠΈΠ½
Π‘Π°Π·ΠΎΠ²Ρ‹ΠΉ курс Π»Π΅ΠΊΡ†ΠΈΠΉ ΠΏΠΎ сопромату, тСория, ΠΏΡ€Π°ΠΊΡ‚ΠΈΠΊΠ°, Π·Π°Π΄Π°Ρ‡ΠΈ.
1. ГСомСтричСскиС характСристики сСчСний.

1.3. ΠœΠΎΠΌΠ΅Π½Ρ‚Ρ‹ ΠΈΠ½Π΅Ρ€Ρ†ΠΈΠΈ простых сСчСний.

1. ΠŸΡ€ΡΠΌΠΎΡƒΠ³ΠΎΠ»ΡŒΠ½ΠΈΠΊ (рис. 1.5,Π°). Вычислим ΠΌΠΎΠΌΠ΅Π½Ρ‚ ΠΈΠ½Π΅Ρ€Ρ†ΠΈΠΈ сСчСния ΠΎΡ‚Π½ΠΎΡΠΈΡ‚Π΅Π»ΡŒΠ½ΠΎ оси Π₯0 , проходящСй Ρ‡Π΅Ρ€Π΅Π· Ρ†Π΅Π½Ρ‚Ρ€ тяТСсти ΠΏΠ°Ρ€Π°Π»Π»Π΅Π»ΡŒΠ½ΠΎ основанию.

Π—Π° dA ΠΏΡ€ΠΈΠΌΠ΅ΠΌ ΠΏΠ»ΠΎΡ‰Π°Π΄ΡŒ бСсконСчно Ρ‚ΠΎΠ½ΠΊΠΎΠ³ΠΎ слоя dA = bdy. Π’ΠΎΠ³Π΄Π°

Π˜Ρ‚Π°ΠΊ,
(1.11)

Аналогично, ΠΏΠΎΠ»ΡƒΡ‡ΠΈΠΌ
(1.12)

2. ΠšΡ€ΡƒΠ³ (рис. 1.5,Π±). Π‘Π½Π°Ρ‡Π°Π»Π° ΠΎΠΏΡ€Π΅Π΄Π΅Π»ΠΈΠΌ полярный ΠΌΠΎΠΌΠ΅Π½Ρ‚ ΠΈΠ½Π΅Ρ€Ρ†ΠΈΠΈ ΠΎΡ‚Π½ΠΎΡΠΈΡ‚Π΅Π»ΡŒΠ½ΠΎ Ρ†Π΅Π½Ρ‚Ρ€Π° ΠΊΡ€ΡƒΠ³Π°

Π—Π° dA ΠΏΡ€ΠΈΠ½ΠΈΠΌΠ°Π΅ΠΌ ΠΏΠ»ΠΎΡ‰Π°Π΄ΡŒ бСсконСчно Ρ‚ΠΎΠ½ΠΊΠΎΠ³ΠΎ ΠΊΠΎΠ»ΡŒΡ†Π° Ρ‚ΠΎΠ»Ρ‰ΠΈΠ½ΠΎΠΉ dp

Ρ‚ΠΎΠ³Π΄Π°

Π‘Π»Π΅Π΄ΠΎΠ²Π°Ρ‚Π΅Π»ΡŒΠ½ΠΎ,
(1.13)

Π’Π΅ΠΏΠ΅Ρ€ΡŒ Π»Π΅Π³ΠΊΠΎ Π½Π°ΠΉΠ΄Π΅ΠΌ Ixo . Π”Π΅ΠΉΡΡ‚Π²ΠΈΡ‚Π΅Π»ΡŒΠ½ΠΎ, для ΠΊΡ€ΡƒΠ³Π° согласно Ρ„ΠΎΡ€ΠΌΡƒΠ»Π΅ (1.9.), ΠΈΠΌΠ΅Π΅ΠΌ IΡ€ = 2IΡ…ΠΎ = 2IΡƒΠΎ , ΠΎΡ‚ΠΊΡƒΠ΄Π°
(1.14)

2. ΠšΠΎΠ»ΡŒΡ†ΠΎ (рис. 1.5,Π²). ОсСвой ΠΌΠΎΠΌΠ΅Π½Ρ‚ ΠΈΠ½Π΅Ρ€Ρ†ΠΈΠΈ Π² этом случаС Ρ€Π°Π²Π΅Π½ разности ΠΌΠΎΠΌΠ΅Π½Ρ‚ΠΎΠ² ΠΈΠ½Π΅Ρ€Ρ†ΠΈΠΈ внСшнСго ΠΈ Π²Π½ΡƒΡ‚Ρ€Π΅Π½Π½Π΅Π³ΠΎ ΠΊΡ€ΡƒΠ³ΠΎΠ²
(1. 15)
Π³Π΄Π΅ c = d/D.

Аналогично полярный ΠΌΠΎΠΌΠ΅Π½Ρ‚ ΠΈΠ½Π΅Ρ€Ρ†ΠΈΠΈ
(1.16)

2. Π’Ρ€Π΅ΡƒΠ³ΠΎΠ»ΡŒΠ½ΠΈΠΊ (рис. 1.5,Π³). ΠžΠΏΡ€Π΅Π΄Π΅Π»ΠΈΠΌ ΠΌΠΎΠΌΠ΅Π½Ρ‚ ΠΈΠ½Π΅Ρ€Ρ†ΠΈΠΈ ΠΎΡ‚Π½ΠΎΡΠΈΡ‚Π΅Π»ΡŒΠ½ΠΎ оси x1 , ΠΏΠ°Ρ€Π°Π»Π»Π΅Π»ΡŒΠ½ΠΎΠΉ основанию ΠΈ проходящСй Ρ‡Π΅Ρ€Π΅Π· Π²Π΅Ρ€ΡˆΠΈΠ½Ρƒ Ρ‚Ρ€Π΅ΡƒΠ³ΠΎΠ»ΡŒΠ½ΠΈΠΊΠ°

Π—Π° dA ΠΏΡ€ΠΈΠΌΠ΅ΠΌ ΠΏΠ»ΠΎΡ‰Π°Π΄ΡŒ бСсконСчно Ρ‚ΠΎΠ½ΠΊΠΎΠΉ Ρ‚Ρ€Π°ΠΏΠ΅Ρ†ΠΈΠΈ KBDE, ΠΏΠ»ΠΎΡ‰Π°Π΄ΡŒ ΠΊΠΎΡ‚ΠΎΡ€ΠΎΠΉ ΠΌΠΎΠΆΠ½ΠΎ ΡΡ‡ΠΈΡ‚Π°Ρ‚ΡŒ Ρ€Π°Π²Π½ΠΎΠΉ ΠΏΠ»ΠΎΡ‰Π°Π΄ΠΈ ΠΏΡ€ΡΠΌΠΎΡƒΠ³ΠΎΠ»ΡŒΠ½ΠΈΠΊΠ°:

DA = by dy,

Π“Π΄Π΅ by – Π΄Π»ΠΈΠ½Π° ΠΏΡ€ΡΠΌΠΎΡƒΠ³ΠΎΠ»ΡŒΠ½ΠΈΠΊΠ°.

ΠžΠŸΠ Π•Π”Π•Π›Π•ΠΠ˜Π•

ΠžΡΠ΅Π²Ρ‹ΠΌ (ΠΈΠ»ΠΈ ΡΠΊΠ²Π°Ρ‚ΠΎΡ€ΠΈΠ°Π»ΡŒΠ½Ρ‹ΠΌ) ΠΌΠΎΠΌΠ΅Π½Ρ‚ΠΎΠΌ ΠΈΠ½Π΅Ρ€Ρ†ΠΈΠΈ сСчСния ΠΎΡ‚Π½ΠΎΡΠΈΡ‚Π΅Π»ΡŒΠ½ΠΎ оси называСтся Π²Π΅Π»ΠΈΡ‡ΠΈΠ½Π°, ΠΊΠΎΡ‚ΠΎΡ€ΡƒΡŽ ΠΎΠΏΡ€Π΅Π΄Π΅Π»ΡΡŽΡ‚ ΠΊΠ°ΠΊ:

Π’Ρ‹Ρ€Π°ΠΆΠ΅Π½ΠΈΠ΅ (1) ΠΎΠ±ΠΎΠ·Π½Π°Ρ‡Π°Π΅Ρ‚, для вычислСния осСвого ΠΌΠΎΠΌΠ΅Π½Ρ‚Π° ΠΈΠ½Π΅Ρ€Ρ†ΠΈΠΈ бСрСтся ΠΏΠΎ всСй ΠΏΠ»ΠΎΡ‰Π°Π΄ΠΈ S сумма ΠΏΡ€ΠΎΠΈΠ·Π²Π΅Π΄Π΅Π½ΠΈΠΉ бСсконСчно ΠΌΠ°Π»Ρ‹Ρ… ΠΏΠ»ΠΎΡ‰Π°Π΄ΠΎΠΊ () ΡƒΠΌΠ½ΠΎΠΆΠ΅Π½Π½Ρ‹Ρ… Π½Π° ΠΊΠ²Π°Π΄Ρ€Π°Ρ‚Ρ‹ расстояний ΠΎΡ‚ Π½ΠΈΡ… Π΄ΠΎ оси вращСния:

Π‘ΡƒΠΌΠΌΠ° осСвых ΠΌΠΎΠΌΠ΅Π½Ρ‚ΠΎΠ² ΠΈΠ½Π΅Ρ€Ρ†ΠΈΠΈ сСчСния ΠΎΡ‚Π½ΠΎΡΠΈΡ‚Π΅Π»ΡŒΠ½ΠΎ Π²Π·Π°ΠΈΠΌΠ½ΠΎ пСрпСндикулярных осСй (Π½Π°ΠΏΡ€ΠΈΠΌΠ΅Ρ€, ΠΎΡ‚Π½ΠΎΡΠΈΡ‚Π΅Π»ΡŒΠ½ΠΎ осСй X ΠΈ Y Π² Π΄Π΅ΠΊΠ°Ρ€Ρ‚ΠΎΠ²ΠΎΠΉ систСмС ΠΊΠΎΠΎΡ€Π΄ΠΈΠ½Π°Ρ‚) Π΄Π°ΡŽΡ‚ полярный ΠΌΠΎΠΌΠ΅Π½Ρ‚ ΠΈΠ½Π΅Ρ€Ρ†ΠΈΠΈ () ΠΎΡ‚Π½ΠΎΡΠΈΡ‚Π΅Π»ΡŒΠ½ΠΎ Ρ‚ΠΎΡ‡ΠΊΠΈ пСрСсСчСния этих осСй:

ΠžΠŸΠ Π•Π”Π•Π›Π•ΠΠ˜Π•

ΠŸΠΎΠ»ΡΡ€Π½Ρ‹ΠΌ ΠΌΠΎΠΌΠ΅Π½Ρ‚ΠΎΠΌ ΠΈΠ½Π΅Ρ€Ρ†ΠΈΠΈ Π½Π°Π·Ρ‹Π²Π°ΡŽΡ‚ ΠΌΠΎΠΌΠ΅Π½Ρ‚ ΠΈΠ½Π΅Ρ€Ρ†ΠΈΠΈ сСчСниСм ΠΏΠΎ ΠΎΡ‚Π½ΠΎΡˆΠ΅Π½ΠΈΡŽ ΠΊ Π½Π΅ΠΊΠΎΡ‚ΠΎΡ€ΠΎΠΉ Ρ‚ΠΎΡ‡ΠΊΠ΅.

ΠžΡΠ΅Π²Ρ‹Π΅ ΠΌΠΎΠΌΠ΅Π½Ρ‚Ρ‹ ΠΈΠ½Π΅Ρ€Ρ†ΠΈΠΈ всСгда большС нуля, Ρ‚Π°ΠΊ ΠΊΠ°ΠΊ Π² ΠΈΡ… опрСдСлСниях (1) ΠΏΠΎΠ΄ Π·Π½Π°ΠΊΠΎΠΌ ΠΈΠ½Ρ‚Π΅Π³Ρ€Π°Π»Π° стоят Π²Π΅Π»ΠΈΡ‡ΠΈΠ½Π° ΠΏΠ»ΠΎΡ‰Π°Π΄ΠΈ элСмСнтарной ΠΏΠ»ΠΎΡ‰Π°Π΄ΠΊΠΈ (), всСгда ΠΏΠΎΠ»ΠΎΠΆΠΈΡ‚Π΅Π»ΡŒΠ½Π°Ρ ΠΈ ΠΊΠ²Π°Π΄Ρ€Π°Ρ‚ расстояния ΠΎΡ‚ этой ΠΏΠ»ΠΎΡ‰Π°Π΄ΠΊΠΈ Π΄ΠΎ оси.

Если ΠΌΡ‹ ΠΈΠΌΠ΅Π΅ΠΌ Π΄Π΅Π»ΠΎ с сСчСниСм слоТной Ρ„ΠΎΡ€ΠΌΡ‹, Ρ‚ΠΎ часто ΠΏΡ€ΠΈ расчСтах ΠΈΡΠΏΠΎΠ»ΡŒΠ·ΡƒΡŽΡ‚ Ρ‚ΠΎ, Ρ‡Ρ‚ΠΎ осСвой ΠΌΠΎΠΌΠ΅Π½Ρ‚ ΠΈΠ½Π΅Ρ€Ρ†ΠΈΠΈ слоТного сСчСния ΠΏΠΎ ΠΎΡ‚Π½ΠΎΡˆΠ΅Π½ΠΈΡŽ ΠΊ оси Ρ€Π°Π²Π΅Π½ суммС осСвых ΠΌΠΎΠΌΠ΅Π½Ρ‚ΠΎΠ² ΠΈΠ½Π΅Ρ€Ρ†ΠΈΠΈ частСй этого сСчСния ΠΎΡ‚Π½ΠΎΡΠΈΡ‚Π΅Π»ΡŒΠ½ΠΎ Ρ‚ΠΎΠΉ ΠΆΠ΅ оси. Однако слСдуСт ΠΏΠΎΠΌΠ½ΠΈΡ‚ΡŒ, Ρ‡Ρ‚ΠΎ нСльзя ΡΡƒΠΌΠΌΠΈΡ€ΠΎΠ²Π°Ρ‚ΡŒ ΠΌΠΎΠΌΠ΅Π½Ρ‚Ρ‹ ΠΈΠ½Π΅Ρ€Ρ†ΠΈΠΈ, ΠΊΠΎΡ‚ΠΎΡ€Ρ‹Π΅ Π½Π°ΠΉΠ΄Π΅Π½Ρ‹ ΠΎΡ‚Π½ΠΎΡΠΈΡ‚Π΅Π»ΡŒΠ½ΠΎ Ρ€Π°Π·Π½Ρ‹Ρ… осСй ΠΈ Ρ‚ΠΎΡ‡Π΅ΠΊ.

ОсСвой ΠΌΠΎΠΌΠ΅Π½Ρ‚ ΠΈΠ½Π΅Ρ€Ρ†ΠΈΠΈ ΠΎΡ‚Π½ΠΎΡΠΈΡ‚Π΅Π»ΡŒΠ½ΠΎ оси проходящСй Ρ‡Π΅Ρ€Π΅Π· Ρ†Π΅Π½Ρ‚Ρ€ тяТСсти сСчСния ΠΈΠΌΠ΅Π΅Ρ‚ наимСньшСС Π·Π½Π°Ρ‡Π΅Π½ΠΈΠ΅ ΠΈΠ· всСх ΠΌΠΎΠΌΠ΅Π½Ρ‚ΠΎΠ² ΠΎΡ‚Π½ΠΎΡΠΈΡ‚Π΅Π»ΡŒΠ½ΠΎ ΠΏΠ°Ρ€Π°Π»Π»Π΅Π»ΡŒΠ½Ρ‹Ρ… с Π½Π΅ΠΉ осСй. ΠœΠΎΠΌΠ΅Π½Ρ‚ ΠΈΠ½Π΅Ρ€Ρ†ΠΈΠΈ ΠΎΡ‚Π½ΠΎΡΠΈΡ‚Π΅Π»ΡŒΠ½ΠΎ любой оси () ΠΏΡ€ΠΈ условии Π΅Π΅ ΠΏΠ°Ρ€Π°Π»Π»Π΅Π»ΡŒΠ½ΠΎΡΡ‚ΠΈ с осью, проходящСй Ρ‡Π΅Ρ€Π΅Π· Ρ†Π΅Π½Ρ‚Ρ€ тяТСсти Ρ€Π°Π²Π΅Π½:

Π³Π΄Π΅ – ΠΌΠΎΠΌΠ΅Π½Ρ‚ ΠΈΠ½Π΅Ρ€Ρ†ΠΈΠΈ сСчСния ΠΎΡ‚Π½ΠΎΡΠΈΡ‚Π΅Π»ΡŒΠ½ΠΎ оси проходящСй Ρ‡Π΅Ρ€Π΅Π· Ρ†Π΅Π½Ρ‚Ρ€ тяТСсти сСчСния; – ΠΏΠ»ΠΎΡ‰Π°Π΄ΡŒ сСчСния; – расстояниС ΠΌΠ΅ΠΆΠ΄Ρƒ осями.

ΠŸΡ€ΠΈΠΌΠ΅Ρ€Ρ‹ Ρ€Π΅ΡˆΠ΅Π½ΠΈΡ Π·Π°Π΄Π°Ρ‡

ΠŸΠ Π˜ΠœΠ•Π  1

Π—Π°Π΄Π°Π½ΠΈΠ΅Π§Π΅ΠΌΡƒ Ρ€Π°Π²Π΅Π½ осСвой ΠΌΠΎΠΌΠ΅Π½Ρ‚ ΠΈΠ½Π΅Ρ€Ρ†ΠΈΠΈ Ρ€Π°Π²Π½ΠΎΠ±Π΅Π΄Ρ€Π΅Π½Π½ΠΎΠ³ΠΎ Ρ‚Ρ€Π΅ΡƒΠ³ΠΎΠ»ΡŒΠ½ΠΎΠ³ΠΎ сСчСния ΠΎΡ‚Π½ΠΎΡΠΈΡ‚Π΅Π»ΡŒΠ½ΠΎ оси Z, проходящСй Ρ‡Π΅Ρ€Π΅Π· Ρ†Π΅Π½Ρ‚Ρ€ тяТСсти () Ρ‚Ρ€Π΅ΡƒΠ³ΠΎΠ»ΡŒΠ½ΠΈΠΊΠ°, ΠΏΠ°Ρ€Π°Π»Π»Π΅Π»ΡŒΠ½ΠΎ Π΅Π³ΠΎ основанию? Высота Ρ‚Ρ€Π΅ΡƒΠ³ΠΎΠ»ΡŒΠ½ΠΈΠΊΠ° Ρ€Π°Π²Π½Π° .

Π Π΅ΡˆΠ΅Π½ΠΈΠ΅Π’Ρ‹Π΄Π΅Π»ΠΈΠΌ Π½Π° Ρ‚Ρ€Π΅ΡƒΠ³ΠΎΠ»ΡŒΠ½ΠΎΠΌ сСчСнии ΠΏΡ€ΡΠΌΠΎΡƒΠ³ΠΎΠ»ΡŒΠ½ΡƒΡŽ ΡΠ»Π΅ΠΌΠ΅Π½Ρ‚Π°Ρ€Π½ΡƒΡŽ ΠΏΠ»ΠΎΡ‰Π°Π΄ΠΊΡƒ (см. рис.1). Она находится Π½Π° расстоянии ΠΎΡ‚ оси вращСния, Π΄Π»ΠΈΠ½Π° ΠΎΠ΄Π½ΠΎΠΉ Π΅Π΅ стороны , другая сторона . Из рис.1 слСдуСт, Ρ‡Ρ‚ΠΎ:

ΠŸΠ»ΠΎΡ‰Π°Π΄ΡŒ Π²Ρ‹Π΄Π΅Π»Π΅Π½Π½ΠΎΠ³ΠΎ ΠΏΡ€ΡΠΌΠΎΡƒΠ³ΠΎΠ»ΡŒΠ½ΠΈΠΊΠ° с ΡƒΡ‡Π΅Ρ‚ΠΎΠΌ (1.1) Ρ€Π°Π²Π½Π°:

Для нахоТдСния осСвого ΠΌΠΎΠΌΠ΅Π½Ρ‚Π° ΠΈΠ½Π΅Ρ€Ρ†ΠΈΠΈ ΠΈΡΠΏΠΎΠ»ΡŒΠ·ΡƒΠ΅ΠΌ Π΅Π³ΠΎ ΠΎΠΏΡ€Π΅Π΄Π΅Π»Π΅Π½ΠΈΠ΅ Π² Π²ΠΈΠ΄Π΅:

ΠžΡ‚Π²Π΅Ρ‚

ΠŸΠ Π˜ΠœΠ•Π  2

ЗаданиСНайдитС осСвыС ΠΌΠΎΠΌΠ΅Π½Ρ‚Ρ‹ ΠΈΠ½Π΅Ρ€Ρ†ΠΈΠΈ ΠΎΡ‚Π½ΠΎΡΠΈΡ‚Π΅Π»ΡŒΠ½ΠΎ пСрпСндикулярных осСй X ΠΈ Y (рис.2) сСчСния Π² Π²ΠΈΠ΄Π΅ ΠΊΡ€ΡƒΠ³Π° Π΄ΠΈΠ°ΠΌΠ΅Ρ‚Ρ€ ΠΊΠΎΡ‚ΠΎΡ€ΠΎΠ³ΠΎ Ρ€Π°Π²Π΅Π½ d.

Π Π΅ΡˆΠ΅Π½ΠΈΠ΅Π”Π»Ρ Ρ€Π΅ΡˆΠ΅Π½ΠΈΡ Π·Π°Π΄Π°Ρ‡ΠΈ ΡƒΠ΄ΠΎΠ±Π½Π΅Π΅ Π½Π°Ρ‡Π°Ρ‚ΡŒ с нахоТдСния полярного ΠΌΠΎΠΌΠ΅Π½Ρ‚Π° ΠΎΡ‚Π½ΠΎΡΠΈΡ‚Π΅Π»ΡŒΠ½ΠΎ Ρ†Π΅Π½Ρ‚Ρ€Π° сСчСния (). ВсС сСчСниС Ρ€Π°Π·ΠΎΠ±ΡŒΠ΅ΠΌ Π½Π° бСсконСчно Ρ‚ΠΎΠ½ΠΊΠΈΠ΅ ΠΊΠΎΠ»ΡŒΡ†Π° Ρ‚ΠΎΠ»Ρ‰ΠΈΠ½ΠΎΠΉ , радиус ΠΊΠΎΡ‚ΠΎΡ€Ρ‹Ρ… ΠΎΠ±ΠΎΠ·Π½Π°Ρ‡ΠΈΠΌ . Π’ΠΎΠ³Π΄Π° ΡΠ»Π΅ΠΌΠ΅Π½Ρ‚Π°Ρ€Π½ΡƒΡŽ ΠΏΠ»ΠΎΡ‰Π°Π΄ΡŒ Π½Π°ΠΉΠ΄Π΅ΠΌ ΠΊΠ°ΠΊ:

Рассматривая Π² ΠΏΡ€Π΅Π΄Ρ‹Π΄ΡƒΡ‰ΠΈΡ… Ρ€Π°Π·Π΄Π΅Π»Π°Ρ… ΠΏΡ€ΠΎΡΡ‚Π΅ΠΉΡˆΠΈΠ΅ Π²ΠΈΠ΄Ρ‹ Π΄Π΅Ρ„ΠΎΡ€ΠΌΠ°Ρ†ΠΈΠΉ – осСвоС растяТСниС ΠΈ сТатиС, смятиС, скалываниС – ΠΌΡ‹ выяснили, Ρ‡Ρ‚ΠΎ ΠΈΡ… сопротивлСниС Π΄Π΅ΠΉΡΡ‚Π²ΡƒΡŽΡ‰Π΅ΠΉ силС ΠΏΡ€ΠΎΠΏΠΎΡ€Ρ†ΠΈΠΎΠ½Π°Π»ΡŒΠ½ΠΎ Ρ‚ΠΎΠ»ΡŒΠΊΠΎ Ρ€Π°Π·ΠΌΠ΅Ρ€Π°ΠΌ ΠΏΠ»ΠΎΡ‰Π°Π΄ΠΈ ΠΏΠΎΠΏΠ΅Ρ€Π΅Ρ‡Π½ΠΎΠ³ΠΎ сСчСния элСмСнта, Π½Π° ΠΊΠΎΡ‚ΠΎΡ€Ρ‹ΠΉ дСйствуСт сила. Π’Π°ΠΊ, ΠΏΡ€ΠΈ ΠΎΠ΄ΠΈΠ½Π°ΠΊΠΎΠ²ΠΎΠΉ ΠΏΠ»ΠΎΡ‰Π°Π΄ΠΈ сСчСния, ΠΎΠ΄Π½ΠΎΠΌ ΠΈ Ρ‚ΠΎΠΌ ΠΆΠ΅ ΠΌΠ°Ρ‚Π΅Ρ€ΠΈΠ°Π»Π΅ ΠΈ ΠΎΠ΄ΠΈΠ½Π°ΠΊΠΎΠ²ΠΎΠΉ силС, Π΄Π΅ΠΉΡΡ‚Π²ΡƒΡŽΡ‰Π΅ΠΉ Π½Π° ΠΊΠ°ΠΆΠ΄Ρ‹ΠΉ ΠΈΠ· стСрТнСй, ΠΈΠ·ΠΎΠ±Ρ€Π°ΠΆΠ΅Π½Π½Ρ‹ΠΉ Π½Π° рис. 9.14, Π² Π½ΠΈΡ… Π²ΠΎΠ·Π½ΠΈΠΊΠ½ΡƒΡ‚ Ρ€Π°Π²Π½Ρ‹Π΅ напряТСния.
ΠŸΠ΅Ρ€Π΅Ρ…ΠΎΠ΄Ρ Π΄Π°Π»Π΅Π΅ ΠΊ ΠΈΠ·ΡƒΡ‡Π΅Π½ΠΈΡŽ Π΄Ρ€ΡƒΠ³ΠΈΡ… Π±ΠΎΠ»Π΅Π΅ слоТных Π²ΠΈΠ΄ΠΎΠ² Π΄Π΅Ρ„ΠΎΡ€ΠΌΠ°Ρ†ΠΈΠΉ (ΠΊΡ€ΡƒΡ‡Π΅Π½ΠΈΠ΅, ΠΈΠ·Π³ΠΈΠ±, Π²Π½Π΅Ρ†Π΅Π½Ρ‚Ρ€Π΅Π½Π½ΠΎΠ΅ сТатиС ΠΈ Π΄Ρ€.) ΠΌΡ‹ ΡƒΠ²ΠΈΠ΄ΠΈΠΌ, Ρ‡Ρ‚ΠΎ Π² этих случаях сопротивлСниС элСмСнта конструкции внСшним силам зависит Π½Π΅ Ρ‚ΠΎΠ»ΡŒΠΊΠΎ ΠΎΡ‚ ΠΏΠ»ΠΎΡ‰Π°Π΄ΠΈ Π΅Π³ΠΎ ΠΏΠΎΠΏΠ΅Ρ€Π΅Ρ‡Π½ΠΎΠ³ΠΎ сСчСния, Π½ΠΎ ΠΈ ΠΎΡ‚ распрСдСлСния этой ΠΏΠ»ΠΎΡ‰Π°Π΄ΠΈ Π² плоскости сСчСния, Ρ‚. Π΅. ΠΎΡ‚ Ρ„ΠΎΡ€ΠΌΡ‹ сСчСния.
Из ΠΎΠ±Ρ‹Π΄Π΅Π½Π½ΠΎΠ³ΠΎ ΠΎΠΏΡ‹Ρ‚Π° ясно, Ρ‡Ρ‚ΠΎ ΡΠΎΠ³Π½ΡƒΡ‚ΡŒ ΡΡ‚Π΅Ρ€ΠΆΠ΅Π½ΡŒ 4 Π² Π²Π΅Ρ€Ρ‚ΠΈΠΊΠ°Π»ΡŒΠ½ΠΎΠΌ Π½Π°ΠΏΡ€Π°Π²Π»Π΅Π½ΠΈΠΈ Ρ‚Ρ€ΡƒΠ΄Π½Π΅Π΅, Ρ‡Π΅ΠΌ ΡΡ‚Π΅Ρ€ΠΆΠ΅Π½ΡŒ 5, Π° ΡΡ‚Π΅Ρ€ΠΆΠ΅Π½ΡŒ 6 ΠΈΠΌΠ΅Π΅Ρ‚ Π΅Ρ‰Π΅ Π±ΠΎΠ»ΡŒΡˆΡƒΡŽ ΠΆΠ΅ΡΡ‚ΠΊΠΎΡΡ‚ΡŒ, хотя ΠΏΠ»ΠΎΡ‰Π°Π΄ΠΈ сСчСний всСх этих стСрТнСй ΠΎΠ΄ΠΈΠ½Π°ΠΊΠΎΠ²Ρ‹Π΅ (рис. 9.14).

ΠŸΠ°Ρ€Π°ΠΌΠ΅Ρ‚Ρ€Π°ΠΌΠΈ, Ρ…Π°Ρ€Π°ΠΊΡ‚Π΅Ρ€ΠΈΠ·ΡƒΡŽΡ‰ΠΈΠΌΠΈ гСомСтричСскиС свойства Ρ€Π°Π·Π»ΠΈΡ‡Π½Ρ‹Ρ… плоских Ρ„ΠΈΠ³ΡƒΡ€, ΠΊΡ€ΠΎΠΌΠ΅ ΠΏΠ»ΠΎΡ‰Π°Π΄ΠΈ, ΡΠ²Π»ΡΡŽΡ‚ΡΡ: статичСскиС ΠΌΠΎΠΌΠ΅Π½Ρ‚Ρ‹, ΠΌΠΎΠΌΠ΅Π½Ρ‚Ρ‹ ΠΈΠ½Π΅Ρ€Ρ†ΠΈΠΈ, ΠΌΠΎΠΌΠ΅Π½Ρ‚Ρ‹ сопротивлСния ΠΈ радиусы ΠΈΠ½Π΅Ρ€Ρ†ΠΈΠΈ.
БтатичСский ΠΌΠΎΠΌΠ΅Π½Ρ‚ ΠΏΠ»ΠΎΡ‰Π°Π΄ΠΈ . ΠŸΡ€Π΅Π΄ΡΡ‚Π°Π²ΠΈΠΌ брус с ΠΏΡ€ΠΎΠΈΠ·Π²ΠΎΠ»ΡŒΠ½ΠΎΠΉ Ρ„ΠΎΡ€ΠΌΠΎΠΉ ΠΏΠΎΠΏΠ΅Ρ€Π΅Ρ‡Π½ΠΎΠ³ΠΎ сСчСния ΠΏΠ»ΠΎΡ‰Π°Π΄ΡŒΡŽ F , Π² плоскости ΠΊΠΎΡ‚ΠΎΡ€ΠΎΠ³ΠΎ ΠΏΡ€ΠΎΠ²Π΅Π΄Π΅Π½Π° ось Ρ… (рис. 9.15). Π’Ρ‹Π΄Π΅Π»ΠΈΠΌ элСмСнт ΠΏΠ»ΠΎΡ‰Π°Π΄ΠΈ dF , располоТСнный Π½Π° расстоянии Ρƒ ΠΎΡ‚ оси Ρ… .. БтатичСским ΠΌΠΎΠΌΠ΅Π½Ρ‚ΠΎΠΌ элСмСнтарной ΠΏΠ»ΠΎΡ‰Π°Π΄ΠΊΠΈ , ΠΎΡ‚Π½ΠΎΡΠΈΡ‚Π΅Π»ΡŒΠ½ΠΎ оси Ρ… Π½Π°Π·Ρ‹Π²Π°ΡŽΡ‚ ΠΏΡ€ΠΎΠΈΠ·Π²Π΅Π΄Π΅Π½ΠΈΠ΅ этой ΠΏΠ»ΠΎΡ‰Π°Π΄ΠΊΠΈ Π½Π° Π΅Π΅ расстояниС Π΄ΠΎ оси:


БтатичСский ΠΌΠΎΠΌΠ΅Π½Ρ‚ всСй ΠΏΠ»ΠΎΡ‰Π°Π΄ΠΈ F ΠΎΡ‚Π½ΠΎΡΠΈΡ‚Π΅Π»ΡŒΠ½ΠΎ оси Ρ… Ρ€Π°Π²Π΅Π½ суммС статичСских ΠΌΠΎΠΌΠ΅Π½Ρ‚ΠΎΠ² всСх элСмСнтарных ΠΏΠ»ΠΎΡ‰Π°Π΄ΠΎΠΊ, ΠΊΠΎΡ‚ΠΎΡ€Ρ‹Π΅ ΠΌΠΎΠ³ΡƒΡ‚ Π±Ρ‹Ρ‚ΡŒ Π²Ρ‹Π΄Π΅Π»Π΅Π½Ρ‹ Π½Π° рассматриваСмой ΠΏΠ»ΠΎΡ‰Π°Π΄ΠΈ:


Из тСорСтичСской ΠΌΠ΅Ρ…Π°Π½ΠΈΠΊΠΈ извСстно, Ρ‡Ρ‚ΠΎ ΠΊΠΎΠΎΡ€Π΄ΠΈΠ½Π°Ρ‚Ρ‹ Ρ†Π΅Π½Ρ‚Ρ€Π° тяТСсти ΠΏΠ»ΠΎΡ‰Π°Π΄ΠΈ Ρ„ΠΈΠ³ΡƒΡ€Ρ‹ ΠΎΠΏΡ€Π΅Π΄Π΅Π»ΡΡŽΡ‚ ΠΏΠΎ Ρ„ΠΎΡ€ΠΌΡƒΠ»Π°ΠΌ:

ΠŸΠΎΡΡ‚ΠΎΠΌΡƒ

Π‘Π»Π΅Π΄ΠΎΠ²Π°Ρ‚Π΅Π»ΡŒΠ½ΠΎ, статичСский ΠΌΠΎΠΌΠ΅Π½Ρ‚ Ρ„ΠΈΠ³ΡƒΡ€Ρ‹ ΠΏΠ»ΠΎΡ‰Π°Π΄ΡŒΡŽ F ΠΎΡ‚Π½ΠΎΡΠΈΡ‚Π΅Π»ΡŒΠ½ΠΎ ΠΊΠ°ΠΊΠΎΠΉ-Π½ΠΈΠ±ΡƒΠ΄ΡŒ оси Ρ€Π°Π²Π΅Π½ ΠΏΡ€ΠΎΠΈΠ·Π²Π΅Π΄Π΅Π½ΠΈΡŽ ΠΏΠ»ΠΎΡ‰Π°Π΄ΠΈ Π½Π° расстояниС Ρ†Π΅Π½Ρ‚Ρ€Π° тяТСсти Ρ„ΠΈΠ³ΡƒΡ€Ρ‹ Π΄ΠΎ этой оси. Π Π°Π·ΠΌΠ΅Ρ€Π½ΠΎΡΡ‚ΡŒ статичСского ΠΌΠΎΠΌΠ΅Π½Ρ‚Π° – Π΅Π΄ΠΈΠ½ΠΈΡ†Π° Π΄Π»ΠΈΠ½Ρ‹ Π² ΠΊΡƒΠ±Π΅ ( , ).
Оси, проходящиС Ρ‡Π΅Ρ€Π΅Π· Ρ†Π΅Π½Ρ‚Ρ€ тяТСсти сСчСния, Π½Π°Π·Ρ‹Π²Π°ΡŽΡ‚ Ρ†Π΅Π½Ρ‚Ρ€Π°Π»ΡŒΠ½Ρ‹ΠΌΠΈ.Если Ρ„ΠΈΠ³ΡƒΡ€Π° ΠΈΠΌΠ΅Π΅Ρ‚ ось симмСтрии, Ρ‚ΠΎ послСдняя всСгда ΠΏΡ€ΠΎΡ…ΠΎΠ΄ΠΈΡ‚ Ρ‡Π΅Ρ€Π΅Π· Ρ†Π΅Π½Ρ‚Ρ€ тяТСсти Ρ„ΠΈΠ³ΡƒΡ€Ρ‹, Ρ‚. Π΅. оси симмСтрии ΠΎΠ΄Π½ΠΎΠ²Ρ€Π΅ΠΌΠ΅Π½Π½ΠΎ ΡΠ²Π»ΡΡŽΡ‚ΡΡ ΠΈ Ρ†Π΅Π½Ρ‚Ρ€Π°Π»ΡŒΠ½Ρ‹ΠΌΠΈ осями.
Π‘ΡƒΠ΄Π΅ΠΌ Ρ‚Π°ΠΊΠΆΠ΅ ΠΈΠΌΠ΅Ρ‚ΡŒ Π² Π²ΠΈΠ΄Ρƒ, Ρ‡Ρ‚ΠΎ статичСский ΠΌΠΎΠΌΠ΅Π½Ρ‚ слоТной Ρ„ΠΈΠ³ΡƒΡ€Ρ‹ ΠΎΡ‚Π½ΠΎΡΠΈΡ‚Π΅Π»ΡŒΠ½ΠΎ Π½Π΅ΠΊΠΎΡ‚ΠΎΡ€ΠΎΠΉ оси Ρ€Π°Π²Π΅Π½ суммС статичСских ΠΌΠΎΠΌΠ΅Π½Ρ‚ΠΎΠ² ΠΎΡ‚Π½ΠΎΡΠΈΡ‚Π΅Π»ΡŒΠ½ΠΎ Ρ‚ΠΎΠΉ ΠΆΠ΅ оси простых Ρ„ΠΈΠ³ΡƒΡ€, Π½Π° ΠΊΠΎΡ‚ΠΎΡ€Ρ‹Π΅ ΠΌΠΎΠΆΠ΅Ρ‚ Π±Ρ‹Ρ‚ΡŒ Ρ€Π°Π·Π±ΠΈΡ‚Π° исходная слоТная Ρ„ΠΈΠ³ΡƒΡ€Π°:
Рис. 9.16. Π‘Ρ…Π΅ΠΌΠ° ΠΊ ΠΎΠΏΡ€Π΅Π΄Π΅Π»Π΅Π½ΠΈΡŽ ΠΊΠΎΠΎΡ€Π΄ΠΈΠ½Π°Ρ‚ Ρ†Π΅Π½Ρ‚Ρ€Π° тяТСсти слоТной Ρ„ΠΈΠ³ΡƒΡ€Ρ‹.

Для Ρ€Π΅ΡˆΠ΅Π½ΠΈΡ этой Π·Π°Π΄Π°Ρ‡ΠΈ Π²Ρ‹Π±Π΅Ρ€Π΅ΠΌ Π΄Π²Π΅ оси ΠΊΠΎΠΎΡ€Π΄ΠΈΠ½Π°Ρ‚ Ρ… ΠΈ Ρƒ , ΡΠΎΠ²ΠΏΠ°Π΄Π°ΡŽΡ‰ΠΈΠ΅ со сторонами Ρ„ΠΈΠ³ΡƒΡ€Ρ‹. РазобьСм Ρ„ΠΈΠ³ΡƒΡ€Ρƒ, всС Ρ€Π°Π·ΠΌΠ΅Ρ€Ρ‹ ΠΊΠΎΡ‚ΠΎΡ€ΠΎΠΉ Π΄ΠΎΠ»ΠΆΠ½Ρ‹ Π±Ρ‹Ρ‚ΡŒ извСстны, Π½Π° элСмСнтарныС части – ΠΏΡ€ΡΠΌΠΎΡƒΠ³ΠΎΠ»ΡŒΠ½ΠΈΠΊΠΈ – ΠΊΠΎΠΎΡ€Π΄ΠΈΠ½Π°Ρ‚Ρ‹ Ρ†Π΅Π½Ρ‚Ρ€ΠΎΠ² тяТСсти ΠΊΠΎΡ‚ΠΎΡ€Ρ‹Ρ… ΠΎΡ‡Π΅Π²ΠΈΠ΄Π½Ρ‹, Ρ‚Π°ΠΊ ΠΊΠ°ΠΊ эти части симмСтричны. Боставим Ρ‚Π΅ΠΏΠ΅Ρ€ΡŒ выраТСния для вычислСния статичСского ΠΌΠΎΠΌΠ΅Π½Ρ‚Π° всСй ΠΏΠ»ΠΎΡ‰Π°Π΄ΠΈ, Π½Π°ΠΏΡ€ΠΈΠΌΠ΅Ρ€ ΠΎΡ‚Π½ΠΎΡΠΈΡ‚Π΅Π»ΡŒΠ½ΠΎ оси Ρƒ . Π­Ρ‚ΠΎ ΠΌΠΎΠΆΠ½ΠΎ ΡΠ΄Π΅Π»Π°Ρ‚ΡŒ двумя способами:
Π°) Π²Π·ΡΡ‚ΡŒ сумму статичСских ΠΌΠΎΠΌΠ΅Π½Ρ‚ΠΎΠ² ΠΎΡ‚Π΄Π΅Π»ΡŒΠ½Ρ‹Ρ… ΠΏΠ»ΠΎΡ‰Π°Π΄Π΅ΠΉ
Π’ этих выраТСниях F – ΠΏΠ»ΠΎΡ‰Π°Π΄ΡŒ всСй Ρ„ΠΈΠ³ΡƒΡ€Ρ‹; – ΠΊΠΎΠΎΡ€Π΄ΠΈΠ½Π°Ρ‚Π° Π΅Π΅ Ρ†Π΅Π½Ρ‚Ρ€Π° тяТСсти; – ΠΏΠ»ΠΎΡ‰Π°Π΄ΠΈ ΠΎΡ‚Π΄Π΅Π»ΡŒΠ½Ρ‹Ρ… частСй Ρ„ΠΈΠ³ΡƒΡ€Ρ‹, Π° – ΠΊΠΎΠΎΡ€Π΄ΠΈΠ½Π°Ρ‚Ρ‹ ΠΈΡ… Ρ†Π΅Π½Ρ‚Ρ€ΠΎΠ² тяТСсти.
ΠŸΡ€ΠΈΡ€Π°Π²Π½ΠΈΠ²Π°Ρ Π΄Ρ€ΡƒΠ³ ΠΊ Π΄Ρ€ΡƒΠ³Ρƒ написанныС Π²Ρ‹ΡˆΠ΅ Ρ„ΠΎΡ€ΠΌΡƒΠ»Ρ‹, ΠΏΠΎΠ»ΡƒΡ‡ΠΈΠΌ ΡƒΡ€Π°Π²Π½Π΅Π½ΠΈΠ΅ с ΠΎΠ΄Π½ΠΎΠΉ нСизвСстной :
Аналогично этому расстояниС Ρ†Π΅Π½Ρ‚Ρ€Π° тяТСсти Ρ„ΠΈΠ³ΡƒΡ€Ρ‹ ΠΎΡ‚ оси Ρ… ΠΌΠΎΠΆΠ΅Ρ‚ Π±Ρ‹Ρ‚ΡŒ Π²Ρ‹Ρ€Π°ΠΆΠ΅Π½ΠΎ Ρ‚Π°ΠΊ:

Боставляя ΠΈΠ½Ρ‚Π΅Π³Ρ€Π°Π», Π² ΠΊΠΎΡ‚ΠΎΡ€ΠΎΠΌ ΠΏΠΎΠ΄Ρ‹Π½Ρ‚Π΅Π³Ρ€Π°Π»ΡŒΠ½ΠΎΠ΅ Π²Ρ‹Ρ€Π°ΠΆΠ΅Π½ΠΈΠ΅ прСдставляСт собой ΠΏΡ€ΠΎΠΈΠ·Π²Π΅Π΄Π΅Π½ΠΈΠ΅ элСмСнта ΠΏΠ»ΠΎΡ‰Π°Π΄ΠΈ Π½Π° ΠΊΠ²Π°Π΄Ρ€Π°Ρ‚ расстояния Π΄ΠΎ Π½Π°Ρ‡Π°Π»Π° ΠΊΠΎΠΎΡ€Π΄ΠΈΠ½Π°Ρ‚ (рис. 9.17), ΠΏΠΎΠ»ΡƒΡ‡ΠΈΠΌ полярный ΠΌΠΎΠΌΠ΅Π½Ρ‚ ΠΈΠ½Π΅Ρ€Ρ†ΠΈΠΈ :
ΠžΡ‚ΠΌΠ΅Ρ‚ΠΈΠΌ Π΅Ρ‰Π΅ ΠΎΠ΄Π½Ρƒ характСристику, Π² ΠΊΠΎΡ‚ΠΎΡ€ΠΎΠΉ ΠΏΠ»ΠΎΡ‰Π°Π΄ΠΊΠ° dF умноТаСтся Π½Π° ΠΏΡ€ΠΎΠΈΠ·Π²Π΅Π΄Π΅Π½ΠΈΠ΅ ΠΊΠΎΠΎΡ€Π΄ΠΈΠ½Π°Ρ‚


Π­Ρ‚Ρƒ Π²Π΅Π»ΠΈΡ‡ΠΈΠ½Ρƒ Π½Π°Π·Ρ‹Π²Π°ΡŽΡ‚ Ρ†Π΅Π½Ρ‚Ρ€ΠΎΠ±Π΅ΠΆΠ½Ρ‹ΠΌ ΠΌΠΎΠΌΠ΅Π½Ρ‚ΠΎΠΌ ΠΈΠ½Π΅Ρ€Ρ†ΠΈΠΈ . ΠŸΡ€ΠΈΠ²Π΅Π΄Π΅Π½Π½Ρ‹Π΅ ΠΌΠΎΠΌΠ΅Π½Ρ‚Ρ‹ ΠΈΠ½Π΅Ρ€Ρ†ΠΈΠΈ ΠΈΠ·ΠΌΠ΅Ρ€ΡΡŽΡ‚ΡΡ Π² Π΅Π΄ΠΈΠ½ΠΈΡ†Π°Ρ… Π΄Π»ΠΈΠ½Ρ‹” взятой Π² Ρ‡Π΅Ρ‚Π²Π΅Ρ€Ρ‚ΠΎΠΉ стСпСни (, ).
ΠžΡΠ΅Π²Ρ‹Π΅ ΠΈ полярныС ΠΌΠΎΠΌΠ΅Π½Ρ‚Ρ‹ ΠΈΠ½Π΅Ρ€Ρ†ΠΈΠΈ Ρ„ΠΈΠ³ΡƒΡ€Ρ‹ – Π²Π΅Π»ΠΈΡ‡ΠΈΠ½Ρ‹ ΠΏΠΎΠ»ΠΎΠΆΠΈΡ‚Π΅Π»ΡŒΠ½Ρ‹Π΅ ΠΈ Π½Π΅ ΠΌΠΎΠ³ΡƒΡ‚ Π±Ρ‹Ρ‚ΡŒ Ρ€Π°Π²Π½Ρ‹ΠΌΠΈ Π½ΡƒΠ»ΡŽ. Π¦Π΅Π½Ρ‚Ρ€ΠΎΠ±Π΅ΠΆΠ½Ρ‹ΠΉ ΠΌΠΎΠΌΠ΅Π½Ρ‚ ΠΈΠ½Π΅Ρ€Ρ†ΠΈΠΈ Π² зависимости ΠΎΡ‚ полоТСния осСй ΠΌΠΎΠΆΠ΅Ρ‚ Π±Ρ‹Ρ‚ΡŒ ΠΏΠΎΠ»ΠΎΠΆΠΈΡ‚Π΅Π»ΡŒΠ½Ρ‹ΠΌ ΠΈΠ»ΠΈ ΠΎΡ‚Ρ€ΠΈΡ†Π°Ρ‚Π΅Π»ΡŒΠ½Ρ‹ΠΌ, Π° Ρ‚Π°ΠΊΠΆΠ΅ Ρ€Π°Π²Π½Ρ‹ΠΌ Π½ΡƒΠ»ΡŽ. Π”Π²Π΅ Π²Π·Π°ΠΈΠΌΠ½ΠΎ пСрпСндикулярныС оси, ΠΎΡ‚Π½ΠΎΡΠΈΡ‚Π΅Π»ΡŒΠ½ΠΎ ΠΊΠΎΡ‚ΠΎΡ€Ρ‹Ρ… Ρ†Π΅Π½Ρ‚Ρ€ΠΎΠ±Π΅ΠΆΠ½Ρ‹ΠΉ ΠΌΠΎΠΌΠ΅Π½Ρ‚ ΠΈΠ½Π΅Ρ€Ρ†ΠΈΠΈ Ρ€Π°Π²Π΅Π½ Π½ΡƒΠ»ΡŽ, Π½Π°Π·Ρ‹Π²Π°ΡŽΡ‚ Π³Π»Π°Π²Π½Ρ‹ΠΌΠΈ осями ΠΈΠ½Π΅Ρ€Ρ†ΠΈΠΈ ΠΈ ΠΎΠ±ΠΎΠ·Π½Π°Ρ‡Π°ΡŽΡ‚ΡΡ . Для симмСтричной Ρ„ΠΈΠ³ΡƒΡ€Ρ‹ ось симмСтрии являСтся ΠΈ Π³Π»Π°Π²Π½ΠΎΠΉ осью.
ΠžΡΠ΅Π²Ρ‹Π΅ ΠΌΠΎΠΌΠ΅Π½Ρ‚Ρ‹ ΠΈΠ½Π΅Ρ€Ρ†ΠΈΠΈ, ΠΎΠΏΡ€Π΅Π΄Π΅Π»Π΅Π½Π½Ρ‹Π΅ ΠΎΡ‚Π½ΠΎΡΠΈΡ‚Π΅Π»ΡŒΠ½ΠΎ Π³Π»Π°Π²Π½Ρ‹Ρ… осСй, ΠΈΠΌΠ΅ΡŽΡ‚ максимальноС ΠΈ минимальноС значСния.
Π’Π°ΠΊ ΠΆΠ΅ ΠΊΠ°ΠΊ ΠΈ для статичСского ΠΌΠΎΠΌΠ΅Π½Ρ‚Π°, ΠΌΠΎΠΌΠ΅Π½Ρ‚ ΠΈΠ½Π΅Ρ€Ρ†ΠΈΠΈ слоТной Ρ„ΠΈΠ³ΡƒΡ€Ρ‹ Ρ€Π°Π²Π΅Π½ суммС ΠΌΠΎΠΌΠ΅Π½Ρ‚ΠΎΠ² ΠΈΠ½Π΅Ρ€Ρ†ΠΈΠΈ ΠΎΠ±Ρ€Π°Π·ΡƒΡŽΡ‰ΠΈΡ… Π΅Π΅ Ρ„ΠΈΠ³ΡƒΡ€. ΠŸΠΎΠ΄Ρ‡Π΅Ρ€ΠΊΠ½Π΅ΠΌ, Ρ‡Ρ‚ΠΎ сказанноС справСдливо Π² Ρ‚ΠΎΠΌ случаС, ΠΊΠΎΠ³Π΄Π° всС ΠΌΠΎΠΌΠ΅Π½Ρ‚Ρ‹ ΠΈΠ½Π΅Ρ€Ρ†ΠΈΠΈ Π²Ρ‹Ρ‡ΠΈΡΠ»ΡΡŽΡ‚ΡΡ ΠΎΡ‚Π½ΠΎΡΠΈΡ‚Π΅Π»ΡŒΠ½ΠΎ ΠΎΠ΄Π½ΠΎΠΉ ΠΈ Ρ‚ΠΎΠΉ ΠΆΠ΅ оси.
Для ΠΌΠΎΠΌΠ΅Π½Ρ‚ΠΎΠ² ΠΈΠ½Π΅Ρ€Ρ†ΠΈΠΈ сущСствуСт Π΅Ρ‰Π΅ ΠΎΠ΄Π½ΠΎ ΠΏΡ€Π°Π²ΠΈΠ»ΠΎ, часто ΠΈΡΠΏΠΎΠ»ΡŒΠ·ΡƒΠ΅ΠΌΠΎΠ΅ Π² расчСтах. ΠŸΡ€ΠΈΠΌΠ΅Π½ΠΈΡ‚Π΅Π»ΡŒΠ½ΠΎ ΠΊ осСвым ΠΌΠΎΠΌΠ΅Π½Ρ‚Π°ΠΌ ΠΎΠ½ΠΎ “формулируСтся ΡΠ»Π΅Π΄ΡƒΡŽΡ‰ΠΈΠΌ ΠΎΠ±Ρ€Π°Π·ΠΎΠΌ: ΠΌΠΎΠΌΠ΅Π½Ρ‚ ΠΈΠ½Π΅Ρ€Ρ†ΠΈΠΈ Ρ„ΠΈΠ³ΡƒΡ€Ρ‹ ΠΎΡ‚Π½ΠΎΡΠΈΡ‚Π΅Π»ΡŒΠ½ΠΎ оси, ΠΏΠ°Ρ€Π°Π»Π»Π΅Π»ΡŒΠ½ΠΎΠΉ Ρ†Π΅Π½Ρ‚Ρ€Π°Π»ΡŒΠ½ΠΎΠΉ, Ρ€Π°Π²Π΅Π½ ΠΌΠΎΠΌΠ΅Π½Ρ‚Ρƒ ΠΈΠ½Π΅Ρ€Ρ†ΠΈΠΈ ΠΎΡ‚Π½ΠΎΡΠΈΡ‚Π΅Π»ΡŒΠ½ΠΎ Ρ†Π΅Π½Ρ‚Ρ€Π°Π»ΡŒΠ½ΠΎΠΉ оси плюс ΠΏΡ€ΠΎΠΈΠ·Π²Π΅Π΄Π΅Π½ΠΈΠ΅ ΠΏΠ»ΠΎΡ‰Π°Π΄ΠΈ Ρ„ΠΈΠ³ΡƒΡ€Ρ‹, Π½Π° ΠΊΠ²Π°Π΄Ρ€Π°Ρ‚ расстояния ΠΌΠ΅ΠΆΠ΄Ρƒ осями (рис. 9.18):
Для Ρ†Π΅Π½Ρ‚Ρ€ΠΎΠ±Π΅ΠΆΠ½Ρ‹Ρ… ΠΌΠΎΠΌΠ΅Π½Ρ‚ΠΎΠ² ΠΈΠ½Π΅Ρ€Ρ†ΠΈΠΈ ΡΠΎΠΎΡ‚Π²Π΅Ρ‚ΡΡ‚Π²ΡƒΡŽΡ‰Π΅Π΅ ΠΏΡ€Π°Π²ΠΈΠ»ΠΎ Π² аналитичСском Π²ΠΈΠ΄Π΅ выглядит Ρ‚Π°ΠΊ:


Для получСния значСния ΠΌΠΎΠΌΠ΅Π½Ρ‚Π° ΠΈΠ½Π΅Ρ€Ρ†ΠΈΠΈ ΠΊΠΎΠ½ΠΊΡ€Π΅Ρ‚Π½ΠΎΠΉ Ρ„ΠΈΠ³ΡƒΡ€Ρ‹ Π² ΠΏΡ€ΠΈΠ½Ρ†ΠΈΠΏΠ΅ Π½Π°Π΄ΠΎ Ρ€Π΅ΡˆΠΈΡ‚ΡŒ ΡΠΎΠΎΡ‚Π²Π΅Ρ‚ΡΡ‚Π²ΡƒΡŽΡ‰ΠΈΠΉ ΠΈΠ½Ρ‚Π΅Π³Ρ€Π°Π» ΠΏΠΎ ΠΏΠ»ΠΎΡ‰Π°Π΄ΠΈ этой Ρ„ΠΈΠ³ΡƒΡ€Ρ‹. Однако с Ρ†Π΅Π»ΡŒΡŽ облСгчСния ΠΈΠ½ΠΆΠ΅Π½Π΅Ρ€Π½Ρ‹Ρ… расчСтов Ρ‚Π°ΠΊΠΈΠ΅ ΠΈΠ½Ρ‚Π΅Π³Ρ€Π°Π»Ρ‹ для Π½Π°ΠΈΠ±ΠΎΠ»Π΅Π΅ распространСнных Ρ„ΠΎΡ€ΠΌ ΠΏΠΎΠΏΠ΅Ρ€Π΅Ρ‡Π½Ρ‹Ρ… сСчСний ΡΡ‚Ρ€ΠΎΠΈΡ‚Π΅Π»ΡŒΠ½Ρ‹Ρ… элСмСнтов ΡƒΠΆΠ΅ Ρ€Π΅ΡˆΠ΅Π½Ρ‹ ΠΈ Ρ€Π΅Π·ΡƒΠ»ΡŒΡ‚Π°Ρ‚Ρ‹ Ρ€Π΅ΡˆΠ΅Π½ΠΈΠΉ Π² Π²ΠΈΠ΄Π΅ Ρ„ΠΎΡ€ΠΌΡƒΠ» прСдставлСны Π² Ρ‚Π°Π±Π»ΠΈΡ†Π°Ρ…, ΠΎΠ΄Π½Π° ΠΈΠ· ΠΊΠΎΡ‚ΠΎΡ€Ρ‹Ρ… ΠΏΠΎΠΌΠ΅Ρ‰Π΅Π½Π° Π² ΠΏΡ€ΠΈΠ»ΠΎΠΆΠ΅Π½ΠΈΠΈ 3.
ΠšΡ€ΠΎΠΌΠ΅ Ρ‚ΠΎΠ³ΠΎ, Π² Π“ΠžΠ‘Π’Π°Ρ… Π½Π° всС стандартныС ΠΏΡ€ΠΎΡ„ΠΈΠ»ΠΈ ΠΏΡ€ΠΎΠΊΠ°Ρ‚Π°, выпускаСмыС Π² нашСй странС (ΡƒΠ³ΠΎΠ»ΠΊΠΈ, Π΄Π²ΡƒΡ‚Π°Π²Ρ€Ρ‹ ΠΈ Π΄Ρ€.), Π΄Π°ΡŽΡ‚ΡΡ значСния осСвых ΠΌΠΎΠΌΠ΅Π½Ρ‚ΠΎΠ² ΠΈΠ½Π΅Ρ€Ρ†ΠΈΠΈ ΠΈ Π΄Ρ€ΡƒΠ³ΠΈΡ… гСомСтричСских характСристик для ΠΊΠ°ΠΆΠ΄ΠΎΠ³ΠΎ Ρ‚ΠΈΠΏΠΎΡ€Π°Π·ΠΌΠ΅Ρ€Π° ΠΏΡ€ΠΎΠΊΠ°Ρ‚Π° (см. ΠΏΡ€ΠΈΠ»ΠΎΠΆΠ΅Π½ΠΈΠ΅ 4).
НаконСц, для слоТных ΠΏΠΎ Ρ„ΠΎΡ€ΠΌΠ΅ сСчСний ΠΌΠΎΠΌΠ΅Π½Ρ‚Ρ‹ ΠΈΠ½Π΅Ρ€Ρ†ΠΈΠΈ ΠΎΠΏΡ€Π΅Π΄Π΅Π»ΡΡŽΡ‚, ΠΈΡΠΏΠΎΠ»ΡŒΠ·ΡƒΡ ΠΈΠ·Π»ΠΎΠΆΠ΅Π½Π½Ρ‹Π΅ Π²Ρ‹ΡˆΠ΅ Π΄Π²Π° ΠΏΡ€Π°Π²ΠΈΠ»Π°: ΠΎ слоТСнии ΠΌΠΎΠΌΠ΅Π½Ρ‚ΠΎΠ² ΠΈΠ½Π΅Ρ€Ρ†ΠΈΠΈ ΠΈ ΠΎ пСрСсчСтС ΠΌΠΎΠΌΠ΅Π½Ρ‚ΠΎΠ² ΠΈΠ½Π΅Ρ€Ρ†ΠΈΠΈ ΠΎΡ‚Π½ΠΎΡΠΈΡ‚Π΅Π»ΡŒΠ½ΠΎ ΠΎΠ΄Π½ΠΈΡ… осСй Π½Π° Π΄Ρ€ΡƒΠ³ΠΈΠ΅ оси.
ΠœΠΎΠΌΠ΅Π½Ρ‚ сопротивлСния . ΠžΡΠ΅Π²Ρ‹ΠΌ ΠΌΠΎΠΌΠ΅Π½Ρ‚ΠΎΠΌ сопротивлСния плоской Ρ„ΠΈΠ³ΡƒΡ€Ρ‹ ΠΎΡ‚Π½ΠΎΡΠΈΡ‚Π΅Π»ΡŒΠ½ΠΎ ΠΊΠ°ΠΊΠΎΠΉ-Π»ΠΈΠ±ΠΎ оси, Π»Π΅ΠΆΠ°Ρ‰Π΅ΠΉ Π² плоскости Ρ„ΠΈΠ³ΡƒΡ€Ρ‹, называСтся частноС ΠΎΡ‚ дСлСния ΠΌΠΎΠΌΠ΅Π½Ρ‚Π° ΠΈΠ½Π΅Ρ€Ρ†ΠΈΠΈ ΠΎΡ‚Π½ΠΎΡΠΈΡ‚Π΅Π»ΡŒΠ½ΠΎ Ρ‚ΠΎΠΉ ΠΆΠ΅ оси Π½Π° расстояниС Π΄ΠΎ Π½Π°ΠΈΠ±ΠΎΠ»Π΅Π΅ ΡƒΠ΄Π°Π»Π΅Π½Π½ΠΎΠΉ Ρ‚ΠΎΡ‡ΠΊΠΈ Ρ„ΠΈΠ³ΡƒΡ€Ρ‹ (см. рис. 9.17):
ΠœΠΎΠΌΠ΅Π½Ρ‚Ρ‹ сопротивлСния ΠΈΠΌΠ΅ΡŽΡ‚ Ρ€Π°Π·ΠΌΠ΅Ρ€Π½ΠΎΡΡ‚ΡŒ Π΄Π»ΠΈΠ½Ρ‹ Π² ΠΊΡƒΠ±Π΅ (, ).
Π€ΠΎΡ€ΠΌΡƒΠ»Ρ‹ для расчСтов осСвых ΠΌΠΎΠΌΠ΅Π½Ρ‚ΠΎΠ² сопротивлСний Π½Π°ΠΈΠ±ΠΎΠ»Π΅Π΅ часто Π²ΡΡ‚Ρ€Π΅Ρ‡Π°ΡŽΡ‰ΠΈΡ…ΡΡ Ρ„ΠΈΠ³ΡƒΡ€ ΠΏΡ€ΠΈΠ²Π΅Π΄Π΅Π½Ρ‹ Π² ΠΏΡ€ΠΈΠ»ΠΎΠΆΠ΅Π½ΠΈΠΈ 3, Π° ΠΊΠΎΠ½ΠΊΡ€Π΅Ρ‚Π½Ρ‹Π΅ значСния этой характСристики для ΠΏΡ€ΠΎΡ„ΠΈΠ»Π΅ΠΉ ΡΡ‚Π°Π»ΡŒΠ½ΠΎΠ³ΠΎ ΠΏΡ€ΠΎΠΊΠ°Ρ‚Π° Π΄Π°Π½Ρ‹ Π² Π“ΠžΠ‘Π’Π°Ρ… (ΠΏΡ€ΠΈΠ»ΠΎΠΆΠ΅Π½ΠΈΠ΅ 4). ΠžΡ‚ΠΌΠ΅Ρ‚ΠΈΠΌ, Ρ‡Ρ‚ΠΎ Π² ΠΎΡ‚Π»ΠΈΡ‡ΠΈΠ΅ ΠΎΡ‚ ΠΌΠΎΠΌΠ΅Π½Ρ‚ΠΎΠ² ΠΈΠ½Π΅Ρ€Ρ†ΠΈΠΈ ΠΌΠΎΠΌΠ΅Π½Ρ‚Ρ‹ сопротивлСния ΡΠΊΠ»Π°Π΄Ρ‹Π²Π°Ρ‚ΡŒ нСльзя.
Радиус ΠΈΠ½Π΅Ρ€Ρ†ΠΈΠΈ . Радиусом ΠΈΠ½Π΅Ρ€Ρ†ΠΈΠΈ называСтся Π²Π΅Π»ΠΈΡ‡ΠΈΠ½Π°, получаСмая ΠΏΠΎ Ρ„ΠΎΡ€ΠΌΡƒΠ»Π΅
Π° для ΠΊΡ€ΡƒΠ³Π° Π΄ΠΈΠ°ΠΌΠ΅Ρ‚Ρ€ΠΎΠΌ d радиус ΠΈΠ½Π΅Ρ€Ρ†ΠΈΠΈ ΠΎΡ‚Π½ΠΎΡΠΈΡ‚Π΅Π»ΡŒΠ½ΠΎ оси, проходящСй Ρ‡Π΅Ρ€Π΅Π· Ρ†Π΅Π½Ρ‚Ρ€ ΠΊΡ€ΡƒΠ³Π°, Ρ€Π°Π²Π΅Π½
Π‘Ρ„Π΅Ρ€Π° примСнСния рассмотрСнных Π²Ρ‹ΡˆΠ΅ гСомСтричСских характСристик сСчСний Π±ΡƒΠ΄Π΅Ρ‚ раскрыта ΠΏΡ€ΠΈ ΠΈΠ·ΡƒΡ‡Π΅Π½ΠΈΠΈ Π²ΠΈΠ΄ΠΎΠ² Π΄Π΅Ρ„ΠΎΡ€ΠΌΠ°Ρ†ΠΈΠΉ, ΠΊΠΎΡ‚ΠΎΡ€Ρ‹ΠΌ посвящСны ΡΠ»Π΅Π΄ΡƒΡŽΡ‰ΠΈΠ΅ ΠΏΠΎΠ΄Ρ€Π°Π·Π΄Π΅Π»Ρ‹ настоящСй Π³Π»Π°Π²Ρ‹.

I = βˆ‘r i 2 dF i =∫r 2 dF (1.1)

Π’ ΠΏΡ€ΠΈΠ½Ρ†ΠΈΠΏΠ΅ ΠΈ ΠΎΠΏΡ€Π΅Π΄Π΅Π»Π΅Π½ΠΈΠ΅ ΠΈ Ρ„ΠΎΡ€ΠΌΡƒΠ»Π°, Π΅Π³ΠΎ ΠΎΠΏΠΈΡΡ‹Π²Π°ΡŽΡ‰Π°Ρ, Π½Π΅ слоТныС ΠΈ Π·Π°ΠΏΠΎΠΌΠ½ΠΈΡ‚ΡŒ ΠΈΡ… Π½Π°ΠΌΠ½ΠΎΠ³ΠΎ Π»Π΅Π³Ρ‡Π΅, Ρ‡Π΅ΠΌ Π²Π½ΠΈΠΊΠ½ΡƒΡ‚ΡŒ Π² ΡΡƒΡ‚ΡŒ. Но всС-Ρ‚Π°ΠΊΠΈ ΠΏΠΎΠΏΡ€ΠΎΠ±ΡƒΠ΅ΠΌ Ρ€Π°Π·ΠΎΠ±Ρ€Π°Ρ‚ΡŒΡΡ, Ρ‡Ρ‚ΠΎ ΠΆΠ΅ Ρ‚Π°ΠΊΠΎΠ΅ ΠΌΠΎΠΌΠ΅Π½Ρ‚ ΠΈΠ½Π΅Ρ€Ρ†ΠΈΠΈ ΠΈ ΠΎΡ‚ΠΊΡƒΠ΄Π° ΠΎΠ½ взялся.

ΠŸΠΎΠ½ΡΡ‚ΠΈΠ΅ ΠΌΠΎΠΌΠ΅Π½Ρ‚ ΠΈΠ½Π΅Ρ€Ρ†ΠΈΠΈ ΠΏΡ€ΠΈΡˆΠ»ΠΎ Π² сопромат ΠΈ ΡΡ‚Ρ€ΠΎΠΈΡ‚Π΅Π»ΡŒΠ½ΡƒΡŽ ΠΌΠ΅Ρ…Π°Π½ΠΈΠΊΡƒ ΠΈΠ· Π΄Ρ€ΡƒΠ³ΠΎΠ³ΠΎ Ρ€Π°Π·Π΄Π΅Π»Π° Ρ„ΠΈΠ·ΠΈΠΊΠΈ, ΠΈΠ·ΡƒΡ‡Π°ΡŽΡ‰Π΅Π³ΠΎ ΠΊΠΈΠ½Π΅ΠΌΠ°Ρ‚ΠΈΠΊΡƒ двиТСния, Π² частности Π²Ρ€Π°Ρ‰Π°Ρ‚Π΅Π»ΡŒΠ½ΠΎΠ΅ Π΄Π²ΠΈΠΆΠ΅Π½ΠΈΠ΅. Но всС Ρ€Π°Π²Π½ΠΎ Π½Π°Ρ‡Π½Π΅ΠΌ ΠΈΠ·Π΄Π°Π»Π΅ΠΊΠ°.

Π― Ρ‚ΠΎΡ‡Π½ΠΎ Π½Π΅ знаю, ΡƒΠΏΠ°Π»ΠΎ Π»ΠΈ Π˜ΡΠ°Π°ΠΊΡƒ ΠΡŒΡŽΡ‚ΠΎΠ½Ρƒ Π½Π° Π³ΠΎΠ»ΠΎΠ²Ρƒ яблоко, ΡƒΠΏΠ°Π»ΠΎ ΠΎΠ½ΠΎ рядом, ΠΈΠ»ΠΈ Π²ΠΎΠΎΠ±Ρ‰Π΅ Π½Π΅ ΠΏΠ°Π΄Π°Π»ΠΎ, тСория вСроятности допускаСт всС эти Π²Π°Ρ€ΠΈΠ°Π½Ρ‚Ρ‹ (ΠΊ Ρ‚ΠΎΠΌΡƒ ΠΆΠ΅ Π² этом яблокС слишком ΠΌΠ½ΠΎΠ³ΠΎ ΠΎΡ‚ библСйской Π»Π΅Π³Π΅Π½Π΄Ρ‹ ΠΎ Π΄Ρ€Π΅Π²Π΅ познания), ΠΎΠ΄Π½Π°ΠΊΠΎ я ΡƒΠ²Π΅Ρ€Π΅Π½, Ρ‡Ρ‚ΠΎ ΠΡŒΡŽΡ‚ΠΎΠ½ Π±Ρ‹Π» Π½Π°Π±Π»ΡŽΠ΄Π°Ρ‚Π΅Π»ΡŒΠ½Ρ‹ΠΌ Ρ‡Π΅Π»ΠΎΠ²Π΅ΠΊΠΎΠΌ, способным Π΄Π΅Π»Π°Ρ‚ΡŒ Π²Ρ‹Π²ΠΎΠ΄Ρ‹ ΠΈΠ· своих наблюдСний. Π’Π°ΠΊ Π½Π°Π±Π»ΡŽΠ΄Π°Ρ‚Π΅Π»ΡŒΠ½ΠΎΡΡ‚ΡŒ ΠΈ Π²ΠΎΠΎΠ±Ρ€Π°ΠΆΠ΅Π½ΠΈΠ΅ ΠΏΠΎΠ·Π²ΠΎΠ»ΠΈΠ»ΠΈ ΠΡŒΡŽΡ‚ΠΎΠ½Ρƒ ΡΡ„ΠΎΡ€ΠΌΡƒΠ»ΠΈΡ€ΠΎΠ²Π°Ρ‚ΡŒ основной Π·Π°ΠΊΠΎΠ½ Π΄ΠΈΠ½Π°ΠΌΠΈΠΊΠΈ (Π²Ρ‚ΠΎΡ€ΠΎΠΉ Π·Π°ΠΊΠΎΠ½ ΠΡŒΡŽΡ‚ΠΎΠ½Π°), согласно ΠΊΠΎΡ‚ΠΎΡ€ΠΎΠΌΡƒ масса Ρ‚Π΅Π»Π° m , умноТСнная Π½Π° ускорСниС a , Ρ€Π°Π²Π½Π° Π΄Π΅ΠΉΡΡ‚Π²ΡƒΡŽΡ‰Π΅ΠΉ силС Q (Π²ΠΎΠΎΠ±Ρ‰Π΅-Ρ‚ΠΎ Π±ΠΎΠ»Π΅Π΅ ΠΏΡ€ΠΈΠ²Ρ‹Ρ‡Π½Ρ‹ΠΌ для силы являСтся ΠΎΠ±ΠΎΠ·Π½Π°Ρ‡Π΅Π½ΠΈΠ΅ F, Π½ΠΎ Ρ‚Π°ΠΊ ΠΊΠ°ΠΊ дальшС ΠΌΡ‹ Π±ΡƒΠ΄Π΅ΠΌ ΠΈΠΌΠ΅Ρ‚ΡŒ Π΄Π΅Π»ΠΎ с ΠΏΠ»ΠΎΡ‰Π°Π΄ΡŒΡŽ, которая Ρ‚Π°ΠΊΠΆΠ΅ часто обозначаСтся ΠΊΠ°ΠΊ F, Ρ‚ΠΎ я ΠΈΡΠΏΠΎΠ»ΡŒΠ·ΡƒΡŽ для внСшнСй силы, рассматриваСмой Π² тСорСтичСской ΠΌΠ΅Ρ…Π°Π½ΠΈΠΊΠ΅ ΠΊΠ°ΠΊ сосрСдоточСнная Π½Π°Π³Ρ€ΡƒΠ·ΠΊΠ°, ΠΎΠ±ΠΎΠ·Π½Π°Ρ‡Π΅Π½ΠΈΠ΅ Q, сути Π΄Π΅Π»Π° это Π½Π΅ мСняСт):

Q = ma (1.2)

По ΠΌΠ½Π΅ Π²Π΅Π»ΠΈΡ‡ΠΈΠ΅ ΠΡŒΡŽΡ‚ΠΎΠ½Π° ΠΈΠΌΠ΅Π½Π½ΠΎ Π² простотС ΠΈ понятности Π΄Π°Π½Π½ΠΎΠ³ΠΎ опрСдСлСния. А Π΅Ρ‰Π΅, Ссли ΡƒΡ‡Π΅ΡΡ‚ΡŒ, Ρ‡Ρ‚ΠΎ ΠΏΡ€ΠΈ равноускорСнном Π΄Π²ΠΈΠΆΠ΅Π½ΠΈΠΈ ускорСниС Π° Ρ€Π°Π²Π½ΠΎ ΠΎΡ‚Π½ΠΎΡˆΠ΅Π½ΠΈΡŽ приращСния скорости Ξ”V ΠΊ ΠΏΠ΅Ρ€ΠΈΠΎΠ΄Ρƒ Π²Ρ€Π΅ΠΌΠ΅Π½ΠΈ Ξ”t , Π·Π° ΠΊΠΎΡ‚ΠΎΡ€Ρ‹ΠΉ ΡΠΊΠΎΡ€ΠΎΡΡ‚ΡŒ измСнилась:

a = Ξ”v/Ξ”t = (v – v ΠΎ)/t (1. 3.1)

ΠΏΡ€ΠΈ V ΠΎ = 0 a = v/t (1.3.2)

Ρ‚ΠΎ ΠΌΠΎΠΆΠ½ΠΎ ΠΎΠΏΡ€Π΅Π΄Π΅Π»ΠΈΡ‚ΡŒ основныС ΠΏΠ°Ρ€Π°ΠΌΠ΅Ρ‚Ρ€Ρ‹ двиТСния, Ρ‚Π°ΠΊΠΈΠ΅ ΠΊΠ°ΠΊ расстояниС, ΡΠΊΠΎΡ€ΠΎΡΡ‚ΡŒ, врСмя ΠΈ Π΄Π°ΠΆΠ΅ ΠΈΠΌΠΏΡƒΠ»ΡŒΡ Ρ€ , Ρ…Π°Ρ€Π°ΠΊΡ‚Π΅Ρ€ΠΈΠ·ΡƒΡŽΡ‰ΠΈΠΉ количСство двиТСния:

p = mv (1.4)

НапримСр, яблоко, ΠΏΠ°Π΄Π°ΡŽΡ‰Π΅Π΅ с Ρ€Π°Π·Π½ΠΎΠΉ высоты ΠΏΠΎΠ΄ дСйствиСм Ρ‚ΠΎΠ»ΡŒΠΊΠΎ силы тяТСсти, Π±ΡƒΠ΄Π΅Ρ‚ ΠΏΠ°Π΄Π°Ρ‚ΡŒ Π΄ΠΎ Π·Π΅ΠΌΠ»ΠΈ Ρ€Π°Π·Π½ΠΎΠ΅ врСмя, ΠΈΠΌΠ΅Ρ‚ΡŒ Ρ€Π°Π·Π½ΡƒΡŽ ΡΠΊΠΎΡ€ΠΎΡΡ‚ΡŒ Π² ΠΌΠΎΠΌΠ΅Π½Ρ‚ призСмлСния ΠΈ соотвСтствСнно Ρ€Π°Π·Π½Ρ‹ΠΉ ΠΈΠΌΠΏΡƒΠ»ΡŒΡ. Π”Ρ€ΡƒΠ³ΠΈΠΌΠΈ словами, яблоко, ΠΏΠ°Π΄Π°ΡŽΡ‰Π΅Π΅ с бóльшСй высоты, Π±ΡƒΠ΄Π΅Ρ‚ дольшС Π»Π΅Ρ‚Π΅Ρ‚ΡŒ ΠΈ сильнСС трСснСт ΠΏΠΎ Π»Π±Ρƒ Π½Π΅Π·Π°Π΄Π°Ρ‡Π»ΠΈΠ²ΠΎΠ³ΠΎ Π½Π°Π±Π»ΡŽΠ΄Π°Ρ‚Π΅Π»Ρ. И всС это ΠΡŒΡŽΡ‚ΠΎΠ½ свСл ΠΊ простой ΠΈ понятной Ρ„ΠΎΡ€ΠΌΡƒΠ»Π΅.

А Π΅Ρ‰Π΅ ΠΡŒΡŽΡ‚ΠΎΠ½ сформулировал Π·Π°ΠΊΠΎΠ½ ΠΈΠ½Π΅Ρ€Ρ†ΠΈΠΈ (ΠΏΠ΅Ρ€Π²Ρ‹ΠΉ Π·Π°ΠΊΠΎΠ½ ΠΡŒΡŽΡ‚ΠΎΠ½Π°): Ссли ускорСниС Π° = 0 , Ρ‚ΠΎ Π² ΠΈΠ½Π΅Ρ€Ρ†ΠΈΠ°Π»ΡŒΠ½ΠΎΠΉ систСмС отсчСта Π½Π΅Π²ΠΎΠ·ΠΌΠΎΠΆΠ½ΠΎ ΠΎΠΏΡ€Π΅Π΄Π΅Π»ΠΈΡ‚ΡŒ, находится Π»ΠΈ наблюдаСмоС Ρ‚Π΅Π»ΠΎ, Π½Π° ΠΊΠΎΡ‚ΠΎΡ€ΠΎΠ΅ Π½Π΅ Π΄Π΅ΠΉΡΡ‚Π²ΡƒΡŽΡ‚ внСшниС силы, Π² состоянии покоя ΠΈΠ»ΠΈ двиТСтся прямолинСйно с постоянной ΡΠΊΠΎΡ€ΠΎΡΡ‚ΡŒΡŽ. Π­Ρ‚ΠΎ свойство ΠΌΠ°Ρ‚Π΅Ρ€ΠΈΠ°Π»ΡŒΠ½Ρ‹Ρ… Ρ‚Π΅Π» ΡΠΎΡ…Ρ€Π°Π½ΡΡ‚ΡŒ свою ΡΠΊΠΎΡ€ΠΎΡΡ‚ΡŒ, ΠΏΡƒΡΡ‚ΡŒ Π΄Π°ΠΆΠ΅ ΠΈ Π½ΡƒΠ»Π΅Π²ΡƒΡŽ, называСтся ΠΈΠ½Π΅Ρ€Ρ‚Π½ΠΎΡΡ‚ΡŒΡŽ. ΠœΠ΅Ρ€ΠΎΠΉ инСртности являСтся инСрционная масса Ρ‚Π΅Π»Π°. Иногда инСрционная масса называСтся ΠΈΠ½Π΅Ρ€Ρ‚Π½ΠΎΠΉ, Π½ΠΎ сути Π΄Π΅Π»Π° это Π½Π΅ мСняСт. БчитаСтся, Ρ‡Ρ‚ΠΎ инСрционная масса Ρ€Π°Π²Π½Π° Π³Ρ€Π°Π²ΠΈΡ‚Π°Ρ†ΠΈΠΎΠ½Π½ΠΎΠΉ массС ΠΈ ΠΏΠΎΡ‚ΠΎΠΌΡƒ часто Π½Π΅ уточняСтся, какая ΠΈΠΌΠ΅Π½Π½ΠΎ масса имССтся Π² Π²ΠΈΠ΄Ρƒ, Π° упоминаСтся просто масса Ρ‚Π΅Π»Π°.

НС ΠΌΠ΅Π½Π΅Π΅ Π²Π°ΠΆΠ½Ρ‹ΠΌ ΠΈ Π·Π½Π°Ρ‡ΠΈΠΌΡ‹ΠΌ являСтся ΠΈ Ρ‚Ρ€Π΅Ρ‚ΠΈΠΉ Π·Π°ΠΊΠΎΠ½ ΠΡŒΡŽΡ‚ΠΎΠ½Π°, согласно ΠΊΠΎΡ‚ΠΎΡ€ΠΎΠΌΡƒ сила дСйствия Ρ€Π°Π²Π½Π° силС противодСйствия, Ссли силы Π½Π°ΠΏΡ€Π°Π²Π»Π΅Π½Ρ‹ ΠΏΠΎ ΠΎΠ΄Π½ΠΎΠΉ прямой, Π½ΠΎ ΠΏΡ€ΠΈ этом Π² ΠΏΡ€ΠΎΡ‚ΠΈΠ²ΠΎΠΏΠΎΠ»ΠΎΠΆΠ½Ρ‹Π΅ стороны . НС смотря, Π½Π° ΠΊΠ°ΠΆΡƒΡ‰ΡƒΡŽΡΡ простоту, ΠΈ этот Π²Ρ‹Π²ΠΎΠ΄ ΠΡŒΡŽΡ‚ΠΎΠ½Π° Π³Π΅Π½ΠΈΠ°Π»Π΅Π½ ΠΈ Π·Π½Π°Ρ‡Π΅Π½ΠΈΠ΅ этого Π·Π°ΠΊΠΎΠ½Π° Ρ‚Ρ€ΡƒΠ΄Π½ΠΎ ΠΏΠ΅Ρ€Π΅ΠΎΡ†Π΅Π½ΠΈΡ‚ΡŒ. Об ΠΎΠ΄Π½ΠΎΠΌ ΠΈΠ· ΠΏΡ€ΠΈΠΌΠ΅Π½Π΅Π½ΠΈΠΉ этого Π·Π°ΠΊΠΎΠ½Π° Ρ‡ΡƒΡ‚ΡŒ Π½ΠΈΠΆΠ΅.

Однако Π΄Π°Π½Π½Ρ‹Π΅ полоТСния справСдливы Ρ‚ΠΎΠ»ΡŒΠΊΠΎ для Ρ‚Π΅Π», двиТущихся ΠΏΠΎΡΡ‚ΡƒΠΏΠ°Ρ‚Π΅Π»ΡŒΠ½ΠΎ, Ρ‚.Π΅. ΠΏΠΎ прямолинСйной Ρ‚Ρ€Π°Π΅ΠΊΡ‚ΠΎΡ€ΠΈΠΈ ΠΈ ΠΏΡ€ΠΈ этом всС ΠΌΠ°Ρ‚Π΅Ρ€ΠΈΠ°Π»ΡŒΠ½Ρ‹Π΅ Ρ‚ΠΎΡ‡ΠΊΠΈ Ρ‚Π°ΠΊΠΈΡ… Ρ‚Π΅Π» Π΄Π²ΠΈΠ³Π°ΡŽΡ‚ΡΡ с ΠΎΠ΄ΠΈΠ½Π°ΠΊΠΎΠ²ΠΎΠΉ ΡΠΊΠΎΡ€ΠΎΡΡ‚ΡŒΡŽ ΠΈΠ»ΠΈ ΠΎΠ΄ΠΈΠ½Π°ΠΊΠΎΠ²Ρ‹ΠΌ ускорСниСм. ΠŸΡ€ΠΈ ΠΊΡ€ΠΈΠ²ΠΎΠ»ΠΈΠ½Π΅ΠΉΠ½ΠΎΠΌ Π΄Π²ΠΈΠΆΠ΅Π½ΠΈΠΈ ΠΈ Π² частности ΠΏΡ€ΠΈ Π²Ρ€Π°Ρ‰Π°Ρ‚Π΅Π»ΡŒΠ½ΠΎΠΌ Π΄Π²ΠΈΠΆΠ΅Π½ΠΈΠΈ, Π½Π°ΠΏΡ€ΠΈΠΌΠ΅Ρ€, ΠΊΠΎΠ³Π΄Π° Ρ‚Π΅Π»ΠΎ вращаСтся Π²ΠΎΠΊΡ€ΡƒΠ³ своСй оси симмСтрии, ΠΌΠ°Ρ‚Π΅Ρ€ΠΈΠ°Π»ΡŒΠ½Ρ‹Π΅ Ρ‚ΠΎΡ‡ΠΊΠΈ Ρ‚Π°ΠΊΠΎΠ³ΠΎ Ρ‚Π΅Π»Π° ΠΏΠ΅Ρ€Π΅ΠΌΠ΅Ρ‰Π°ΡŽΡ‚ΡΡ Π² пространствС с ΠΎΠ΄ΠΈΠ½Π°ΠΊΠΎΠ²ΠΎΠΉ ΡƒΠ³Π»ΠΎΠ²ΠΎΠΉ ΡΠΊΠΎΡ€ΠΎΡΡ‚ΡŒΡŽ w , Π½ΠΎ ΠΏΡ€ΠΈ этом линСйная ΡΠΊΠΎΡ€ΠΎΡΡ‚ΡŒ v Ρƒ Ρ€Π°Π·Π»ΠΈΡ‡Π½Ρ‹Ρ… Ρ‚ΠΎΡ‡Π΅ΠΊ Π±ΡƒΠ΄Π΅Ρ‚ разная ΠΈ эта линСйная ΡΠΊΠΎΡ€ΠΎΡΡ‚ΡŒ прямо ΠΏΡ€ΠΎΠΏΠΎΡ€Ρ†ΠΈΠΎΠ½Π°Π»ΡŒΠ½Π° Ρ€Π°ΡΡΡ‚ΠΎΡΠ½ΠΈΡŽ r ΠΎΡ‚ оси вращСния Π΄ΠΎ этой Ρ‚ΠΎΡ‡ΠΊΠΈ:

v = wr (1. 5)

ΠΏΡ€ΠΈ этом угловая ΡΠΊΠΎΡ€ΠΎΡΡ‚ΡŒ Ρ€Π°Π²Π½Π° ΠΎΡ‚Π½ΠΎΡˆΠ΅Π½ΠΈΡŽ приращСния ΡƒΠ³Π»Π° ΠΏΠΎΠ²ΠΎΡ€ΠΎΡ‚Π° Δφ ΠΊ ΠΏΠ΅Ρ€ΠΈΠΎΠ΄Ρƒ Π²Ρ€Π΅ΠΌΠ΅Π½ΠΈ Ξ”t , Π·Π° ΠΊΠΎΡ‚ΠΎΡ€Ρ‹ΠΉ ΡƒΠ³ΠΎΠ» ΠΏΠΎΠ²ΠΎΡ€ΠΎΡ‚Π° измСнился:

w = Δφ/Ξ”t = (Ο† – Ο† ΠΎ)/t (1.6.1)

ΠΏΡ€ΠΈ Ο† ΠΎ = 0 w = Ο†/t (1.7.2)

соотвСтствСнно Π½ΠΎΡ€ΠΌΠ°Π»ΡŒΠ½ΠΎΠ΅ ускорСниС Π° n ΠΏΡ€ΠΈ Π²Ρ€Π°Ρ‰Π°Ρ‚Π΅Π»ΡŒΠ½ΠΎΠΌ Π΄Π²ΠΈΠΆΠ΅Π½ΠΈΠΈ Ρ€Π°Π²Π½ΠΎ:

a n = v 2 /r = w 2 r (1.8)

И получаСтся, Ρ‡Ρ‚ΠΎ для Π²Ρ€Π°Ρ‰Π°Ρ‚Π΅Π»ΡŒΠ½ΠΎΠ³ΠΎ двиТСния ΠΌΡ‹ Π½Π΅ ΠΌΠΎΠΆΠ΅ΠΌ прямо ΠΈΡΠΏΠΎΠ»ΡŒΠ·ΠΎΠ²Π°Ρ‚ΡŒ Ρ„ΠΎΡ€ΠΌΡƒΠ»Ρƒ (1.2), Ρ‚Π°ΠΊ ΠΊΠ°ΠΊ ΠΏΡ€ΠΈ Π²Ρ€Π°Ρ‰Π°Ρ‚Π΅Π»ΡŒΠ½ΠΎΠΌ Π΄Π²ΠΈΠΆΠ΅Π½ΠΈΠΈ ΠΎΠ΄Π½ΠΎΠ³ΠΎ Ρ‚ΠΎΠ»ΡŒΠΊΠΎ значСния массы Ρ‚Π΅Π»Π° нСдостаточно, трСбуСтся Π΅Ρ‰Π΅ Π·Π½Π°Ρ‚ΡŒ распрСдСлСниС этой массы Π² Ρ‚Π΅Π»Π΅. ΠŸΠΎΠ»ΡƒΡ‡Π°Π΅Ρ‚ΡΡ, Ρ‡Ρ‚ΠΎ Ρ‡Π΅ΠΌ Π±Π»ΠΈΠΆΠ΅ ΠΌΠ°Ρ‚Π΅Ρ€ΠΈΠ°Π»ΡŒΠ½Ρ‹Π΅ Ρ‚ΠΎΡ‡ΠΊΠΈ Ρ‚Π΅Π»Π° ΠΊ оси вращСния, Ρ‚Π΅ΠΌ ΠΌΠ΅Π½ΡŒΡˆΡƒΡŽ силу трСбуСтся ΠΏΡ€ΠΈΠ»ΠΎΠΆΠΈΡ‚ΡŒ, Ρ‡Ρ‚ΠΎΠ±Ρ‹ Π·Π°ΡΡ‚Π°Π²ΠΈΡ‚ΡŒ Ρ‚Π΅Π»ΠΎ Π²Ρ€Π°Ρ‰Π°Ρ‚ΡŒΡΡ ΠΈ Π½Π°ΠΎΠ±ΠΎΡ€ΠΎΡ‚, Ρ‡Π΅ΠΌ дальшС ΠΌΠ°Ρ‚Π΅Ρ€ΠΈΠ°Π»ΡŒΠ½Ρ‹Π΅ Ρ‚ΠΎΡ‡ΠΊΠΈ Ρ‚Π΅Π»Π° ΠΎΡ‚ оси вращСния, Ρ‚Π΅ΠΌ Π±ΠΎΠ»ΡŒΡˆΡƒΡŽ силу Π½ΡƒΠΆΠ½ΠΎ ΠΏΡ€ΠΈΠ»ΠΎΠΆΠΈΡ‚ΡŒ, Ρ‡Ρ‚ΠΎΠ±Ρ‹ Π·Π°ΡΡ‚Π°Π²ΠΈΡ‚ΡŒ Ρ‚Π΅Π»ΠΎ Π²Ρ€Π°Ρ‰Π°Ρ‚ΡŒΡΡ (Π² Π΄Π°Π½Π½ΠΎΠΌ случаС Ρ€Π΅Ρ‡ΡŒ ΠΈΠ΄Π΅Ρ‚ ΠΎ ΠΏΡ€ΠΈΠ»ΠΎΠΆΠ΅Π½ΠΈΠΈ силы Π² ΠΎΠ΄Π½ΠΎΠΉ ΠΈ Ρ‚ΠΎΠΉ ΠΆΠ΅ Ρ‚ΠΎΡ‡ΠΊΠ΅). К Ρ‚ΠΎΠΌΡƒ ΠΆΠ΅ ΠΏΡ€ΠΈ Π²Ρ€Π°Ρ‰Π΅Π½ΠΈΠΈ Ρ‚Π΅Π»Π° Π±ΠΎΠ»Π΅Π΅ ΡƒΠ΄ΠΎΠ±Π½ΠΎ Ρ€Π°ΡΡΠΌΠ°Ρ‚Ρ€ΠΈΠ²Π°Ρ‚ΡŒ Π½Π΅ Π΄Π΅ΠΉΡΡ‚Π²ΡƒΡŽΡ‰ΡƒΡŽ силу, Π° Π²Ρ€Π°Ρ‰Π°ΡŽΡ‰ΠΈΠΉ ΠΌΠΎΠΌΠ΅Π½Ρ‚, Ρ‚Π°ΠΊ ΠΊΠ°ΠΊ ΠΏΡ€ΠΈ Π²Ρ€Π°Ρ‰Π°Ρ‚Π΅Π»ΡŒΠ½ΠΎΠΌ Π΄Π²ΠΈΠΆΠ΅Π½ΠΈΠΈ Ρ‚ΠΎΡ‡ΠΊΠ° прилоТСния силы Ρ‚Π°ΠΊΠΆΠ΅ ΠΈΠΌΠ΅Π΅Ρ‚ большоС Π·Π½Π°Ρ‡Π΅Π½ΠΈΠ΅.

ΠŸΠΎΡ€Π°Π·ΠΈΡ‚Π΅Π»ΡŒΠ½Ρ‹Π΅ свойства ΠΌΠΎΠΌΠ΅Π½Ρ‚Π° Π½Π°ΠΌ извСстны со Π²Ρ€Π΅ΠΌΠ΅Π½ АрхимСда ΠΈ Ссли ΠΏΡ€ΠΈΠΌΠ΅Π½ΠΈΡ‚ΡŒ понятиС ΠΌΠΎΠΌΠ΅Π½Ρ‚Π° ΠΊ Π²Ρ€Π°Ρ‰Π°Ρ‚Π΅Π»ΡŒΠ½ΠΎΠΌΡƒ двиТСнию, Ρ‚ΠΎ Π·Π½Π°Ρ‡Π΅Π½ΠΈΠ΅ ΠΌΠΎΠΌΠ΅Π½Ρ‚Π° М Π±ΡƒΠ΄Π΅Ρ‚ Ρ‚Π΅ΠΌ большС, Ρ‡Π΅ΠΌ большС расстояниС r ΠΎΡ‚ оси вращСния Π΄ΠΎ Ρ‚ΠΎΡ‡ΠΊΠΈ прилоТСния силы F (Π² ΡΡ‚Ρ€ΠΎΠΈΡ‚Π΅Π»ΡŒΠ½ΠΎΠΉ ΠΌΠ΅Ρ…Π°Π½ΠΈΠΊΠ΅ внСшняя сила часто обозначаСтся ΠΊΠ°ΠΊ Π  ΠΈΠ»ΠΈ Q ):

М = Qr (1.9)

Из этой Ρ‚Π°ΠΊΠΆΠ΅ Π½Π΅ ΠΎΡ‡Π΅Π½ΡŒ слоТной Ρ„ΠΎΡ€ΠΌΡƒΠ»Ρ‹ Π²Ρ‹Ρ…ΠΎΠ΄ΠΈΡ‚, Ρ‡Ρ‚ΠΎ Ссли сила Π±ΡƒΠ΄Π΅Ρ‚ ΠΏΡ€ΠΈΠ»ΠΎΠΆΠ΅Π½Π° ΠΏΠΎ оси вращСния, Ρ‚ΠΎ Π½ΠΈΠΊΠ°ΠΊΠΎΠ³ΠΎ вращСния Π½Π΅ Π±ΡƒΠ΄Π΅Ρ‚, Ρ‚Π°ΠΊ ΠΊΠ°ΠΊ r = 0, Π° Ссли сила Π±ΡƒΠ΄Π΅Ρ‚ ΠΏΡ€ΠΈΠ»ΠΎΠΆΠ΅Π½Π° Π½Π° максимальном ΡƒΠ΄Π°Π»Π΅Π½ΠΈΠΈ ΠΎΡ‚ оси вращСния, Ρ‚ΠΎ ΠΈ Π·Π½Π°Ρ‡Π΅Π½ΠΈΠ΅ ΠΌΠΎΠΌΠ΅Π½Ρ‚Π° Π±ΡƒΠ΄Π΅Ρ‚ ΠΌΠ°ΠΊΡΠΈΠΌΠ°Π»ΡŒΠ½Ρ‹ΠΌ. А Ссли ΠΌΡ‹ подставим Π² Ρ„ΠΎΡ€ΠΌΡƒΠ»Ρƒ (1.9) Π·Π½Π°Ρ‡Π΅Π½ΠΈΠ΅ силы ΠΈΠ· Ρ„ΠΎΡ€ΠΌΡƒΠ»Ρ‹ (1.2) ΠΈ Π·Π½Π°Ρ‡Π΅Π½ΠΈΠ΅ Π½ΠΎΡ€ΠΌΠ°Π»ΡŒΠ½ΠΎΠ³ΠΎ ускорСния ΠΈ Ρ„ΠΎΡ€ΠΌΡƒΠ»Ρ‹ (1.8), Ρ‚ΠΎ ΠΏΠΎΠ»ΡƒΡ‡ΠΈΠΌ ΡΠ»Π΅Π΄ΡƒΡŽΡ‰Π΅Π΅ ΡƒΡ€Π°Π²Π½Π΅Π½ΠΈΠ΅:

М = mw 2 r·r = mw 2 r 2 (1.10)

Π’ частном случаС ΠΊΠΎΠ³Π΄Π° Ρ‚Π΅Π»ΠΎ являСтся ΠΌΠ°Ρ‚Π΅Ρ€ΠΈΠ°Π»ΡŒΠ½ΠΎΠΉ Ρ‚ΠΎΡ‡ΠΊΠΎΠΉ, ΠΈΠΌΠ΅ΡŽΡ‰Π΅ΠΉ Ρ€Π°Π·ΠΌΠ΅Ρ€Ρ‹ Π½Π°ΠΌΠ½ΠΎΠ³ΠΎ мСньшС, Ρ‡Π΅ΠΌ расстояниС ΠΎΡ‚ этой Ρ‚ΠΎΡ‡ΠΊΠΈ Π΄ΠΎ оси вращСния, ΡƒΡ€Π°Π²Π½Π΅Π½ΠΈΠ΅ (1.10) ΠΏΡ€ΠΈΠΌΠ΅Π½ΠΈΠΌΠΎ Π² чистом Π²ΠΈΠ΄Π΅. Однако для Ρ‚Π΅Π»Π°, Π²Ρ€Π°Ρ‰Π°ΡŽΡ‰Π΅Π³ΠΎΡΡ Π²ΠΎΠΊΡ€ΡƒΠ³ ΠΎΠ΄Π½ΠΎΠΉ ΠΈΠ· своих осСй симмСтрии, расстояниС ΠΎΡ‚ ΠΊΠ°ΠΆΠ΄ΠΎΠΉ ΠΌΠ°Ρ‚Π΅Ρ€ΠΈΠ°Π»ΡŒΠ½ΠΎΠΉ Ρ‚ΠΎΡ‡ΠΊΠΈ ΡΠΎΡΡ‚Π°Π²Π»ΡΡŽΡ‰Π΅ΠΉ Π΄Π°Π½Π½ΠΎΠ΅ Ρ‚Π΅Π»ΠΎ, всСгда мСньшС ΠΎΠ΄Π½ΠΎΠ³ΠΎ ΠΈΠ· гСомСтричСских Ρ€Π°Π·ΠΌΠ΅Ρ€ΠΎΠ² Ρ‚Π΅Π»Π° ΠΈ ΠΏΠΎΡ‚ΠΎΠΌΡƒ распрСдСлСниС массы Ρ‚Π΅Π»Π° ΠΈΠΌΠ΅Π΅Ρ‚ большоС Π·Π½Π°Ρ‡Π΅Π½ΠΈΠ΅, Π² этом случаС трСбуСтся ΡƒΡ‡Π΅ΡΡ‚ΡŒ эти расстояния ΠΎΡ‚Π΄Π΅Π»ΡŒΠ½ΠΎ для ΠΊΠ°ΠΆΠ΄ΠΎΠΉ Ρ‚ΠΎΡ‡ΠΊΠΈ:

M = βˆ‘r i 2 w 2 m i (1.11.1)

М с = w 2 ∫r 2 dm

И Ρ‚ΠΎΠ³Π΄Π° получаСтся, Ρ‡Ρ‚ΠΎ согласно Ρ‚Ρ€Π΅Ρ‚ΡŒΠ΅ΠΌΡƒ Π·Π°ΠΊΠΎΠ½Ρƒ ΠΡŒΡŽΡ‚ΠΎΠ½Π° Π² ΠΎΡ‚Π²Π΅Ρ‚ Π½Π° дСйствиС Π²Ρ€Π°Ρ‰Π°ΡŽΡ‰Π΅Π³ΠΎ ΠΌΠΎΠΌΠ΅Π½Ρ‚Π° Π±ΡƒΠ΄Π΅Ρ‚ Π²ΠΎΠ·Π½ΠΈΠΊΠ°Ρ‚ΡŒ Ρ‚Π°ΠΊ Π½Π°Π·Ρ‹Π²Π°Π΅ΠΌΡ‹ΠΉ ΠΌΠΎΠΌΠ΅Π½Ρ‚ ΠΈΠ½Π΅Ρ€Ρ†ΠΈΠΈ I . ΠŸΡ€ΠΈ этом значСния Π²Ρ€Π°Ρ‰Π°ΡŽΡ‰Π΅Π³ΠΎ ΠΌΠΎΠΌΠ΅Π½Ρ‚Π° ΠΈ ΠΌΠΎΠΌΠ΅Π½Ρ‚Π° ΠΈΠ½Π΅Ρ€Ρ†ΠΈΠΈ Π±ΡƒΠ΄ΡƒΡ‚ Ρ€Π°Π²Π½Ρ‹, Π° сами ΠΌΠΎΠΌΠ΅Π½Ρ‚Ρ‹ Π½Π°ΠΏΡ€Π°Π²Π»Π΅Π½Ρ‹ Π² ΠΏΡ€ΠΎΡ‚ΠΈΠ²ΠΎΠΏΠΎΠ»ΠΎΠΆΠ½Ρ‹Π΅ стороны. ΠŸΡ€ΠΈ постоянной ΡƒΠ³Π»ΠΎΠ²ΠΎΠΉ скорости вращСния, Π½Π°ΠΏΡ€ΠΈΠΌΠ΅Ρ€ w = 1, основными Π²Π΅Π»ΠΈΡ‡ΠΈΠ½Π°ΠΌΠΈ, Ρ…Π°Ρ€Π°ΠΊΡ‚Π΅Ρ€ΠΈΠ·ΡƒΡŽΡ‰ΠΈΠΌΠΈ Π²Ρ€Π°Ρ‰Π°ΡŽΡ‰ΠΈΠΉ ΠΌΠΎΠΌΠ΅Π½Ρ‚ ΠΈΠ»ΠΈ ΠΌΠΎΠΌΠ΅Π½Ρ‚ ΠΈΠ½Π΅Ρ€Ρ†ΠΈΠΈ Π±ΡƒΠ΄ΡƒΡ‚ масса ΠΌΠ°Ρ‚Π΅Ρ€ΠΈΠ°Π»ΡŒΠ½Ρ‹Ρ… Ρ‚ΠΎΡ‡Π΅ΠΊ, ΡΠΎΡΡ‚Π°Π²Π»ΡΡŽΡ‰ΠΈΡ… Ρ‚Π΅Π»ΠΎ, ΠΈ расстояния ΠΎΡ‚ этих Ρ‚ΠΎΡ‡Π΅ΠΊ Π΄ΠΎ оси вращСния. Π’ ΠΈΡ‚ΠΎΠ³Π΅ Ρ„ΠΎΡ€ΠΌΡƒΠ»Π° ΠΌΠΎΠΌΠ΅Π½Ρ‚Π° ΠΈΠ½Π΅Ρ€Ρ†ΠΈΠΈ ΠΏΡ€ΠΈΠΌΠ΅Ρ‚ ΡΠ»Π΅Π΄ΡƒΡŽΡ‰ΠΈΠΉ Π²ΠΈΠ΄:

[- М] = I = βˆ‘r i 2 m i (1. 12.1)

I c = ∫r 2 dm (1.11.2) – ΠΏΡ€ΠΈ Π²Ρ€Π°Ρ‰Π΅Π½ΠΈΠΈ Ρ‚Π΅Π»Π° Π²ΠΎΠΊΡ€ΡƒΠ³ оси симмСтрии

Π³Π΄Π΅ I – общСпринятоС ΠΎΠ±ΠΎΠ·Π½Π°Ρ‡Π΅Π½ΠΈΠ΅ ΠΌΠΎΠΌΠ΅Π½Ρ‚Π° ΠΈΠ½Π΅Ρ€Ρ†ΠΈΠΈ, I c – ΠΎΠ±ΠΎΠ·Π½Π°Ρ‡Π΅Π½ΠΈΠ΅ осСвого ΠΌΠΎΠΌΠ΅Π½Ρ‚Π° ΠΈΠ½Π΅Ρ€Ρ†ΠΈΠΈ Ρ‚Π΅Π»Π°, ΠΊΠ³/ΠΌ 2 . Для ΠΎΠ΄Π½ΠΎΡ€ΠΎΠ΄Π½ΠΎΠ³ΠΎ Ρ‚Π΅Π»Π°, ΠΈΠΌΠ΅ΡŽΡ‰Π΅Π³ΠΎ ΠΎΠ΄ΠΈΠ½Π°ΠΊΠΎΠ²ΡƒΡŽ ΠΏΠ»ΠΎΡ‚Π½ΠΎΡΡ‚ΡŒ ρ ΠΏΠΎ всСму ΠΎΠ±ΡŠΠ΅ΠΌΡƒ Ρ‚Π΅Π»Π° V Ρ„ΠΎΡ€ΠΌΡƒΠ»Ρƒ осСвого ΠΌΠΎΠΌΠ΅Π½Ρ‚Π° ΠΈΠ½Π΅Ρ€Ρ†ΠΈΠΈ Ρ‚Π΅Π»Π° ΠΌΠΎΠΆΠ½ΠΎ Π·Π°ΠΏΠΈΡΠ°Ρ‚ΡŒ Ρ‚Π°ΠΊ:

I c = ∫ρr 2 dV (1.13)

Π’Π°ΠΊΠΈΠΌ ΠΎΠ±Ρ€Π°Π·ΠΎΠΌ ΠΌΠΎΠΌΠ΅Π½Ρ‚ ΠΈΠ½Π΅Ρ€Ρ†ΠΈΠΈ являСтся ΠΌΠ΅Ρ€ΠΎΠΉ инСртности Ρ‚Π΅Π»Π° ΠΏΡ€ΠΈ Π²Ρ€Π°Ρ‰Π°Ρ‚Π΅Π»ΡŒΠ½ΠΎΠΌ Π΄Π²ΠΈΠΆΠ΅Π½ΠΈΠΈ, ΠΏΠΎΠ΄ΠΎΠ±Π½ΠΎ Ρ‚ΠΎΠΌΡƒ ΠΊΠ°ΠΊ масса являСтся ΠΌΠ΅Ρ€ΠΎΠΉ инСртности Ρ‚Π΅Π»Π° ΠΏΡ€ΠΈ ΠΏΠΎΡΡ‚ΡƒΠΏΠ°Ρ‚Π΅Π»ΡŒΠ½ΠΎΠΌ прямолинСйном Π΄Π²ΠΈΠΆΠ΅Π½ΠΈΠΈ .

ВсС ΠΊΡ€ΡƒΠ³ замкнулся. И Ρ‚ΡƒΡ‚ ΠΌΠΎΠΆΠ΅Ρ‚ Π²ΠΎΠ·Π½ΠΈΠΊΠ½ΡƒΡ‚ΡŒ вопрос, ΠΊΠ°ΠΊΠΎΠ΅ ΠΎΡ‚Π½ΠΎΡˆΠ΅Π½ΠΈΠ΅ всС эти Π·Π°ΠΊΠΎΠ½Ρ‹ Π΄ΠΈΠ½Π°ΠΌΠΈΠΊΠΈ ΠΈ ΠΊΠΈΠ½Π΅ΠΌΠ°Ρ‚ΠΈΠΊΠΈ ΠΈΠΌΠ΅ΡŽΡ‚ ΠΊ расчСту статичСских ΡΡ‚Ρ€ΠΎΠΈΡ‚Π΅Π»ΡŒΠ½Ρ‹Ρ… конструкций? ΠžΠΊΠ°Π·Ρ‹Π²Π°Π΅Ρ‚ΡΡ, Ρ‡Ρ‚ΠΎ Π½ΠΈ Π½Π° Π΅ΡΡ‚ΡŒ самоС прямоС ΠΈ нСпосрСдствСнноС. Π’ΠΎ-ΠΏΠ΅Ρ€Π²Ρ‹Ρ… ΠΏΠΎΡ‚ΠΎΠΌΡƒ, Ρ‡Ρ‚ΠΎ всС эти Ρ„ΠΎΡ€ΠΌΡƒΠ»Ρ‹ Π²Ρ‹Π²ΠΎΠ΄ΠΈΠ»ΠΈΡΡŒ Ρ„ΠΈΠ·ΠΈΠΊΠ°ΠΌΠΈ ΠΈ ΠΌΠ°Ρ‚Π΅ΠΌΠ°Ρ‚ΠΈΠΊΠ°ΠΌΠΈ Π² Ρ‚Π΅ Π΄Π°Π»Π΅ΠΊΠΈΠ΅ Π²Ρ€Π΅ΠΌΠ΅Π½Π°, ΠΊΠΎΠ³Π΄Π° Ρ‚Π°ΠΊΠΈΡ… дисциплин, ΠΊΠ°ΠΊ “ВСорСтичСская ΠΌΠ΅Ρ…Π°Π½ΠΈΠΊΠ°” ΠΈΠ»ΠΈ “ВСория сопротивлСния ΠΌΠ°Ρ‚Π΅Ρ€ΠΈΠ°Π»ΠΎΠ²” попросту Π½Π΅ сущСствовало. А Π²ΠΎ-Π²Ρ‚ΠΎΡ€Ρ‹Ρ… ΠΏΠΎΡ‚ΠΎΠΌΡƒ, Ρ‡Ρ‚ΠΎ вСсь расчСт ΡΡ‚Ρ€ΠΎΠΈΡ‚Π΅Π»ΡŒΠ½Ρ‹Ρ… конструкций ΠΈ построСн Π½Π° основС ΡƒΠΊΠ°Π·Π°Π½Π½Ρ‹Ρ… Π·Π°ΠΊΠΎΠ½ΠΎΠ² ΠΈ Ρ„ΠΎΡ€ΠΌΡƒΠ»ΠΈΡ€ΠΎΠ²ΠΎΠΊ ΠΈ ΠΏΠΎΠΊΠ° Π½ΠΈ ΠΊΠ΅ΠΌ Π½Π΅ ΠΎΠΏΡ€ΠΎΠ²Π΅Ρ€Π³Π½ΡƒΡ‚ΠΎΠΌ ΡƒΡ‚Π²Π΅Ρ€ΠΆΠ΅Π½ΠΈΠΈ ΠΎ равСнствС Π³Ρ€Π°Π²ΠΈΡ‚Π°Ρ†ΠΈΠΎΠ½Π½ΠΎΠΉ ΠΈ ΠΈΠ½Π΅Ρ€Ρ‚ΠΎΠΉ масс. Π’ΠΎΡ‚ Ρ‚ΠΎΠ»ΡŒΠΊΠΎ Π² Ρ‚Π΅ΠΎΡ€ΠΈΠΈ сопротивлСния ΠΌΠ°Ρ‚Π΅Ρ€ΠΈΠ°Π»ΠΎΠ² всС Π΅Ρ‰Π΅ ΠΏΡ€ΠΎΡ‰Π΅, ΠΊΠ°ΠΊ Π½ΠΈ ΠΏΠ°Ρ€Π°Π΄ΠΎΠΊΡΠ°Π»ΡŒΠ½ΠΎ это Π·Π²ΡƒΡ‡ΠΈΡ‚.

А ΠΏΡ€ΠΎΡ‰Π΅ ΠΏΠΎΡ‚ΠΎΠΌΡƒ, Ρ‡Ρ‚ΠΎ ΠΏΡ€ΠΈ Ρ€Π΅ΡˆΠ΅Π½ΠΈΠΈ ΠΎΠΏΡ€Π΅Π΄Π΅Π»Π΅Π½Π½Ρ‹Ρ… Π·Π°Π΄Π°Ρ‡ ΠΌΠΎΠΆΠ΅Ρ‚ Ρ€Π°ΡΡΠΌΠ°Ρ‚Ρ€ΠΈΠ²Π°Ρ‚ΡŒΡΡ Π½Π΅ всС Ρ‚Π΅Π»ΠΎ, Π° Ρ‚ΠΎΠ»ΡŒΠΊΠΎ Π΅Π³ΠΎ ΠΏΠΎΠΏΠ΅Ρ€Π΅Ρ‡Π½ΠΎΠ΅ сСчСниС, Π° ΠΏΡ€ΠΈ нСобходимости нСсколько ΠΏΠΎΠΏΠ΅Ρ€Π΅Ρ‡Π½Ρ‹Ρ… сСчСний. Но Π² этих сСчСниях Π΄Π΅ΠΉΡΡ‚Π²ΡƒΡŽΡ‚ Ρ‚Π°ΠΊΠΈΠ΅ ΠΆΠ΅ физичСскиС силы, ΠΏΡ€Π°Π²Π΄Π° ΠΈΠΌΠ΅ΡŽΡ‰ΠΈΠ΅ нСсколько ΠΈΠ½ΡƒΡŽ ΠΏΡ€ΠΈΡ€ΠΎΠ΄Ρƒ. Π’Π°ΠΊΠΈΠΌ ΠΎΠ±Ρ€Π°Π·ΠΎΠΌ, Ссли Ρ€Π°ΡΡΠΌΠ°Ρ‚Ρ€ΠΈΠ²Π°Ρ‚ΡŒ Π½Π΅ΠΊΠΎΠ΅ Ρ‚Π΅Π»ΠΎ, Π΄Π»ΠΈΠ½Π° ΠΊΠΎΡ‚ΠΎΡ€ΠΎΠ³ΠΎ постоянна, Π° само Ρ‚Π΅Π»ΠΎ являСтся ΠΎΠ΄Π½ΠΎΡ€ΠΎΠ΄Π½Ρ‹ΠΌ, Ρ‚ΠΎ Ссли Π½Π΅ ΡƒΡ‡ΠΈΡ‚Ρ‹Π²Π°Ρ‚ΡŒ постоянныС ΠΏΠ°Ρ€Π°ΠΌΠ΅Ρ‚Ρ€Ρ‹ – Π΄Π»ΠΈΠ½Ρƒ ΠΈ ΠΏΠ»ΠΎΡ‚Π½ΠΎΡΡ‚ΡŒ (l = const, ρ = const ) – ΠΌΡ‹ ΠΏΠΎΠ»ΡƒΡ‡ΠΈΠΌ модСль ΠΏΠΎΠΏΠ΅Ρ€Π΅Ρ‡Π½ΠΎΠ³ΠΎ сСчСния. Для Ρ‚Π°ΠΊΠΎΠ³ΠΎ ΠΏΠΎΠΏΠ΅Ρ€Π΅Ρ‡Π½ΠΎΠ³ΠΎ сСчСния с матСматичСской Ρ‚ΠΎΡ‡ΠΊΠΈ зрСния Π±ΡƒΠ΄Π΅Ρ‚ справСдливым ΡƒΡ€Π°Π²Π½Π΅Π½ΠΈΠ΅:

I Ρ€ = ∫r 2 dF (2.1) β†’ (1.1)

Π³Π΄Π΅ I p – полярный ΠΌΠΎΠΌΠ΅Π½Ρ‚ ΠΈΠ½Π΅Ρ€Ρ†ΠΈΠΈ ΠΏΠΎΠΏΠ΅Ρ€Π΅Ρ‡Π½ΠΎΠ³ΠΎ сСчСния, ΠΌ 4 . Π’ ΠΈΡ‚ΠΎΠ³Π΅ ΠΌΡ‹ ΠΏΠΎΠ»ΡƒΡ‡ΠΈΠ»ΠΈ Ρ„ΠΎΡ€ΠΌΡƒΠ»Ρƒ, с ΠΊΠΎΡ‚ΠΎΡ€ΠΎΠΉ Π½Π°Ρ‡ΠΈΠ½Π°Π»ΠΈ (Π° Π²ΠΎΡ‚ стало Π»ΠΈ понятнСС, Ρ‡Ρ‚ΠΎ Ρ‚Π°ΠΊΠΎΠ΅ ΠΌΠΎΠΌΠ΅Π½Ρ‚ ΠΈΠ½Π΅Ρ€Ρ†ΠΈΠΈ сСчСния, Π½Π΅ знаю).

Π’Π°ΠΊ ΠΊΠ°ΠΊ Π² Ρ‚Π΅ΠΎΡ€ΠΈΠΈ сопротивлСния ΠΌΠ°Ρ‚Π΅Ρ€ΠΈΠ°Π»ΠΎΠ² часто Ρ€Π°ΡΡΠΌΠ°Ρ‚Ρ€ΠΈΠ²Π°ΡŽΡ‚ΡΡ ΠΏΡ€ΡΠΌΠΎΡƒΠ³ΠΎΠ»ΡŒΠ½Ρ‹Π΅ сСчСния, Π΄Π° ΠΈ ΠΏΡ€ΡΠΌΠΎΡƒΠ³ΠΎΠ»ΡŒΠ½Π°Ρ систСма ΠΊΠΎΠΎΡ€Π΄ΠΈΠ½Π°Ρ‚ Π±ΠΎΠ»Π΅Π΅ ΡƒΠ΄ΠΎΠ±Π½Π°, Ρ‚ΠΎ ΠΏΡ€ΠΈ Ρ€Π΅ΡˆΠ΅Π½ΠΈΠΈ Π·Π°Π΄Π°Ρ‡ ΠΎΠ±Ρ‹Ρ‡Π½ΠΎ Ρ€Π°ΡΡΠΌΠ°Ρ‚Ρ€ΠΈΠ²Π°ΡŽΡ‚ΡΡ Π΄Π²Π° осСвых ΠΌΠΎΠΌΠ΅Π½Ρ‚Π° ΠΈΠ½Π΅Ρ€Ρ†ΠΈΠΈ ΠΏΠΎΠΏΠ΅Ρ€Π΅Ρ‡Π½ΠΎΠ³ΠΎ сСчСния:

I z = ∫y 2 dF (2.2.1)

I y = ∫z 2 dF (2.2.2)

Рисунок 1 . ЗначСния ΠΊΠΎΠΎΡ€Π΄ΠΈΠ½Π°Ρ‚ ΠΏΡ€ΠΈ ΠΎΠΏΡ€Π΅Π΄Π΅Π»Π΅Π½ΠΈΠΈ осСвых ΠΌΠΎΠΌΠ΅Π½Ρ‚ΠΎΠ² ΠΈΠ½Π΅Ρ€Ρ†ΠΈΠΈ.

Π’ΡƒΡ‚ ΠΌΠΎΠΆΠ΅Ρ‚ Π²ΠΎΠ·Π½ΠΈΠΊΠ½ΡƒΡ‚ΡŒ вопрос, ΠΏΠΎΡ‡Π΅ΠΌΡƒ ΠΈΡΠΏΠΎΠ»ΡŒΠ·ΠΎΠ²Π°Π½Ρ‹ оси z ΠΈ Ρƒ , Π° Π½Π΅ Π±ΠΎΠ»Π΅Π΅ ΠΏΡ€ΠΈΠ²Ρ‹Ρ‡Π½Ρ‹Π΅ Ρ… ΠΈ Ρƒ ? Π’Π°ΠΊ ΡƒΠΆ слоТилось, Ρ‡Ρ‚ΠΎ ΠΎΠΏΡ€Π΅Π΄Π΅Π»Π΅Π½ΠΈΠ΅ усилий Π² ΠΏΠΎΠΏΠ΅Ρ€Π΅Ρ‡Π½ΠΎΠΌ сСчСнии ΠΈ ΠΏΠΎΠ΄Π±ΠΎΡ€ сСчСния, Π²Ρ‹Π΄Π΅Ρ€ΠΆΠΈΠ²Π°ΡŽΡ‰Π΅Π³ΠΎ Π΄Π΅ΠΉΡΡ‚Π²ΡƒΡŽΡ‰ΠΈΠ΅ напряТСния, Ρ€Π°Π²Π½Ρ‹Π΅ ΠΏΡ€ΠΈΠ»ΠΎΠΆΠ΅Π½Π½Ρ‹ΠΌ усилиям – Π΄Π²Π΅ Ρ€Π°Π·Π½Ρ‹Π΅ Π·Π°Π΄Π°Ρ‡ΠΈ. ΠŸΠ΅Ρ€Π²ΡƒΡŽ Π·Π°Π΄Π°Ρ‡Ρƒ – ΠΎΠΏΡ€Π΅Π΄Π΅Π»Π΅Π½ΠΈΠ΅ усилий – Ρ€Π΅ΡˆΠ°Π΅Ρ‚ ΡΡ‚Ρ€ΠΎΠΈΡ‚Π΅Π»ΡŒΠ½Π°Ρ ΠΌΠ΅Ρ…Π°Π½ΠΈΠΊΠ°, Π²Ρ‚ΠΎΡ€ΡƒΡŽ Π·Π°Π΄Π°Ρ‡Ρƒ – ΠΏΠΎΠ΄Π±ΠΎΡ€ сСчСния – тСория сопротивлСния ΠΌΠ°Ρ‚Π΅Ρ€ΠΈΠ°Π»ΠΎΠ². ΠŸΡ€ΠΈ этом Π² ΡΡ‚Ρ€ΠΎΠΈΡ‚Π΅Π»ΡŒΠ½ΠΎΠΉ ΠΌΠ΅Ρ…Π°Π½ΠΈΠΊΠ΅ рассматриваСтся ΠΏΡ€ΠΈ Ρ€Π΅ΡˆΠ΅Π½ΠΈΠΈ простых Π·Π°Π΄Π°Ρ‡ достаточно часто ΡΡ‚Π΅Ρ€ΠΆΠ΅Π½ΡŒ (для прямолинСйных конструкций), ΠΈΠΌΠ΅ΡŽΡ‰ΠΈΠΉ ΠΎΠΏΡ€Π΅Π΄Π΅Π»Π΅Π½Π½ΡƒΡŽ Π΄Π»ΠΈΠ½Ρƒ l , Π° высота ΠΈ ΡˆΠΈΡ€ΠΈΠ½Π° сСчСния Π½Π΅ ΡƒΡ‡ΠΈΡ‚Ρ‹Π²Π°ΡŽΡ‚ΡΡ, ΠΏΡ€ΠΈ этом считаСтся, Ρ‡Ρ‚ΠΎ ось Ρ… ΠΊΠ°ΠΊ Ρ€Π°Π· ΠΈ ΠΏΡ€ΠΎΡ…ΠΎΠ΄ΠΈΡ‚ Ρ‡Π΅Ρ€Π΅Π· Ρ†Π΅Π½Ρ‚Ρ€Ρ‹ тяТСсти всСх ΠΏΠΎΠΏΠ΅Ρ€Π΅Ρ‡Π½Ρ‹Ρ… сСчСний ΠΈ Ρ‚Π°ΠΊΠΈΠΌ ΠΎΠ±Ρ€Π°Π·ΠΎΠΌ ΠΏΡ€ΠΈ построСнии ΡΠΏΡŽΡ€ (ΠΏΠΎΡ€ΠΎΠΉ достаточно слоТных) Π΄Π»ΠΈΠ½Π° l ΠΊΠ°ΠΊ Ρ€Π°Π· ΠΈ откладываСтся ΠΏΠΎ оси Ρ… , Π° ΠΏΠΎ оси Ρƒ ΠΎΡ‚ΠΊΠ»Π°Π΄Ρ‹Π²Π°ΡŽΡ‚ΡΡ значСния ΡΠΏΡŽΡ€. Π’ Ρ‚ΠΎ ΠΆΠ΅ врСмя тСория сопротивлСния ΠΌΠ°Ρ‚Π΅Ρ€ΠΈΠ°Π»ΠΎΠ² рассматриваСт ΠΈΠΌΠ΅Π½Π½ΠΎ ΠΏΠΎΠΏΠ΅Ρ€Π΅Ρ‡Π½ΠΎΠ΅ сСчСниС, для ΠΊΠΎΡ‚ΠΎΡ€ΠΎΠ³ΠΎ Π²Π°ΠΆΠ½Ρ‹ ΡˆΠΈΡ€ΠΈΠ½Π° ΠΈ высота, Π° Π΄Π»ΠΈΠ½Π° Π½Π΅ учитываСтся. Π‘Π°ΠΌΠΎ собой ΠΏΡ€ΠΈ Ρ€Π΅ΡˆΠ΅Π½ΠΈΠΈ Π·Π°Π΄Π°Ρ‡ Ρ‚Π΅ΠΎΡ€ΠΈΠΈ сопротивлСния ΠΌΠ°Ρ‚Π΅Ρ€ΠΈΠ°Π»ΠΎΠ², Ρ‚Π°ΠΊΠΆΠ΅ ΠΏΠΎΡ€ΠΎΠΉ достаточно слоТных ΠΈΡΠΏΠΎΠ»ΡŒΠ·ΡƒΡŽΡ‚ΡΡ всС Ρ‚Π΅ ΠΆΠ΅ ΠΏΡ€ΠΈΠ²Ρ‹Ρ‡Π½Ρ‹Π΅ оси Ρ… ΠΈ Ρƒ . МнС Ρ‚Π°ΠΊΠΎΠ΅ ΠΏΠΎΠ»ΠΎΠΆΠ΅Π½ΠΈΠ΅ Π΄Π΅Π» каТСтся Π½Π΅ совсСм ΠΏΡ€Π°Π²ΠΈΠ»ΡŒΠ½Ρ‹ΠΌ, Ρ‚Π°ΠΊ ΠΊΠ°ΠΊ Π½Π΅ смотря Π½Π° Ρ€Π°Π·Π½ΠΈΡ†Ρƒ, это всС ΠΆΠ΅ смСТныС Π·Π°Π΄Π°Ρ‡ΠΈ ΠΈ ΠΏΠΎΡ‚ΠΎΠΌΡƒ Π±ΡƒΠ΄Π΅Ρ‚ Π±ΠΎΠ»Π΅Π΅ цСлСсообразным использованиС Π΅Π΄ΠΈΠ½Ρ‹Ρ… осСй для рассчитываСмой конструкции.

Π—Π½Π°Ρ‡Π΅Π½ΠΈΠ΅ полярного ΠΌΠΎΠΌΠ΅Π½Ρ‚Π° ΠΈΠ½Π΅Ρ€Ρ†ΠΈΠΈ Π² ΠΏΡ€ΡΠΌΠΎΡƒΠ³ΠΎΠ»ΡŒΠ½ΠΎΠΉ систСмС ΠΊΠΎΠΎΡ€Π΄ΠΈΠ½Π°Ρ‚ Π±ΡƒΠ΄Π΅Ρ‚:

I Ρ€ = ∫r 2 dF = ∫y 2 dF + ∫z 2 dF (2.3)

Π’Π°ΠΊ ΠΊΠ°ΠΊ Π² ΠΏΡ€ΡΠΌΠΎΡƒΠ³ΠΎΠ»ΡŒΠ½ΠΎΠΉ систСмС ΠΊΠΎΠΎΡ€Π΄ΠΈΠ½Π°Ρ‚ радиус – это Π³ΠΈΠΏΠΎΡ‚Π΅Π½ΡƒΠ·Π° ΠΏΡ€ΡΠΌΠΎΡƒΠ³ΠΎΠ»ΡŒΠ½ΠΎΠ³ΠΎ Ρ‚Ρ€Π΅ΡƒΠ³ΠΎΠ»ΡŒΠ½ΠΈΠΊΠ°, Π° ΠΊΠ°ΠΊ извСстно ΠΊΠ²Π°Π΄Ρ€Π°Ρ‚ Π³ΠΈΠΏΠΎΡ‚Π΅Π½ΡƒΠ·Ρ‹ Ρ€Π°Π²Π΅Π½ суммС ΠΊΠ²Π°Π΄Ρ€Π°Ρ‚ΠΎΠ² ΠΊΠ°Ρ‚Π΅Ρ‚ΠΎΠ². А Π΅Ρ‰Π΅ сущСствуСт понятиС Ρ†Π΅Π½Ρ‚Ρ€ΠΎΠ±Π΅ΠΆΠ½ΠΎΠ³ΠΎ ΠΌΠΎΠΌΠ΅Π½Ρ‚Π° ΠΈΠ½Π΅Ρ€Ρ†ΠΈΠΈ ΠΏΠΎΠΏΠ΅Ρ€Π΅Ρ‡Π½ΠΎΠ³ΠΎ сСчСния:

I xz = ∫xzdF (2.4)

Π‘Ρ€Π΅Π΄ΠΈ осСй ΠΏΡ€ΡΠΌΠΎΡƒΠ³ΠΎΠ»ΡŒΠ½ΠΎΠΉ систСмы ΠΊΠΎΠΎΡ€Π΄ΠΈΠ½Π°Ρ‚, проходящих Ρ‡Π΅Ρ€Π΅Π· Ρ†Π΅Π½Ρ‚Ρ€ тяТСсти ΠΏΠΎΠΏΠ΅Ρ€Π΅Ρ‡Π½ΠΎΠ³ΠΎ сСчСния, Π΅ΡΡ‚ΡŒ Π΄Π²Π΅ Π²Π·Π°ΠΈΠΌΠ½ΠΎ-пСрпСндикулярныС оси, ΠΎΡ‚Π½ΠΎΡΠΈΡ‚Π΅Π»ΡŒΠ½ΠΎ ΠΊΠΎΡ‚ΠΎΡ€Ρ‹Ρ… осСвыС ΠΌΠΎΠΌΠ΅Π½Ρ‚Ρ‹ ΠΈΠ½Π΅Ρ€Ρ†ΠΈΠΈ ΠΏΡ€ΠΈΠ½ΠΈΠΌΠ°ΡŽΡ‚ максимальноС ΠΈ минимальноС Π·Π½Π°Ρ‡Π΅Π½ΠΈΠ΅, ΠΏΡ€ΠΈ этом Ρ†Π΅Π½Ρ‚Ρ€ΠΎΠ±Π΅ΠΆΠ½Ρ‹ΠΉ ΠΌΠΎΠΌΠ΅Π½Ρ‚ ΠΈΠ½Π΅Ρ€Ρ†ΠΈΠΈ сСчСния I zy = 0 . Π’Π°ΠΊΠΈΠ΅ оси Π½Π°Π·Ρ‹Π²Π°ΡŽΡ‚ Π³Π»Π°Π²Π½Ρ‹ΠΌΠΈ Ρ†Π΅Π½Ρ‚Ρ€Π°Π»ΡŒΠ½Ρ‹ΠΌΠΈ осями ΠΏΠΎΠΏΠ΅Ρ€Π΅Ρ‡Π½ΠΎΠ³ΠΎ сСчСния, Π° ΠΌΠΎΠΌΠ΅Π½Ρ‚Ρ‹ ΠΈΠ½Π΅Ρ€Ρ†ΠΈΠΈ ΠΎΡ‚Π½ΠΎΡΠΈΡ‚Π΅Π»ΡŒΠ½ΠΎ Ρ‚Π°ΠΊΠΈΡ… осСй – Π³Π»Π°Π²Π½Ρ‹ΠΌΠΈ Ρ†Π΅Π½Ρ‚Ρ€Π°Π»ΡŒΠ½Ρ‹ΠΌΠΈ ΠΌΠΎΠΌΠ΅Π½Ρ‚Π°ΠΌΠΈ ΠΈΠ½Π΅Ρ€Ρ†ΠΈΠΈ

Когда Π² Ρ‚Π΅ΠΎΡ€ΠΈΠΈ сопротивлСния ΠΌΠ°Ρ‚Π΅Ρ€ΠΈΠ°Π»ΠΎΠ² Ρ€Π΅Ρ‡ΡŒ Π·Π°Ρ…ΠΎΠ΄ΠΈΡ‚ ΠΎ ΠΌΠΎΠΌΠ΅Π½Ρ‚Π°Ρ… ΠΈΠ½Π΅Ρ€Ρ†ΠΈΠΈ, Ρ‚ΠΎ ΠΊΠ°ΠΊ ΠΏΡ€Π°Π²ΠΈΠ»ΠΎ Π² Π²ΠΈΠ΄Ρƒ ΠΈΠΌΠ΅ΡŽΡ‚ΡΡ ΠΈΠΌΠ΅Π½Π½ΠΎ Π³Π»Π°Π²Π½Ρ‹Π΅ Ρ†Π΅Π½Ρ‚Ρ€Π°Π»ΡŒΠ½Ρ‹Π΅ ΠΌΠΎΠΌΠ΅Π½Ρ‚Ρ‹ ΠΈΠ½Π΅Ρ€Ρ†ΠΈΠΈ ΠΏΠΎΠΏΠ΅Ρ€Π΅Ρ‡Π½ΠΎΠ³ΠΎ сСчСния. Для ΠΊΠ²Π°Π΄Ρ€Π°Ρ‚Π½Ρ‹Ρ…, ΠΏΡ€ΡΠΌΠΎΡƒΠ³ΠΎΠ»ΡŒΠ½Ρ‹Ρ…, ΠΊΡ€ΡƒΠ³Π»Ρ‹Ρ… сСчСний Π³Π»Π°Π²Π½Ρ‹Π΅ оси Π±ΡƒΠ΄ΡƒΡ‚ ΡΠΎΠ²ΠΏΠ°Π΄Π°Ρ‚ΡŒ с осями симмСтрии. ΠœΠΎΠΌΠ΅Π½Ρ‚Ρ‹ ΠΈΠ½Π΅Ρ€Ρ†ΠΈΠΈ ΠΏΠΎΠΏΠ΅Ρ€Π΅Ρ‡Π½ΠΎΠ³ΠΎ сСчСния Ρ‚Π°ΠΊΠΆΠ΅ Π½Π°Π·Ρ‹Π²Π°ΡŽΡ‚ гСомСтричСскими ΠΌΠΎΠΌΠ΅Π½Ρ‚Π°ΠΌΠΈ ΠΈΠ½Π΅Ρ€Ρ†ΠΈΠΈ ΠΈΠ»ΠΈ ΠΌΠΎΠΌΠ΅Π½Ρ‚Π°ΠΌΠΈ ΠΈΠ½Π΅Ρ€Ρ†ΠΈΠΈ ΠΏΠ»ΠΎΡ‰Π°Π΄ΠΈ, Π½ΠΎ ΡΡƒΡ‚ΡŒ ΠΎΡ‚ этого Π½Π΅ измСняСтся.

Π’ ΠΏΡ€ΠΈΠ½Ρ†ΠΈΠΏΠ΅ самому ΠΎΠΏΡ€Π΅Π΄Π΅Π»ΡΡ‚ΡŒ значСния Π³Π»Π°Π²Π½Ρ‹Ρ… Ρ†Π΅Π½Ρ‚Ρ€Π°Π»ΡŒΠ½Ρ‹Ρ… ΠΌΠΎΠΌΠ΅Π½Ρ‚ΠΎΠ² ΠΈΠ½Π΅Ρ€Ρ†ΠΈΠΈ для ΠΏΠΎΠΏΠ΅Ρ€Π΅Ρ‡Π½Ρ‹Ρ… сСчСний Π½Π°ΠΈΠ±ΠΎΠ»Π΅Π΅ распространСнных гСомСтричСских Ρ„ΠΎΡ€ΠΌ – ΠΊΠ²Π°Π΄Ρ€Π°Ρ‚Π°, ΠΏΡ€ΡΠΌΠΎΡƒΠ³ΠΎΠ»ΡŒΠ½ΠΈΠΊΠ°, ΠΊΡ€ΡƒΠ³Π°, Ρ‚Ρ€ΡƒΠ±Ρ‹, Ρ‚Ρ€Π΅ΡƒΠ³ΠΎΠ»ΡŒΠ½ΠΈΠΊΠ° ΠΈ Π½Π΅ΠΊΠΎΡ‚ΠΎΡ€Ρ‹Ρ… Π΄Ρ€ΡƒΠ³ΠΈΡ… – большой нСобходимости Π½Π΅Ρ‚. Π’Π°ΠΊΠΈΠ΅ ΠΌΠΎΠΌΠ΅Π½Ρ‚Ρ‹ ΠΈΠ½Π΅Ρ€Ρ†ΠΈΠΈ Π΄Π°Π²Π½ΠΎ ΠΎΠΏΡ€Π΅Π΄Π΅Π»Π΅Π½Ρ‹ ΠΈ ΡˆΠΈΡ€ΠΎΠΊΠΎ извСстны. А ΠΏΡ€ΠΈ расчСтС осСвых ΠΌΠΎΠΌΠ΅Π½Ρ‚ΠΎΠ² ΠΈΠ½Π΅Ρ€Ρ†ΠΈΠΈ для сСчСний слоТной гСомСтричСской Ρ„ΠΎΡ€ΠΌΡ‹ справСдлива Ρ‚Π΅ΠΎΡ€Π΅ΠΌΠ° Π“ΡŽΠΉΠ³Π΅Π½ΡΠ°-Π¨Ρ‚Π΅ΠΉΠ½Π΅Ρ€Π°:

I = I c + r 2 F (2.5)

Ρ‚Π°ΠΊΠΈΠΌ ΠΎΠ±Ρ€Π°Π·ΠΎΠΌ, Ссли извСстны ΠΏΠ»ΠΎΡ‰Π°Π΄ΠΈ ΠΈ Ρ†Π΅Π½Ρ‚Ρ€Ρ‹ тяТСсти простых гСомСтричСских Ρ„ΠΈΠ³ΡƒΡ€, ΡΠΎΡΡ‚Π°Π²Π»ΡΡŽΡ‰ΠΈΡ… слоТноС сСчСниС, Ρ‚ΠΎ ΠΎΠΏΡ€Π΅Π΄Π΅Π»ΠΈΡ‚ΡŒ Π·Π½Π°Ρ‡Π΅Π½ΠΈΠ΅ осСвого ΠΌΠΎΠΌΠ΅Π½Ρ‚Π° ΠΈΠ½Π΅Ρ€Ρ†ΠΈΠΈ всСго сСчСния Π½Π΅ составит Ρ‚Ρ€ΡƒΠ΄Π°. А для Ρ‚ΠΎΠ³ΠΎ, Ρ‡Ρ‚ΠΎΠ±Ρ‹ ΠΎΠΏΡ€Π΅Π΄Π΅Π»ΠΈΡ‚ΡŒ Ρ†Π΅Π½Ρ‚Ρ€ тяТСсти слоТного сСчСния, ΠΈΡΠΏΠΎΠ»ΡŒΠ·ΡƒΡŽΡ‚ΡΡ статичСскиС ΠΌΠΎΠΌΠ΅Π½Ρ‚Ρ‹ ΠΏΠΎΠΏΠ΅Ρ€Π΅Ρ‡Π½ΠΎΠ³ΠΎ сСчСния. Π‘ΠΎΠ»Π΅Π΅ ΠΏΠΎΠ΄Ρ€ΠΎΠ±Π½ΠΎ статичСскиС ΠΌΠΎΠΌΠ΅Π½Ρ‚Ρ‹ Ρ€Π°ΡΡΠΌΠ°Ρ‚Ρ€ΠΈΠ²Π°ΡŽΡ‚ΡΡ Π² Π΄Ρ€ΡƒΠ³ΠΎΠΉ ΡΡ‚Π°Ρ‚ΡŒΠ΅, здСсь лишь добавлю. ЀизичСский смысл статичСского ΠΌΠΎΠΌΠ΅Π½Ρ‚Π° ΡΠ»Π΅Π΄ΡƒΡŽΡ‰ΠΈΠΉ: статичСский ΠΌΠΎΠΌΠ΅Π½Ρ‚ Ρ‚Π΅Π»Π° – это сумма ΠΌΠΎΠΌΠ΅Π½Ρ‚ΠΎΠ² для ΠΌΠ°Ρ‚Π΅Ρ€ΠΈΠ°Π»ΡŒΠ½Ρ‹Ρ… Ρ‚ΠΎΡ‡Π΅ΠΊ, ΡΠΎΡΡ‚Π°Π²Π»ΡΡŽΡ‰ΠΈΡ… Ρ‚Π΅Π»ΠΎ, ΠΎΡ‚Π½ΠΎΡΠΈΡ‚Π΅Π»ΡŒΠ½ΠΎ Π½Π΅ΠΊΠΎΡ‚ΠΎΡ€ΠΎΠΉ Ρ‚ΠΎΡ‡ΠΊΠΈ (полярный статичСский ΠΌΠΎΠΌΠ΅Π½Ρ‚) ΠΈΠ»ΠΈ ΠΎΡ‚Π½ΠΎΡΠΈΡ‚Π΅Π»ΡŒΠ½ΠΎ оси (осСвой статичСский ΠΌΠΎΠΌΠ΅Π½Ρ‚), Π° Ρ‚Π°ΠΊ ΠΊΠ°ΠΊ ΠΌΠΎΠΌΠ΅Π½Ρ‚ – это ΠΏΡ€ΠΎΠΈΠ·Π²Π΅Π΄Π΅Π½ΠΈΠ΅ силы Π½Π° ΠΏΠ»Π΅Ρ‡ΠΎ (1.9), Ρ‚ΠΎ ΠΈ опрСдСляСтся статичСский ΠΌΠΎΠΌΠ΅Π½Ρ‚ Ρ‚Π΅Π»Π° соотвСтствСнно:

S = βˆ‘M = βˆ‘r i m i = ∫rdm (2.6)

ΠΈ Ρ‚ΠΎΠ³Π΄Π° полярный статичСский ΠΌΠΎΠΌΠ΅Π½Ρ‚ ΠΏΠΎΠΏΠ΅Ρ€Π΅Ρ‡Π½ΠΎΠ³ΠΎ сСчСния Π±ΡƒΠ΄Π΅Ρ‚:

S Ρ€ = ∫rdF (2.7)

Как Π²ΠΈΠ΄ΠΈΠΌ, ΠΎΠΏΡ€Π΅Π΄Π΅Π»Π΅Π½ΠΈΠ΅ статичСского ΠΌΠΎΠΌΠ΅Π½Ρ‚Π° сходно с ΠΎΠΏΡ€Π΅Π΄Π΅Π»Π΅Π½ΠΈΠ΅ΠΌ ΠΌΠΎΠΌΠ΅Π½Ρ‚Π° ΠΈΠ½Π΅Ρ€Ρ†ΠΈΠΈ. Но Π΅ΡΡ‚ΡŒ ΠΈ ΠΏΡ€ΠΈΠ½Ρ†ΠΈΠΏΠΈΠ°Π»ΡŒΠ½Π°Ρ Ρ€Π°Π·Π½ΠΈΡ†Π°. БтатичСский ΠΌΠΎΠΌΠ΅Π½Ρ‚ ΠΏΠΎΡ‚ΠΎΠΌΡƒ ΠΈ называСтся статичСским, Ρ‡Ρ‚ΠΎ для Ρ‚Π΅Π»Π°, Π½Π° ΠΊΠΎΡ‚ΠΎΡ€ΠΎΠ΅ дСйствуСт сила тяТСсти, статичСский ΠΌΠΎΠΌΠ΅Π½Ρ‚ Ρ€Π°Π²Π΅Π½ Π½ΡƒΠ»ΡŽ ΠΎΡ‚Π½ΠΎΡΠΈΡ‚Π΅Π»ΡŒΠ½ΠΎ Ρ†Π΅Π½Ρ‚Ρ€Π° тяТСсти. Π”Ρ€ΡƒΠ³ΠΈΠΌΠΈ словами Ρ‚Π°ΠΊΠΎΠ΅ Ρ‚Π΅Π»ΠΎ находится Π² состоянии равновСсия, Ссли ΠΎΠΏΠΎΡ€Π° ΠΏΡ€ΠΈΠ»ΠΎΠΆΠ΅Π½Π° ΠΊ Ρ†Π΅Π½Ρ‚Ρ€Ρƒ тяТСсти Ρ‚Π΅Π»Π°. А согласно ΠΏΠ΅Ρ€Π²ΠΎΠΌΡƒ Π·Π°ΠΊΠΎΠ½Ρƒ ΠΡŒΡŽΡ‚ΠΎΠ½Π° Ρ‚Π°ΠΊΠΎΠ΅ Ρ‚Π΅Π»ΠΎ ΠΈΠ»ΠΈ находится Π² состоянии покоя ΠΈΠ»ΠΈ двиТСтся с постоянной ΡΠΊΠΎΡ€ΠΎΡΡ‚ΡŒΡŽ, Ρ‚.Π΅. ускорСниС = 0. А Π΅Ρ‰Π΅ с чисто матСматичСской Ρ‚ΠΎΡ‡ΠΊΠΈ зрСния статичСский ΠΌΠΎΠΌΠ΅Π½Ρ‚ ΠΌΠΎΠΆΠ΅Ρ‚ Π±Ρ‹Ρ‚ΡŒ Ρ€Π°Π²Π΅Π½ Π½ΡƒΠ»ΡŽ ΠΏΠΎ Ρ‚ΠΎΠΉ простой ΠΏΡ€ΠΈΡ‡ΠΈΠ½Π΅, Ρ‡Ρ‚ΠΎ ΠΏΡ€ΠΈ ΠΎΠΏΡ€Π΅Π΄Π΅Π»Π΅Π½ΠΈΠΈ статичСского ΠΌΠΎΠΌΠ΅Π½Ρ‚Π° Π½Π΅ΠΎΠ±Ρ…ΠΎΠ΄ΠΈΠΌΠΎ ΡƒΡ‡ΠΈΡ‚Ρ‹Π²Π°Ρ‚ΡŒ Π½Π°ΠΏΡ€Π°Π²Π»Π΅Π½ΠΈΠ΅ дСйствия ΠΌΠΎΠΌΠ΅Π½Ρ‚Π°. НапримСр ΠΎΡ‚Π½ΠΎΡΠΈΡ‚Π΅Π»ΡŒΠ½ΠΎ осСй ΠΊΠΎΠΎΡ€Π΄ΠΈΠ½Π°Ρ‚, проходящих Ρ‡Π΅Ρ€Π΅Π· Ρ†Π΅Π½Ρ‚Ρ€ тяТСсти ΠΏΡ€ΡΠΌΠΎΡƒΠ³ΠΎΠ»ΡŒΠ½ΠΈΠΊΠ°, ΠΏΠ»ΠΎΡ‰Π°Π΄ΠΈ Π²Π΅Ρ€Ρ…Π½Π΅ΠΉ части ΠΈ Π½ΠΈΠΆΠ½Π΅ΠΉ части ΠΏΡ€ΡΠΌΠΎΡƒΠ³ΠΎΠ»ΡŒΠ½ΠΈΠΊΠ° Π±ΡƒΠ΄ΡƒΡ‚ ΠΏΠΎΠ»ΠΎΠΆΠΈΡ‚Π΅Π»ΡŒΠ½Ρ‹ΠΌΠΈ Ρ‚Π°ΠΊ ΠΊΠ°ΠΊ ΡΠΈΠΌΠ²ΠΎΠ»ΠΈΠ·ΠΈΡ€ΡƒΡŽΡ‚ силу тяТСсти, Π΄Π΅ΠΉΡΡ‚Π²ΡƒΡŽΡ‰ΡƒΡŽ Π² ΠΎΠ΄Π½ΠΎΠΌ Π½Π°ΠΏΡ€Π°Π²Π»Π΅Π½ΠΈΠΈ. ΠŸΡ€ΠΈ этом расстояниС ΠΎΡ‚ оси Π΄ΠΎ Ρ†Π΅Π½Ρ‚Ρ€Π° тяТСсти ΠΌΠΎΠΆΠ½ΠΎ Ρ€Π°ΡΡΠΌΠ°Ρ‚Ρ€ΠΈΠ²Π°Ρ‚ΡŒ ΠΊΠ°ΠΊ ΠΏΠΎΠ»ΠΎΠΆΠΈΡ‚Π΅Π»ΡŒΠ½ΠΎΠ΅ (условно: ΠΌΠΎΠΌΠ΅Π½Ρ‚ ΠΎΡ‚ силы тяТСсти Π²Π΅Ρ€Ρ…Π½Π΅ΠΉ части ΠΏΡ€ΡΠΌΠΎΡƒΠ³ΠΎΠ»ΡŒΠ½ΠΈΠΊΠ° пытаСтся Π²Ρ€Π°Ρ‰Π°Ρ‚ΡŒ сСчСниС ΠΏΠΎ часовой стрСлкС), Π° Π΄ΠΎ Ρ†Π΅Π½Ρ‚Ρ€Π° тяТСсти Π½ΠΈΠΆΠ½Π΅ΠΉ части – ΠΊΠ°ΠΊ ΠΎΡ‚Ρ€ΠΈΡ†Π°Ρ‚Π΅Π»ΡŒΠ½ΠΎΠ΅ (условно: ΠΌΠΎΠΌΠ΅Π½Ρ‚ ΠΎΡ‚ силы тяТСсти Π½ΠΈΠΆΠ½Π΅ΠΉ части ΠΏΡ€ΡΠΌΠΎΡƒΠ³ΠΎΠ»ΡŒΠ½ΠΈΠΊΠ° пытаСтся Π²Ρ€Π°Ρ‰Π°Ρ‚ΡŒ сСчСниС ΠΏΡ€ΠΎΡ‚ΠΈΠ² часовой стрСлки). А Ρ‚Π°ΠΊ ΠΊΠ°ΠΊ Ρ‚Π°ΠΊΠΈΠ΅ ΠΏΠ»ΠΎΡ‰Π°Π΄ΠΈ числСнно Ρ€Π°Π²Π½Ρ‹ ΠΈ Ρ€Π°Π²Π½Ρ‹ расстояния ΠΎΡ‚ Ρ†Π΅Π½Ρ‚Ρ€ΠΎΠ² тяТСсти Π²Π΅Ρ€Ρ…Π½Π΅ΠΉ части ΠΏΡ€ΡΠΌΠΎΡƒΠ³ΠΎΠ»ΡŒΠ½ΠΈΠΊΠ° ΠΈ Π½ΠΈΠΆΠ½Π΅ΠΉ части ΠΏΡ€ΡΠΌΠΎΡƒΠ³ΠΎΠ»ΡŒΠ½ΠΈΠΊΠ°, Ρ‚ΠΎ сумма Π΄Π΅ΠΉΡΡ‚Π²ΡƒΡŽΡ‰ΠΈΡ… ΠΌΠΎΠΌΠ΅Π½Ρ‚ΠΎΠ² ΠΈ составит искомый 0.

S z = ∫ydF = 0 (2.8)

А Π΅Ρ‰Π΅ этот Π²Π΅Π»ΠΈΠΊΠΈΠΉ ноль позволяСт ΠΎΠΏΡ€Π΅Π΄Π΅Π»ΡΡ‚ΡŒ ΠΎΠΏΠΎΡ€Π½Ρ‹Π΅ Ρ€Π΅Π°ΠΊΡ†ΠΈΠΈ ΡΡ‚Ρ€ΠΎΠΈΡ‚Π΅Π»ΡŒΠ½Ρ‹Ρ… конструкций. Если Ρ€Π°ΡΡΠΌΠ°Ρ‚Ρ€ΠΈΠ²Π°Ρ‚ΡŒ ΡΡ‚Ρ€ΠΎΠΈΡ‚Π΅Π»ΡŒΠ½ΡƒΡŽ ΠΊΠΎΠ½ΡΡ‚Ρ€ΡƒΠΊΡ†ΠΈΡŽ, ΠΊ ΠΊΠΎΡ‚ΠΎΡ€ΠΎΠΉ ΠΏΡ€ΠΈΠ»ΠΎΠΆΠ΅Π½Π° Π½Π°ΠΏΡ€ΠΈΠΌΠ΅Ρ€ сосрСдоточСнная Π½Π°Π³Ρ€ΡƒΠ·ΠΊΠ° Q Π² Π½Π΅ΠΊΠΎΡ‚ΠΎΡ€ΠΎΠΉ Ρ‚ΠΎΡ‡ΠΊΠ΅, Ρ‚ΠΎ Ρ‚Π°ΠΊΡƒΡŽ ΡΡ‚Ρ€ΠΎΠΈΡ‚Π΅Π»ΡŒΠ½ΡƒΡŽ ΠΊΠΎΠ½ΡΡ‚Ρ€ΡƒΠΊΡ†ΠΈΡŽ ΠΌΠΎΠΆΠ½ΠΎ Ρ€Π°ΡΡΠΌΠ°Ρ‚Ρ€ΠΈΠ²Π°Ρ‚ΡŒ, ΠΊΠ°ΠΊ Ρ‚Π΅Π»ΠΎ с Ρ†Π΅Π½Ρ‚Ρ€ΠΎΠΌ тяТСсти Π² Ρ‚ΠΎΡ‡ΠΊΠ΅ прилоТСния силы, Π° ΠΎΠΏΠΎΡ€Π½Ρ‹Π΅ Ρ€Π΅Π°ΠΊΡ†ΠΈΠΈ Π² этом случаС Ρ€Π°ΡΡΠΌΠ°Ρ‚Ρ€ΠΈΠ²Π°ΡŽΡ‚ΡΡ, ΠΊΠ°ΠΊ силы ΠΏΡ€ΠΈΠ»ΠΎΠΆΠ΅Π½Π½Ρ‹Π΅ Π² Ρ‚ΠΎΡ‡ΠΊΠ°Ρ… ΠΎΠΏΠΎΡ€. Π’Π°ΠΊΠΈΠΌ ΠΎΠ±Ρ€Π°Π·ΠΎΠΌ зная Π·Π½Π°Ρ‡Π΅Π½ΠΈΠ΅ сосрСдоточСнной Π½Π°Π³Ρ€ΡƒΠ·ΠΊΠΈ Q ΠΈ расстояния ΠΎΡ‚ Ρ‚ΠΎΡ‡ΠΊΠΈ прилоТСния Π½Π°Π³Ρ€ΡƒΠ·ΠΊΠΈ Π΄ΠΎ ΠΎΠΏΠΎΡ€ ΡΡ‚Ρ€ΠΎΠΈΡ‚Π΅Π»ΡŒΠ½ΠΎΠΉ конструкции, ΠΌΠΎΠΆΠ½ΠΎ ΠΎΠΏΡ€Π΅Π΄Π΅Π»ΠΈΡ‚ΡŒ ΠΎΠΏΠΎΡ€Π½Ρ‹Π΅ Ρ€Π΅Π°ΠΊΡ†ΠΈΠΈ. НапримСр для ΡˆΠ°Ρ€Π½ΠΈΡ€Π½ΠΎ ΠΎΠΏΠ΅Ρ€Ρ‚ΠΎΠΉ Π±Π°Π»ΠΊΠΈ Π½Π° Π΄Π²ΡƒΡ… ΠΎΠΏΠΎΡ€Π°Ρ… Π·Π½Π°Ρ‡Π΅Π½ΠΈΠ΅ ΠΎΠΏΠΎΡ€Π½Ρ‹Ρ… Ρ€Π΅Π°ΠΊΡ†ΠΈΠΉ Π±ΡƒΠ΄Π΅Ρ‚ ΠΏΡ€ΠΎΠΏΠΎΡ€Ρ†ΠΈΠΎΠ½Π°Π»ΡŒΠ½ΠΎ Ρ€Π°ΡΡΡ‚ΠΎΡΠ½ΠΈΡŽ Π΄ΠΎ Ρ‚ΠΎΡ‡ΠΊΠΈ прилоТСния силы, Π° сумма Ρ€Π΅Π°ΠΊΡ†ΠΈΠΉ ΠΎΠΏΠΎΡ€ Π±ΡƒΠ΄Π΅Ρ‚ Ρ€Π°Π²Π½Π° ΠΏΡ€ΠΈΠ»ΠΎΠΆΠ΅Π½Π½ΠΎΠΉ Π½Π°Π³Ρ€ΡƒΠ·ΠΊΠ΅. Но ΠΊΠ°ΠΊ ΠΏΡ€Π°Π²ΠΈΠ»ΠΎ ΠΏΡ€ΠΈ ΠΎΠΏΡ€Π΅Π΄Π΅Π»Π΅Π½ΠΈΠΈ ΠΎΠΏΠΎΡ€Π½Ρ‹Ρ… Ρ€Π΅Π°ΠΊΡ†ΠΈΠΉ ΠΏΠΎΡΡ‚ΡƒΠΏΠ°ΡŽΡ‚ Π΅Ρ‰Π΅ ΠΏΡ€ΠΎΡ‰Π΅: Π·Π° Ρ†Π΅Π½Ρ‚Ρ€ тяТСсти принимаСтся ΠΎΠ΄Π½Π° ΠΈΠ· ΠΎΠΏΠΎΡ€ ΠΈ Ρ‚ΠΎΠ³Π΄Π° сумма ΠΌΠΎΠΌΠ΅Π½Ρ‚ΠΎΠ² ΠΎΡ‚ ΠΏΡ€ΠΈΠ»ΠΎΠΆΠ΅Π½Π½ΠΎΠΉ Π½Π°Π³Ρ€ΡƒΠ·ΠΊΠΈ ΠΈ ΠΎΡ‚ ΠΎΡΡ‚Π°Π»ΡŒΠ½Ρ‹Ρ… ΠΎΠΏΠΎΡ€Π½Ρ‹Ρ… Ρ€Π΅Π°ΠΊΡ†ΠΈΠΉ всС Ρ€Π°Π²Π½ΠΎ Ρ€Π°Π²Π½Π° Π½ΡƒΠ»ΡŽ. Π’ этом случаС ΠΌΠΎΠΌΠ΅Π½Ρ‚ ΠΎΡ‚ ΠΎΠΏΠΎΡ€Π½ΠΎΠΉ Ρ€Π΅Π°ΠΊΡ†ΠΈΠΈ ΠΎΡ‚Π½ΠΎΡΠΈΡ‚Π΅Π»ΡŒΠ½ΠΎ ΠΊΠΎΡ‚ΠΎΡ€ΠΎΠΉ составляСтся ΡƒΡ€Π°Π²Π½Π΅Π½ΠΈΠ΅ ΠΌΠΎΠΌΠ΅Π½Ρ‚ΠΎΠ², Ρ€Π°Π²Π΅Π½ Π½ΡƒΠ»ΡŽ, Ρ‚Π°ΠΊ ΠΊΠ°ΠΊ ΠΏΠ»Π΅Ρ‡ΠΎ дСйствия силы = 0, Π° Π·Π½Π°Ρ‡ΠΈΡ‚ Π² суммС ΠΌΠΎΠΌΠ΅Π½Ρ‚ΠΎΠ² ΠΎΡΡ‚Π°ΡŽΡ‚ΡΡ Ρ‚ΠΎΠ»ΡŒΠΊΠΎ Π΄Π²Π΅ силы: прилоТСнная Π½Π°Π³Ρ€ΡƒΠ·ΠΊΠ° ΠΈ нСизвСстная опорная рСакция (для статичСски ΠΎΠΏΡ€Π΅Π΄Π΅Π»ΠΈΠΌΡ‹Ρ… конструкций).

Π’Π°ΠΊΠΈΠΌ ΠΎΠ±Ρ€Π°Π·ΠΎΠΌ ΠΏΡ€ΠΈΠ½Ρ†ΠΈΠΏΠΈΠ°Π»ΡŒΠ½Π°Ρ Ρ€Π°Π·Π½ΠΈΡ†Π° ΠΌΠ΅ΠΆΠ΄Ρƒ статичСским ΠΌΠΎΠΌΠ΅Π½Ρ‚ΠΎΠΌ ΠΈ ΠΌΠΎΠΌΠ΅Π½Ρ‚ΠΎΠΌ ΠΈΠ½Π΅Ρ€Ρ†ΠΈΠΈ Π² Ρ‚ΠΎΠΌ, Ρ‡Ρ‚ΠΎ статичСский ΠΌΠΎΠΌΠ΅Π½Ρ‚ Ρ…Π°Ρ€Π°ΠΊΡ‚Π΅Ρ€ΠΈΠ·ΡƒΠ΅Ρ‚ сСчСниС, ΠΊΠΎΡ‚ΠΎΡ€ΠΎΠ΅ сила тяТСсти ΠΊΠ°ΠΊ Π±Ρ‹ пытаСтся ΡΠ»ΠΎΠΌΠ°Ρ‚ΡŒ ΠΏΠΎΠΏΠΎΠ»Π°ΠΌ ΠΎΡ‚Π½ΠΎΡΠΈΡ‚Π΅Π»ΡŒΠ½ΠΎ Ρ†Π΅Π½Ρ‚Ρ€Π° тяТСсти ΠΈΠ»ΠΈ оси симмСтрии, Π° ΠΌΠΎΠΌΠ΅Π½Ρ‚ ΠΈΠ½Π΅Ρ€Ρ†ΠΈΠΈ Ρ…Π°Ρ€Π°ΠΊΡ‚Π΅Ρ€ΠΈΠ·ΡƒΠ΅Ρ‚ Ρ‚Π΅Π»ΠΎ, всС ΠΌΠ°Ρ‚Π΅Ρ€ΠΈΠ°Π»ΡŒΠ½Ρ‹Π΅ Ρ‚ΠΎΡ‡ΠΊΠΈ ΠΊΠΎΡ‚ΠΎΡ€ΠΎΠ³ΠΎ ΠΏΠ΅Ρ€Π΅ΠΌΠ΅Ρ‰Π°ΡŽΡ‚ΡΡ (ΠΈΠ»ΠΈ ΠΏΡ‹Ρ‚Π°ΡŽΡ‚ΡΡ ΠΏΠ΅Ρ€Π΅ΠΌΠ΅ΡΡ‚ΠΈΡ‚ΡŒΡΡ Π² ΠΎΠ΄Π½ΠΎΠΌ Π½Π°ΠΏΡ€Π°Π²Π»Π΅Π½ΠΈΠΈ). Π’ΠΎΠ·ΠΌΠΎΠΆΠ½ΠΎ, Π±ΠΎΠ»Π΅Π΅ наглядно ΠΏΡ€Π΅Π΄ΡΡ‚Π°Π²ΠΈΡ‚ΡŒ сСбС эту Ρ€Π°Π·Π½ΠΈΡ†Ρƒ ΠΏΠΎΠΌΠΎΠ³ΡƒΡ‚ ΡΠ»Π΅Π΄ΡƒΡŽΡ‰ΠΈΠ΅ достаточно условныС расчСтныС схСмы для ΠΏΡ€ΡΠΌΠΎΡƒΠ³ΠΎΠ»ΡŒΠ½ΠΎΠ³ΠΎ сСчСния:

Рисунок 2 . Наглядная Ρ€Π°Π·Π½ΠΈΡ†Π° ΠΌΠ΅ΠΆΠ΄Ρƒ статичСским ΠΌΠΎΠΌΠ΅Π½Ρ‚ΠΎΠΌ ΠΈ ΠΌΠΎΠΌΠ΅Π½Ρ‚ΠΎΠΌ ΠΈΠ½Π΅Ρ€Ρ†ΠΈΠΈ.

А Ρ‚Π΅ΠΏΠ΅Ρ€ΡŒ вСрнСмся Π΅Ρ‰Π΅ Ρ€Π°Π· ΠΊ ΠΊΠΈΠ½Π΅ΠΌΠ°Ρ‚ΠΈΠΊΠ΅ двиТСния. Если ΠΏΡ€ΠΎΠ²ΠΎΠ΄ΠΈΡ‚ΡŒ Π°Π½Π°Π»ΠΎΠ³ΠΈΠΈ ΠΌΠ΅ΠΆΠ΄Ρƒ напряТСниями, Π²ΠΎΠ·Π½ΠΈΠΊΠ°ΡŽΡ‰ΠΈΠΌΠΈ Π² ΠΏΠΎΠΏΠ΅Ρ€Π΅Ρ‡Π½Ρ‹Ρ… сСчСниях ΡΡ‚Ρ€ΠΎΠΈΡ‚Π΅Π»ΡŒΠ½Ρ‹Ρ… конструкций, ΠΈ Ρ€Π°Π·Π»ΠΈΡ‡Π½Ρ‹ΠΌΠΈ Π²ΠΈΠ΄Π°ΠΌΠΈ двиТСния, Ρ‚ΠΎ Π² Ρ†Π΅Π½Ρ‚Ρ€Π°Π»ΡŒΠ½ΠΎ растягиваСмых ΠΈ Ρ†Π΅Π½Ρ‚Ρ€Π°Π»ΡŒΠ½ΠΎ сТатых элСмСнтах Π²ΠΎΠ·Π½ΠΈΠΊΠ°ΡŽΡ‚ напряТСния Ρ€Π°Π²Π½ΠΎΠΌΠ΅Ρ€Π½Ρ‹Π΅ ΠΏΠΎ всСй ΠΏΠ»ΠΎΡ‰Π°Π΄ΠΈ сСчСния. Π­Ρ‚ΠΈ напряТСния ΠΌΠΎΠΆΠ½ΠΎ ΡΡ€Π°Π²Π½ΠΈΡ‚ΡŒ с дСйствиСм Π½Π΅ΠΊΠΎΡ‚ΠΎΡ€ΠΎΠΉ силы Π½Π° Ρ‚Π΅Π»ΠΎ, ΠΏΡ€ΠΈ ΠΊΠΎΡ‚ΠΎΡ€ΠΎΠΌ Ρ‚Π΅Π»ΠΎ Π±ΡƒΠ΄Π΅Ρ‚ Π΄Π²ΠΈΠ³Π°Ρ‚ΡŒΡΡ прямолинСйно ΠΈ ΠΏΠΎΡΡ‚ΡƒΠΏΠ°Ρ‚Π΅Π»ΡŒΠ½ΠΎ. А самоС интСрСсноС, это Ρ‚ΠΎ, Ρ‡Ρ‚ΠΎ ΠΏΠΎΠΏΠ΅Ρ€Π΅Ρ‡Π½Ρ‹Π΅ сСчСния Ρ†Π΅Π½Ρ‚Ρ€Π°Π»ΡŒΠ½ΠΎ-растянутых ΠΈΠ»ΠΈ Ρ†Π΅Π½Ρ‚Ρ€Π°Π»ΡŒΠ½ΠΎ сТатых элСмСнтов Π΄Π΅ΠΉΡΡ‚Π²ΠΈΡ‚Π΅Π»ΡŒΠ½ΠΎ двиТутся, Ρ‚Π°ΠΊ ΠΊΠ°ΠΊ Π΄Π΅ΠΉΡΡ‚Π²ΡƒΡŽΡ‰ΠΈΠ΅ напряТСния Π²Ρ‹Π·Ρ‹Π²Π°ΡŽΡ‚ Π΄Π΅Ρ„ΠΎΡ€ΠΌΠ°Ρ†ΠΈΠΈ. И Π²Π΅Π»ΠΈΡ‡ΠΈΠ½Ρƒ Ρ‚Π°ΠΊΠΈΡ… Π΄Π΅Ρ„ΠΎΡ€ΠΌΠ°Ρ†ΠΈΠΉ ΠΌΠΎΠΆΠ½ΠΎ ΠΎΠΏΡ€Π΅Π΄Π΅Π»ΠΈΡ‚ΡŒ для любого ΠΏΠΎΠΏΠ΅Ρ€Π΅Ρ‡Π½ΠΎΠ³ΠΎ сСчСния конструкции. Для этого достаточно Π·Π½Π°Ρ‚ΡŒ Π·Π½Π°Ρ‡Π΅Π½ΠΈΠ΅ Π΄Π΅ΠΉΡΡ‚Π²ΡƒΡŽΡ‰ΠΈΡ… напряТСний, Π΄Π»ΠΈΠ½Ρƒ элСмСнта, ΠΏΠ»ΠΎΡ‰Π°Π΄ΡŒ сСчСния ΠΈ ΠΌΠΎΠ΄ΡƒΠ»ΡŒ упругости ΠΌΠ°Ρ‚Π΅Ρ€ΠΈΠ°Π»Π°, ΠΈΠ· ΠΊΠΎΡ‚ΠΎΡ€ΠΎΠ³ΠΎ ΠΈΠ·Π³ΠΎΡ‚ΠΎΠ²Π»Π΅Π½Π° конструкция.

Π£ ΠΈΠ·Π³ΠΈΠ±Π°Π΅ΠΌΡ‹Ρ… элСмСнтов ΠΏΠΎΠΏΠ΅Ρ€Π΅Ρ‡Π½Ρ‹Π΅ сСчСния Ρ‚Π°ΠΊΠΆΠ΅ Π½Π΅ ΠΎΡΡ‚Π°ΡŽΡ‚ΡΡ Π½Π° мСстС, Π° ΠΏΠ΅Ρ€Π΅ΠΌΠ΅Ρ‰Π°ΡŽΡ‚ΡΡ, ΠΏΡ€ΠΈ этом ΠΏΠ΅Ρ€Π΅ΠΌΠ΅Ρ‰Π΅Π½ΠΈΠ΅ ΠΏΠΎΠΏΠ΅Ρ€Π΅Ρ‡Π½Ρ‹Ρ… сСчСний ΠΈΠ·Π³ΠΈΠ±Π°Π΅ΠΌΡ‹Ρ… элСмСнтов ΠΏΠΎΠ΄ΠΎΠ±Π½ΠΎ Π²Ρ€Π°Ρ‰Π΅Π½ΠΈΡŽ Π½Π΅ΠΊΠΎΠ΅Π³ΠΎ Ρ‚Π΅Π»Π° ΠΎΡ‚Π½ΠΎΡΠΈΡ‚Π΅Π»ΡŒΠ½ΠΎ Π½Π΅ΠΊΠΎΡ‚ΠΎΡ€ΠΎΠΉ оси. Как Π²Ρ‹ ΡƒΠΆΠ΅ Π½Π°Π²Π΅Ρ€Π½ΠΎΠ΅ догадались, ΠΌΠΎΠΌΠ΅Π½Ρ‚ ΠΈΠ½Π΅Ρ€Ρ†ΠΈΠΈ позволяСт ΠΎΠΏΡ€Π΅Π΄Π΅Π»ΠΈΡ‚ΡŒ ΠΈ ΡƒΠ³ΠΎΠ» Π½Π°ΠΊΠ»ΠΎΠ½Π° ΠΏΠΎΠΏΠ΅Ρ€Π΅Ρ‡Π½ΠΎΠ³ΠΎ сСчСния ΠΈ ΠΏΠ΅Ρ€Π΅ΠΌΠ΅Ρ‰Π΅Π½ΠΈΠ΅ Ξ”l для ΠΊΡ€Π°ΠΉΠ½ΠΈΡ… Ρ‚ΠΎΡ‡Π΅ΠΊ сСчСния. Π­Ρ‚ΠΈ ΠΊΡ€Π°ΠΉΠ½ΠΈΠ΅ Ρ‚ΠΎΡ‡ΠΊΠΈ для ΠΏΡ€ΡΠΌΠΎΡƒΠ³ΠΎΠ»ΡŒΠ½ΠΎΠ³ΠΎ сСчСния находятся Π½Π° расстоянии, Ρ€Π°Π²Π½ΠΎΠΌ ΠΏΠΎΠ»ΠΎΠ²ΠΈΠ½Π΅ высоты сСчСния (ΠΏΠΎΡ‡Π΅ΠΌΡƒ – достаточно ΠΏΠΎΠ΄Ρ€ΠΎΠ±Π½ΠΎ описано Π² ΡΡ‚Π°Ρ‚ΡŒΠ΅ “ΠžΡΠ½ΠΎΠ²Ρ‹ сопромата. ΠžΠΏΡ€Π΅Π΄Π΅Π»Π΅Π½Π΅Π½ΠΈΠ΅ ΠΏΡ€ΠΎΠ³ΠΈΠ±Π° “). А это Π² свою ΠΎΡ‡Π΅Ρ€Π΅Π΄ΡŒ позволяСт ΠΎΠΏΡ€Π΅Π΄Π΅Π»ΠΈΡ‚ΡŒ ΠΏΡ€ΠΎΠ³ΠΈΠ± конструкции.

А Π΅Ρ‰Π΅ ΠΌΠΎΠΌΠ΅Π½Ρ‚ ΠΈΠ½Π΅Ρ€Ρ†ΠΈΠΈ позволяСт ΠΎΠΏΡ€Π΅Π΄Π΅Π»ΠΈΡ‚ΡŒ ΠΌΠΎΠΌΠ΅Π½Ρ‚ сопротивлСния сСчСния . Для этого ΠΌΠΎΠΌΠ΅Π½Ρ‚ ΠΈΠ½Π΅Ρ€Ρ†ΠΈΠΈ Π½ΡƒΠΆΠ½ΠΎ просто Ρ€Π°Π·Π΄Π΅Π»ΠΈΡ‚ΡŒ Π½Π° расстояниС ΠΎΡ‚ Ρ†Π΅Π½Ρ‚Ρ€Π° тяТСсти сСчСния Π΄ΠΎ Π½Π°ΠΈΠ±ΠΎΠ»Π΅Π΅ ΡƒΠ΄Π°Π»Π΅Π½Π½ΠΎΠΉ Ρ‚ΠΎΡ‡ΠΊΠΈ сСчСния, для ΠΏΡ€ΡΠΌΠΎΡƒΠ³ΠΎΠ»ΡŒΠ½ΠΎΠ³ΠΎ сСчСния Π½Π° h/2. А Ρ‚Π°ΠΊ ΠΊΠ°ΠΊ исслСдуСмыС сСчСния Π½Π΅ всСгда симмСтричны, Ρ‚ΠΎ Π·Π½Π°Ρ‡Π΅Π½ΠΈΠ΅ ΠΌΠΎΠΌΠ΅Π½Ρ‚Π° сопротивлСния ΠΌΠΎΠΆΠ΅Ρ‚ Π±Ρ‹Ρ‚ΡŒ Ρ€Π°Π·Π½Ρ‹ΠΌ для Ρ€Π°Π·Π½Ρ‹Ρ… частСй сСчСния.

А Π½Π°Ρ‡Π°Π»ΠΎΡΡŒ всС с банального яблока… хотя Π½Π΅Ρ‚, Π½Π°Ρ‡ΠΈΠ½Π°Π»ΠΎΡΡŒ всС со слова.

Π Π΅Π·ΡƒΠ»ΡŒΡ‚Π°Ρ‚ расчСтов зависит Π½Π΅ Ρ‚ΠΎΠ»ΡŒΠΊΠΎ ΠΎΡ‚ ΠΏΠ»ΠΎΡ‰Π°Π΄ΠΈ сСчСния, поэтому ΠΏΡ€ΠΈ Ρ€Π΅ΡˆΠ΅Π½ΠΈΠΈ Π·Π°Π΄Π°Ρ‡ ΠΏΠΎ сопромату Π½Π΅ ΠΎΠ±ΠΎΠΉΡ‚ΠΈΡΡŒ Π±Π΅Π· опрСдСлСния гСомСтричСских характСристик Ρ„ΠΈΠ³ΡƒΡ€ : статичСских, осСвых, полярного ΠΈ Ρ†Π΅Π½Ρ‚Ρ€ΠΎΠ±Π΅ΠΆΠ½ΠΎΠ³ΠΎ ΠΌΠΎΠΌΠ΅Π½Ρ‚ΠΎΠ² ΠΈΠ½Π΅Ρ€Ρ†ΠΈΠΈ. ΠžΠ±ΡΠ·Π°Ρ‚Π΅Π»ΡŒΠ½ΠΎ Π½Π΅ΠΎΠ±Ρ…ΠΎΠ΄ΠΈΠΌΠΎ ΡƒΠΌΠ΅Ρ‚ΡŒ ΠΎΠΏΡ€Π΅Π΄Π΅Π»ΡΡ‚ΡŒ ΠΏΠΎΠ»ΠΎΠΆΠ΅Π½ΠΈΠ΅ Ρ†Π΅Π½Ρ‚Ρ€Π° тяТСсти сСчСния (ΠΎΡ‚ полоТСния Ρ†Π΅Π½Ρ‚Ρ€Π° тяТСсти зависят пСрСчислСнныС гСомСтричСскиС характСристики). К дополнСнию ΠΊ гСомСтричСским характСристикам простых Ρ„ΠΈΠ³ΡƒΡ€: ΠΏΡ€ΡΠΌΠΎΡƒΠ³ΠΎΠ»ΡŒΠ½ΠΈΠΊΠ°, ΠΊΠ²Π°Π΄Ρ€Π°Ρ‚Π°, Ρ€Π°Π²Π½ΠΎΠ±Π΅Π΄Ρ€Π΅Π½Π½ΠΎΠ³ΠΎ ΠΈ ΠΏΡ€ΡΠΌΠΎΡƒΠ³ΠΎΠ»ΡŒΠ½ΠΎΠ³ΠΎ Ρ‚Ρ€Π΅ΡƒΠ³ΠΎΠ»ΡŒΠ½ΠΈΠΊΠΎΠ², ΠΊΡ€ΡƒΠ³Π°, ΠΏΠΎΠ»ΡƒΠΊΡ€ΡƒΠ³Π° . Π£ΠΊΠ°Π·Π°Π½Ρ‹ Ρ†Π΅Π½Ρ‚Ρ€ тяТСсти ΠΈ ΠΏΠΎΠ»ΠΎΠΆΠ΅Π½ΠΈΠ΅ Π³Π»Π°Π²Π½Ρ‹Ρ… Ρ†Π΅Π½Ρ‚Ρ€Π°Π»ΡŒΠ½Ρ‹Ρ… осСй, ΠΈ ΠΎΠΏΡ€Π΅Π΄Π΅Π»Π΅Π½Ρ‹ ΠΎΡ‚Π½ΠΎΡΠΈΡ‚Π΅Π»ΡŒΠ½ΠΎ Π½ΠΈΡ… гСомСтричСскиС характСристики ΠΏΡ€ΠΈ условии, Ρ‡Ρ‚ΠΎ ΠΌΠ°Ρ‚Π΅Ρ€ΠΈΠ°Π» Π±Π°Π»ΠΊΠΈ ΠΎΠ΄Π½ΠΎΡ€ΠΎΠ΄Π½Ρ‹ΠΉ.

ГСомСтричСскиС характСристики ΠΏΡ€ΡΠΌΠΎΡƒΠ³ΠΎΠ»ΡŒΠ½ΠΈΠΊΠ° ΠΈ ΠΊΠ²Π°Π΄Ρ€Π°Ρ‚Π°

ΠžΡΠ΅Π²Ρ‹Π΅ ΠΌΠΎΠΌΠ΅Π½Ρ‚Ρ‹ ΠΈΠ½Π΅Ρ€Ρ†ΠΈΠΈ ΠΏΡ€ΡΠΌΠΎΡƒΠ³ΠΎΠ»ΡŒΠ½ΠΈΠΊΠ° (ΠΊΠ²Π°Π΄Ρ€Π°Ρ‚Π°)

ГСомСтричСскиС характСристики ΠΏΡ€ΡΠΌΠΎΡƒΠ³ΠΎΠ»ΡŒΠ½ΠΎΠ³ΠΎ Ρ‚Ρ€Π΅ΡƒΠ³ΠΎΠ»ΡŒΠ½ΠΈΠΊΠ°

ΠžΡΠ΅Π²Ρ‹Π΅ ΠΌΠΎΠΌΠ΅Π½Ρ‚Ρ‹ ΠΈΠ½Π΅Ρ€Ρ†ΠΈΠΈ ΠΏΡ€ΡΠΌΠΎΡƒΠ³ΠΎΠ»ΡŒΠ½ΠΎΠ³ΠΎ Ρ‚Ρ€Π΅ΡƒΠ³ΠΎΠ»ΡŒΠ½ΠΈΠΊΠ°

ГСомСтричСскиС характСристики Ρ€Π°Π²Π½ΠΎΠ±Π΅Π΄Ρ€Π΅Π½Π½ΠΎΠ³ΠΎ Ρ‚Ρ€Π΅ΡƒΠ³ΠΎΠ»ΡŒΠ½ΠΈΠΊΠ°

ΠžΡΠ΅Π²Ρ‹Π΅ ΠΌΠΎΠΌΠ΅Π½Ρ‚Ρ‹ ΠΈΠ½Π΅Ρ€Ρ†ΠΈΠΈ Ρ€Π°Π²Π½ΠΎΠ±Π΅Π΄Ρ€Π΅Π½Π½ΠΎΠ³ΠΎ Ρ‚Ρ€Π΅ΡƒΠ³ΠΎΠ»ΡŒΠ½ΠΈΠΊΠ°

MYsopromat.ru: ΠœΠΎΠΌΠ΅Π½Ρ‚Ρ‹ ΠΈΠ½Π΅Ρ€Ρ†ΠΈΠΈ ΠΏΡ€ΠΎΡΡ‚Π΅ΠΉΡˆΠΈΡ… Ρ„ΠΈΠ³ΡƒΡ€


Π’ расчСтной ΠΏΡ€Π°ΠΊΡ‚ΠΈΠΊΠ΅ часто Π²ΡΡ‚Ρ€Π΅Ρ‡Π°ΡŽΡ‚ΡΡ сСчСния Π² Π²ΠΈΠ΄Π΅ ΠΏΡ€ΠΎΡΡ‚Π΅ΠΉΡˆΠΈΡ… Ρ„ΠΈΠ³ΡƒΡ€ (ΠΏΡ€ΡΠΌΠΎΡƒΠ³ΠΎΠ»ΡŒΠ½ΠΈΠΊΠΎΠ², ΠΊΡ€ΡƒΠ³ΠΎΠ², Ρ‚Ρ€Π΅ΡƒΠ³ΠΎΠ»ΡŒΠ½ΠΈΠΊΠΎΠ² ΠΈ Ρ‚. ΠΏ.) ΠΈΠ»ΠΈ ΠΈΡ… ΠΊΠΎΠΌΠ±ΠΈΠ½Π°Ρ†ΠΈΠΉ. ΠŸΡ€ΠΈ вычислСнии ΠΌΠΎΠΌΠ΅Π½Ρ‚ΠΎΠ² ΠΈΠ½Π΅Ρ€Ρ†ΠΈΠΈ Ρ‚Π°ΠΊΠΈΡ… Ρ„ΠΈΠ³ΡƒΡ€ ΠΎΠ±Ρ‹Ρ‡Π½ΠΎ ΠΏΠΎΠ»ΡŒΠ·ΡƒΡŽΡ‚ΡΡ Π·Π°Ρ€Π°Π½Π΅Π΅ Π²Ρ‹Π²Π΅Π΄Π΅Π½Π½Ρ‹ΠΌΠΈ расчСтными Ρ„ΠΎΡ€ΠΌΡƒΠ»Π°ΠΌΠΈ. Рассмотрим Π½Π΅ΠΊΠΎΡ‚ΠΎΡ€Ρ‹Π΅ ΠΈΠ· Ρ„ΠΈΠ³ΡƒΡ€.

ΠŸΡ€ΡΠΌΠΎΡƒΠ³ΠΎΠ»ΡŒΠ½ΠΈΠΊ ΠΈ ΠΏΠ°Ρ€Π°Π»Π»Π΅Π»ΠΎΠ³Ρ€Π°ΠΌΠΌ (рис. 6.4). Π’Ρ‹Π΄Π΅Π»ΠΈΠΌ ΡΠ»Π΅ΠΌΠ΅Π½Ρ‚Π°Ρ€Π½ΡƒΡŽ полоску ΠΏΠ»ΠΎΡ‰Π°Π΄ΡŒΡŽ dF=bdy ΠΈ подставим это Π·Π½Π°Ρ‡Π΅Π½ΠΈΠ΅ dF ΠΏΠΎΠ΄ Π·Π½Π°ΠΊ ΠΈΠ½Ρ‚Π΅Π³Ρ€Π°Π»Π° (6.5):

Рис. 6.4

Рис. 6.5

.

Π‘Π»Π΅Π΄ΠΎΠ²Π°Ρ‚Π΅Π»ΡŒΠ½ΠΎ, ΠΌΠΎΠΌΠ΅Π½Ρ‚ ΠΈΠ½Π΅Ρ€Ρ†ΠΈΠΈ ΠΏΡ€ΡΠΌΠΎΡƒΠ³ΠΎΠ»ΡŒΠ½ΠΈΠΊΠ° ΠΈ ΠΏΠ°Ρ€Π°Π»Π»Π΅Π»ΠΎΠ³Ρ€Π°ΠΌΠΌΠ° с основаниСм b ΠΈ высотой h ΠΎΡ‚Π½ΠΎΡΠΈΡ‚Π΅Π»ΡŒΠ½ΠΎ Ρ†Π΅Π½Ρ‚Ρ€Π°Π»ΡŒΠ½ΠΎΠΉ оси, ΠΏΠ°Ρ€Π°Π»Π»Π΅Π»ΡŒΠ½ΠΎΠΉ основанию,

.

(6.16)

ΠœΠΎΠΌΠ΅Π½Ρ‚Ρ‹ ΠΈΠ½Π΅Ρ€Ρ†ΠΈΠΈ этих Ρ„ΠΈΠ³ΡƒΡ€ ΠΎΡ‚Π½ΠΎΡΠΈΡ‚Π΅Π»ΡŒΠ½ΠΎ осСй, проходящих Ρ‡Π΅Ρ€Π΅Π· основаниС, Π½Π°Ρ…ΠΎΠ΄ΠΈΠΌ ΠΏΠΎ Ρ„ΠΎΡ€ΠΌΡƒΠ»Π΅ (6.13):

.

(6.17)

ΠœΠΎΠΌΠ΅Π½Ρ‚Ρ‹ ΠΈΠ½Π΅Ρ€Ρ†ΠΈΠΈ ΠΏΡ€ΡΠΌΠΎΡƒΠ³ΠΎΠ»ΡŒΠ½ΠΈΠΊΠ° ΠΎΡ‚Π½ΠΎΡΠΈΡ‚Π΅Π»ΡŒΠ½ΠΎ осСй yc ΠΈ y Π²Ρ‹Ρ‡ΠΈΡΠ»ΡΡŽΡ‚ΡΡ ΠΏΠΎ Ρ„ΠΎΡ€ΠΌΡƒΠ»Π°ΠΌ (6.16) ΠΈ (6.17), Π³Π΄Π΅ b замСняСтся Π½Π° h, Π° h Π½Π° b:

.

(6.18)

.

(6.19)

Π’Ρ€Π΅ΡƒΠ³ΠΎΠ»ΡŒΠ½ΠΈΠΊ с основаниСм b ΠΈ высотой h (рис. 6.5).

РазобьСм Ρ‚Ρ€Π΅ΡƒΠ³ΠΎΠ»ΡŒΠ½ΠΈΠΊ Π½Π° элСмСнтарныС полоски, ΠΏΠ°Ρ€Π°Π»Π»Π΅Π»ΡŒΠ½Ρ‹Π΅ Π΅Π³ΠΎ основанию. ΠŸΠ»ΠΎΡ‰Π°Π΄ΡŒ Ρ‚Π°ΠΊΠΎΠΉ полоски

.

Π’ΠΎΠ³Π΄Π° ΠΌΠΎΠΌΠ΅Π½Ρ‚ ΠΈΠ½Π΅Ρ€Ρ†ΠΈΠΈ Ρ‚Ρ€Π΅ΡƒΠ³ΠΎΠ»ΡŒΠ½ΠΈΠΊΠ° ΠΎΡ‚Π½ΠΎΡΠΈΡ‚Π΅Π»ΡŒΠ½ΠΎ оси, проходящСй Ρ‡Π΅Ρ€Π΅Π· основаниС,

.

(6.20)

ΠŸΠΎΠ΄ΡΡ‡ΠΈΡ‚Ρ‹Π²Π°Ρ ΠΏΠΎ Ρ„ΠΎΡ€ΠΌΡƒΠ»Π°ΠΌ пСрСноса ΠΌΠΎΠΌΠ΅Π½Ρ‚ ΠΈΠ½Π΅Ρ€Ρ†ΠΈΠΈ Ρ‚Ρ€Π΅ΡƒΠ³ΠΎΠ»ΡŒΠ½ΠΈΠΊΠ° ΠΎΡ‚Π½ΠΎΡΠΈΡ‚Π΅Π»ΡŒΠ½ΠΎ Ρ†Π΅Π½Ρ‚Ρ€Π°Π»ΡŒΠ½ΠΎΠΉ оси, ΠΏΠ°Ρ€Π°Π»Π»Π΅Π»ΡŒΠ½ΠΎΠΉ основанию, ΠΏΠΎΠ»ΡƒΡ‡Π°Π΅ΠΌ

.

(6.21)

ΠšΡ€ΡƒΠ³ ΠΈ ΠΏΠΎΠ»ΡƒΠΊΡ€ΡƒΠ³ Π΄ΠΈΠ°ΠΌΠ΅Ρ‚Ρ€Π° d (рис. 6.6). ΠŸΠΎΠ΄ΡΡ‡ΠΈΡ‚Ρ‹Π²Π°Π΅ΠΌ сначала полярный ΠΌΠΎΠΌΠ΅Π½Ρ‚ ΠΈΠ½Π΅Ρ€Ρ†ΠΈΠΈ ΠΊΡ€ΡƒΠ³Π°. Для этого Π²Ρ‹Π΄Π΅Π»ΠΈΠΌ Π² сСчСнии окруТностями радиуса ρ ΠΈ ρ+dρ элСмСнтарноС ΠΊΠΎΠ»ΡŒΡ†ΠΎ ΠΏΠ»ΠΎΡ‰Π°Π΄ΡŒΡŽ dF=2πρdρ ΠΈ вычислим Iy ΠΏΠΎ Ρ„ΠΎΡ€ΠΌΡƒΠ»Π΅ (6.7):

.

(6.22)

Рис. 6.6.

ΠžΠ±Ρ‹Ρ‡Π½ΠΎ Ρ€Π°Π·ΠΌΠ΅Ρ€Ρ‹ ΠΊΡ€ΡƒΠ³Π»ΠΎΠ³ΠΎ сСчСния Π²Ρ‹Ρ€Π°ΠΆΠ°ΡŽΡ‚ Ρ‡Π΅Ρ€Π΅Π· Π΄ΠΈΠ°ΠΌΠ΅Ρ‚Ρ€ d ΠΈ ΠΏΠΎΠ΄ΡΡ‡ΠΈΡ‚Ρ‹Π²Π°ΡŽΡ‚ Ip ΠΏΠΎ Ρ„ΠΎΡ€ΠΌΡƒΠ»Π΅

.

(6.23)

ΠžΡΠ΅Π²Ρ‹Π΅ ΠΌΠΎΠΌΠ΅Π½Ρ‚Ρ‹ ΠΈΠ½Π΅Ρ€Ρ†ΠΈΠΈ ΠΊΡ€ΡƒΠ³Π° Π½Π°ΠΉΠ΄Π΅ΠΌ с ΠΏΠΎΠΌΠΎΡ‰ΡŒΡŽ ΡΠΎΠΎΡ‚Π½ΠΎΡˆΠ΅Π½ΠΈΡ (6.8). ЗамСчая, Ρ‡Ρ‚ΠΎ Π² силу симмСтрии ΠΊΡ€ΡƒΠ³Π° Iz=Iy, ΠΏΠΎΠ»ΡƒΡ‡Π°Π΅ΠΌ для осСвых ΠΌΠΎΠΌΠ΅Π½Ρ‚ΠΎΠ² ΠΈΠ½Π΅Ρ€Ρ†ΠΈΠΈ ΠΊΡ€ΡƒΠ³Π° Π²Ρ‹Ρ€Π°ΠΆΠ΅Π½ΠΈΠ΅

.

(6.24)

Π¦Π΅Π½Ρ‚Ρ€Π°Π»ΡŒΠ½Ρ‹Π΅ оси y ΠΈ z дСлят ΠΊΡ€ΡƒΠ³ Π½Π° Ρ‡Π΅Ρ‚Ρ‹Ρ€Π΅ ΡΠΎΠ²Π΅Ρ€ΡˆΠ΅Π½Π½ΠΎ ΠΎΠ΄ΠΈΠ½Π°ΠΊΠΎΠ²Ρ‹Π΅ части с Ρ€Π°Π²Π½Ρ‹ΠΌΠΈ ΠΌΠΎΠΌΠ΅Π½Ρ‚Π°ΠΌΠΈ ΠΈΠ½Π΅Ρ€Ρ†ΠΈΠΈ ΠΎΡ‚Π½ΠΎΡΠΈΡ‚Π΅Π»ΡŒΠ½ΠΎ этих осСй. Π‘Π»Π΅Π΄ΠΎΠ²Π°Ρ‚Π΅Π»ΡŒΠ½ΠΎ, ΠΌΠΎΠΌΠ΅Π½Ρ‚Ρ‹ ΠΈΠ½Π΅Ρ€Ρ†ΠΈΠΈ ΠΊΡ€ΡƒΠ³Π° ΠΈ ΠΏΠΎΠ»ΡƒΠΊΡ€ΡƒΠ³Π° ΠΎΡ‚Π½ΠΎΡΠΈΡ‚Π΅Π»ΡŒΠ½ΠΎ осСй y ΠΈ z Π΄ΠΎΠ»ΠΆΠ½Ρ‹ Π±Ρ‹Ρ‚ΡŒ Ρ€Π°Π²Π½Ρ‹ соотвСтствСнно ΡƒΡ‡Π΅Ρ‚Π²Π΅Ρ€Π΅Π½Π½Ρ‹ΠΌ ΠΈ ΡƒΠ΄Π²ΠΎΠ΅Π½Π½Ρ‹ΠΌ ΠΌΠΎΠΌΠ΅Π½Ρ‚Π°ΠΌ ΠΈΠ½Π΅Ρ€Ρ†ΠΈΠΈ ΠΎΡ‚Π½ΠΎΡΠΈΡ‚Π΅Π»ΡŒΠ½ΠΎ Ρ‚Π΅Ρ… ΠΆΠ΅ осСй ΠΎΠ΄Π½ΠΎΠΉ Ρ‡Π΅Ρ‚Π²Π΅Ρ€Ρ‚ΠΈ ΠΊΡ€ΡƒΠ³Π°. Из сказанного слСдуСт, Ρ‡Ρ‚ΠΎ ΠΌΠΎΠΌΠ΅Π½Ρ‚Ρ‹ ΠΈΠ½Π΅Ρ€Ρ†ΠΈΠΈ ΠΏΠΎΠ»ΡƒΠΊΡ€ΡƒΠ³Π° ΠΎΡ‚Π½ΠΎΡΠΈΡ‚Π΅Π»ΡŒΠ½ΠΎ оси симмСтрии y ΠΈ оси z, проходящСй Ρ‡Π΅Ρ€Π΅Π· Π΅Π³ΠΎ основаниС (рис. 6.2), Π±ΡƒΠ΄ΡƒΡ‚ ΠΎΠ΄ΠΈΠ½Π°ΠΊΠΎΠ²Ρ‹ ΠΈ Ρ€Π°Π²Π½Ρ‹ ΠΏΠΎΠ»ΠΎΠ²ΠΈΠ½Π΅ ΠΌΠΎΠΌΠ΅Π½Ρ‚Π° ΠΈΠ½Π΅Ρ€Ρ†ΠΈΠΈ ΠΊΡ€ΡƒΠ³Π°,

,

(6.25)

Π° ΠΌΠΎΠΌΠ΅Π½Ρ‚Ρ‹ ΠΈΠ½Π΅Ρ€Ρ†ΠΈΠΈ Ρ‡Π΅Ρ‚Π²Π΅Ρ€Ρ‚ΠΈ ΠΊΡ€ΡƒΠ³Π°

.

(6.26)

Β Β Β Β 

Π“Ρ€Π°Ρ„ΠΈΠΊ зависимости ΠΌΠΎΠΌΠ΅Π½Ρ‚Π° ΠΈΠ½Π΅Ρ€Ρ†ΠΈΠΈ ΠΎΡ‚ ΠΊΠ²Π°Π΄Ρ€Π°Ρ‚Π° расстояний

ΠšΠ°Ρ„Π΅Π΄Ρ€Π° ΠΎΠ±Ρ‰Π΅ΠΉ ΠΈ тСхничСской Ρ„ΠΈΠ·ΠΈΠΊΠΈ

ΠžΡ‚Ρ‡Π΅Ρ‚ ΠΏΠΎ Π»Π°Π±ΠΎΡ€Π°Ρ‚ΠΎΡ€Π½ΠΎΠΉ Ρ€Π°Π±ΠΎΡ‚Π΅ β„–6.

По дисциплинС Π€ΠΈΠ·ΠΈΠΊΠ°

Π’Π΅ΠΌΠ°: ΠžΠΏΡ€Π΅Π΄Π΅Π»Π΅Π½ΠΈΠ΅ ΠΌΠΎΠΌΠ΅Π½Ρ‚Π° ΠΈΠ½Π΅Ρ€Ρ†ΠΈΠΈ с ΠΏΠΎΠΌΠΎΡ‰ΡŒΡŽ маятника ΠžΠ±Π΅Ρ€ΠΊΠ΅ΠΊΠ°

Автор: студСнт Π³Ρ€. Π“Π‘-16-1 /Π“Π°Π»ΠΎΠΌΠ·ΠΈΠΊ Π’.А./

(подпись) (Ѐ.И.О.)

ΠŸΠ ΠžΠ’Π•Π Π˜Π›Π΄ΠΎΡ†Π΅Π½Ρ‚ _____________ /Π€ΠΈΡ†Π°ΠΊ Π’.Π’./

(Π΄ΠΎΠ»ΠΆΠ½ΠΎΡΡ‚ΡŒ) (подпись)(Π€.И.О.)

Π‘Π°Π½ΠΊΡ‚-ΠŸΠ΅Ρ‚Π΅Ρ€Π±ΡƒΡ€Π³

ЦСль Ρ€Π°Π±ΠΎΡ‚Ρ‹ – ΠΈΡΡΠ»Π΅Π΄ΠΎΠ²Π°Ρ‚ΡŒ Π·Π°Π²ΠΈΡΠΈΠΌΠΎΡΡ‚ΡŒ ΠΌΠΎΠΌΠ΅Π½Ρ‚Π° ΠΈΠ½Π΅Ρ€Ρ†ΠΈΠΈ крСстовины с Π½Π°Π΄Π΅Ρ‚Ρ‹ΠΌΠΈ Π½Π° Π½Π΅Π΅ Π³Ρ€ΡƒΠ·ΠΈΠΊΠ°ΠΌΠΈ ΠΎΡ‚ распрСдСлСния массы ΠΎΡ‚Π½ΠΎΡΠΈΡ‚Π΅Π»ΡŒΠ½ΠΎ оси вращСния, проходящСй Ρ‡Π΅Ρ€Π΅Π· Ρ†Π΅Π½Ρ‚Ρ€ масс крСстовины.

Π―Π²Π»Π΅Π½ΠΈΠ΅ ΠΈΠ·ΡƒΡ‡Π΅Π½Π½ΠΎΠ΅ Π² процСссС Ρ€Π°Π±ΠΎΡ‚Π΅:

Π’Ρ€Π°Ρ‰Π°Ρ‚Π΅Π»ΡŒΠ½Ρ‹ΠΌ Π΄Π²ΠΈΠΆΠ΅Π½ΠΈΠ΅ΠΌ Ρ‚Π²Π΅Ρ€Π΄ΠΎΠ³ΠΎ Ρ‚Π΅Π»Π° – явлСниС(((Π€ΠΈΡ†Π°ΠΊ сказал, Ρ‡Ρ‚ΠΎ Π±Ρ€Π΅Π΄))), ΠΊΠΎΠ³Π΄Π° ΠΏΡ€ΠΈ Π²Ρ€Π°Ρ‰Π΅Π½ΠΈΠΈ Ρ‚Π²Π΅Ρ€Π΄ΠΎΠ³ΠΎ Ρ‚Π΅Π»Π° всС Π΅Π³ΠΎ Ρ‚ΠΎΡ‡ΠΊΠΈ двиТутся ΠΏΠΎ окруТностям, Ρ†Π΅Π½Ρ‚Ρ€Ρ‹ ΠΊΠΎΡ‚ΠΎΡ€Ρ‹Ρ… Π»Π΅ΠΆΠ°Ρ‚ Π½Π° ΠΎΠ΄Π½ΠΎΠΉ прямой, Π½Π°Π·Ρ‹Π²Π°Π΅ΠΌΠΎΠΉ осью вращСния.

ΠžΡΠ½ΠΎΠ²Π½Ρ‹Π΅ опрСдСлСния физичСских Π²Π΅Π»ΠΈΡ‡ΠΈΠ½, явлСний, процСссов:

ΠœΠΎΠΌΠ΅Π½Ρ‚ ΠΈΠ½Π΅Ρ€Ρ†ΠΈΠΈ тСлаявляСтся ΠΌΠ΅Ρ€ΠΎΠΉ инСртности Ρ‚Π΅Π»Π° ΠΏΡ€ΠΈ Π²Ρ€Π°Ρ‰Π°Ρ‚Π΅Π»ΡŒΠ½ΠΎΠΌ Π΄Π²ΠΈΠΆΠ΅Π½ΠΈΠΈ. ΠœΠΎΠΌΠ΅Π½Ρ‚ ΠΈΠ½Π΅Ρ€Ρ†ΠΈΠΈ Ρ‚Π΅Π»Π° зависит ΠΎΡ‚ Ρ€Π°Π·ΠΌΠ΅Ρ€ΠΎΠ² ΠΈ Ρ„ΠΎΡ€ΠΌΡ‹ Ρ‚Π΅Π» ΠΈ ΠΎΡ‚ распрСдСлСния массы Ρ‚Π΅Π»Π° ΠΎΡ‚Π½ΠΎΡΠΈΡ‚Π΅Π»ΡŒΠ½ΠΎ оси вращСния.

ΠœΠΎΠΌΠ΅Π½Ρ‚ΠΎΠΌ ΠΈΠ½Π΅Ρ€Ρ†ΠΈΠΈ ΠΌΠ°Ρ‚Π΅Ρ€ΠΈΠ°Π»ΡŒΠ½ΠΎΠΉ Ρ‚ΠΎΡ‡ΠΊΠΈ ΠΎΡ‚Π½ΠΎΡΠΈΡ‚Π΅Π»ΡŒΠ½ΠΎ Π½Π΅ΠΊΠΎΡ‚ΠΎΡ€ΠΎΠΉ оси называСтся Π²Π΅Π»ΠΈΡ‡ΠΈΠ½Π° J,равная ΠΏΡ€ΠΎΠΈΠ·Π²Π΅Π΄Π΅Π½ΠΈΡŽ массmΠΌΠ°Ρ‚Π΅Ρ€ΠΈΠ°Π»ΡŒΠ½ΠΎΠΉ Ρ‚ΠΎΡ‡ΠΊΠΈ Π½Π° ΠΊΠ²Π°Π΄Ρ€Π°Ρ‚Ρ‹ Π΅Ρ‘ расстояния ΠΎΡ‚ оси. ,Π³Π΄Π΅ m-масса ΠΌΠ°Ρ‚Π΅Ρ€ΠΈΠ°Π»ΡŒΠ½ΠΎΠΉ Ρ‚ΠΎΡ‡ΠΊΠΈ, r-расстояниС ΠΎΡ‚ ΠΌΠ°Ρ‚Π΅Ρ€ΠΈΠ°Π»ΡŒΠ½ΠΎΠΉ Ρ‚ΠΎΡ‡ΠΊΠΈ Π΄ΠΎ оси.

ΠœΠΎΠΌΠ΅Π½Ρ‚ ΠΈΠ½Π΅Ρ€Ρ†ΠΈΠΈ Ρ‚Π²Π΅Ρ€Π΄ΠΎΠ³ΠΎ Ρ‚Π΅Π»Π° (сплошного)называСтся Π²Π΅Π»ΠΈΡ‡ΠΈΠ½Π°J, равная ΠΏΡ€ΠΎΠΈΠ·Π²Π΅Π΄Π΅Π½ΠΈΡŽΠΌΠ°ΡΡΡ‹Ρ‚Π΅Π»Π°-m Π½Π° радиус-R ΠΎΡ‚Π½ΠΎΡΠΈΡ‚Π΅Π»ΡŒΠ½ΠΎ оси, проходящСй Ρ‡Π΅Ρ€Π΅Π· Ρ†Π΅Π½Ρ‚Ρ€Ρ‹ оснований.

ΠœΠΎΠΌΠ΅Π½Ρ‚ΠΎΠΌ ΠΈΠ½Π΅Ρ€Ρ†ΠΈΠΈ систСмы ΠΌΠ°Ρ‚Π΅Ρ€ΠΈΠ°Π»ΡŒΠ½ΠΎΠΉ Ρ‚ΠΎΡ‡ΠΊΠΈ ΠΎΡ‚Π½ΠΎΡΠΈΡ‚Π΅Π»ΡŒΠ½ΠΎ осиназываСтся Π²Π΅Π»ΠΈΡ‡ΠΈΠ½Π° J,равная суммС ΠΏΡ€ΠΎΠΈΠ·Π²Π΅Π΄Π΅Π½ΠΈΠΉ массвсСх ΠΌΠ°Ρ‚Π΅Ρ€ΠΈΠ°Π»ΡŒΠ½Ρ‹Ρ… Ρ‚ΠΎΡ‡Π΅ΠΊ, ΠΎΠ±Ρ€Π°Π·ΡƒΡŽΡ‰ΠΈΡ…ΠΌΠ΅Ρ…Π°Π½ΠΈΡ‡Π΅ΡΠΊΡƒΡŽΡΠΈΡΡ‚Π΅ΠΌΡƒ, Π½Π° ΠΊΠ²Π°Π΄Ρ€Π°Ρ‚Ρ‹ ΠΈΡ… расстояний ΠΎΡ‚ Π΄Π°Π½Π½ΠΎΠΉ оси.

***Π­Ρ‚Ρƒ страницу Π²Ρ‹ΡƒΡ‡ΠΈΡ‚ΡŒ(Π΅Ρ‰Π΅ ΠΎΠ½ спросил ΠΏΡ€ΠΎ основноС ΡƒΡ€Π°Π²Π½Π΅Π½ΠΈΠ΅ Π²Ρ€Π°Ρ‰Π°Ρ‚Π΅Π»ΡŒΠ½ΠΎΠ³ΠΎ двиТСния)

Π—Π°ΠΊΠΎΠ½ ΠΈ ΡΠΎΠΎΡ‚Π½ΠΎΡˆΠ΅Π½ΠΈΡ

1) ОсновноС ΡƒΡ€Π°Π²Π½Π΅Π½ΠΈΠ΅ Π΄ΠΈΠ½Π°ΠΌΠΈΠΊΠ° Π²Ρ€Π°Ρ‰Π°Ρ‚Π΅Π»ΡŒΠ½ΠΎΠ³ΠΎ Π΄Π²ΠΈΠΆΠ΅Π½ΠΈΠ΅ Ρ‚Π²Π΅Ρ€Π΄ΠΎΠ³ΠΎ Ρ‚Π΅Π»Π°:

Π³Π΄Π΅: – суммарный ΠΌΠΎΠΌΠ΅Π½Ρ‚ Π²Π½Π΅ΡˆΠ½ΠΈΡ… сил; J -ΠΌΠΎΠΌΠ΅Π½Ρ‚ ΠΈΠ½Π΅Ρ€Ρ†ΠΈΠΈ Ρ‚Π΅Π»Π° ΠΎΡ‚Π½ΠΎΡΠΈΡ‚Π΅Π»ΡŒΠ½ΠΎ Ρ‚ΠΎΠΉ ΠΆΠ΅ оси; -ΡƒΠ³Π»ΠΎΠ²ΠΎΠ΅ ускорСниС

[M]=Н*м

,

2) Π—Π°ΠΊΠΎΠ½ ΠΡŒΡŽΡ‚ΠΎΠ½Π°:

; m=const. Π³Π΄Π΅ a – ускорСниС Ρ‚Π΅Π»Π°, m – масса Ρ‚Π΅Π»Π°. F – сила натяТСния

[M]=ΠΊΠ³ [F]=H [a]=

-ΠΈΠ·ΠΌΠ΅Π½Π΅Π½ΠΈΠ΅ ΠΈΠΌΠΏΡƒΠ»ΡŒΡΠ° Ρ‚Π΅Π»Π°

F-сила натяТСния

[P]=Н*с

[F]=Н

ВСорСтичСскиС оТидания. Π Π΅Π·ΡƒΠ»ΡŒΡ‚Π°Ρ‚.

1. РасчётныС Ρ„ΠΎΡ€ΠΌΡƒΠ»Ρ‹:

. Π³Π΄Π΅ – радиус шкива, m- масса Π³Ρ€ΡƒΠ·Π°, h – ΠΏΡƒΡ‚ΡŒ, ΠΏΡ€ΠΎΠΉΠ΄Π΅Π½Π½Ρ‹ΠΉ Π³Ρ€ΡƒΠ·ΠΎΠΌ

Π³Π΄Π΅ J0 – ΠΌΠΎΠΌΠ΅Π½Ρ‚ ΠΈΠ½Π΅Ρ€Ρ†ΠΈΠΈ Ρ‚Π΅Π»Π° ΠΏΡ€ΠΈ r = 0.

Π˜ΡΡ…ΠΎΠ΄Π½Ρ‹Π΅ Π΄Π°Π½Π½Ρ‹Π΅:h=0,43ΠΌ

=0,042ΠΌ; m=0,083ΠΊΠ³

Π€ΠΎΡ€ΠΌΡƒΠ»Ρ‹ ΠΏΠΎΠ³Ρ€Π΅ΡˆΠ½ΠΎΡΡ‚ΠΈ: .

Π’Π°Π±Π»ΠΈΡ†Π° 1(РасчСт ΠΌΠΎΠΌΠ΅Π½Ρ‚Π° ΠΈΠ½Π΅Ρ€Ρ†ΠΈΠΈ Ρ‚Π΅Π»Π°)

β„– r, ΠΌ t, c , c Jэ ΠΊΠ³*ΠΌ2 JΡ€ ΠΊΠ³*ΠΌ2
0,25 8.633 8.632 0.127 0,124235
8.638
8.625
0,22 7.836 7.706 0,0985 0,096176
7.700
7.581
0,19 6.144 6.176 0,0632 0,071699
6.010
6.375
0,16 5.066 5.092 0.0429 0,050804
5.084
5.126
0,13 4.068 4.05 0,0271 0,033491
4.048
4,033
0,10 3.541 3,444 0,0195 0,01976
3,405
3,386
0,07 2.859 2.809 0,0129 0,009611
2.770
2.798

Π’Π°Π±Π»ΠΈΡ†Π° 2(ΠžΠ±Ρ€Π°Π±ΠΎΡ‚ΠΊΠ° Ρ€Π΅Π·ΡƒΠ»ΡŒΡ‚Π°Ρ‚ΠΎΠ² экспСримСнта)

β„– r, ΠΌ xi Ji xiJi
0,25 0,0625 0,12671 0,00391 0,00792
0,22 0,0484 0,101 0,00234 0,004885
0,19 0,0361 0,06479 0,0013 0,00233
0,16 0,0256 0,0440 0,0007 0,00113
0,13 0,0169 0,0278 0,0003 0,00046
0,1 0,01 0,02004 0,0001 0,0002
0,07 0,0049 0,01328 0,00002 0,00007
Β  0,0292 0,0568 0,00123 0,00243

ΠŸΡ€ΠΈΠΌΠ΅Ρ€Ρ‹ расчётов:

m – масса Π³Ρ€ΡƒΠ·Π°, ΠΊΠ³;

r0 – радиус шкива, ΠΌ;

g – ускорСниС свободного падСния;

t – врСмя двиТСния Π³Ρ€ΡƒΠ·Π°;

h – ΠΏΡƒΡ‚ΡŒ, ΠΏΡ€ΠΎΠΉΠ΄Π΅Π½Π½Ρ‹ΠΉ Π³Ρ€ΡƒΠ·ΠΎΠΌ

Π‘Ρ€Π΅Π΄Π½Π΅Π΅ ΠΊΠ²Π°Π΄Ρ€Π°Ρ‚ΠΈΡ‡Π½ΠΎΠ΅ ΠΎΡ‚ΠΊΠ»ΠΎΠ½Π΅Π½ΠΈΠ΅:

ΠžΠΊΠΎΠ½Ρ‡Π°Ρ‚Π΅Π»ΡŒΠ½Ρ‹ΠΉ Ρ€Π΅Π·ΡƒΠ»ΡŒΡ‚Π°Ρ‚:

Π“Ρ€Π°Ρ„ΠΈΠΊ зависимости ΠΌΠΎΠΌΠ΅Π½Ρ‚Π° ΠΈΠ½Π΅Ρ€Ρ†ΠΈΠΈ ΠΎΡ‚ ΠΊΠ²Π°Π΄Ρ€Π°Ρ‚Π° расстояний.

Π’Ρ‹Π²ΠΎΠ΄: ПослС провСдСния ΠΎΠΏΡ‹Ρ‚ΠΎΠ² ΠΈ произвСдСния расчётов Π±Ρ‹Π» ΠΎΠΏΡ€Π΅Π΄Π΅Π»Ρ‘Π½ ΠΌΠΎΠΌΠ΅Π½Ρ‚ ΠΈΠ½Π΅Ρ€Ρ†ΠΈΠΈ с ΠΏΠΎΠΌΠΎΡ‰ΡŒΡŽ ΠΌ


аятника ΠžΠ±Π΅Ρ€Π±Π΅ΠΊΠ°. ΠŸΡ€ΠΈ ΡƒΠΌΠ΅Π½ΡŒΡˆΠ΅Π½ΠΈΠΈ радиуса вращСния врСмя вращСния ΡƒΠΌΠ΅Π½ΡŒΡˆΠ°Π΅Ρ‚ΡΡ. Π§Π΅ΠΌ большС радиус шкива, Ρ‚Π΅ΠΌ большС ΠΌΠΎΠΌΠ΅Π½Ρ‚ ΠΈΠ½Π΅Ρ€Ρ†ΠΈΠΈ систСмы

ΠŸΡ€ΠΎΡ†Π΅Π½Ρ‚ ΠΏΠΎΠ³Ρ€Π΅ΡˆΠ½ΠΎΡΡ‚ΠΈ:

ВычислСниС ΠΌΠΎΠΌΠ΅Π½Ρ‚Π° ΠΈΠ½Π΅Ρ€Ρ†ΠΈΠΈ ΠΊΠ²Π°Π΄Ρ€Π°Ρ‚Π½ΠΎΠΉ пластины

АвторскиС ΠΏΡ€Π°Π²Π° Β© Майкл Π ΠΈΡ‡ΠΌΠΎΠ½Π΄. Π­Ρ‚Π° Ρ€Π°Π±ΠΎΡ‚Π° находится ΠΏΠΎΠ΄ Π»ΠΈΡ†Π΅Π½Π·ΠΈΠ΅ΠΉ Creative Commons License.

Π’Π΅ΠΎΡ€Π΅ΠΌΠ° ΠΎ ΠΏΠ°Ρ€Π°Π»Π»Π΅Π»ΡŒΠ½Ρ‹Ρ… осях позволяСт ΠΈΡΠΏΠΎΠ»ΡŒΠ·ΠΎΠ²Π°Ρ‚ΡŒ ΠŸΠ ΠžΠ‘Π’Π«Π• части. ΡΠΎΠ·Π΄Π°Ρ‚ΡŒ ΠΌΠΎΠΌΠ΅Π½Ρ‚ ΠΈΠ½Π΅Ρ€Ρ†ΠΈΠΈ Π‘Π›ΠžΠ–ΠΠžΠ“Πž Ρ‚Π΅Π»Π°. Π”Π°Π²Π°ΠΉΡ‚Π΅ сдСлаСм ΠΏΡ€ΠΈΠΌΠ΅Ρ€, Ρ‡Ρ‚ΠΎΠ±Ρ‹ ΡƒΠ²ΠΈΠ΄Π΅Ρ‚ΡŒ, ΠΊΠ°ΠΊ это Ρ€Π°Π±ΠΎΡ‚Π°Π΅Ρ‚.

НачнитС с ΠΎΡ‡Π΅Π½ΡŒ простого ΠΏΡ€Π΅Π΄ΠΌΠ΅Ρ‚Π°: Π΄Π»ΠΈΠ½Π½ΠΎΠ³ΠΎ Ρ‚ΠΎΠ½ΠΊΠΎΠ³ΠΎ стСрТня. массой ΠΌ ΠΈ Π΄Π»ΠΈΠ½ΠΎΠΉ L , закрутился Π²ΠΎΠΊΡ€ΡƒΠ³ Π΅Π³ΠΎ Ρ†Π΅Π½Ρ‚Ρ€Π°.

Какой ΠΌΠΎΠΌΠ΅Π½Ρ‚ ΠΈΠ½Π΅Ρ€Ρ†ΠΈΠΈ этого стСрТня?





 

Π’Π΅ΠΏΠ΅Ρ€ΡŒ ΠΏΡ€Π΅Π΄ΠΏΠΎΠ»ΠΎΠΆΠΈΠΌ, Ρ‡Ρ‚ΠΎ я ΠΎΡ‚ΠΎΠ΄Π²ΠΈΠ½ΡƒΠ» ΡΡ‚Π΅Ρ€ΠΆΠ΅Π½ΡŒ ΠΎΡ‚ ось вращСния Π½Π° нСбольшоС расстояниС x , ΠΊΠ°ΠΊ это:

Какой ΠΌΠΎΠΌΠ΅Π½Ρ‚ ΠΈΠ½Π΅Ρ€Ρ†ΠΈΠΈ Ρƒ этого смСщСнного стСрТня, Ссли я ΠΏΡ€ΠΎΠ΄ΠΎΠ»ΠΆΡƒ Π²Ρ€Π°Ρ‰Π°Ρ‚ΡŒ Π΅Π³ΠΎ Π²ΠΎΠΊΡ€ΡƒΠ³ Ρ‚ΠΎΠΉ ΠΆΠ΅ оси?





 

Π˜Ρ‚Π°ΠΊ, Ρ‡Ρ‚ΠΎ, Ссли я объСдиню Π΄Π²Π° стСрТня, Ρ‡Ρ‚ΠΎΠ±Ρ‹ получился составной ΠΎΠ±ΡŠΠ΅ΠΊΡ‚? ΠŸΡ€Π΅Π΄ΠΏΠΎΠ»ΠΎΠΆΠΈΠΌ, я ΠΏΠΎΠΌΠ΅Ρ‰Π°ΡŽ ΠΏΠ΅Ρ€Π²Ρ‹ΠΉ ΡΡ‚Π΅Ρ€ΠΆΠ΅Π½ΡŒ ΠΏΠΎ Ρ†Π΅Π½Ρ‚Ρ€Ρƒ оси, Π° вторая ΡˆΡ‚Π°Π½Π³Π° смСщСна Π½Π° расстояниС Ρ… .Какой Π±Ρ‹ суммарный ΠΌΠΎΠΌΠ΅Π½Ρ‚ ΠΈΠ½Π΅Ρ€Ρ†ΠΈΠΈ этого двухстСрТнСвого комбинация Π±Ρ‹Ρ‚ΡŒ?





 

Но Π·Π°Ρ‡Π΅ΠΌ ΠΎΡΡ‚Π°Π½Π°Π²Π»ΠΈΠ²Π°Ρ‚ΡŒΡΡ Π½Π° Π΄Π²ΡƒΡ… стСрТнях? Π― ΠΌΠΎΠ³Ρƒ ΠΈΡΠΏΠΎΠ»ΡŒΠ·ΠΎΠ²Π°Ρ‚ΡŒ Ρ‚Ρƒ ΠΆΠ΅ Ρ‚Π΅Ρ…Π½ΠΈΠΊΡƒ, Ρ‡Ρ‚ΠΎΠ±Ρ‹ ΠΏΠΎΡΡ‚Ρ€ΠΎΠΈΡ‚ΡŒ Π±ΠΎΠ»ΡŒΡˆΡƒΡŽ ΠΊΠΎΠ½ΡΡ‚Ρ€ΡƒΠΊΡ†ΠΈΡŽ. ΠΈΠ· мноТСства стСрТнСй, Π° Ρ‚Π°ΠΊΠΆΠ΅ Π²Ρ‹Ρ‡ΠΈΡΠ»ΠΈΡ‚ΡŒ Π΅Π³ΠΎ ΠΌΠΎΠΌΠ΅Π½Ρ‚ ΠΈΠ½Π΅Ρ€Ρ†ΠΈΠΈ.

НапримСр, я ΠΌΠΎΠ³ Π±Ρ‹ Ρ€Π°Π·ΠΌΠ΅ΡΡ‚ΠΈΡ‚ΡŒ Ρ†Π΅Π»ΡƒΡŽ ΠΊΡƒΡ‡Ρƒ стСрТнСй рядом с Π΄Ρ€ΡƒΠ³ Π΄Ρ€ΡƒΠ³Π°, Ρ‡Ρ‚ΠΎΠ±Ρ‹ ΠΏΠΎΠ»ΡƒΡ‡ΠΈΠ»Π°ΡΡŒ квадратная Ρ‚Π°Ρ€Π΅Π»ΠΊΠ°.

НазовСм ΠΎΠ±Ρ‰ΡƒΡŽ массу всСй этой ΠΏΡ€ΡΠΌΠΎΡƒΠ³ΠΎΠ»ΡŒΠ½ΠΎΠΉ пластины M . Он ΠΈΠΌΠ΅Π΅Ρ‚ ΠΏΠ»ΠΎΡ‰Π°Π΄ΡŒ повСрхности A = L x L = L 2 .ΠœΡ‹ ΠΌΠΎΠΆΠ΅ΠΌ ΠΎΠΏΡ€Π΅Π΄Π΅Π»ΠΈΡ‚ΡŒ массу Π½Π° Π΅Π΄ΠΈΠ½ΠΈΡ†Ρƒ ΠΏΠ»ΠΎΡ‰Π°Π΄ΠΈ ΠΊΠ°ΠΊ

Π’Π΅ΠΏΠ΅Ρ€ΡŒ, Ρ‡Ρ‚ΠΎΠ±Ρ‹ Π²Ρ‹ΡΡΠ½ΠΈΡ‚ΡŒ ΠΌΠΎΠΌΠ΅Π½Ρ‚ ΠΈΠ½Π΅Ρ€Ρ†ΠΈΠΈ этого слоТная структура, я Ρ€Π°ΡΡΠΌΠ°Ρ‚Ρ€ΠΈΠ²Π°ΡŽ Ρ‚ΠΎΠ»ΡŒΠΊΠΎ ΠΎΠ΄Π½Ρƒ Π΄Π΅Ρ‚Π°Π»ΡŒ Π·Π° Ρ€Π°Π·. ΠŸΠΎΠ·Π²ΠΎΠ»ΡŒΡ‚Π΅ ΠΌΠ½Π΅ Π²Π·ΡΡ‚ΡŒ ΠΎΠ΄ΠΈΠ½ Ρ‚ΠΎΠ½ΠΊΠΈΠΉ ΡΡ‚Π΅Ρ€ΠΆΠ΅Π½ΡŒ, расстояниС x ΠΎΡ‚ оси ΠΈ ΡˆΠΈΡ€ΠΈΠ½ΠΎΠΉ dx .

Масса этого кусочка дМ составляСт

ΠΈ Π²ΠΊΠ»Π°Π΄ этой Π΄Π΅Ρ‚Π°Π»ΠΈ Π² ΠΌΠΎΠΌΠ΅Π½Ρ‚ ΠΈΠ½Π΅Ρ€Ρ†ΠΈΠΈ Π΄ΠΎΠ»ΠΆΠ΅Π½ Π±Ρ‹Ρ‚ΡŒ


Π›Π°Π΄Π½ΠΎ, Ρ‚Π΅ΠΏΠ΅Ρ€ΡŒ твоя ΠΎΡ‡Π΅Ρ€Π΅Π΄ΡŒ: Π²Ρ‹Ρ‡ΠΈΡΠ»ΠΈΡ‚ΡŒ ΠΌΠΎΠΌΠ΅Π½Ρ‚ ΠΈΠ½Π΅Ρ€Ρ†ΠΈΠΈ всСй ΠΊΠ²Π°Π΄Ρ€Π°Ρ‚Π½ΠΎΠΉ пластины, суммируя Π²ΠΊΠ»Π°Π΄Ρ‹ ΠΎΡ‚ всСх ΠΌΠ°Π»Π΅Π½ΡŒΠΊΠΈΡ… стСрТнСй.

  • Π’Ρ‹Ρ€Π°Π·ΠΈΡ‚Π΅ ΠΌΠΎΠΌΠ΅Π½Ρ‚ ΠΈΠ½Π΅Ρ€Ρ†ΠΈΠΈ Ρ‡Π΅Ρ€Π΅Π· общая масса M ΠΈ Π΄Π»ΠΈΠ½Π° стороны L
  • БоотвСтствуСт Π»ΠΈ это Π·Π½Π°Ρ‡Π΅Π½ΠΈΠ΅ Ρ‚ΠΎΠΌΡƒ, Ρ‡Ρ‚ΠΎ ΡƒΠΊΠ°Π·Π°Π½ΠΎ Π² вашСм ΡƒΡ‡Π΅Π±Π½ΠΈΠΊΠ΅?

Π’Ρ‹ ΠΌΠΎΠΆΠ΅Ρ‚Π΅ ΠΏΡ€ΠΎΠ²Π΅Ρ€ΠΈΡ‚ΡŒ свои ΠΎΡ‚Π²Π΅Ρ‚Ρ‹, посмотрСв Π½Π° ΠΌΠΎΠ΅ собствСнноС Ρ€Π΅ΡˆΠ΅Π½ΠΈΠ΅.


АвторскиС ΠΏΡ€Π°Π²Π° Β© Майкл Π ΠΈΡ‡ΠΌΠΎΠ½Π΄. Π­Ρ‚Π° Ρ€Π°Π±ΠΎΡ‚Π° находится ΠΏΠΎΠ΄ Π»ΠΈΡ†Π΅Π½Π·ΠΈΠ΅ΠΉ Creative Commons License.

ΠœΠΎΠΌΠ΅Π½Ρ‚ ΠΈΠ½Π΅Ρ€Ρ†ΠΈΠΈ ΠΏΠ»ΠΎΡ‰Π°Π΄ΠΈ – Ρ‚ΠΈΠΏΠΈΡ‡Π½Ρ‹Π΅ ΠΏΠΎΠΏΠ΅Ρ€Π΅Ρ‡Π½Ρ‹Π΅ сСчСния I

ΠœΠΎΠΌΠ΅Π½Ρ‚ ΠΈΠ½Π΅Ρ€Ρ†ΠΈΠΈ ΠΏΠ»ΠΎΡ‰Π°Π΄ΠΈ ΠΈΠ»ΠΈ ΠœΠΎΠΌΠ΅Π½Ρ‚ ΠΈΠ½Π΅Ρ€Ρ†ΠΈΠΈ ΠΏΠ»ΠΎΡ‰Π°Π΄ΠΈ – , Ρ‚Π°ΠΊΠΆΠ΅ извСстный ΠΊΠ°ΠΊ сСкундный ΠΌΠΎΠΌΠ΅Π½Ρ‚ ΠΏΠ»ΠΎΡ‰Π°Π΄ΠΈ I , это свойство Ρ„ΠΎΡ€ΠΌΡ‹, ΠΊΠΎΡ‚ΠΎΡ€ΠΎΠ΅ ΠΈΡΠΏΠΎΠ»ΡŒΠ·ΡƒΠ΅Ρ‚ΡΡ для прогнозирования ΠΏΡ€ΠΎΠ³ΠΈΠ±Π°, ΠΈΠ·Π³ΠΈΠ±Π° ΠΈ напряТСния Π² Π±Π°Π»ΠΊΠ°Ρ….

ΠœΠΎΠΌΠ΅Π½Ρ‚ ΠΈΠ½Π΅Ρ€Ρ†ΠΈΠΈ ΠΏΠ»ΠΎΡ‰Π°Π΄ΠΈ – БританскиС Π΅Π΄ΠΈΠ½ΠΈΡ†Ρ‹

ΠœΠΎΠΌΠ΅Π½Ρ‚ ΠΈΠ½Π΅Ρ€Ρ†ΠΈΠΈ ΠΏΠ»ΠΎΡ‰Π°Π΄ΠΈ – ΠœΠ΅Ρ‚Ρ€ΠΈΡ‡Π΅ΡΠΊΠΈΠ΅ Π΅Π΄ΠΈΠ½ΠΈΡ†Ρ‹

ΠŸΡ€Π΅ΠΎΠ±Ρ€Π°Π·ΠΎΠ²Π°Π½ΠΈΠ΅ ΠΌΠ΅ΠΆΠ΄Ρƒ Π΅Π΄ΠΈΠ½ΠΈΡ†Π°ΠΌΠΈ измСрСния

  • 1 см 4 = 10 -8 ΠΌ 4 = 10 4 ΠΌΠΌ 4
  • 1 дюйм 4 = 4,16×10 5 ΠΌΠΌ 4 = 41,6 см 4
ΠŸΡ€ΠΈΠΌΠ΅Ρ€ – ΠŸΡ€Π΅ΠΎΠ±Ρ€Π°Π·ΠΎΠ²Π°Π½ΠΈΠ΅ Π΅Π΄ΠΈΠ½ΠΈΡ† измСрСния ΠΌΠΎΠΌΠ΅Π½Ρ‚Π° ΠΈΠ½Π΅Ρ€Ρ†ΠΈΠΈ

9240 см 4 ΠΌΠΎΠΆΠ½ΠΎ ΠΏΡ€Π΅ΠΎΠ±Ρ€Π°Π·ΠΎΠ²Π°Ρ‚ΡŒ Π² ΠΌΠΌ 4 ΡƒΠΌΠ½ΠΎΠΆΠ΅Π½ΠΈΠ΅ΠΌ Π½Π° 10 4

(9240 см 4 ) 10 4 = 9.24 10 7 мм 4

ΠœΠΎΠΌΠ΅Π½Ρ‚ ΠΈΠ½Π΅Ρ€Ρ†ΠΈΠΈ ΠΏΠ»ΠΎΡ‰Π°Π΄ΠΈ (ΠΌΠΎΠΌΠ΅Π½Ρ‚ ΠΈΠ½Π΅Ρ€Ρ†ΠΈΠΈ ΠΏΠ»ΠΎΡ‰Π°Π΄ΠΈ ΠΈΠ»ΠΈ Π²Ρ‚ΠΎΡ€ΠΎΠΉ ΠΌΠΎΠΌΠ΅Π½Ρ‚ ΠΏΠ»ΠΎΡ‰Π°Π΄ΠΈ)

для ΠΈΠ·Π³ΠΈΠ±Π° Π²ΠΎΠΊΡ€ΡƒΠ³ оси x ΠΌΠΎΠΆΠ½ΠΎ Π²Ρ‹Ρ€Π°Π·ΠΈΡ‚ΡŒ ΠΊΠ°ΠΊ

I x = ∫ y 2 dA (1)

Π³Π΄Π΅

I x = ΠΌΠΎΠΌΠ΅Π½Ρ‚ ΠΈΠ½Π΅Ρ€Ρ†ΠΈΠΈ ΠΏΠ»ΠΎΡ‰Π°Π΄ΠΈ ΠΎΡ‚Π½ΠΎΡΠΈΡ‚Π΅Π»ΡŒΠ½ΠΎ оси x ( ΠΌ 4 , ΠΌΠΌ 4 , Π΄ΡŽΠΉΠΌΡ‹ 4 )

y = пСрпСндикулярноС расстояниС ΠΎΡ‚ оси x Π΄ΠΎ элСмСнта dA (ΠΌ, ΠΌΠΌ, дюймов )

dA = ΠΏΠ»ΠΎΡ‰Π°Π΄ΡŒ элСмСнта ( ΠΌ 2 , ΠΌΠΌ 2 , дюймов 2 )

ΠœΠΎΠΌΠ΅Π½Ρ‚ ΠΈΠ½Π΅Ρ€Ρ†ΠΈΠΈ ΠΈΠ·Π³ΠΈΠ±Π° Π²ΠΎΠΊΡ€ΡƒΠ³ оси y ΠΌΠΎΠΆΠ½ΠΎ Π²Ρ‹Ρ€Π°Π·ΠΈΡ‚ΡŒ ΠΊΠ°ΠΊ

90 094 I y = ∫ x 2 dA (2)

Π³Π΄Π΅

I y = ΠΌΠΎΠΌΠ΅Π½Ρ‚ ΠΈΠ½Π΅Ρ€Ρ†ΠΈΠΈ ΠΏΠ»ΠΎΡ‰Π°Π΄ΠΈ ΠΎΡ‚Π½ΠΎΡΠΈΡ‚Π΅Π»ΡŒΠ½ΠΎ оси y ( ΠΌ 4 , ΠΌΠΌ 4 , Π΄ΡŽΠΉΠΌΡ‹ 4 )

x = пСрпСндикулярноС расстояниС ΠΎΡ‚ оси y Π΄ΠΎ элСмСнта dA (ΠΌ, ΠΌΠΌ, дюймов )

ΠŸΠ»ΠΎΡ‰Π°Π΄ΡŒ ΠœΠΎΠΌΠ΅Π½Ρ‚ ΠΈΠ½Π΅Ρ€Ρ†ΠΈΠΈ для Ρ‚ΠΈΠΏΠΈΡ‡Π½ΠΎΠ³ΠΎ ΠΏΠΎΠΏΠ΅Ρ€Π΅Ρ‡Π½ΠΎΠ³ΠΎ сСчСния I

БплошноС ΠΊΠ²Π°Π΄Ρ€Π°Ρ‚Π½ΠΎΠ΅ ΠΏΠΎΠΏΠ΅Ρ€Π΅Ρ‡Π½ΠΎΠ΅ сСчСниС

ΠœΠΎΠΌΠ΅Π½Ρ‚ ΠΈΠ½Π΅Ρ€Ρ†ΠΈΠΈ ΠΏΠ»ΠΎΡ‰Π°Π΄ΠΈ для сплошного ΠΊΠ²Π°Π΄Ρ€Π°Ρ‚Π½ΠΎΠ³ΠΎ сСчСния ΠΌΠΎΠΆΠ½ΠΎ Ρ€Π°ΡΡΡ‡ΠΈΡ‚Π°Ρ‚ΡŒ ΠΊΠ°ΠΊ

I x = a 4 /12 (2)

Π³Π΄Π΅

a = сторона (ΠΌΠΌ, ΠΌ, дюйм..)

I y = a 4 /12 (2b)

БплошноС ΠΏΡ€ΡΠΌΠΎΡƒΠ³ΠΎΠ»ΡŒΠ½ΠΎΠ΅ сСчСниС

ΠœΠΎΠΌΠ΅Π½Ρ‚ ΠΏΠ»ΠΎΡ‰Π°Π΄ΠΈ Ineria для ΠΏΡ€ΡΠΌΠΎΡƒΠ³ΠΎΠ»ΡŒΠ½ΠΎΠ³ΠΎ сСчСния ΠΌΠΎΠΆΠ½ΠΎ Ρ€Π°ΡΡΡ‡ΠΈΡ‚Π°Ρ‚ΡŒ ΠΊΠ°ΠΊ

I x = bh 3 /12 (3)

Π³Π΄Π΅

b = ΡˆΠΈΡ€ΠΈΠ½Π°

h = высота

I y = b 3 h / 12 (3b)

БплошноС ΠΊΡ€ΡƒΠ³Π»ΠΎΠ΅ сСчСниС

ΠœΠΎΠΌΠ΅Π½Ρ‚ ΠΈΠ½Π΅Ρ€Ρ†ΠΈΠΈ ΠΏΠ»ΠΎΡ‰Π°Π΄ΠΈ для сплошного цилиндричСского сСчСния ΠΌΠΎΠΆΠ½ΠΎ Ρ€Π°ΡΡΡ‡ΠΈΡ‚Π°Ρ‚ΡŒ ΠΊΠ°ΠΊ

I x = Ο€ r 4 /4

= Ο€ d 4 /64 (4)

Π³Π΄Π΅

9 0094 r = радиус

d = Π΄ΠΈΠ°ΠΌΠ΅Ρ‚Ρ€

I y = Ο€ r 4 /4

= Ο€ d 4 /64 (4b)

ΠΏΠΎΠ»Ρ‹ΠΉ ЦилиндричСскоС ΠΏΠΎΠΏΠ΅Ρ€Π΅Ρ‡Π½ΠΎΠ΅ сСчСниС

ΠœΠΎΠΌΠ΅Π½Ρ‚ ΠΈΠ½Π΅Ρ€Ρ†ΠΈΠΈ ΠΏΠ»ΠΎΡ‰Π°Π΄ΠΈ для ΠΏΠΎΠ»ΠΎΠ³ΠΎ цилиндричСского профиля ΠΌΠΎΠΆΠ½ΠΎ Ρ€Π°ΡΡΡ‡ΠΈΡ‚Π°Ρ‚ΡŒ ΠΊΠ°ΠΊ

I x = Ο€ (d o 4 – d i 4 ) / 64 ( 5)

, Π³Π΄Π΅

d o = внСшний Π΄ΠΈΠ°ΠΌΠ΅Ρ‚Ρ€ Ρ†ΠΈΠ»ΠΈΠ½Π΄Ρ€Π°

d i = Π²Π½ΡƒΡ‚Ρ€Π΅Π½Π½ΠΈΠΉ Π΄ΠΈΠ°ΠΌΠ΅Ρ‚Ρ€ Ρ†ΠΈΠ»ΠΈΠ½Π΄Ρ€Π°

I y = Ο€ (d o 9017 4 – d i 4 ) / 64 (5b)

ΠšΠ²Π°Π΄Ρ€Π°Ρ‚Π½ΠΎΠ΅ сСчСниС – Π΄ΠΈΠ°Π³ΠΎΠ½Π°Π»ΡŒΠ½Ρ‹Π΅ ΠΌΠΎΠΌΠ΅Π½Ρ‚Ρ‹

90 002 ΠœΠΎΠΌΠ΅Π½Ρ‚Ρ‹ ΠΈΠ½Π΅Ρ€Ρ†ΠΈΠΈ диагональной ΠΏΠ»ΠΎΡ‰Π°Π΄ΠΈ для ΠΊΠ²Π°Π΄Ρ€Π°Ρ‚Π½ΠΎΠ³ΠΎ сСчСния ΠΌΠΎΠΆΠ½ΠΎ Ρ€Π°ΡΡΡ‡ΠΈΡ‚Π°Ρ‚ΡŒ ΠΊΠ°ΠΊ

I x = I y = a 4 /12 (6)

ΠŸΡ€ΡΠΌΠΎΡƒΠ³ΠΎΠ»ΡŒΠ½ΠΎΠ΅ сСчСниС – ΠœΠΎΠΌΠ΅Π½Ρ‚Ρ‹ ΠΏΠ»ΠΎΡ‰Π°Π΄ΠΈ Π½Π° любой Π»ΠΈΠ½ΠΈΠΈ, проходящСй Ρ‡Π΅Ρ€Π΅Π· Ρ†Π΅Π½Ρ‚Ρ€ силы тяТСсти

ΠŸΡ€ΡΠΌΠΎΡƒΠ³ΠΎΠ»ΡŒΠ½ΠΎΠ΅ сСчСниС ΠΈ ΠΏΠ»ΠΎΡ‰Π°Π΄ΡŒ ΠΌΠΎΠΌΠ΅Π½Ρ‚Π° Π½Π° Π»ΠΈΠ½ΠΈΠΈ, проходящСй Ρ‡Π΅Ρ€Π΅Π· Ρ†Π΅Π½Ρ‚Ρ€ тяТСсти, ΠΌΠΎΠΆΠ½ΠΎ Ρ€Π°ΡΡΡ‡ΠΈΡ‚Π°Ρ‚ΡŒ ΠΊΠ°ΠΊ

I x = (bh / 12) (h 2 cos 2 a + b 2 sin 2 a) (7)

БиммСтричная Ρ„ΠΎΡ€ΠΌΠ°

ΠœΠΎΠΌΠ΅Π½Ρ‚ ΠΈΠ½Π΅Ρ€Ρ†ΠΈΠΈ ΠΏΠ»ΠΎΡ‰Π°Π΄ΠΈ для сСчСния симмСтричной Ρ„ΠΎΡ€ΠΌΡ‹ ΠΌΠΎΠΆΠ½ΠΎ Ρ€Π°ΡΡΡ‡ΠΈΡ‚Π°Ρ‚ΡŒ ΠΊΠ°ΠΊ

I x = (ah 3 /12) + (b / 12) (H 3 – h 3 ) (8)

I y = (a 3 h / 12) + (b 3 /12) (H – h) ( 8b)

НС симмСтричная Ρ„ΠΎΡ€ΠΌΠ°

ΠŸΠ»ΠΎΡ‰Π°Π΄ΡŒ ΠœΠΎΠΌΠ΅Π½Ρ‚ ΠΈΠ½Π΅Ρ€Ρ†ΠΈΠΈ для нСсиммСтричного профиля ΠΌΠΎΠΆΠ½ΠΎ Ρ€Π°ΡΡΡ‡ΠΈΡ‚Π°Ρ‚ΡŒ ΠΊΠ°ΠΊ

I x = (1/3) (B y b 3 – B 1 h b 3 + by t 3 – b1 h t 3 ) (9)

ΠŸΠ»ΠΎΡ‰Π°Π΄ΡŒ ΠœΠΎΠΌΠ΅Π½Ρ‚ ΠΈΠ½Π΅Ρ€Ρ†ΠΈΠΈ Π² зависимости ΠΎΡ‚Π—Π°Π²ΠΈΡΠΈΠΌΠΎΡΡ‚ΡŒ полярного ΠΌΠΎΠΌΠ΅Π½Ρ‚Π° ΠΈΠ½Π΅Ρ€Ρ†ΠΈΠΈ ΠΎΡ‚ ΠΌΠΎΠΌΠ΅Π½Ρ‚Π° ΠΈΠ½Π΅Ρ€Ρ†ΠΈΠΈ

  • Β«ΠœΠΎΠΌΠ΅Π½Ρ‚ ΠΈΠ½Π΅Ρ€Ρ†ΠΈΠΈ ΠΏΠ»ΠΎΡ‰Π°Π΄ΠΈΒ» – это свойство Ρ„ΠΎΡ€ΠΌΡ‹, ΠΊΠΎΡ‚ΠΎΡ€ΠΎΠ΅ ΠΈΡΠΏΠΎΠ»ΡŒΠ·ΡƒΠ΅Ρ‚ΡΡ для прогнозирования ΠΏΡ€ΠΎΠ³ΠΈΠ±Π°, ΠΈΠ·Π³ΠΈΠ±Π° ΠΈ напряТСния Π² Π±Π°Π»ΠΊΠ°Ρ…. 2 $, Π° Π½Π΅ ΠΎΡ‚ $ r $? (физичСская ΠΏΡ€ΠΈΡ‡ΠΈΠ½Π°)

    Π­Ρ‚ΠΎ частично основано Π½Π° ΠΎΡ‚Π²Π΅Ρ‚Π΅ TZDZ.{net} \ tau_i = \ alpha I

    долл. БША

    ΠΌΠΎΠΆΠ½ΠΎ ΠΎΡ‚ΠΎΠ±Ρ€Π°Π·ΠΈΡ‚ΡŒ Ρ‚Π΅ΠΌ ΠΆΠ΅ способом.

    Π‘ этим связаны большиС вопросы: ΠΏΠΎΡ‡Π΅ΠΌΡƒ крутящий ΠΌΠΎΠΌΠ΅Π½Ρ‚ Π²Π°ΠΆΠ΅Π½ ΠΈ ΠΏΠΎΡ‡Π΅ΠΌΡƒ ΠΎΠ½ Ρ‚Π°ΠΊ сильно отличаСтся ΠΎΡ‚ всСго, ΠΎ Ρ‡Π΅ΠΌ ΡƒΠ·Π½Π°Π»ΠΈ Ρ€Π°Π½Π΅Π΅? ΠšΡ€ΠΎΠΌΠ΅ Ρ‚ΠΎΠ³ΠΎ, ΠΊΠΎΠ³Π΄Π° слСдуСт ΠΈΡΠΏΠΎΠ»ΡŒΠ·ΠΎΠ²Π°Ρ‚ΡŒ крутящий ΠΌΠΎΠΌΠ΅Π½Ρ‚, Π² ΠΎΡ‚Π»ΠΈΡ‡ΠΈΠ΅ ΠΎΡ‚ Π·Π°ΠΊΠΎΠ½ΠΎΠ² ΠΡŒΡŽΡ‚ΠΎΠ½Π° ΠΈ кинСматичСских ΡƒΡ€Π°Π²Π½Π΅Π½ΠΈΠΉ?

    ΠžΡ‚Π²Π΅Ρ‚ Π½Π° ΠΏΠ΅Ρ€Π²Ρ‹ΠΉ вопрос:

    Π’ Π±ΠΎΠ»ΡŒΡˆΠΈΠ½ΡΡ‚Π²Π΅ Π²Π²ΠΎΠ΄Π½Ρ‹Ρ… курсов Ρ„ΠΈΠ·ΠΈΠΊΠΈ инСрция вводится сразу послС Π³Π»Π°Π²Ρ‹ ΠΎ Ρ†Π΅Π½Ρ‚Ρ€Π΅ масс, которая слСдуСт Π·Π° нСсколькими Π³Π»Π°Π²Π°ΠΌΠΈ ΠΎ Π΄Π²ΠΈΠΆΠ΅Π½ΠΈΠΈ Ρ‚Π²Π΅Ρ€Π΄Ρ‹Ρ… Ρ‚Π΅Π», Π² ΠΊΠΎΡ‚ΠΎΡ€Ρ‹Ρ… каТдая частица двиТСтся с ΠΎΠ΄ΠΈΠ½Π°ΠΊΠΎΠ²Ρ‹ΠΌ ΠΎΠ±Ρ‰ΠΈΠΌ Π΄Π²ΠΈΠΆΠ΅Π½ΠΈΠ΅ΠΌ.УравнСния ΠΊΠΈΠ½Π΅ΠΌΠ°Ρ‚ΠΈΠΊΠΈ ΠΌΠΎΠ³ΡƒΡ‚ Π±Ρ‹Ρ‚ΡŒ ΠΏΡ€ΠΈΠΌΠ΅Π½Π΅Π½Ρ‹ ΠΊ Ρ‚Π²Π΅Ρ€Π΄Ρ‹ΠΌ ΠΎΠ±ΡŠΠ΅ΠΊΡ‚Π°ΠΌ Π² этих ΠΏΡ€Π΅Π΄Ρ‹Π΄ΡƒΡ‰ΠΈΡ… Π³Π»Π°Π²Π°Ρ…, ΠΏΠΎΡ‚ΠΎΠΌΡƒ Ρ‡Ρ‚ΠΎ каТдая частица Π² ΠΎΠ±ΡŠΠ΅ΠΊΡ‚Π΅ двиТСтся ΠΏΡ€ΠΈΠΌΠ΅Ρ€Π½ΠΎ с Ρ‚ΠΎΠΉ ΠΆΠ΅ ΡΠΊΠΎΡ€ΠΎΡΡ‚ΡŒΡŽ, Ρ‡Ρ‚ΠΎ ΠΈ любая другая частица Π² ΠΎΠ±ΡŠΠ΅ΠΊΡ‚Π΅. Когда Π²Ρ‹ Π΄ΠΎΠΉΠ΄Π΅Ρ‚Π΅ Π΄ΠΎ Π³Π»Π°Π², посвящСнных Ρ†Π΅Π½Ρ‚Ρ€Ρƒ масс, это ΡƒΠΆΠ΅ Π½Π΅ Ρ‚Π°ΠΊ, Π½ΠΎ ΠΈΠ·-Π·Π° ΠΏΡ€ΠΈΡ€ΠΎΠ΄Ρ‹ Ρ†Π΅Π½Ρ‚Ρ€Π° масс кинСматичСскиС уравнСния всС Π΅Ρ‰Π΅ ΠΌΠΎΠ³ΡƒΡ‚ ΠΏΡ€ΠΈΠΌΠ΅Π½ΡΡ‚ΡŒΡΡ. Π—Π°Ρ‚Π΅ΠΌ, Π² Π³Π»Π°Π²Π°Ρ… ΠΎ Π²Ρ€Π°Ρ‰Π΅Π½ΠΈΠΈ, становятся Π½Π΅ΠΎΠ±Ρ…ΠΎΠ΄ΠΈΠΌΡ‹ΠΌΠΈ понятия крутящСго ΠΌΠΎΠΌΠ΅Π½Ρ‚Π°, ΠΈΠ½Π΅Ρ€Ρ†ΠΈΠΈ ΠΈ ΡƒΠ³Π»ΠΎΠ²ΠΎΠ³ΠΎ ΠΌΠΎΠΌΠ΅Π½Ρ‚Π°, ΠΏΠΎΡ‚ΠΎΠΌΡƒ Ρ‡Ρ‚ΠΎ Π½Π΅ каТдая частица Π² ΠΎΠ±ΡŠΠ΅ΠΊΡ‚Π°Ρ…, с ΠΊΠΎΡ‚ΠΎΡ€Ρ‹ΠΌΠΈ Π²Ρ‹ ΠΈΠΌΠ΅Π΅Ρ‚Π΅ Π΄Π΅Π»ΠΎ, двиТСтся с ΠΎΠ΄Π½ΠΎΠΉ ΠΈ Ρ‚ΠΎΠΉ ΠΆΠ΅ ΠΏΠΎΡΡ‚ΡƒΠΏΠ°Ρ‚Π΅Π»ΡŒΠ½ΠΎΠΉ ΡΠΊΠΎΡ€ΠΎΡΡ‚ΡŒΡŽ.Π’Π΅ΠΌ Π½Π΅ ΠΌΠ΅Π½Π΅Π΅, ΠΎΠ½ΠΈ ΠΏΠΎ-ΠΏΡ€Π΅ΠΆΠ½Π΅ΠΌΡƒ двиТутся с рСгулярным Π΄Π²ΠΈΠΆΠ΅Π½ΠΈΠ΅ΠΌ, поэтому ΠΈΡ… ΠΌΠΎΠΆΠ½ΠΎ ΠΌΠΎΠ΄Π΅Π»ΠΈΡ€ΠΎΠ²Π°Ρ‚ΡŒ, Π½ΠΎ для этого Ρ‚Ρ€Π΅Π±ΡƒΡŽΡ‚ΡΡ Π΄Ρ€ΡƒΠ³ΠΈΠ΅ ΠΌΠΎΠ΄Π΅Π»ΠΈ.

    ΠžΡ‚Π²Π΅Ρ‚ Π½Π° Π²Ρ‚ΠΎΡ€ΠΎΠΉ вопрос:

    Torque слСдуСт ΠΈΡΠΏΠΎΠ»ΡŒΠ·ΠΎΠ²Π°Ρ‚ΡŒ, ΠΊΠΎΠ³Π΄Π° ΠΎΡ‚Π΄Π΅Π»ΡŒΠ½Ρ‹Π΅ частицы Π² систСмС частиц, ΠΊΠΎΡ‚ΠΎΡ€ΡƒΡŽ Π²Ρ‹ ΠΌΠΎΠ΄Π΅Π»ΠΈΡ€ΡƒΠ΅Ρ‚Π΅, Π½Π΅ всС двиТутся ΠΏΡ€ΠΈΠΌΠ΅Ρ€Π½ΠΎ с ΠΎΠ΄ΠΈΠ½Π°ΠΊΠΎΠ²ΠΎΠΉ ΡΠΊΠΎΡ€ΠΎΡΡ‚ΡŒΡŽ. IOW, Π΅Π³ΠΎ слСдуСт ΠΈΡΠΏΠΎΠ»ΡŒΠ·ΠΎΠ²Π°Ρ‚ΡŒ, ΠΊΠΎΠ³Π΄Π° Π½Π΅ всС частицы Π² систСмС двиТутся ΠΎΠ΄ΠΈΠ½Π°ΠΊΠΎΠ²ΠΎ. Π‘ΠΎΠ»Π΅Π΅ ΠΊΠΎΠ½ΠΊΡ€Π΅Ρ‚Π½ΠΎ, ΠΎΠ΄Π½Π°ΠΊΠΎ, Π΅Π³ΠΎ слСдуСт ΠΈΡΠΏΠΎΠ»ΡŒΠ·ΠΎΠ²Π°Ρ‚ΡŒ, ΠΊΠΎΠ³Π΄Π° систСма Π½Π΅ ΠΈΠΌΠ΅Π΅Ρ‚ Ρ€Π°Π²Π½ΠΎΠΌΠ΅Ρ€Π½ΠΎΠ³ΠΎ ΠΏΠΎΡΡ‚ΡƒΠΏΠ°Ρ‚Π΅Π»ΡŒΠ½ΠΎΠ³ΠΎ двиТСния, Π½ΠΎ ΠΈΠΌΠ΅Π΅Ρ‚ Ρ€Π°Π²Π½ΠΎΠΌΠ΅Ρ€Π½ΠΎΠ΅ ΡƒΠ³Π»ΠΎΠ²ΠΎΠ΅ Π΄Π²ΠΈΠΆΠ΅Π½ΠΈΠ΅.2 $ ΠΎΠ±ΡŠΡΡΠ½ΡΠ΅Ρ‚ΡΡ Ρ‚Π΅ΠΌ, Ρ‡Ρ‚ΠΎ ΠΎΠ΄Π½Π° ΠΈΠ· $ r $ проистСкаСт ΠΈΠ· Ρ‚ΠΎΠ³ΠΎ Ρ„Π°ΠΊΡ‚Π°, Ρ‡Ρ‚ΠΎ $ a_T = r \ alpha $, Π³Π΄Π΅ $ a_T $ – Ρ‚Π°Π½Π³Π΅Π½Ρ†ΠΈΠ°Π»ΡŒΠ½ΠΎΠ΅ ускорСниС. Π”Ρ€ΡƒΠ³ΠΎΠΉ связан с Ρ‚Π΅ΠΌ, Ρ‡Ρ‚ΠΎ $ s = r \ theta $. ΠŸΠΎΡΠΊΠΎΠ»ΡŒΠΊΡƒ крутящий ΠΌΠΎΠΌΠ΅Π½Ρ‚ являСтся ΠΏΡ€ΠΎΠΈΠ·Π²ΠΎΠ΄Π½ΠΎΠΉ $ W_R $ ΠΏΠΎ $ \ theta $, Π° Π½Π΅ ΠΏΠΎ $ s $, $ r $ ΠΈΠ· $ s = r \ theta $ остаСтся Π² ΡƒΡ€Π°Π²Π½Π΅Π½ΠΈΠΈ. Π˜Π½Π΅Ρ€Ρ†ΠΈΡ Ρ‚Π°ΠΊΠΆΠ΅ становится для нас Π²Π΄Π²ΠΎΠΉΠ½Π΅ ΠΏΠΎΠ»Π΅Π·Π½ΠΎΠΉ, ΠΏΠΎΡ‚ΠΎΠΌΡƒ Ρ‡Ρ‚ΠΎ ΠΎΠ½Π° Ρ‚Π°ΠΊΠΆΠ΅ присутствуСт Π² расчСтС кинСтичСской энСргии вращСния, ΠΈ поэтому Ρ‚Π΅ΠΏΠ΅Ρ€ΡŒ это полноцСнная концСпция со своим собствСнным Π½Π°Π·Π²Π°Π½ΠΈΠ΅ΠΌ ΠΈ страницСй Π² Π²ΠΈΠΊΠΈΠΏΠ΅Π΄ΠΈΠΈ.2 dA

    , Π³Π΄Π΅ A – ΠΏΠ»ΠΎΡ‰Π°Π΄ΡŒ Ρ„ΠΎΡ€ΠΌΡ‹, Π° y – расстояниС любой Ρ‚ΠΎΡ‡ΠΊΠΈ Π²Π½ΡƒΡ‚Ρ€ΠΈ области A ΠΎΡ‚ Π·Π°Π΄Π°Π½Π½ΠΎΠΉ оси вращСния. Из опрСдСлСния ΠΎΡ‡Π΅Π²ΠΈΠ΄Π½ΠΎ, Ρ‡Ρ‚ΠΎ ΠΌΠΎΠΌΠ΅Π½Ρ‚ ΠΈΠ½Π΅Ρ€Ρ†ΠΈΠΈ всСгда Π΄ΠΎΠ»ΠΆΠ΅Π½ ΠΈΠΌΠ΅Ρ‚ΡŒ ΠΏΠΎΠ»ΠΎΠΆΠΈΡ‚Π΅Π»ΡŒΠ½ΠΎΠ΅ Π·Π½Π°Ρ‡Π΅Π½ΠΈΠ΅, ΠΏΠΎΡΠΊΠΎΠ»ΡŒΠΊΡƒ Π²Π½ΡƒΡ‚Ρ€ΠΈ ΠΈΠ½Ρ‚Π΅Π³Ρ€Π°Π»Π° Π΅ΡΡ‚ΡŒ Ρ‚ΠΎΠ»ΡŒΠΊΠΎ ΠΊΠ²Π°Π΄Ρ€Π°Ρ‚Π½Ρ‹ΠΉ Ρ‡Π»Π΅Π½.

    ΠšΠΎΠ½Ρ†Π΅ΠΏΡ‚ΡƒΠ°Π»ΡŒΠ½ΠΎ Π²Ρ‚ΠΎΡ€ΠΎΠΉ ΠΌΠΎΠΌΠ΅Π½Ρ‚ ΠΏΠ»ΠΎΡ‰Π°Π΄ΠΈ связан с распрСдСлСниСм ΠΏΠ»ΠΎΡ‰Π°Π΄ΠΈ Ρ„ΠΈΠ³ΡƒΡ€Ρ‹. Π’ частности, Π±ΠΎΠ»Π΅Π΅ высокий ΠΌΠΎΠΌΠ΅Π½Ρ‚ ΡƒΠΊΠ°Π·Ρ‹Π²Π°Π΅Ρ‚ Π½Π° Ρ‚ΠΎ, Ρ‡Ρ‚ΠΎ ΠΏΠ»ΠΎΡ‰Π°Π΄ΡŒ Ρ„ΠΎΡ€ΠΌΡ‹ распрСдСлСна Π΄Π°Π»Π΅ΠΊΠΎ ΠΎΡ‚ оси. Напротив, Π±ΠΎΠ»Π΅Π΅ Π½ΠΈΠ·ΠΊΠΈΠΉ ΠΌΠΎΠΌΠ΅Π½Ρ‚ ΡƒΠΊΠ°Π·Ρ‹Π²Π°Π΅Ρ‚ Π½Π° Π±ΠΎΠ»Π΅Π΅ ΠΊΠΎΠΌΠΏΠ°ΠΊΡ‚Π½ΡƒΡŽ Ρ„ΠΎΡ€ΠΌΡƒ, ΠΏΠ»ΠΎΡ‰Π°Π΄ΡŒ ΠΊΠΎΡ‚ΠΎΡ€ΠΎΠΉ располоТСна Π±Π»ΠΈΠΆΠ΅ ΠΊ оси.НапримСр, Π½Π° ΡΠ»Π΅Π΄ΡƒΡŽΡ‰Π΅ΠΌ рисункС ΠΎΠ±Π΅ Ρ„ΠΎΡ€ΠΌΡ‹ ΠΈΠΌΠ΅ΡŽΡ‚ Ρ€Π°Π²Π½Ρ‹Π΅ ΠΏΠ»ΠΎΡ‰Π°Π΄ΠΈ, Ρ‚ΠΎΠ³Π΄Π° ΠΊΠ°ΠΊ правая Ρ„ΠΎΡ€ΠΌΠ° ΠΈΠΌΠ΅Π΅Ρ‚ Π±ΠΎΠ»Π΅Π΅ высокий Π²Ρ‚ΠΎΡ€ΠΎΠΉ ΠΌΠΎΠΌΠ΅Π½Ρ‚ ΠΏΠ»ΠΎΡ‰Π°Π΄ΠΈ Π²ΠΎΠΊΡ€ΡƒΠ³ красной оси, ΠΏΠΎΡΠΊΠΎΠ»ΡŒΠΊΡƒ, ΠΏΠΎ ΡΡ€Π°Π²Π½Π΅Π½ΠΈΡŽ с Π»Π΅Π²ΠΎΠΉ, Π΅Π΅ ΠΏΠ»ΠΎΡ‰Π°Π΄ΡŒ распрСдСлСна Π·Π½Π°Ρ‡ΠΈΡ‚Π΅Π»ΡŒΠ½ΠΎ дальшС ΠΎΡ‚ оси. .

    ВСрминология

    Π§Π°Ρ‰Π΅ всСго Ρ‚Π΅Ρ€ΠΌΠΈΠ½ ΠΌΠΎΠΌΠ΅Π½Ρ‚ ΠΈΠ½Π΅Ρ€Ρ†ΠΈΠΈ ΠΈΡΠΏΠΎΠ»ΡŒΠ·ΡƒΠ΅Ρ‚ΡΡ для Π²Ρ‚ΠΎΡ€ΠΎΠ³ΠΎ ΠΌΠΎΠΌΠ΅Π½Ρ‚Π° ΠΏΠ»ΠΎΡ‰Π°Π΄ΠΈ, особСнно Π² ΠΈΠ½ΠΆΠ΅Π½Π΅Ρ€Π½Ρ‹Ρ… дисциплинах. Однако Π² Ρ„ΠΈΠ·ΠΈΠΊΠ΅ ΠΌΠΎΠΌΠ΅Π½Ρ‚ ΠΈΠ½Π΅Ρ€Ρ†ΠΈΠΈ связан с распрСдСлСниСм массы Π²ΠΎΠΊΡ€ΡƒΠ³ оси ΠΈ, ΠΊΠ°ΠΊ Ρ‚Π°ΠΊΠΎΠ²ΠΎΠΉ, являСтся свойством ΠΎΠ±ΡŠΠ΅ΠΌΠ½Ρ‹Ρ… ΠΎΠ±ΡŠΠ΅ΠΊΡ‚ΠΎΠ², Π² ΠΎΡ‚Π»ΠΈΡ‡ΠΈΠ΅ ΠΎΡ‚ Π²Ρ‚ΠΎΡ€ΠΎΠ³ΠΎ ΠΌΠΎΠΌΠ΅Π½Ρ‚Π° ΠΏΠ»ΠΎΡ‰Π°Π΄ΠΈ, ΠΊΠΎΡ‚ΠΎΡ€Ρ‹ΠΉ являСтся свойством плоских областСй.На ΠΏΡ€Π°ΠΊΡ‚ΠΈΠΊΠ΅ для описания Π²Ρ‚ΠΎΡ€ΠΎΠ³ΠΎ ΠΌΠΎΠΌΠ΅Π½Ρ‚Π° ΠΏΠ»ΠΎΡ‰Π°Π΄ΠΈ ΠΌΠΎΠΆΠ½ΠΎ ΠΈΡΠΏΠΎΠ»ΡŒΠ·ΠΎΠ²Π°Ρ‚ΡŒ ΡΠ»Π΅Π΄ΡƒΡŽΡ‰ΠΈΠ΅ Ρ‚Π΅Ρ€ΠΌΠΈΠ½Ρ‹:

    • ΠΌΠΎΠΌΠ΅Π½Ρ‚ ΠΈΠ½Π΅Ρ€Ρ†ΠΈΠΈ
    • ΠΌΠΎΠΌΠ΅Π½Ρ‚ ΠΈΠ½Π΅Ρ€Ρ†ΠΈΠΈ ΠΏΠ»ΠΎΡ‰Π°Π΄ΠΈ
    • ΠΌΠΎΠΌΠ΅Π½Ρ‚ ΠΈΠ½Π΅Ρ€Ρ†ΠΈΠΈ ΠΏΠ»ΠΎΡ‰Π°Π΄ΠΈ
    • ΠΌΠΎΠΌΠ΅Π½Ρ‚ ΠΈΠ½Π΅Ρ€Ρ†ΠΈΠΈ ΠΏΠΎΠΏΠ΅Ρ€Π΅Ρ‡Π½ΠΎΠ³ΠΎ сСчСния
    • ΠΌΠΎΠΌΠ΅Π½Ρ‚ ΠΈΠ½Π΅Ρ€Ρ†ΠΈΠΈ Π±Π°Π»ΠΊΠ°

    Π’Ρ‚ΠΎΡ€ΠΎΠΉ ΠΌΠΎΠΌΠ΅Π½Ρ‚ ΠΏΠ»ΠΎΡ‰Π°Π΄ΠΈ (ΠΌΠΎΠΌΠ΅Π½Ρ‚ ΠΈΠ½Π΅Ρ€Ρ†ΠΈΠΈ) ΠΈΠΌΠ΅Π΅Ρ‚ Π·Π½Π°Ρ‡Π΅Π½ΠΈΠ΅ Ρ‚ΠΎΠ»ΡŒΠΊΠΎ Ρ‚ΠΎΠ³Π΄Π°, ΠΊΠΎΠ³Π΄Π° ΠΎΠΏΡ€Π΅Π΄Π΅Π»Π΅Π½Π° ось вращСния. Π’Π΅ΠΌ Π½Π΅ ΠΌΠ΅Π½Π΅Π΅, часто ΠΌΠΎΠΆΠ½ΠΎ ΠΈΡΠΏΠΎΠ»ΡŒΠ·ΠΎΠ²Π°Ρ‚ΡŒ Ρ‚Π΅Ρ€ΠΌΠΈΠ½ Β«ΠΌΠΎΠΌΠ΅Π½Ρ‚ ΠΈΠ½Π΅Ρ€Ρ†ΠΈΠΈ окруТности», ΠΎΡ‚ΡΡƒΡ‚ΡΡ‚Π²ΡƒΡŽΡ‰ΠΈΠΉ для обозначСния оси.Π’ Ρ‚Π°ΠΊΠΈΡ… случаях, вСроятно, подразумСваСтся ось, проходящая Ρ‡Π΅Ρ€Π΅Π· Ρ†Π΅Π½Ρ‚Ρ€ тяТСсти Ρ„ΠΎΡ€ΠΌΡ‹.

    ΠŸΡ€ΠΎΠΈΠ·Π²Π΅Π΄Π΅Π½ΠΈΠ΅ ΠΈΠ½Π΅Ρ€Ρ†ΠΈΠΈ

    ΠŸΡ€ΠΎΠΈΠ·Π²Π΅Π΄Π΅Π½ΠΈΠ΅ ΠΈΠ½Π΅Ρ€Ρ†ΠΈΠΈ плоской Π·Π°ΠΌΠΊΠ½ΡƒΡ‚ΠΎΠΉ области опрСдСляСтся ΠΊΠ°ΠΊ ΠΈΠ½Ρ‚Π΅Π³Ρ€Π°Π» ΠΏΠΎ ΠΏΠ»ΠΎΡ‰Π°Π΄ΠΈ произвСдСния расстояний ΠΎΡ‚ ΠΏΠ°Ρ€Ρ‹ осСй x ΠΈ y:

    I_ {xy} = \ iint_A xy dA

    , Π³Π΄Π΅ A – ΠΏΠ»ΠΎΡ‰Π°Π΄ΡŒ Ρ„ΠΎΡ€ΠΌΡ‹, Π° x, y – расстояния любой Ρ‚ΠΎΡ‡ΠΊΠΈ Π²Π½ΡƒΡ‚Ρ€ΠΈ области A ΠΎΡ‚ ΡΠΎΠΎΡ‚Π²Π΅Ρ‚ΡΡ‚Π²ΡƒΡŽΡ‰ΠΈΡ… осСй.

    Если ΠΎΠ΄Π½Π° ΠΈΠ· Π΄Π²ΡƒΡ… осСй Ρ‚Π°ΠΊΠΆΠ΅ являСтся осью симмСтрии, Ρ‚ΠΎ I_ {xy} = 0.

    Π’Π°ΠΊΠΆΠ΅ ΠΎΠ±Ρ€Π°Ρ‚ΠΈΡ‚Π΅ Π²Π½ΠΈΠΌΠ°Π½ΠΈΠ΅, Ρ‡Ρ‚ΠΎ Π² ΠΎΡ‚Π»ΠΈΡ‡ΠΈΠ΅ ΠΎΡ‚ Π²Ρ‚ΠΎΡ€ΠΎΠ³ΠΎ ΠΌΠΎΠΌΠ΅Π½Ρ‚Π° ΠΏΠ»ΠΎΡ‰Π°Π΄ΠΈ, ΠΏΡ€ΠΎΠΈΠ·Π²Π΅Π΄Π΅Π½ΠΈΠ΅ ΠΈΠ½Π΅Ρ€Ρ†ΠΈΠΈ ΠΌΠΎΠΆΠ΅Ρ‚ ΠΏΡ€ΠΈΠ½ΠΈΠΌΠ°Ρ‚ΡŒ ΠΎΡ‚Ρ€ΠΈΡ†Π°Ρ‚Π΅Π»ΡŒΠ½Ρ‹Π΅ значСния.

    Π”ΠΎΠΏΠΎΠ»Π½ΠΈΡ‚Π΅Π»ΡŒΠ½Π°Ρ информация

    ΠŸΠΎΠ½Ρ€Π°Π²ΠΈΠ»Π°ΡΡŒ страница? ПодСлись с Π΄Ρ€ΡƒΠ·ΡŒΡΠΌΠΈ!

    Как Ρ€Π°ΡΡΡ‡ΠΈΡ‚Π°Ρ‚ΡŒ ΠΌΠΎΠΌΠ΅Π½Ρ‚ ΠΈΠ½Π΅Ρ€Ρ†ΠΈΠΈ Π±Π°Π»ΠΊΠΈ?

    Ρ€Π°Π·ΠΌΠ΅Ρ€ ΡˆΡ€ΠΈΡ„Ρ‚Π°: 15 пиксСлСй;
    }
    ]]>

    Как Ρ€Π°ΡΡΡ‡ΠΈΡ‚Π°Ρ‚ΡŒ ΠΌΠΎΠΌΠ΅Π½Ρ‚ ΠΈΠ½Π΅Ρ€Ρ†ΠΈΠΈ сСкции Π±Π°Π»ΠΊΠΈ
    (Π²Ρ‚ΠΎΡ€ΠΎΠΉ ΠΌΠΎΠΌΠ΅Π½Ρ‚ ΠΏΠ»ΠΎΡ‰Π°Π΄ΠΈ)

    ΠŸΡ€Π΅ΠΆΠ΄Π΅ Ρ‡Π΅ΠΌ ΠΌΡ‹ Π½Π°ΠΉΠ΄Π΅ΠΌ ΠΌΠΎΠΌΠ΅Π½Ρ‚ ΠΈΠ½Π΅Ρ€Ρ†ΠΈΠΈ (ΠΈΠ»ΠΈ Π²Ρ‚ΠΎΡ€ΠΎΠΉ ΠΌΠΎΠΌΠ΅Π½Ρ‚ ΠΏΠ»ΠΎΡ‰Π°Π΄ΠΈ) сСчСния Π±Π°Π»ΠΊΠΈ, Π½Π΅ΠΎΠ±Ρ…ΠΎΠ΄ΠΈΠΌΠΎ Π·Π½Π°Ρ‚ΡŒ Π΅Π΅ Ρ†Π΅Π½Ρ‚Ρ€ΠΎΠΈΠ΄ (ΠΈΠ»ΠΈ Ρ†Π΅Π½Ρ‚Ρ€ масс).НапримСр, Ссли трСбуСтся ΠΌΠΎΠΌΠ΅Π½Ρ‚ ΠΈΠ½Π΅Ρ€Ρ†ΠΈΠΈ сСкции ΠΎΡ‚Π½ΠΎΡΠΈΡ‚Π΅Π»ΡŒΠ½ΠΎ Π΅Π΅ Π³ΠΎΡ€ΠΈΠ·ΠΎΠ½Ρ‚Π°Π»ΡŒΠ½ΠΎΠΉ (XX) оси, Ρ‚ΠΎΠ³Π΄Π° сначала потрСбуСтся Π²Π΅Ρ€Ρ‚ΠΈΠΊΠ°Π»ΡŒΠ½Ρ‹ΠΉ (y) Ρ†Π΅Π½Ρ‚Ρ€ΠΎΠΈΠ΄ (поТалуйста, просмотритС нашС Π£Ρ‡Π΅Π±Π½ΠΎΠ΅ пособиС ΠΎ Ρ‚ΠΎΠΌ, ΠΊΠ°ΠΊ Ρ€Π°ΡΡΡ‡ΠΈΡ‚Π°Ρ‚ΡŒ Ρ†Π΅Π½Ρ‚Ρ€ΠΎΠΈΠ΄ сСкции Π±Π°Π»ΠΊΠΈ).

    ΠŸΡ€Π΅ΠΆΠ΄Π΅ Ρ‡Π΅ΠΌ ΠΌΡ‹ Π½Π°Ρ‡Π½Π΅ΠΌ, Ссли Π²Ρ‹ искали наш ΠΊΠ°Π»ΡŒΠΊΡƒΠ»ΡΡ‚ΠΎΡ€ свободного ΠΌΠΎΠΌΠ΅Π½Ρ‚Π° ΠΈΠ½Π΅Ρ€Ρ†ΠΈΠΈ, Ρ‰Π΅Π»ΠΊΠ½ΠΈΡ‚Π΅ ссылку, Ρ‡Ρ‚ΠΎΠ±Ρ‹ ΡƒΠ·Π½Π°Ρ‚ΡŒ большС. Π­Ρ‚ΠΎ вычислит Ρ†Π΅Π½Ρ‚Ρ€ΠΎΠΈΠ΄, moi ΠΈ Π΄Ρ€ΡƒΠ³ΠΈΠ΅ Ρ€Π΅Π·ΡƒΠ»ΡŒΡ‚Π°Ρ‚Ρ‹ ΠΈ Π΄Π°ΠΆΠ΅ ΠΏΠΎΠΊΠ°ΠΆΠ΅Ρ‚ Π²Π°ΠΌ ΠΏΠΎΡˆΠ°Π³ΠΎΠ²Ρ‹Π΅ вычислСния! А ΠΏΠΎΠΊΠ° Π΄Π°Π²Π°ΠΉΡ‚Π΅ посмотрим Π½Π° пошаговоС руководство ΠΈ ΠΏΡ€ΠΈΠΌΠ΅Ρ€ Ρ‚ΠΎΠ³ΠΎ, ΠΊΠ°ΠΊ Ρ€Π°ΡΡΡ‡ΠΈΡ‚Π°Ρ‚ΡŒ ΠΌΠΎΠΌΠ΅Π½Ρ‚ ΠΈΠ½Π΅Ρ€Ρ†ΠΈΠΈ:

    Π¨Π°Π³ 1. Π Π°Π·Π΄Π΅Π»ΠΈΡ‚Π΅ ΡΠ΅ΠΊΡ†ΠΈΡŽ Π±Π°Π»ΠΊΠΈ Π½Π° части

    ΠŸΡ€ΠΈ вычислСнии ΠΌΠΎΠΌΠ΅Π½Ρ‚Π° ΠΈΠ½Π΅Ρ€Ρ†ΠΈΠΈ ΠΏΠ»ΠΎΡ‰Π°Π΄ΠΈ ΠΌΡ‹ Π΄ΠΎΠ»ΠΆΠ½Ρ‹ Π²Ρ‹Ρ‡ΠΈΡΠ»ΠΈΡ‚ΡŒ ΠΌΠΎΠΌΠ΅Π½Ρ‚ ΠΈΠ½Π΅Ρ€Ρ†ΠΈΠΈ ΠΌΠ΅Π½ΡŒΡˆΠΈΡ… сСгмСнтов.ΠŸΠΎΠΏΡ€ΠΎΠ±ΡƒΠΉΡ‚Π΅ Ρ€Π°Π·Π±ΠΈΡ‚ΡŒ ΠΈΡ… Π½Π° простыС ΠΏΡ€ΡΠΌΠΎΡƒΠ³ΠΎΠ»ΡŒΠ½Ρ‹Π΅ сСкции. НапримСр, рассмотрим ΡΠ΅ΠΊΡ†ΠΈΡŽ Π΄Π²ΡƒΡ‚Π°Π²Ρ€ΠΎΠ²ΠΎΠΉ Π±Π°Π»ΠΊΠΈ Π½ΠΈΠΆΠ΅, которая Ρ‚Π°ΠΊΠΆΠ΅ Π±Ρ‹Π»Π° прСдставлСна ​​в нашСм руководствС ΠΏΠΎ Ρ†Π΅Π½Ρ‚Ρ€ΠΎΠΈΠ΄Π°ΠΌ. ΠœΡ‹ Ρ€Π΅ΡˆΠΈΠ»ΠΈ Ρ€Π°Π·Π΄Π΅Π»ΠΈΡ‚ΡŒ эту ΡΠ΅ΠΊΡ†ΠΈΡŽ Π½Π° 3 ΠΏΡ€ΡΠΌΠΎΡƒΠ³ΠΎΠ»ΡŒΠ½Ρ‹Ρ… сСгмСнта:

    Π¨Π°Π³ 2: РасчСт Π½Π΅ΠΉΡ‚Ρ€Π°Π»ΡŒΠ½ΠΎΠΉ оси (NA)

    ΠΠ΅ΠΉΡ‚Ρ€Π°Π»ΡŒΠ½Π°Ρ ось (NA) ΠΈΠ»ΠΈ Π³ΠΎΡ€ΠΈΠ·ΠΎΠ½Ρ‚Π°Π»ΡŒΠ½Π°Ρ ось XX располоТСна Π² Ρ†Π΅Π½Ρ‚Ρ€Π΅ тяТСсти ΠΈΠ»ΠΈ Ρ†Π΅Π½Ρ‚Ρ€Π΅ масс. Π’ нашСм руководствС ΠΏΠΎ Ρ†Π΅Π½Ρ‚Ρ€ΠΎΠΈΠ΄Π°ΠΌ Ρ†Π΅Π½Ρ‚Ρ€ тяТСсти этой сСкции Ρ€Π°Π½Π΅Π΅ находился Π½Π° расстоянии 216,29 ΠΌΠΌ ΠΎΡ‚ Π½ΠΈΠΆΠ½Π΅ΠΉ части сСкции.

    Π¨Π°Π³ 3: РасчСт ΠΌΠΎΠΌΠ΅Π½Ρ‚Π° ΠΈΠ½Π΅Ρ€Ρ†ΠΈΠΈ

    Для расчСта ΠΏΠΎΠ»Π½ΠΎΠ³ΠΎ ΠΌΠΎΠΌΠ΅Π½Ρ‚Π° ΠΈΠ½Π΅Ρ€Ρ†ΠΈΠΈ сСкции Π½Π°ΠΌ Π½Π΅ΠΎΠ±Ρ…ΠΎΠ΄ΠΈΠΌΠΎ ΠΈΡΠΏΠΎΠ»ΡŒΠ·ΠΎΠ²Π°Ρ‚ΡŒ Β«Π’Π΅ΠΎΡ€Π΅ΠΌΡƒ ΠΎ ΠΏΠ°Ρ€Π°Π»Π»Π΅Π»ΡŒΠ½ΠΎΡΡ‚ΠΈ оси»:

    ΠŸΠΎΡΠΊΠΎΠ»ΡŒΠΊΡƒ ΠΌΡ‹ Ρ€Π°Π·Π΄Π΅Π»ΠΈΠ»ΠΈ Π΅Π³ΠΎ Π½Π° Ρ‚Ρ€ΠΈ ΠΏΡ€ΡΠΌΠΎΡƒΠ³ΠΎΠ»ΡŒΠ½Ρ‹Π΅ части, ΠΌΡ‹ Π΄ΠΎΠ»ΠΆΠ½Ρ‹ Π²Ρ‹Ρ‡ΠΈΡΠ»ΠΈΡ‚ΡŒ ΠΌΠΎΠΌΠ΅Π½Ρ‚ ΠΈΠ½Π΅Ρ€Ρ†ΠΈΠΈ ΠΊΠ°ΠΆΠ΄ΠΎΠΉ ΠΈΠ· этих частСй. Π¨ΠΈΡ€ΠΎΠΊΠΎ извСстно, Ρ‡Ρ‚ΠΎ ΡƒΡ€Π°Π²Π½Π΅Π½ΠΈΠ΅ ΠΌΠΎΠΌΠ΅Π½Ρ‚Π° ΠΈΠ½Π΅Ρ€Ρ†ΠΈΠΈ ΠΏΡ€ΡΠΌΠΎΡƒΠ³ΠΎΠ»ΡŒΠ½ΠΈΠΊΠ° ΠΎΡ‚Π½ΠΎΡΠΈΡ‚Π΅Π»ΡŒΠ½ΠΎ Π΅Π³ΠΎ Ρ†Π΅Π½Ρ‚Ρ€Π°Π»ΡŒΠ½ΠΎΠΉ оси ΠΈΠΌΠ΅Π΅Ρ‚ простой Π²ΠΈΠ΄:

    ΠœΠΎΠΌΠ΅Π½Ρ‚ ΠΈΠ½Π΅Ρ€Ρ†ΠΈΠΈ Π΄Ρ€ΡƒΠ³ΠΈΡ… Ρ„ΠΎΡ€ΠΌ часто указываСтся Π½Π° Π»ΠΈΡ†Π΅Π²ΠΎΠΉ / ΠΎΠ±ΠΎΡ€ΠΎΡ‚Π½ΠΎΠΉ сторонС ΡƒΡ‡Π΅Π±Π½ΠΈΠΊΠΎΠ² ΠΈΠ»ΠΈ Π² этом руководствС ΠΏΠΎ Ρ„ΠΎΡ€ΠΌΠ°ΠΌ ΠΌΠΎΠΌΠ΅Π½Ρ‚Π° ΠΈΠ½Π΅Ρ€Ρ†ΠΈΠΈ.Однако ΠΏΡ€ΡΠΌΠΎΡƒΠ³ΠΎΠ»ΡŒΠ½Π°Ρ Ρ„ΠΎΡ€ΠΌΠ° ΠΎΡ‡Π΅Π½ΡŒ Ρ…Π°Ρ€Π°ΠΊΡ‚Π΅Ρ€Π½Π° для сСчСний Π±Π°Π»ΠΎΠΊ, поэтому, Π½Π°Π²Π΅Ρ€Π½ΠΎΠ΅, стоит Π·Π°ΠΏΠΎΠΌΠ½ΠΈΡ‚ΡŒ.

    Π’Π΅ΠΏΠ΅Ρ€ΡŒ Ρƒ нас Π΅ΡΡ‚ΡŒ вся информация, нСобходимая для использования Β«Π’Π΅ΠΎΡ€Π΅ΠΌΡ‹ ΠΎ ΠΏΠ°Ρ€Π°Π»Π»Π΅Π»ΡŒΠ½ΠΎΠΉ оси» ΠΈ опрСдСлСния ΠΏΠΎΠ»Π½ΠΎΠ³ΠΎ ΠΌΠΎΠΌΠ΅Π½Ρ‚Π° ΠΈΠ½Π΅Ρ€Ρ†ΠΈΠΈ Π΄Π²ΡƒΡ‚Π°Π²Ρ€ΠΎΠ²ΠΎΠΉ Π±Π°Π»ΠΊΠΈ. Π’ нашСм ΠΏΡ€ΠΈΠΌΠ΅Ρ€Π΅ ΠΌΠΎΠΌΠ΅Π½Ρ‚Π° ΠΈΠ½Π΅Ρ€Ρ†ΠΈΠΈ:

    Π˜Ρ‚Π°ΠΊ, Ρƒ вас Π΅ΡΡ‚ΡŒ руководство ΠΏΠΎ расчСту ΠΏΠ»ΠΎΡ‰Π°Π΄ΠΈ ΠΌΠΎΠΌΠ΅Π½Ρ‚Π° для сСкций Π±Π°Π»ΠΊΠΈ. Π­Ρ‚ΠΎΡ‚ Ρ€Π΅Π·ΡƒΠ»ΡŒΡ‚Π°Ρ‚ ΠΈΠΌΠ΅Π΅Ρ‚ Ρ€Π΅ΡˆΠ°ΡŽΡ‰Π΅Π΅ Π·Π½Π°Ρ‡Π΅Π½ΠΈΠ΅ ΠΏΡ€ΠΈ ΠΏΡ€ΠΎΠ΅ΠΊΡ‚ΠΈΡ€ΠΎΠ²Π°Π½ΠΈΠΈ конструкций ΠΈ являСтся Π²Π°ΠΆΠ½Ρ‹ΠΌ Ρ„Π°ΠΊΡ‚ΠΎΡ€ΠΎΠΌ отклонСния Π±Π°Π»ΠΊΠΈ.ΠœΡ‹ надССмся, Ρ‡Ρ‚ΠΎ Π²Π°ΠΌ ΠΏΠΎΠ½Ρ€Π°Π²ΠΈΠ»ΠΎΡΡŒ это руководство, ΠΈ с Π½Π΅Ρ‚Π΅Ρ€ΠΏΠ΅Π½ΠΈΠ΅ΠΌ ΠΆΠ΄Π΅ΠΌ Π²Π°ΡˆΠΈΡ… ΠΊΠΎΠΌΠΌΠ΅Π½Ρ‚Π°Ρ€ΠΈΠ΅Π².

    Π‘ΠžΠΠ£Π‘: ИспользованиС нашСго ΠΊΠ°Π»ΡŒΠΊΡƒΠ»ΡΡ‚ΠΎΡ€Π° ΠΌΠΎΠΌΠ΅Π½Ρ‚Π° ΠΈΠ½Π΅Ρ€Ρ†ΠΈΠΈ УчСтная запись

    SkyCiv ΠΏΠΎΠΊΠ°Π·Ρ‹Π²Π°Π΅Ρ‚ ΠΏΠΎΠ»Π½Ρ‹Π΅ расчСты ΠΌΠΎΠΌΠ΅Π½Ρ‚Π° ΠΈΠ½Π΅Ρ€Ρ†ΠΈΠΈ. Π­Ρ‚ΠΎΡ‚ ΠΈΠ½Ρ‚Π΅Ρ€Π°ΠΊΡ‚ΠΈΠ²Π½Ρ‹ΠΉ ΠΌΠΎΠ΄ΡƒΠ»ΡŒ ΠΏΠΎΠΊΠ°ΠΆΠ΅Ρ‚ Π²Π°ΠΌ ΠΏΠΎΡˆΠ°Π³ΠΎΠ²Ρ‹Π΅ расчСты Ρ‚ΠΎΠ³ΠΎ, ΠΊΠ°ΠΊ Π½Π°ΠΉΡ‚ΠΈ ΠΌΠΎΠΌΠ΅Π½Ρ‚ ΠΈΠ½Π΅Ρ€Ρ†ΠΈΠΈ:

    Π’Ρ‹ Ρ‚Π°ΠΊΠΆΠ΅ ΠΌΠΎΠΆΠ΅Ρ‚Π΅ ΠΏΠΎΡΠΌΠΎΡ‚Ρ€Π΅Ρ‚ΡŒ Ρ€Π΅Π·ΡƒΠ»ΡŒΡ‚Π°Ρ‚Ρ‹ нашСго ΠΊΠ°Π»ΡŒΠΊΡƒΠ»ΡΡ‚ΠΎΡ€Π° свободного ΠΌΠΎΠΌΠ΅Π½Ρ‚Π° ΠΈΠ½Π΅Ρ€Ρ†ΠΈΠΈ, Ρ‡Ρ‚ΠΎΠ±Ρ‹ ΠΏΡ€ΠΎΠ²Π΅Ρ€ΠΈΡ‚ΡŒ свою Ρ€Π°Π±ΠΎΡ‚Ρƒ. Π­Ρ‚ΠΎ вычислит всС свойства вашСго ΠΏΠΎΠΏΠ΅Ρ€Π΅Ρ‡Π½ΠΎΠ³ΠΎ сСчСния ΠΈ являСтся ΠΏΠΎΠ»Π΅Π·Π½Ρ‹ΠΌ справочным ΠΌΠ°Ρ‚Π΅Ρ€ΠΈΠ°Π»ΠΎΠΌ для расчСта Ρ†Π΅Π½Ρ‚Ρ€ΠΎΠΈΠ΄Π°, ΠΏΠ»ΠΎΡ‰Π°Π΄ΠΈ ΠΈ ΠΌΠΎΠΌΠ΅Π½Ρ‚Π° ΠΈΠ½Π΅Ρ€Ρ†ΠΈΠΈ сСчСний вашСй Π±Π°Π»ΠΊΠΈ!

    ΠšΠ°Π»ΡŒΠΊΡƒΠ»ΡΡ‚ΠΎΡ€ свободного ΠΌΠΎΠΌΠ΅Π½Ρ‚Π° ΠΈΠ½Π΅Ρ€Ρ†ΠΈΠΈ

    Π€ΠΎΡ€ΠΌΡƒΠ»Π° ΠΌΠΎΠΌΠ΅Π½Ρ‚Π° ΠΈΠ½Π΅Ρ€Ρ†ΠΈΠΈ ΠΈ Π΄Ρ€ΡƒΠ³ΠΈΠ΅ физичСскиС Ρ„ΠΎΡ€ΠΌΡƒΠ»Ρ‹

    ΠœΠΎΠΌΠ΅Π½Ρ‚ ΠΈΠ½Π΅Ρ€Ρ†ΠΈΠΈ ΠΎΠ±ΡŠΠ΅ΠΊΡ‚Π° – это числовоС Π·Π½Π°Ρ‡Π΅Π½ΠΈΠ΅, ΠΊΠΎΡ‚ΠΎΡ€ΠΎΠ΅ ΠΌΠΎΠΆΠ΅Ρ‚ Π±Ρ‹Ρ‚ΡŒ вычислСно для любого Ρ‚Π²Π΅Ρ€Π΄ΠΎΠ³ΠΎ Ρ‚Π΅Π»Π°, ΠΊΠΎΡ‚ΠΎΡ€ΠΎΠ΅ ΡΠΎΠ²Π΅Ρ€ΡˆΠ°Π΅Ρ‚ физичСскоС Π²Ρ€Π°Ρ‰Π΅Π½ΠΈΠ΅ Π²ΠΎΠΊΡ€ΡƒΠ³ фиксированной оси.Он основан Π½Π΅ Ρ‚ΠΎΠ»ΡŒΠΊΠΎ Π½Π° физичСской Ρ„ΠΎΡ€ΠΌΠ΅ ΠΎΠ±ΡŠΠ΅ΠΊΡ‚Π° ΠΈ Π΅Π³ΠΎ распрСдСлСнии массы, Π½ΠΎ ΠΈ Π½Π° ΠΊΠΎΠ½ΠΊΡ€Π΅Ρ‚Π½ΠΎΠΉ ΠΊΠΎΠ½Ρ„ΠΈΠ³ΡƒΡ€Π°Ρ†ΠΈΠΈ Ρ‚ΠΎΠ³ΠΎ, ΠΊΠ°ΠΊ ΠΎΠ±ΡŠΠ΅ΠΊΡ‚ вращаСтся. Π’Π°ΠΊΠΈΠΌ ΠΎΠ±Ρ€Π°Π·ΠΎΠΌ, ΠΎΠ΄ΠΈΠ½ ΠΈ Ρ‚ΠΎΡ‚ ΠΆΠ΅ ΠΎΠ±ΡŠΠ΅ΠΊΡ‚, Π²Ρ€Π°Ρ‰Π°ΡŽΡ‰ΠΈΠΉΡΡ ΠΏΠΎ-Ρ€Π°Π·Π½ΠΎΠΌΡƒ, Π±ΡƒΠ΄Π΅Ρ‚ ΠΈΠΌΠ΅Ρ‚ΡŒ Ρ€Π°Π·Π½Ρ‹ΠΉ ΠΌΠΎΠΌΠ΅Π½Ρ‚ ΠΈΠ½Π΅Ρ€Ρ†ΠΈΠΈ Π² ΠΊΠ°ΠΆΠ΄ΠΎΠΉ ситуации.

    ΠžΠ±Ρ‰Π°Ρ Ρ„ΠΎΡ€ΠΌΡƒΠ»Π°

    ΠžΠ±Ρ‰Π°Ρ Ρ„ΠΎΡ€ΠΌΡƒΠ»Π° для опрСдСлСния ΠΌΠΎΠΌΠ΅Π½Ρ‚Π° ΠΈΠ½Π΅Ρ€Ρ†ΠΈΠΈ. Π­Π½Π΄Ρ€ΡŽ Π¦ΠΈΠΌΠΌΠ΅Ρ€ΠΌΠ°Π½ ДТонс

    ΠžΠ±Ρ‰Π°Ρ Ρ„ΠΎΡ€ΠΌΡƒΠ»Π° прСдставляСт собой самоС основноС ΠΊΠΎΠ½Ρ†Π΅ΠΏΡ‚ΡƒΠ°Π»ΡŒΠ½ΠΎΠ΅ ΠΏΠΎΠ½ΠΈΠΌΠ°Π½ΠΈΠ΅ ΠΌΠΎΠΌΠ΅Π½Ρ‚Π° ΠΈΠ½Π΅Ρ€Ρ†ΠΈΠΈ. Π’ ΠΏΡ€ΠΈΠ½Ρ†ΠΈΠΏΠ΅, для любого Π²Ρ€Π°Ρ‰Π°ΡŽΡ‰Π΅Π³ΠΎΡΡ ΠΎΠ±ΡŠΠ΅ΠΊΡ‚Π° ΠΌΠΎΠΌΠ΅Π½Ρ‚ ΠΈΠ½Π΅Ρ€Ρ†ΠΈΠΈ ΠΌΠΎΠΆΠ½ΠΎ Ρ€Π°ΡΡΡ‡ΠΈΡ‚Π°Ρ‚ΡŒ, взяв расстояниС ΠΊΠ°ΠΆΠ΄ΠΎΠΉ частицы ΠΎΡ‚ оси вращСния ( r Π² ΡƒΡ€Π°Π²Π½Π΅Π½ΠΈΠΈ), возвСдя это Π·Π½Π°Ρ‡Π΅Π½ΠΈΠ΅ Π² ΠΊΠ²Π°Π΄Ρ€Π°Ρ‚ (это Ρ‡Π»Π΅Π½ r 2 ), ΠΈ умноТая Π΅Π³ΠΎ Π½Π° массу этой частицы.Π’Ρ‹ Π΄Π΅Π»Π°Π΅Ρ‚Π΅ это для всСх частиц, ΡΠΎΡΡ‚Π°Π²Π»ΡΡŽΡ‰ΠΈΡ… Π²Ρ€Π°Ρ‰Π°ΡŽΡ‰ΠΈΠΉΡΡ ΠΎΠ±ΡŠΠ΅ΠΊΡ‚, Π° Π·Π°Ρ‚Π΅ΠΌ складываСтС эти значСния вмСстС, ΠΈ это Π΄Π°Π΅Ρ‚ ΠΌΠΎΠΌΠ΅Π½Ρ‚ ΠΈΠ½Π΅Ρ€Ρ†ΠΈΠΈ.

    БлСдствиСм этой Ρ„ΠΎΡ€ΠΌΡƒΠ»Ρ‹ являСтся Ρ‚ΠΎ, Ρ‡Ρ‚ΠΎ ΠΎΠ΄ΠΈΠ½ ΠΈ Ρ‚ΠΎΡ‚ ΠΆΠ΅ ΠΎΠ±ΡŠΠ΅ΠΊΡ‚ ΠΏΠΎΠ»ΡƒΡ‡Π°Π΅Ρ‚ Ρ€Π°Π·Π½ΠΎΠ΅ Π·Π½Π°Ρ‡Π΅Π½ΠΈΠ΅ ΠΌΠΎΠΌΠ΅Π½Ρ‚Π° ΠΈΠ½Π΅Ρ€Ρ†ΠΈΠΈ Π² зависимости ΠΎΡ‚ Ρ‚ΠΎΠ³ΠΎ, ΠΊΠ°ΠΊ ΠΎΠ½ вращаСтся. Новая ось вращСния заканчиваСтся Π΄Ρ€ΡƒΠ³ΠΎΠΉ Ρ„ΠΎΡ€ΠΌΡƒΠ»ΠΎΠΉ, Π΄Π°ΠΆΠ΅ Ссли физичСская Ρ„ΠΎΡ€ΠΌΠ° ΠΎΠ±ΡŠΠ΅ΠΊΡ‚Π° остаСтся ΠΏΡ€Π΅ΠΆΠ½Π΅ΠΉ.

    Π­Ρ‚Π° Ρ„ΠΎΡ€ΠΌΡƒΠ»Π° прСдставляСт собой Π½Π°ΠΈΠ±ΠΎΠ»Π΅Π΅ Π³Ρ€ΡƒΠ±Ρ‹ΠΉ ΠΏΠΎΠ΄Ρ…ΠΎΠ΄ ΠΊ Π²Ρ‹Ρ‡ΠΈΡΠ»Π΅Π½ΠΈΡŽ ΠΌΠΎΠΌΠ΅Π½Ρ‚Π° ΠΈΠ½Π΅Ρ€Ρ†ΠΈΠΈ.Π”Ρ€ΡƒΠ³ΠΈΠ΅ ΠΏΡ€ΠΈΠ²Π΅Π΄Π΅Π½Π½Ρ‹Π΅ Ρ„ΠΎΡ€ΠΌΡƒΠ»Ρ‹ ΠΎΠ±Ρ‹Ρ‡Π½ΠΎ Π±ΠΎΠ»Π΅Π΅ ΠΏΠΎΠ»Π΅Π·Π½Ρ‹ ΠΈ ΠΏΡ€Π΅Π΄ΡΡ‚Π°Π²Π»ΡΡŽΡ‚ собой Π½Π°ΠΈΠ±ΠΎΠ»Π΅Π΅ распространСнныС ситуации, с ΠΊΠΎΡ‚ΠΎΡ€Ρ‹ΠΌΠΈ ΡΡ‚Π°Π»ΠΊΠΈΠ²Π°ΡŽΡ‚ΡΡ Ρ„ΠΈΠ·ΠΈΠΊΠΈ.

    Π˜Π½Ρ‚Π΅Π³Ρ€Π°Π»ΡŒΠ½Π°Ρ Ρ„ΠΎΡ€ΠΌΡƒΠ»Π°

    ΠžΠ±Ρ‰Π°Ρ Ρ„ΠΎΡ€ΠΌΡƒΠ»Π° ΠΏΠΎΠ»Π΅Π·Π½Π°, Ссли ΠΎΠ±ΡŠΠ΅ΠΊΡ‚ ΠΌΠΎΠΆΠ½ΠΎ Ρ€Π°ΡΡΠΌΠ°Ρ‚Ρ€ΠΈΠ²Π°Ρ‚ΡŒ ΠΊΠ°ΠΊ Π½Π°Π±ΠΎΡ€ дискрСтных Ρ‚ΠΎΡ‡Π΅ΠΊ, ΠΊΠΎΡ‚ΠΎΡ€Ρ‹Π΅ ΠΌΠΎΠΆΠ½ΠΎ ΡΠΊΠ»Π°Π΄Ρ‹Π²Π°Ρ‚ΡŒ. Однако для Π±ΠΎΠ»Π΅Π΅ слоТного ΠΎΠ±ΡŠΠ΅ΠΊΡ‚Π° ΠΌΠΎΠΆΠ΅Ρ‚ ΠΏΠΎΡ‚Ρ€Π΅Π±ΠΎΠ²Π°Ρ‚ΡŒΡΡ ΠΏΡ€ΠΈΠΌΠ΅Π½ΠΈΡ‚ΡŒ исчислСниС, Ρ‡Ρ‚ΠΎΠ±Ρ‹ Π²Π·ΡΡ‚ΡŒ ΠΈΠ½Ρ‚Π΅Π³Ρ€Π°Π» ΠΏΠΎ всСму ΠΎΠ±ΡŠΠ΅ΠΌΡƒ. ΠŸΠ΅Ρ€Π΅ΠΌΠ΅Π½Π½Π°Ρ r – это радиус-Π²Π΅ΠΊΡ‚ΠΎΡ€ ΠΎΡ‚ Ρ‚ΠΎΡ‡ΠΊΠΈ Π΄ΠΎ оси вращСния.Π€ΠΎΡ€ΠΌΡƒΠ»Π° p ( r ) прСдставляСт собой Ρ„ΡƒΠ½ΠΊΡ†ΠΈΡŽ плотности массы Π² ΠΊΠ°ΠΆΠ΄ΠΎΠΉ Ρ‚ΠΎΡ‡ΠΊΠ΅ r:

    I-sub-P Ρ€Π°Π²Π½ΠΎ суммС i ΠΎΡ‚ 1 Π΄ΠΎ N количСства m-sub-i, ΡƒΠΌΠ½ΠΎΠΆΠ΅Π½Π½ΠΎΠ³ΠΎ Π½Π° r-sub-i Π² ΠΊΠ²Π°Π΄Ρ€Π°Ρ‚Π΅.

    ВвСрдая сфСра

    ВвСрдая сфСра, Π²Ρ€Π°Ρ‰Π°ΡŽΡ‰Π°ΡΡΡ Π²ΠΎΠΊΡ€ΡƒΠ³ оси, проходящСй Ρ‡Π΅Ρ€Π΅Π· Ρ†Π΅Π½Ρ‚Ρ€ сфСры, с массой M ΠΈ радиусом R , ΠΈΠΌΠ΅Π΅Ρ‚ ΠΌΠΎΠΌΠ΅Π½Ρ‚ ΠΈΠ½Π΅Ρ€Ρ†ΠΈΠΈ, опрСдСляСмый ΠΏΠΎ Ρ„ΠΎΡ€ΠΌΡƒΠ»Π΅:

    I = (2/5) MR 2

    Полая тонкостСнная сфСра

    Полая сфСра с Ρ‚ΠΎΠ½ΠΊΠΎΠΉ Π½Π΅Π·Π½Π°Ρ‡ΠΈΡ‚Π΅Π»ΡŒΠ½ΠΎΠΉ стСнкой, Π²Ρ€Π°Ρ‰Π°ΡŽΡ‰Π΅ΠΉΡΡ Π²ΠΎΠΊΡ€ΡƒΠ³ оси, проходящСй Ρ‡Π΅Ρ€Π΅Π· Ρ†Π΅Π½Ρ‚Ρ€ сфСры, с массой M ΠΈ радиусом R , ΠΈΠΌΠ΅Π΅Ρ‚ ΠΌΠΎΠΌΠ΅Π½Ρ‚ ΠΈΠ½Π΅Ρ€Ρ†ΠΈΠΈ, опрСдСляСмый ΠΏΠΎ Ρ„ΠΎΡ€ΠΌΡƒΠ»Π΅:

    I = (2/3) MR 2

    Π¦Π΅Π»ΡŒΠ½Ρ‹ΠΉ Ρ†ΠΈΠ»ΠΈΠ½Π΄Ρ€

    Бплошной Ρ†ΠΈΠ»ΠΈΠ½Π΄Ρ€, Π²Ρ€Π°Ρ‰Π°ΡŽΡ‰ΠΈΠΉΡΡ Π²ΠΎΠΊΡ€ΡƒΠ³ оси, проходящСй Ρ‡Π΅Ρ€Π΅Π· Ρ†Π΅Π½Ρ‚Ρ€ Ρ†ΠΈΠ»ΠΈΠ½Π΄Ρ€Π°, с массой M ΠΈ радиусом R , ΠΈΠΌΠ΅Π΅Ρ‚ ΠΌΠΎΠΌΠ΅Π½Ρ‚ ΠΈΠ½Π΅Ρ€Ρ†ΠΈΠΈ, опрСдСляСмый ΠΏΠΎ Ρ„ΠΎΡ€ΠΌΡƒΠ»Π΅:

    I = (1/2) MR 2

    ΠŸΠΎΠ»Ρ‹ΠΉ тонкостСнный Ρ†ΠΈΠ»ΠΈΠ½Π΄Ρ€

    ΠŸΠΎΠ»Ρ‹ΠΉ Ρ†ΠΈΠ»ΠΈΠ½Π΄Ρ€ с Ρ‚ΠΎΠ½ΠΊΠΎΠΉ Π½Π΅Π·Π½Π°Ρ‡ΠΈΡ‚Π΅Π»ΡŒΠ½ΠΎΠΉ стСнкой, Π²Ρ€Π°Ρ‰Π°ΡŽΡ‰Π΅ΠΉΡΡ Π²ΠΎΠΊΡ€ΡƒΠ³ оси, проходящСй Ρ‡Π΅Ρ€Π΅Π· Ρ†Π΅Π½Ρ‚Ρ€ Ρ†ΠΈΠ»ΠΈΠ½Π΄Ρ€Π°, с массой M ΠΈ радиусом R , ΠΈΠΌΠ΅Π΅Ρ‚ ΠΌΠΎΠΌΠ΅Π½Ρ‚ ΠΈΠ½Π΅Ρ€Ρ†ΠΈΠΈ, опрСдСляСмый ΠΏΠΎ Ρ„ΠΎΡ€ΠΌΡƒΠ»Π΅:

    Π― = MR 2

    ΠŸΠΎΠ»Ρ‹ΠΉ Ρ†ΠΈΠ»ΠΈΠ½Π΄Ρ€

    ΠŸΠΎΠ»Ρ‹ΠΉ Ρ†ΠΈΠ»ΠΈΠ½Π΄Ρ€ с Π²Ρ€Π°Ρ‰Π°ΡŽΡ‰Π΅ΠΉΡΡ Π²ΠΎΠΊΡ€ΡƒΠ³ оси, проходящСй Ρ‡Π΅Ρ€Π΅Π· Ρ†Π΅Π½Ρ‚Ρ€ Ρ†ΠΈΠ»ΠΈΠ½Π΄Ρ€Π°, с массой M , Π²Π½ΡƒΡ‚Ρ€Π΅Π½Π½ΠΈΠΌ радиусом R 1 ΠΈ внСшним радиусом R 2 , ΠΈΠΌΠ΅Π΅Ρ‚ ΠΌΠΎΠΌΠ΅Π½Ρ‚ ΠΈΠ½Π΅Ρ€Ρ†ΠΈΠΈ, опрСдСляСмый Ρ„ΠΎΡ€ΠΌΡƒΠ»Π°:

    I = (1/2) M ( R 1 2 + R 2 2 )

    ΠŸΡ€ΠΈΠΌΠ΅Ρ‡Π°Π½ΠΈΠ΅: Если Π²Ρ‹ взяли эту Ρ„ΠΎΡ€ΠΌΡƒΠ»Ρƒ ΠΈ установили R 1 = R 2 = R (ΠΈΠ»ΠΈ, Ρ‡Ρ‚ΠΎ Π±ΠΎΠ»Π΅Π΅ умСстно, взяли матСматичСский ΠΏΡ€Π΅Π΄Π΅Π» ΠΊΠ°ΠΊ R 1 ΠΈ R 2 с ΠΎΠ±Ρ‰ΠΈΠΌ радиусом R ), Π²Ρ‹ ΠΏΠΎΠ»ΡƒΡ‡ΠΈΡ‚Π΅ Ρ„ΠΎΡ€ΠΌΡƒΠ»Ρƒ для ΠΌΠΎΠΌΠ΅Π½Ρ‚Π° ΠΈΠ½Π΅Ρ€Ρ†ΠΈΠΈ ΠΏΠΎΠ»ΠΎΠ³ΠΎ тонкостСнного Ρ†ΠΈΠ»ΠΈΠ½Π΄Ρ€Π°.

    ΠŸΡ€ΡΠΌΠΎΡƒΠ³ΠΎΠ»ΡŒΠ½Π°Ρ пластина, Ρ†Π΅Π½Ρ‚Ρ€Π°Π»ΡŒΠ½Π°Ρ ось

    Вонкая ΠΏΡ€ΡΠΌΠΎΡƒΠ³ΠΎΠ»ΡŒΠ½Π°Ρ пластина, Π²Ρ€Π°Ρ‰Π°ΡŽΡ‰Π°ΡΡΡ Π²ΠΎΠΊΡ€ΡƒΠ³ оси, пСрпСндикулярной Ρ†Π΅Π½Ρ‚Ρ€Ρƒ пластины, с массой M ΠΈ Π΄Π»ΠΈΠ½Π°ΠΌΠΈ сторон a ΠΈ b , ΠΈΠΌΠ΅Π΅Ρ‚ ΠΌΠΎΠΌΠ΅Π½Ρ‚ ΠΈΠ½Π΅Ρ€Ρ†ΠΈΠΈ, опрСдСляСмый ΠΏΠΎ Ρ„ΠΎΡ€ΠΌΡƒΠ»Π΅:

    I = (1/12) M ( a 2 + Π± 2 )

    ΠŸΡ€ΡΠΌΠΎΡƒΠ³ΠΎΠ»ΡŒΠ½Π°Ρ пластина, ось вдоль ΠΊΡ€ΠΎΠΌΠΊΠΈ

    Вонкая ΠΏΡ€ΡΠΌΠΎΡƒΠ³ΠΎΠ»ΡŒΠ½Π°Ρ пластина, Π²Ρ€Π°Ρ‰Π°ΡŽΡ‰Π°ΡΡΡ Π²ΠΎΠΊΡ€ΡƒΠ³ оси вдоль ΠΎΠ΄Π½ΠΎΠ³ΠΎ края пластины, с массой M ΠΈ Π΄Π»ΠΈΠ½Π°ΠΌΠΈ сторон a ΠΈ b , Π³Π΄Π΅ a – расстояниС, пСрпСндикулярноС оси вращСния, ΠΈΠΌΠ΅Π΅Ρ‚ ΠΌΠΎΠΌΠ΅Π½Ρ‚ инСрция опрСдСляСтся ΠΏΠΎ Ρ„ΠΎΡ€ΠΌΡƒΠ»Π΅:

    I = (1/3) ΠΌΠ»Π½ Π»Π΅Ρ‚ 2

    Вонкая ΡˆΡ‚Π°Π½Π³Π°, Ρ†Π΅Π½Ρ‚Ρ€Π°Π»ΡŒΠ½Π°Ρ ось

    Π’ΠΎΠ½ΠΊΠΈΠΉ ΡΡ‚Π΅Ρ€ΠΆΠ΅Π½ΡŒ, Π²Ρ€Π°Ρ‰Π°ΡŽΡ‰ΠΈΠΉΡΡ Π²ΠΎΠΊΡ€ΡƒΠ³ оси, проходящСй Ρ‡Π΅Ρ€Π΅Π· Ρ†Π΅Π½Ρ‚Ρ€ стСрТня (пСрпСндикулярно Π΅Π³ΠΎ Π΄Π»ΠΈΠ½Π΅), с массой M ΠΈ Π΄Π»ΠΈΠ½ΠΎΠΉ L , ΠΈΠΌΠ΅Π΅Ρ‚ ΠΌΠΎΠΌΠ΅Π½Ρ‚ ΠΈΠ½Π΅Ρ€Ρ†ΠΈΠΈ, опрСдСляСмый ΠΏΠΎ Ρ„ΠΎΡ€ΠΌΡƒΠ»Π΅:

    I = (1/12) ΠΌΠ» 2

    Вонкая ΡˆΡ‚Π°Π½Π³Π°, сквозная ось с ΠΎΠ΄Π½ΠΎΠ³ΠΎ ΠΊΠΎΠ½Ρ†Π°

    Π’ΠΎΠ½ΠΊΠΈΠΉ ΡΡ‚Π΅Ρ€ΠΆΠ΅Π½ΡŒ, Π²Ρ€Π°Ρ‰Π°ΡŽΡ‰ΠΈΠΉΡΡ Π½Π° оси, проходящСй Ρ‡Π΅Ρ€Π΅Π· ΠΊΠΎΠ½Π΅Ρ† стСрТня (пСрпСндикулярно Π΅Π³ΠΎ Π΄Π»ΠΈΠ½Π΅), с массой M ΠΈ Π΄Π»ΠΈΠ½ΠΎΠΉ L , ΠΈΠΌΠ΅Π΅Ρ‚ ΠΌΠΎΠΌΠ΅Π½Ρ‚ ΠΈΠ½Π΅Ρ€Ρ†ΠΈΠΈ, опрСдСляСмый ΠΏΠΎ Ρ„ΠΎΡ€ΠΌΡƒΠ»Π΅:

    I = (1/3) ΠΌΠ» 2

    (ΠΌΠΎΠΌΠ΅Π½Ρ‚ ΠΈΠ½Π΅Ρ€Ρ†ΠΈΠΈ – TotalConstructionHelp)


    Π¦Π΅Π½Ρ‚Ρ€ΠΎΠΈΠ΄Ρ‹ ΠΈ ΠΌΠΎΠΌΠ΅Π½Ρ‚ ΠΈΠ½Π΅Ρ€Ρ†ΠΈΠΈ

    Π¦Π΅Π½Ρ‚Ρ€ΠΎΠΈΠ΄ Π΄Π²ΡƒΠΌΠ΅Ρ€Π½ΠΎΠΉ повСрхности (Π½Π°ΠΏΡ€ΠΈΠΌΠ΅Ρ€, ΠΏΠΎΠΏΠ΅Ρ€Π΅Ρ‡Π½ΠΎΠ΅ сСчСниС структурной Ρ„ΠΎΡ€ΠΌΡ‹) – это Ρ‚ΠΎΡ‡ΠΊΠ°, которая соотвСтствуСт Ρ†Π΅Π½Ρ‚Ρ€Ρƒ тяТСсти ΠΎΡ‡Π΅Π½ΡŒ Ρ‚ΠΎΠ½ΠΊΠΎΠΉ ΠΎΠ΄Π½ΠΎΡ€ΠΎΠ΄Π½ΠΎΠΉ пластины Ρ‚ΠΎΠΉ ΠΆΠ΅ ΠΏΠ»ΠΎΡ‰Π°Π΄ΠΈ ΠΈ Ρ„ΠΎΡ€ΠΌΡ‹.Плоская ΠΏΠΎΠ²Π΅Ρ€Ρ…Π½ΠΎΡΡ‚ΡŒ (ΠΈΠ»ΠΈ рисунок) ΠΌΠΎΠΆΠ΅Ρ‚ ΠΏΡ€Π΅Π΄ΡΡ‚Π°Π²Π»ΡΡ‚ΡŒ Ρ„Π°ΠΊΡ‚ΠΈΡ‡Π΅ΡΠΊΡƒΡŽ ΠΏΠ»ΠΎΡ‰Π°Π΄ΡŒ (Π½Π°ΠΏΡ€ΠΈΠΌΠ΅Ρ€, ΠΏΠ»ΠΎΡ‰Π°Π΄ΡŒ ΠΏΡ€ΠΈΡ‚ΠΎΠΊΠ° ΠΈΠ»ΠΈ ΠΏΠΎΠΏΠ΅Ρ€Π΅Ρ‡Π½ΠΎΠ΅ сСчСниС Π±Π°Π»ΠΊΠΈ) ΠΈΠ»ΠΈ ΠΎΠ±Ρ€Π°Π·Π½ΡƒΡŽ Π΄ΠΈΠ°Π³Ρ€Π°ΠΌΠΌΡƒ (Π½Π°ΠΏΡ€ΠΈΠΌΠ΅Ρ€, Π΄ΠΈΠ°Π³Ρ€Π°ΠΌΠΌΡƒ Π½Π°Π³Ρ€ΡƒΠ·ΠΊΠΈ ΠΈΠ»ΠΈ ΠΈΠ·Π³ΠΈΠ±Π°ΡŽΡ‰Π΅Π³ΠΎ ΠΌΠΎΠΌΠ΅Π½Ρ‚Π°). Π’ любом случаС часто Π±Ρ‹Π²Π°Π΅Ρ‚ ΠΏΠΎΠ»Π΅Π·Π½ΠΎ ΠΎΠΏΡ€Π΅Π΄Π΅Π»ΠΈΡ‚ΡŒ Ρ†Π΅Π½Ρ‚Ρ€ тяТСсти области.

    БиммСтрия ΠΌΠΎΠΆΠ΅Ρ‚ Π±Ρ‹Ρ‚ΡŒ ΠΎΡ‡Π΅Π½ΡŒ ΠΏΠΎΠ»Π΅Π·Π½ΠΎΠΉ для опрСдСлСния мСстополоТСния Ρ†Π΅Π½Ρ‚Ρ€Π° тяТСсти области. Если ΠΎΠ±Π»Π°ΡΡ‚ΡŒ (ΠΈΠ»ΠΈ сСчСниС, ΠΈΠ»ΠΈ Ρ‚Π΅Π»ΠΎ) ΠΈΠΌΠ΅Π΅Ρ‚ ΠΎΠ΄Π½Ρƒ линию симмСтрии, Ρ†Π΅Π½Ρ‚Ρ€ тяТСсти Π±ΡƒΠ΄Π΅Ρ‚ Π»Π΅ΠΆΠ°Ρ‚ΡŒ Π³Π΄Π΅-Ρ‚ΠΎ вдоль Π»ΠΈΠ½ΠΈΠΈ симмСтрии. Π­Ρ‚ΠΎ ΠΎΠ·Π½Π°Ρ‡Π°Π΅Ρ‚, Ρ‡Ρ‚ΠΎ Ссли Π±Ρ‹ Ρ‚Ρ€Π΅Π±ΠΎΠ²Π°Π»ΠΎΡΡŒ ΡƒΡ€Π°Π²Π½ΠΎΠ²Π΅ΡΠΈΡ‚ΡŒ ΠΎΠ±Π»Π°ΡΡ‚ΡŒ (ΠΈΠ»ΠΈ Ρ‚Π΅Π»ΠΎ, ΠΈΠ»ΠΈ ΡΠ΅ΠΊΡ†ΠΈΡŽ) Π² Π³ΠΎΡ€ΠΈΠ·ΠΎΠ½Ρ‚Π°Π»ΡŒΠ½ΠΎΠΌ ΠΏΠΎΠ»ΠΎΠΆΠ΅Π½ΠΈΠΈ, ΠΏΠΎΠ΄Π»ΠΎΠΆΠΈΠ² ΠΏΠΎΠ΄ Π½Π΅Π΅ ΠΊΠ°Ρ€Π°Π½Π΄Π°Ρˆ ΠΈΠ»ΠΈ ΠΊΡ€Π°ΠΉ, Ρ‚ΠΎ ΠΊΠ°Ρ€Π°Π½Π΄Π°Ρˆ Π»ΡƒΡ‡ΡˆΠ΅ всСго ΠΏΠΎΠ»ΠΎΠΆΠΈΡ‚ΡŒ прямо ΠΏΠΎΠ΄ Π»ΠΈΠ½ΠΈΠ΅ΠΉ симмСтрии.

    Если Ρ‚Π΅Π»ΠΎ (ΠΈΠ»ΠΈ ΠΎΠ±Π»Π°ΡΡ‚ΡŒ, ΠΈΠ»ΠΈ сСчСниС) ΠΈΠΌΠ΅Π΅Ρ‚ Π΄Π²Π΅ (ΠΈΠ»ΠΈ Π±ΠΎΠ»Π΅Π΅) Π»ΠΈΠ½ΠΈΠΈ симмСтрии, Ρ†Π΅Π½Ρ‚Ρ€ тяТСсти Π΄ΠΎΠ»ΠΆΠ΅Π½ Π»Π΅ΠΆΠ°Ρ‚ΡŒ Π³Π΄Π΅-Ρ‚ΠΎ вдоль ΠΊΠ°ΠΆΠ΄ΠΎΠΉ ΠΈΠ· этих Π»ΠΈΠ½ΠΈΠΉ. Π’Π°ΠΊΠΈΠΌ ΠΎΠ±Ρ€Π°Π·ΠΎΠΌ, Ρ†Π΅Π½Ρ‚Ρ€ тяТСсти находится Π² Ρ‚ΠΎΡ‡ΠΊΠ΅ пСрСсСчСния Π»ΠΈΠ½ΠΈΠΉ. Π­Ρ‚ΠΎ ΠΎΠ·Π½Π°Ρ‡Π°Π΅Ρ‚, Ρ‡Ρ‚ΠΎ Ссли Π±Ρ‹ Ρ‚Ρ€Π΅Π±ΠΎΠ²Π°Π»ΠΎΡΡŒ ΡƒΡ€Π°Π²Π½ΠΎΠ²Π΅ΡΠΈΡ‚ΡŒ ΠΎΠ±Π»Π°ΡΡ‚ΡŒ (ΠΈΠ»ΠΈ Ρ‚Π΅Π»ΠΎ, ΠΈΠ»ΠΈ ΡΠ΅ΠΊΡ†ΠΈΡŽ) Π² Π³ΠΎΡ€ΠΈΠ·ΠΎΠ½Ρ‚Π°Π»ΡŒΠ½ΠΎΠΌ ΠΏΠΎΠ»ΠΎΠΆΠ΅Π½ΠΈΠΈ, помСстив ΠΏΠΎΠ΄ Π½Π΅Π΅ гвоздь, Ρ‚ΠΎ ΠΊΠΎΠ½Ρ‡ΠΈΠΊ гвоздя Π»ΡƒΡ‡ΡˆΠ΅ всСго Ρ€Π°ΡΠΏΠΎΠ»ΠΎΠΆΠΈΡ‚ΡŒ нСпосрСдствСнно ΠΏΠΎΠ΄ Ρ‚ΠΎΡ‡ΠΊΠΎΠΉ, Π³Π΄Π΅ Π²ΡΡ‚Ρ€Π΅Ρ‡Π°ΡŽΡ‚ΡΡ Π»ΠΈΠ½ΠΈΠΈ симмСтрии. Π­Ρ‚ΠΎ ΠΌΠΎΠΆΠ΅Ρ‚ ΠΏΠΎΠΊΠ°Π·Π°Ρ‚ΡŒΡΡ ΠΎΡ‡Π΅Π²ΠΈΠ΄Π½Ρ‹ΠΌ, Π½ΠΎ понятиС Ρ†Π΅Π½Ρ‚Ρ€ΠΎΠΈΠ΄Π° ΠΎΡ‡Π΅Π½ΡŒ Π²Π°ΠΆΠ½ΠΎ ΠΏΠΎΠ½ΠΈΠΌΠ°Ρ‚ΡŒ ΠΊΠ°ΠΊ графичСски, Ρ‚Π°ΠΊ ΠΈ числСнно.ПолоТСниС Ρ†Π΅Π½Ρ‚Ρ€Π° тяТСсти Π½Π΅ΠΊΠΎΡ‚ΠΎΡ€Ρ‹Ρ… простых Ρ„ΠΎΡ€ΠΌ Π»Π΅Π³ΠΊΠΎ опрСдСляСтся ΠΏΡ€ΠΈ осмотрС. Π˜Π·Π²Π΅ΡΡ‚Π½ΠΎ, Ρ‡Ρ‚ΠΎ Ρ†Π΅Π½Ρ‚Ρ€ тяТСсти ΠΊΡ€ΡƒΠ³Π° находится Π² Π΅Π³ΠΎ Ρ†Π΅Π½Ρ‚Ρ€Π΅, Π° Ρ†Π΅Π½Ρ‚Ρ€ тяТСсти ΠΊΠ²Π°Π΄Ρ€Π°Ρ‚Π° находится Π½Π° пСрСсСчСнии Π΄Π²ΡƒΡ… Π»ΠΈΠ½ΠΈΠΉ, ΡΠΎΠ΅Π΄ΠΈΠ½ΡΡŽΡ‰ΠΈΡ… сСрСдины ΠΏΠ°Ρ€Π°Π»Π»Π΅Π»ΡŒΠ½Ρ‹Ρ… сторон. Π£ ΠΊΡ€ΡƒΠ³Π° бСсконСчноС количСство Π»ΠΈΠ½ΠΈΠΉ симмСтрии, Π° Ρƒ ΠΊΠ²Π°Π΄Ρ€Π°Ρ‚Π° – Ρ‡Π΅Ρ‚Ρ‹Ρ€Π΅.

    Π¦Π΅Π½Ρ‚Ρ€ΠΎΠΈΠ΄ сСчСния Π½Π΅ всСгда находится Π² ΠΏΡ€Π΅Π΄Π΅Π»Π°Ρ… ΠΏΠ»ΠΎΡ‰Π°Π΄ΠΈ ΠΈΠ»ΠΈ ΠΌΠ°Ρ‚Π΅Ρ€ΠΈΠ°Π»Π° сСчСния. ΠŸΠΎΠ»Ρ‹Π΅ Ρ‚Ρ€ΡƒΠ±Ρ‹, L-ΠΎΠ±Ρ€Π°Π·Π½Ρ‹Π΅ ΠΈ Π½Π΅ΠΊΠΎΡ‚ΠΎΡ€Ρ‹Π΅ сСкции Π½Π΅ΠΏΡ€Π°Π²ΠΈΠ»ΡŒΠ½ΠΎΠΉ Ρ„ΠΎΡ€ΠΌΡ‹ ΠΈΠΌΠ΅ΡŽΡ‚ Ρ†Π΅Π½Ρ‚Ρ€ΠΎΠΈΠ΄, располоТСнный Π²Π½Π΅ ΠΌΠ°Ρ‚Π΅Ρ€ΠΈΠ°Π»Π° сСкции.Π­Ρ‚ΠΎ Π½Π΅ ΠΏΡ€ΠΎΠ±Π»Π΅ΠΌΠ°, ΠΏΠΎΡΠΊΠΎΠ»ΡŒΠΊΡƒ Ρ†Π΅Π½Ρ‚Ρ€ΠΎΠΈΠ΄ Π½Π° самом Π΄Π΅Π»Π΅ ΠΈΡΠΏΠΎΠ»ΡŒΠ·ΡƒΠ΅Ρ‚ΡΡ Ρ‚ΠΎΠ»ΡŒΠΊΠΎ ΠΊΠ°ΠΊ Ρ‚ΠΎΡ‡ΠΊΠ° отсчСта, ΠΎΡ‚ ΠΊΠΎΡ‚ΠΎΡ€ΠΎΠΉ ΠΈΠ·ΠΌΠ΅Ρ€ΡΡŽΡ‚ΡΡ расстояния. Π’ΠΎΡ‡Π½ΠΎΠ΅ ΠΏΠΎΠ»ΠΎΠΆΠ΅Π½ΠΈΠ΅ Ρ†Π΅Π½Ρ‚Ρ€ΠΎΠΈΠ΄Π° ΠΌΠΎΠΆΠ½ΠΎ ΠΎΠΏΡ€Π΅Π΄Π΅Π»ΠΈΡ‚ΡŒ, ΠΊΠ°ΠΊ описано Π²Ρ‹ΡˆΠ΅, с ΠΏΠΎΠΌΠΎΡ‰ΡŒΡŽ графичСской статики ΠΈΠ»ΠΈ числСнно.

    Π¦Π΅Π½Ρ‚Ρ€ΠΎΠΈΠ΄ любой области ΠΌΠΎΠΆΠ½ΠΎ Π½Π°ΠΉΡ‚ΠΈ, взяв ΠΌΠΎΠΌΠ΅Π½Ρ‚Ρ‹ ΠΈΠ΄Π΅Π½Ρ‚ΠΈΡ„ΠΈΡ†ΠΈΡ€ΡƒΠ΅ΠΌΡ‹Ρ… областСй (Π½Π°ΠΏΡ€ΠΈΠΌΠ΅Ρ€, ΠΏΡ€ΡΠΌΠΎΡƒΠ³ΠΎΠ»ΡŒΠ½ΠΈΠΊΠΎΠ² ΠΈΠ»ΠΈ Ρ‚Ρ€Π΅ΡƒΠ³ΠΎΠ»ΡŒΠ½ΠΈΠΊΠΎΠ²) Π²ΠΎΠΊΡ€ΡƒΠ³ любой оси. Π­Ρ‚ΠΎ дСлаСтся Ρ‚Π°ΠΊ ΠΆΠ΅, ΠΊΠ°ΠΊ Ρ†Π΅Π½Ρ‚Ρ€ тяТСсти ΠΌΠΎΠΆΠ½ΠΎ Π½Π°ΠΉΡ‚ΠΈ, взяв ΠΌΠΎΠΌΠ΅Π½Ρ‚Ρ‹ вСса. ΠœΠΎΠΌΠ΅Π½Ρ‚ большой ΠΏΠ»ΠΎΡ‰Π°Π΄ΠΈ ΠΎΡ‚Π½ΠΎΡΠΈΡ‚Π΅Π»ΡŒΠ½ΠΎ любой оси Ρ€Π°Π²Π΅Π½ алгСбраичСской суммС ΠΌΠΎΠΌΠ΅Π½Ρ‚ΠΎΠ² ΡΠΎΡΡ‚Π°Π²Π»ΡΡŽΡ‰ΠΈΡ… Π΅Π΅ ΠΏΠ»ΠΎΡ‰Π°Π΄Π΅ΠΉ.Π­Ρ‚ΠΎ выраТаСтся ΡΠ»Π΅Π΄ΡƒΡŽΡ‰ΠΈΠΌ ΡƒΡ€Π°Π²Π½Π΅Π½ΠΈΠ΅ΠΌ:

    Π‘ΡƒΠΌΠΌΠ° MAtotal = MA1 + MA2 + MA3 + …

    ΠœΠΎΠΌΠ΅Π½Ρ‚ любой области опрСдСляСтся ΠΊΠ°ΠΊ ΠΏΡ€ΠΎΠΈΠ·Π²Π΅Π΄Π΅Π½ΠΈΠ΅ ΠΏΠ»ΠΎΡ‰Π°Π΄ΠΈ ΠΈ пСрпСндикулярного расстояния ΠΎΡ‚ Ρ†Π΅Π½Ρ‚Ρ€Π° тяТСсти ΠΏΠ»ΠΎΡ‰Π°Π΄ΠΈ Π΄ΠΎ оси ΠΌΠΎΠΌΠ΅Π½Ρ‚Π°. Π‘ ΠΏΠΎΠΌΠΎΡ‰ΡŒΡŽ этого ΠΏΡ€ΠΈΠ½Ρ†ΠΈΠΏΠ° ΠΌΡ‹ ΠΌΠΎΠΆΠ΅ΠΌ Π½Π°ΠΉΡ‚ΠΈ Ρ†Π΅Π½Ρ‚Ρ€ тяТСсти любой простой ΠΈΠ»ΠΈ составной области.

    Π¦Π΅Π½Ρ‚Ρ€ тяТСсти:

    Π”Π°Π½ΠΎ: пластина, показанная Π½Π° схСмС, ΠΈΠΌΠ΅Π΅Ρ‚ вСс 1 # / дюйм 2 (1 Ρ„ΡƒΠ½Ρ‚ Π½Π° ΠΊΠ²Π°Π΄Ρ€Π°Ρ‚Π½Ρ‹ΠΉ дюйм) Π³ΠΎΡ€ΠΈΠ·ΠΎΠ½Ρ‚Π°Π»ΡŒΠ½ΠΎΠΉ повСрхности.

    ΠžΠΏΡ€Π΅Π΄Π΅Π»ΠΈΡ‚ΡŒ:
    Ρ†Π΅Π½Ρ‚Ρ€ тяТСсти пластины, зная, Ρ‡Ρ‚ΠΎ ΠΎΠ½Π° симмСтрична ΠΎΡ‚Π½ΠΎΡΠΈΡ‚Π΅Π»ΡŒΠ½ΠΎ оси X-X.

    РСшСниС: ΠŸΡ€ΠΈΠ½Ρ†ΠΈΠΏ ΠΌΠΎΠΌΠ΅Π½Ρ‚ΠΎΠ² гласит, Ρ‡Ρ‚ΠΎ ΠΎΠ±Ρ‰ΠΈΠΉ вСс Π²ΠΎΠΊΡ€ΡƒΠ³ оси Ρ€Π°Π²Π΅Π½ суммС ΠΌΠΎΠΌΠ΅Π½Ρ‚ΠΎΠ² вСсов ΠΊΠΎΠΌΠΏΠΎΠ½Π΅Π½Ρ‚ΠΎΠ² ΠΎΡ‚Π½ΠΎΡΠΈΡ‚Π΅Π»ΡŒΠ½ΠΎ этой ΠΆΠ΅ оси. Π’Π°ΠΊΠΈΠΌ ΠΎΠ±Ρ€Π°Π·ΠΎΠΌ, ΠΏΠ΅Ρ€Π²ΠΎΠ΅, Ρ‡Ρ‚ΠΎ Π½ΡƒΠΆΠ½ΠΎ ΡΠ΄Π΅Π»Π°Ρ‚ΡŒ, – это Ρ€Π°Π·Π΄Π΅Π»ΠΈΡ‚ΡŒ Ρ‚Π°Ρ€Π΅Π»ΠΊΡƒ Π½Π° нСсколько простых частСй. Π—Π°Ρ‚Π΅ΠΌ ΠΎΠΏΡ€Π΅Π΄Π΅Π»ΠΈΡ‚Π΅ ΠΏΠ»ΠΎΡ‰Π°Π΄ΡŒ ΠΈ Ρ†Π΅Π½Ρ‚Ρ€ тяТСсти (ΠΈΠ»ΠΈ Ρ†Π΅Π½Ρ‚Ρ€ΠΎΠΈΠ΄) для ΠΊΠ°ΠΆΠ΄ΠΎΠΉ ΠΈΠ· составных частСй. ПослС этого ΠΈΠ·ΠΌΠ΅Ρ€ΡŒΡ‚Π΅ ΠΌΠΎΠΌΠ΅Π½Ρ‚Ρ‹ ΠΊΠ°ΠΆΠ΄ΠΎΠΉ ΠΈΠ· частСй Π²ΠΎΠΊΡ€ΡƒΠ³ ΡƒΠ΄ΠΎΠ±Π½ΠΎΠΉ оси (Π² этом случаС Π²Ρ‹Π±Π΅Ρ€ΠΈΡ‚Π΅ ось Z-Z, Π²ΠΎΠΊΡ€ΡƒΠ³ ΠΊΠΎΡ‚ΠΎΡ€ΠΎΠΉ Π±ΡƒΠ΄ΡƒΡ‚ ΠΈΠ·ΠΌΠ΅Ρ€ΡΡ‚ΡŒΡΡ эти ΠΌΠΎΠΌΠ΅Π½Ρ‚Ρ‹).Ось Z-Z здСсь ΠΎΠ±ΠΎΠ·Π½Π°Ρ‡Π΅Π½Π° ΠΊΠ°ΠΊ Ref Axis.


    Π‘ΡƒΠΌΠΌΠ° MAtotal = MA1 + MA2 + MA3

    Π­Ρ‚ΠΎ простоС ΡƒΡ€Π°Π²Π½Π΅Π½ΠΈΠ΅ ΠΌΠΎΠΆΠ½ΠΎ ΠΏΠ΅Ρ€Π΅ΠΏΠΈΡΠ°Ρ‚ΡŒ ΡΠ»Π΅Π΄ΡƒΡŽΡ‰ΠΈΠΌ ΠΎΠ±Ρ€Π°Π·ΠΎΠΌ, Π² ΠΊΠΎΡ‚ΠΎΡ€ΠΎΠΌ описана каТдая ΠΈΠ· составных частСй:

    (Atotal) (расстояниС ΠΎΡ‚ исходной оси Π΄ΠΎ Ρ†Π΅Π½Ρ‚Ρ€Π°Π»ΡŒΠ½ΠΎΠΉ оси) = (A1) (расстояниС ΠΎΡ‚ Ρ†Π΅Π½Ρ‚Ρ€Π° тяТСсти A1 Π΄ΠΎ исходной оси) + (A2) (расстояниС ΠΎΡ‚ Ρ†Π΅Π½Ρ‚Ρ€Π° тяТСсти A2 Π΄ΠΎ исходной оси) + (A3) (расстояниС ΠΎΡ‚ Ρ†Π΅Π½Ρ‚Ρ€ΠΎΠΈΠ΄Π° ΠΎΡ‚ A3 Π΄ΠΎ оси отсчСта)

    , Π° Π·Π°Ρ‚Π΅ΠΌ Ρ€Π΅ΡˆΠΈΡ‚ΡŒ ΠΎΡ‚Π½ΠΎΡΠΈΡ‚Π΅Π»ΡŒΠ½ΠΎ y…. Ρ†Π΅Π½Ρ‚Ρ€Π°Π»ΡŒΠ½Π°Ρ ось находится Π½Π° расстоянии 7,3 дюйма ΠΎΡ‚ исходной оси.

    ЀактичСский Ρ†Π΅Π½Ρ‚Ρ€ тяТСсти находится Π½Π° ΠΏΠΎΠ»ΠΏΡƒΡ‚ΠΈ ΠΏΠΎ Π³Π»ΡƒΠ±ΠΈΠ½Π΅ пластины Π² Ρ‚ΠΎΡ‡ΠΊΠ΅, рассчитанной Π²Ρ‹ΡˆΠ΅. ΠŸΡ€ΠΈ ΡƒΠΌΠ΅Π½ΡŒΡˆΠ΅Π½ΠΈΠΈ Ρ‚ΠΎΠ»Ρ‰ΠΈΠ½Ρ‹ пластины линия дСйствия Ρ†Π΅Π½Ρ‚Ρ€Π° тяТСсти останСтся, Π² Ρ‚ΠΎ врСмя ΠΊΠ°ΠΊ Ρ†Π΅Π½Ρ‚Ρ€ тяТСсти пСрСмСщаСтся ΠΏΡ€ΠΎΠΏΠΎΡ€Ρ†ΠΈΠΎΠ½Π°Π»ΡŒΠ½ΠΎ этой Π»ΠΈΠ½ΠΈΠΈ дСйствия, всСгда Π΄Π΅ΠΉΡΡ‚Π²ΡƒΡŽΡ‰Π΅ΠΉ Π² срСднСй Ρ‚ΠΎΡ‡ΠΊΠ΅ Π³Π»ΡƒΠ±ΠΈΠ½Ρ‹ пластины. Если Ρ‚ΠΎΠ»Ρ‰ΠΈΠ½Π° пластины ΡƒΠΌΠ΅Π½ΡŒΡˆΠ°Π΅Ρ‚ΡΡ Π΄ΠΎ нуля, ΠΎΠ½Π° Π½Π΅ ΠΈΠΌΠ΅Π΅Ρ‚ вСса, ΠΈ ΠΏΡ€Π΅ΠΆΠ½Π΅Π΅ ΠΏΠΎΠ»ΠΎΠΆΠ΅Π½ΠΈΠ΅ Ρ†Π΅Π½Ρ‚Ρ€Π° тяТСсти Ρ‚Π΅ΠΏΠ΅Ρ€ΡŒ называСтся Ρ†Π΅Π½Ρ‚Ρ€ΠΎΠΈΠ΄ΠΎΠΌ ΠΏΠ»ΠΎΡ‰Π°Π΄ΠΈ.

    ΠœΠΎΠΌΠ΅Π½Ρ‚ ΠΈΠ½Π΅Ρ€Ρ†ΠΈΠΈ (I) – это Ρ‚Π΅Ρ€ΠΌΠΈΠ½, ΠΈΡΠΏΠΎΠ»ΡŒΠ·ΡƒΠ΅ΠΌΡ‹ΠΉ для описания способности ΠΏΠΎΠΏΠ΅Ρ€Π΅Ρ‡Π½ΠΎΠ³ΠΎ сСчСния ΡΠΎΠΏΡ€ΠΎΡ‚ΠΈΠ²Π»ΡΡ‚ΡŒΡΡ ΠΈΠ·Π³ΠΈΠ±Ρƒ. Он всСгда учитываСтся ΠΎΡ‚Π½ΠΎΡΠΈΡ‚Π΅Π»ΡŒΠ½ΠΎ Π±Π°Π·ΠΎΠ²ΠΎΠΉ оси, Ρ‚Π°ΠΊΠΎΠΉ ΠΊΠ°ΠΊ X-X ΠΈΠ»ΠΈ Y-Y. Π­Ρ‚ΠΎ матСматичСскоС свойство сСчСния, связанноС с ΠΏΠ»ΠΎΡ‰Π°Π΄ΡŒΡŽ повСрхности ΠΈ Ρ‚Π΅ΠΌ, ΠΊΠ°ΠΊ эта ΠΏΠ»ΠΎΡ‰Π°Π΄ΡŒ распрСдСляСтся ΠΎΡ‚Π½ΠΎΡΠΈΡ‚Π΅Π»ΡŒΠ½ΠΎ Π±Π°Π·ΠΎΠ²ΠΎΠΉ оси. Π‘Π°Π·ΠΎΠ²ΠΎΠΉ осью ΠΎΠ±Ρ‹Ρ‡Π½ΠΎ являСтся Ρ†Π΅Π½Ρ‚Ρ€Π°Π»ΡŒΠ½Π°Ρ ось.

    ΠœΠΎΠΌΠ΅Π½Ρ‚ ΠΈΠ½Π΅Ρ€Ρ†ΠΈΠΈ Ρ‚Π°ΠΊΠΆΠ΅ извСстСн ΠΊΠ°ΠΊ Π’Ρ‚ΠΎΡ€ΠΎΠΉ ΠΌΠΎΠΌΠ΅Π½Ρ‚ области ΠΈ матСматичСски выраТаСтся ΠΊΠ°ΠΊ:

    Ixx = Π‘ΡƒΠΌΠΌΠ° (A) (y 2 )

    Π’ ΠΊΠΎΡ‚ΠΎΡ€ΠΎΠΌ:

    Ixx = ΠΌΠΎΠΌΠ΅Π½Ρ‚ ΠΈΠ½Π΅Ρ€Ρ†ΠΈΠΈ Π²ΠΎΠΊΡ€ΡƒΠ³ оси x
    A = ΠΏΠ»ΠΎΡ‰Π°Π΄ΡŒ плоскости ΠΎΠ±ΡŠΠ΅ΠΊΡ‚Π°
    y = расстояниС ΠΌΠ΅ΠΆΠ΄Ρƒ Ρ†Π΅Π½Ρ‚Ρ€ΠΎΠΌ тяТСсти ΠΎΠ±ΡŠΠ΅ΠΊΡ‚Π° ΠΈ осью x

    ΠœΠΎΠΌΠ΅Π½Ρ‚ ΠΈΠ½Π΅Ρ€Ρ†ΠΈΠΈ – это Π²Π°ΠΆΠ½ΠΎΠ΅ Π·Π½Π°Ρ‡Π΅Π½ΠΈΠ΅, ΠΊΠΎΡ‚ΠΎΡ€ΠΎΠ΅ ΠΈΡΠΏΠΎΠ»ΡŒΠ·ΡƒΠ΅Ρ‚ΡΡ для опрСдСлСния напряТСнного состояния Π² сСчСнии, расчСта сопротивлСния ΠΏΡ€ΠΎΠ΄ΠΎΠ»ΡŒΠ½ΠΎΠΌΡƒ ΠΈΠ·Π³ΠΈΠ±Ρƒ ΠΈ опрСдСлСния Π²Π΅Π»ΠΈΡ‡ΠΈΠ½Ρ‹ ΠΏΡ€ΠΎΠ³ΠΈΠ±Π° Π±Π°Π»ΠΊΠΈ.
    НапримСр, Ссли ΠΏΡ€ΠΎΠ΅ΠΊΡ‚ΠΈΡ€ΠΎΠ²Ρ‰ΠΈΠΊΡƒ даСтся ΠΎΠΏΡ€Π΅Π΄Π΅Π»Π΅Π½Π½Ρ‹ΠΉ Π½Π°Π±ΠΎΡ€ ΠΎΠ³Ρ€Π°Π½ΠΈΡ‡Π΅Π½ΠΈΠΉ для структурной ΠΏΡ€ΠΎΠ±Π»Π΅ΠΌΡ‹ (Ρ‚.Π΅. Π½Π°Π³Ρ€ΡƒΠ·ΠΊΠΈ, ΠΏΡ€ΠΎΠ»Π΅Ρ‚Ρ‹ ΠΈ ΠΊΠΎΠ½Π΅Ρ‡Π½Ρ‹Π΅ условия), ΠΌΠΎΠΆΠ΅Ρ‚ Π±Ρ‹Ρ‚ΡŒ ΠΎΠΏΡ€Π΅Π΄Π΅Π»Π΅Π½ΠΎ Β«Ρ‚Ρ€Π΅Π±ΡƒΠ΅ΠΌΠΎΠ΅Β» Π·Π½Π°Ρ‡Π΅Π½ΠΈΠ΅ ΠΌΠΎΠΌΠ΅Π½Ρ‚Π° ΠΈΠ½Π΅Ρ€Ρ†ΠΈΠΈ. Π’ΠΎΠ³Π΄Π° любой структурный элСмСнт, ΠΊΠΎΡ‚ΠΎΡ€Ρ‹ΠΉ ΠΈΠΌΠ΅Π΅Ρ‚ хотя Π±Ρ‹ этот ΠΊΠΎΠ½ΠΊΡ€Π΅Ρ‚Π½Ρ‹ΠΉ ΠΌΠΎΠΌΠ΅Π½Ρ‚ ΠΈΠ½Π΅Ρ€Ρ†ΠΈΠΈ, ΠΌΠΎΠΆΠ½ΠΎ Π±ΡƒΠ΄Π΅Ρ‚ ΠΈΡΠΏΠΎΠ»ΡŒΠ·ΠΎΠ²Π°Ρ‚ΡŒ Π² конструкции. Π”Ρ€ΡƒΠ³ΠΎΠΉ ΠΏΡ€ΠΈΠΌΠ΅Ρ€ ΠΌΠΎΠΆΠ΅Ρ‚ Π±Ρ‹Ρ‚ΡŒ, Ссли Π²Π΅Ρ€Π½ΠΎ ΠΎΠ±Ρ€Π°Ρ‚Π½ΠΎΠ΅; ΠΊΠΎΠ½ΠΊΡ€Π΅Ρ‚Π½Ρ‹ΠΉ элСмСнт даСтся Π² Π΄ΠΈΠ·Π°ΠΉΠ½Π΅. Π—Π°Ρ‚Π΅ΠΌ ΠΌΠΎΠΆΠ½ΠΎ Π±Ρ‹Π»ΠΎ ΠΎΠΏΡ€Π΅Π΄Π΅Π»ΠΈΡ‚ΡŒ Π½Π΅ΡΡƒΡ‰ΡƒΡŽ ΡΠΏΠΎΡΠΎΠ±Π½ΠΎΡΡ‚ΡŒ элСмСнта.

    Π”Π°Π²Π°ΠΉΡ‚Π΅ посмотрим Π½Π° Π΄Π²Π΅ доски, Ρ‡Ρ‚ΠΎΠ±Ρ‹ ΠΈΠ½Ρ‚ΡƒΠΈΡ‚ΠΈΠ²Π½ΠΎ ΠΎΠΏΡ€Π΅Π΄Π΅Π»ΠΈΡ‚ΡŒ, какая ΠΈΠ· Π½ΠΈΡ… Π±ΡƒΠ΄Π΅Ρ‚ ΠΎΡ‚ΠΊΠ»ΠΎΠ½ΡΡ‚ΡŒΡΡ большС ΠΈ ΠΏΠΎΡ‡Π΅ΠΌΡƒ.Если Π΄Π²Π΅ доски с фактичСскими Ρ€Π°Π·ΠΌΠ΅Ρ€Π°ΠΌΠΈ 2 дюйма Π½Π° 10 дюймов Π±Ρ‹Π»ΠΈ ΡƒΠ»ΠΎΠΆΠ΅Π½Ρ‹ рядом – ΠΎΠ΄Π½Π° со стороны Π΄Π²ΡƒΡ… дюймов, Π° другая – со стороны восьми дюймов, Ρ‚ΠΎ ΠΏΠ»Π°Ρ‚Π°, которая опираСтся Π½Π° Π΅Π΅ 2-Π΄ΡŽΠΉΠΌΠΎΠ²Ρ‹ΠΉ ΠΊΡ€Π°ΠΉ, Π±ΡƒΠ΄Π΅Ρ‚ Π·Π½Π°Ρ‡ΠΈΡ‚Π΅Π»ΡŒΠ½ΠΎ ТСстчС, Ρ‡Π΅ΠΌ Ρ‚Π°, которая поддСрТиваСтся вдоль. Π΅Π³ΠΎ 10-Π΄ΡŽΠΉΠΌΠΎΠ²Ρ‹ΠΉ ΠΊΡ€Π°ΠΉ. ОбС ΠΏΠ»Π°Ρ‚Ρ‹ ΠΈΠΌΠ΅ΡŽΡ‚ ΠΎΠ΄ΠΈΠ½Π°ΠΊΠΎΠ²ΡƒΡŽ ΠΏΠ»ΠΎΡ‰Π°Π΄ΡŒ ΠΏΠΎΠΏΠ΅Ρ€Π΅Ρ‡Π½ΠΎΠ³ΠΎ сСчСния, Π½ΠΎ ΠΏΠΎ-Ρ€Π°Π·Π½ΠΎΠΌΡƒ распрСдСлСны ΠΎΡ‚Π½ΠΎΡΠΈΡ‚Π΅Π»ΡŒΠ½ΠΎ Π³ΠΎΡ€ΠΈΠ·ΠΎΠ½Ρ‚Π°Π»ΡŒΠ½ΠΎΠΉ Ρ†Π΅Π½Ρ‚Ρ€Π°Π»ΡŒΠ½ΠΎΠΉ оси.


    Ixx = (1/12) (b) (h 3 ) = (1/12) x (b) x (h x h x h)

    Π’ ΠΊΠΎΡ‚ΠΎΡ€ΠΎΠΌ Π·Π½Π°Ρ‡Π΅Π½ΠΈΠ΅ b всСгда принимаСтся Ρ€Π°Π²Π½Ρ‹ΠΌ сторонС, ΠΏΠ°Ρ€Π°Π»Π»Π΅Π»ΡŒΠ½ΠΎΠΉ Π±Π°Π·ΠΎΠ²ΠΎΠΉ оси, Π° h – высотой сСкции.Π­Ρ‚ΠΎ ΠΎΡ‡Π΅Π½ΡŒ Π²Π°ΠΆΠ½ΠΎ ΠΎΡ‚ΠΌΠ΅Ρ‚ΠΈΡ‚ΡŒ! Если ΠΏΡ€ΠΈΠ½ΡΡ‚ΡŒ Π½Π΅ΠΏΡ€Π°Π²ΠΈΠ»ΡŒΠ½ΠΎΠ΅ Π·Π½Π°Ρ‡Π΅Π½ΠΈΠ΅ для значСния b, вычислСния Π±ΡƒΠ΄ΡƒΡ‚ ΡΠΎΠ²Π΅Ρ€ΡˆΠ΅Π½Π½ΠΎ Π½Π΅Π²Π΅Ρ€Π½Ρ‹ΠΌΠΈ.

    ΠœΠΎΠΌΠ΅Π½Ρ‚ ΠΈΠ½Π΅Ρ€Ρ†ΠΈΠΈ

    Π”Π°Π½ΠΎ: ΠΏΠΎΠΏΠ΅Ρ€Π΅Ρ‡Π½ΠΎΠ΅ сСчСниС.
    ΠžΠΏΡ€Π΅Π΄Π΅Π»ΠΈΡ‚Π΅: ΠΌΠΎΠΌΠ΅Π½Ρ‚Ρ‹ ΠΈΠ½Π΅Ρ€Ρ†ΠΈΠΈ, Ixx ΠΈ Iyy этого Ρ€Π°Π·Π΄Π΅Π»Π°.

    РСшСниС:


    ΠœΠΎΠΌΠ΅Π½Ρ‚ ΠΈΠ½Π΅Ρ€Ρ†ΠΈΠΈ ΠΏΡ€ΡΠΌΠΎΡƒΠ³ΠΎΠ»ΡŒΠ½ΠΎΠΉ Ρ„ΠΎΡ€ΠΌΡ‹, Ρ‚Π°ΠΊΠΎΠΉ ΠΊΠ°ΠΊ эта, Π»Π΅Π³ΠΊΠΎ вычисляСтся с ΠΏΠΎΠΌΠΎΡ‰ΡŒΡŽ уравнСния I = 1/12 bh4. Однако ΠΎΡ‡Π΅Π½ΡŒ Π²Π°ΠΆΠ½ΠΎ, Ρ‡Ρ‚ΠΎΠ±Ρ‹ b ΠΈ h Π±Ρ‹Π»ΠΈ присвоСны ΠΏΡ€Π°Π²ΠΈΠ»ΡŒΠ½Ρ‹Π΅ значСния.

    Π’Ρ‹ ΠΌΠΎΠΆΠ΅Ρ‚Π΅ просто ΠΏΠΎΠ²Π΅Ρ€Π½ΡƒΡ‚ΡŒ элСмСнт Π½Π° 90 градусов ΠΈ произвСсти пСрСсчСт, всСгда запоминая исходноС ΠΏΠΎΠ»ΠΎΠΆΠ΅Π½ΠΈΠ΅ элСмСнта.

    Ixx = 1/12 (4 дюйма) (10 дюймов) 3 = 333,2 дюйма 4
    Iyy = 1/12 (10 дюймов) (4 дюйма) 3 = 53,312 дюйм 4

    Π’ этом случаС наблюдСниС ΠΏΠΎΠ΄Ρ‚Π²Π΅Ρ€Π΄ΠΈΡ‚ Π²Ρ‹Π±ΠΎΡ€ для b ΠΈ h. Π›ΠΎΠ³ΠΈΡ‡Π½ΠΎ, Ρ‡Ρ‚ΠΎ Ixx большС, Ρ‡Π΅ΠΌ Iyy, ΠΏΠΎΡ‚ΠΎΠΌΡƒ Ρ‡Ρ‚ΠΎ большая Ρ‡Π°ΡΡ‚ΡŒ ΠΏΡ€ΡΠΌΠΎΡƒΠ³ΠΎΠ»ΡŒΠ½ΠΎΠΉ области находится дальшС ΠΎΡ‚ оси x-x, Ρ‡Π΅ΠΌ ось y-y. Π­Ρ‚ΠΎ ΠΏΡ€ΠΈΠ²ΠΎΠ΄ΠΈΡ‚ ΠΊ Ρ‚ΠΎΠΌΡƒ, Ρ‡Ρ‚ΠΎ Ρ„ΠΎΡ€ΠΌΠ° ΠΈΠΌΠ΅Π΅Ρ‚ большСС сопротивлСниС Π²Ρ€Π°Ρ‰Π΅Π½ΠΈΡŽ Π²ΠΎΠΊΡ€ΡƒΠ³ оси x-x ΠΈ, ΡΠ»Π΅Π΄ΠΎΠ²Π°Ρ‚Π΅Π»ΡŒΠ½ΠΎ, больший ΠΌΠΎΠΌΠ΅Π½Ρ‚ ΠΈΠ½Π΅Ρ€Ρ†ΠΈΠΈ Π²ΠΎΠΊΡ€ΡƒΠ³ этой оси.

    Π’Π°ΠΆΠ½ΠΎΡΡ‚ΡŒ распрСдСлСния ΠΏΠ»ΠΎΡ‰Π°Π΄ΠΈ Π²ΠΎΠΊΡ€ΡƒΠ³ Π΅Π³ΠΎ Ρ†Π΅Π½Ρ‚Ρ€Π°Π»ΡŒΠ½ΠΎΠΉ оси становится ΠΎΡ‡Π΅Π²ΠΈΠ΄Π½ΠΎΠΉ ΠΏΡ€ΠΈ сравнСнии Π·Π½Π°Ρ‡Π΅Π½ΠΈΠΉ ΠΌΠΎΠΌΠ΅Π½Ρ‚Π° ΠΈΠ½Π΅Ρ€Ρ†ΠΈΠΈ ряда Ρ‚ΠΈΠΏΠΈΡ‡Π½Ρ‹Ρ… ΠΊΠΎΠ½Ρ„ΠΈΠ³ΡƒΡ€Π°Ρ†ΠΈΠΉ Π±Π°Π»ΠΊΠΈ. ВсС элСмСнты, ΠΏΠΎΠΊΠ°Π·Π°Π½Π½Ρ‹Π΅ Π½ΠΈΠΆΠ΅, ΠΈΠΌΠ΅ΡŽΡ‚ Ρ€Π°Π·ΠΌΠ΅Ρ€ 2 x 10 дюймов; Π² ΠΏΠΎΠΏΠ΅Ρ€Π΅Ρ‡Π½ΠΎΠΌ сСчСнии, Ρ€Π°Π²Π½ΠΎΠΉ Π΄Π»ΠΈΠ½Ρ‹ ΠΈ ΠΎΠ΄ΠΈΠ½Π°ΠΊΠΎΠ²ΠΎΠΉ Π½Π°Π³Ρ€ΡƒΠ·ΠΊΠΈ.

    ΠΠΠ‘Π’Π ΠžΠ•ΠΠΠ«Π• РАЗДЕЛЫ Часто Π±Ρ‹Π²Π°Π΅Ρ‚ Π²Ρ‹Π³ΠΎΠ΄Π½ΠΎ ΠΎΠ±ΡŠΠ΅Π΄ΠΈΠ½ΠΈΡ‚ΡŒ нСсколько элСмСнтов мСньшСго Ρ€Π°Π·ΠΌΠ΅Ρ€Π°, Ρ‡Ρ‚ΠΎΠ±Ρ‹ ΡΠΎΠ·Π΄Π°Ρ‚ΡŒ Π±Π°Π»ΠΊΡƒ ΠΈΠ»ΠΈ ΠΊΠΎΠ»ΠΎΠ½Π½Ρƒ большСй прочности. ΠœΠΎΠΌΠ΅Π½Ρ‚ ΠΈΠ½Π΅Ρ€Ρ†ΠΈΠΈ Ρ‚Π°ΠΊΠΎΠΉ сборной сСкции опрСдСляСтся слоТСниСм ΠΌΠΎΠΌΠ΅Π½Ρ‚ΠΎΠ² ΠΈΠ½Π΅Ρ€Ρ†ΠΈΠΈ составных частСй.Π­Ρ‚ΠΎ ΠΌΠΎΠΆΠ΅Ρ‚ Π±Ρ‹Ρ‚ΡŒ сдСлано, Ссли ΠΈ Ρ‚ΠΎΠ»ΡŒΠΊΠΎ Ссли ΠΌΠΎΠΌΠ΅Π½Ρ‚Ρ‹ ΠΈΠ½Π΅Ρ€Ρ†ΠΈΠΈ ΠΊΠ°ΠΆΠ΄ΠΎΠΉ ΡΠΎΡΡ‚Π°Π²Π»ΡΡŽΡ‰Π΅ΠΉ области взяты ΠΎΡ‚Π½ΠΎΡΠΈΡ‚Π΅Π»ΡŒΠ½ΠΎ ΠΎΠ±Ρ‰Π΅ΠΉ оси, ΠΈ Ρ‚ΠΎΠ³Π΄Π° ΠΈ Ρ‚ΠΎΠ»ΡŒΠΊΠΎ Ρ‚ΠΎΠ³Π΄Π°, ΠΊΠΎΠ³Π΄Π° Ρ€Π΅Π·ΡƒΠ»ΡŒΡ‚ΠΈΡ€ΡƒΡŽΡ‰Π΅Π΅ сСчСниС дСйствуСт ΠΊΠ°ΠΊ Π΅Π΄ΠΈΠ½ΠΎΠ΅ Ρ†Π΅Π»ΠΎΠ΅.

    ЗастроСнныС сСкции

    Π”Π°Π½ΠΎ:
    ΡΠ»Π΅Π΄ΡƒΡŽΡ‰ΠΈΠ΅ ΠΏΠΎΠΏΠ΅Ρ€Π΅Ρ‡Π½Ρ‹Π΅ сСчСния
    ΠžΠΏΡ€Π΅Π΄Π΅Π»ΠΈΡ‚ΡŒ:
    Ix ΠΊΠ°ΠΆΠ΄ΠΎΠ³ΠΎ Ρ€Π°Π·Π΄Π΅Π»Π° с ΡƒΡ‡Π΅Ρ‚ΠΎΠΌ Π΅Π³ΠΎ составных частСй.

    РСшСниС:
    Π’ этом ΠΏΡ€ΠΈΠΌΠ΅Ρ€Π΅ Box Ρ€Π°Π·Π±ΠΈΡ‚ Π½Π° 4 ΠΎΡ‚Π΄Π΅Π»ΡŒΠ½Ρ‹Ρ… элСмСнта, ΠΈ ΠΏΠΎΠΊΠ°Π·Π°Π½Π° ΠΏΡ€ΠΎΡ†Π΅Π΄ΡƒΡ€Π° вычислСния Ixx.

    РасчСт Π²Ρ€ΡƒΡ‡Π½ΡƒΡŽ с ΠΏΠΎΠΌΠΎΡ‰ΡŒΡŽ ΠΊΠΎΠΌΠΏΡŒΡŽΡ‚Π΅Ρ€Π½ΠΎΠ³ΠΎ расчСта, ΠΏΡ€ΠΈΠ²Π΅Π΄Π΅Π½Π½ΠΎΠ³ΠΎ Π½ΠΈΠΆΠ΅.

    ΠŸΡ€ΠΈΠΌΠ΅Ρ€ Ρ€Π΅Π·ΡƒΠ»ΡŒΡ‚Π°Ρ‚ΠΎΠ² ΠΊΠΎΠΌΠΏΡŒΡŽΡ‚Π΅Ρ€Π½ΠΎΠΉ ΠΏΡ€ΠΎΠ³Ρ€Π°ΠΌΠΌΡ‹, доступной Π² нашСм Ρ€Π°Π·Π΄Π΅Π»Π΅ бСсплатного ΠΏΡ€ΠΎΠ³Ρ€Π°ΠΌΠΌΠ½ΠΎΠ³ΠΎ обСспСчСния




    Π€ΠžΠ ΠœΠ£Π›Π ΠŸΠ•Π Π•Π”ΠΠ§Π˜

    БущСствуСт ΠΌΠ½ΠΎΠ³ΠΎ составных сСкций, Π² ΠΊΠΎΡ‚ΠΎΡ€Ρ‹Ρ… составныС части Π½Π΅ распрСдСлСны симмСтрично ΠΎΡ‚Π½ΠΎΡΠΈΡ‚Π΅Π»ΡŒΠ½ΠΎ Ρ†Π΅Π½Ρ‚Ρ€Π°Π»ΡŒΠ½ΠΎΠΉ оси. Π‘Π°ΠΌΡ‹ΠΉ простой способ ΠΎΠΏΡ€Π΅Π΄Π΅Π»ΠΈΡ‚ΡŒ ΠΌΠΎΠΌΠ΅Π½Ρ‚ ΠΈΠ½Π΅Ρ€Ρ†ΠΈΠΈ Ρ‚Π°ΠΊΠΎΠ³ΠΎ участка – Π½Π°ΠΉΡ‚ΠΈ ΠΌΠΎΠΌΠ΅Π½Ρ‚ ΠΈΠ½Π΅Ρ€Ρ†ΠΈΠΈ составных частСй ΠΎΡ‚Π½ΠΎΡΠΈΡ‚Π΅Π»ΡŒΠ½ΠΎ ΠΈΡ… собствСнной Ρ†Π΅Π½Ρ‚Ρ€Π°Π»ΡŒΠ½ΠΎΠΉ оси, Π° Π·Π°Ρ‚Π΅ΠΌ ΠΏΡ€ΠΈΠΌΠ΅Π½ΠΈΡ‚ΡŒ Ρ„ΠΎΡ€ΠΌΡƒΠ»Ρƒ пСрСноса.Π€ΠΎΡ€ΠΌΡƒΠ»Π° пСрСноса пСрСносит ΠΌΠΎΠΌΠ΅Π½Ρ‚ ΠΈΠ½Π΅Ρ€Ρ†ΠΈΠΈ сСчСния ΠΈΠ»ΠΈ ΠΏΠ»ΠΎΡ‰Π°Π΄ΠΈ с Π΅Π³ΠΎ собствСнной Ρ†Π΅Π½Ρ‚Ρ€Π°Π»ΡŒΠ½ΠΎΠΉ оси Π½Π° Π΄Ρ€ΡƒΠ³ΡƒΡŽ ΠΏΠ°Ρ€Π°Π»Π»Π΅Π»ΡŒΠ½ΡƒΡŽ ось. Из матСматичСского Π°Π½Π°Π»ΠΈΠ·Π° извСстно, Ρ‡Ρ‚ΠΎ это:

    Ix = Ic + Ad 2

    Π“Π΄Π΅:

    Ix = ΠΌΠΎΠΌΠ΅Π½Ρ‚ ΠΈΠ½Π΅Ρ€Ρ†ΠΈΠΈ ΠΎΡ‚Π½ΠΎΡΠΈΡ‚Π΅Π»ΡŒΠ½ΠΎ оси x-x (Π² 4 )
    Ic = ΠΌΠΎΠΌΠ΅Π½Ρ‚ ΠΈΠ½Π΅Ρ€Ρ†ΠΈΠΈ ΠΎΡ‚Π½ΠΎΡΠΈΡ‚Π΅Π»ΡŒΠ½ΠΎ Ρ†Π΅Π½Ρ‚Ρ€Π°Π»ΡŒΠ½ΠΎΠΉ оси c-c, ΠΏΠ°Ρ€Π°Π»Π»Π΅Π»ΡŒΠ½ΠΎΠΉ x-x (Π² 4 )
    A = ΠΏΠ»ΠΎΡ‰Π°Π΄ΡŒ сСчСния (Π² 2 )
    d = расстояниС ΠΏΠΎ пСрпСндикуляру ΠΌΠ΅ΠΆΠ΄Ρƒ ΠΏΠ°Ρ€Π°Π»Π»Π΅Π»ΡŒΠ½Ρ‹ΠΌΠΈ осями x-x ΠΈ c-c (Π΄ΡŽΠΉΠΌΡ‹)

    Π€ΠΎΡ€ΠΌΡƒΠ»Π° ΠΏΠ΅Ρ€Π΅Π΄Π°Ρ‡ΠΈ

    Π”Π°Π½ΠΎ:
    ΠΏΡ€ΠΈΠΊΠ»Π΅Π΅Π½Π½Ρ‹ΠΉ асиммСтричный нарост Π²Π½ΠΈΠ·Ρƒ.

ΠžΡΡ‚Π°Π²ΠΈΡ‚ΡŒ ΠΊΠΎΠΌΠΌΠ΅Π½Ρ‚Π°Ρ€ΠΈΠΉ